322

Click here to load reader

Eserciziario Di Analisi Matematica 2

Embed Size (px)

Citation preview

Page 1: Eserciziario Di Analisi Matematica 2
Page 2: Eserciziario Di Analisi Matematica 2
Page 3: Eserciziario Di Analisi Matematica 2
Page 4: Eserciziario Di Analisi Matematica 2
Page 5: Eserciziario Di Analisi Matematica 2

Mathematics Magazine, Vol. 82, December 2008

1806. Proposed by C. Pohoata (Romania).

Let M be a point on the circumcircle of triangle ABC and lying on the arc BC thatdoes not contain A. Let I be the incenter of ABC, and let E and F be the feet of theperpendiculars from I to lines MB and MC, respectively. Prove that the value of

IE + IF

AM

is independent of the position of M .

Solution proposed by G.R.A.20 Problem Solving Group, Roma, Italy.

Let α, β, γ and θ be respectively the angles CAB, ABC, BCA and BCM and let a,b, c be the sides CB, AC, AB.

Since IB bisects β and CMB = π − α (ABMC is cyclic), then IBE = β/2 + α− θand

IE = IB sin(β/2 + α− θ) = rsin(β/2 + α− θ)

sin(β/2).

Similarly

IF = IC sin(γ/2 + θ) = rsin(γ/2 + θ)

sin(γ/2).

Moreover,CM

sin(α− θ)=

BM

sin(θ)=

a

sin(α)=

b

sin(β)=

c

sin(γ)= 2R

and therefore, since ABMC is cyclic, we have that

AM = CM · ca

+MB · ba

=2R

sin(α)(sin(α− θ) sin(γ) + sin(θ) sin(β)) .

HenceIE + IF

AM=r sin(α) sin(γ) sin(β)

2R sin(β/2) sin(γ/2)· f(θ)

where

f(θ) =sin(β/2 + α− θ) sin(γ/2) + sin(γ/2 + θ) sin(β/2)

sin(α− θ) sin(γ) + sin(θ) sin(β).

Reminding that α = π − β − γ, after some tedious computations we verified thatf(θ) = f(0) for θ ∈ [0, α] that is (IE+ IF )/AM is independent of the position of M .

Page 6: Eserciziario Di Analisi Matematica 2

Mathematics Magazine, Vol. 81, October 2008

1803. Proposed by M.W. Botsko (USA).

Let (X, 〈〉) be a real inner product space, and let

B = x ∈ X : ||x|| ≤ 1

be the unit ball in X, where ||x|| =√〈x, x〉. Let f : B → B be a function satisfying

||f(x) − f(y)|| ≤ ||x − y|| for all x, y ∈ B. Prove that the set of fixed points of f isconvex.

Solution proposed by G.R.A.20 Problem Solving Group, Roma, Italy.

We will show that the set of fixed points of f is convex in the more general settingthat X is a strictly convex normed space, that is

x 6= y, ‖x‖ ≤ 1, ‖y‖ ≤ 1 implies that∥∥x+y

2

∥∥ < 1.

Note that a real inner product space is strictly convex because if x 6= y, ||x|| ≤ 1,||y|| ≤ 1 then∥∥∥∥x+ y

2

∥∥∥∥2

=1

4〈x+ y, x+ y〉 =

1

2||x||2 +

1

2||y||2 −

∥∥∥∥x− y2

∥∥∥∥2

≤ 1−∥∥∥∥x− y2

∥∥∥∥2

< 1.

If the set of fixed points is empty or it contains just one point there is nothing toprove. Assume that x, y are two different fixed points of f and let z = αx+ (1− α)ywith α ∈ [0, 1] then

‖x− f(z)‖ = ‖f(x)− f(z)‖ ≤ ‖x− z‖ ≤ (1− α)‖x− y‖.

If f(z) 6= z then, since X is strictly convex, ‖x − f(z)‖ ≤ (1 − α)‖x − y‖ and‖x− z‖ ≤ (1− α)‖x− y‖ implies that∥∥∥∥x− z + f(z)

2

∥∥∥∥ < (1− α)‖x− y‖.

In a similar way we can show that∥∥∥∥y − z + f(z)

2

∥∥∥∥ < α‖x− y‖.

Hence we get the following contradiction

‖x− y‖ ≤∥∥∥∥x− z + f(z)

2

∥∥∥∥+

∥∥∥∥y − z + f(z)

2

∥∥∥∥ < (1− α)‖x− y‖+ α‖x− y‖ = ‖x− y‖

and therefore f(z) = z, that is any convex combination of x and y is a fixed point off .

Note that if X is not strictly convex than the statement can be false: let X = R2 withthe norm ‖(x1, x2)‖ = max(|x1|, |x2|) and consider the non-expanding map f : B → Bgiven by f(x1, x2) = (x1, |x1|). Then the set of fixed points of f is not convex:x = (1, 1) and y = (−1, 1) are fixed points but (x+ y)/2 = (0, 1) is not.

Page 7: Eserciziario Di Analisi Matematica 2

Mathematics Magazine, Vol. 81, June 2008

1797. Proposed by O. Furdui (USA).

Let a, b and c be nonnegative real numbers. Find the value of

limn→∞

n∑k=1

√n2 + kn + a√

n2 + kn + b√

n2 + kn + c.

Solution proposed by G.R.A.20 Problem Solving Group, Roma, Italy.

Since√

n2 + kn + a = n

√1 +

k

n

√1 +

a

n2 + kn

then, for k = 1, . . . , n (remember that a is nonnegative)

n

√1 +

k

n

√1 +

a

2n2≤√

n2 + kn + a ≤ n

√1 +

k

n

√1 +

a

n2.

Hence, we have an upper bound

n∑k=1

√n2 + kn + a√

n2 + kn + b√

n2 + kn + c≤

√1 + a

2n2√1 + b

n2

√1 + c

n2

· 1

n

n∑k=1

1√1 + k

n

and a lower bound

n∑k=1

√n2 + kn + a√

n2 + kn + b√

n2 + kn + c≥

√1 + a

n2√1 + b

2n2

√1 + c

2n2

· 1

n

n∑k=1

1√1 + k

n

.

Since both the upper bound and the lower bound, as n goes to infinity, tend to∫ 1

0

1√1 + x

dx = 2(√

2− 1)

this is also the value of our limit.

Page 8: Eserciziario Di Analisi Matematica 2

Mathematics Magazine, Vol. 81, June 2008

1796. Proposed by M. McMullen (USA).

A point is selected at random from the region inside of a regular n-gon. What is theprobability that the point is closer to the center of the n-gon than it is to n-gon itself?

Solution proposed by G.R.A.20 Problem Solving Group, Roma, Italy.

Consider the right triangle with vertices: the center of the n-gon, the midpoint of aside, a vertex of that side. We can assume that its cartesian coordinates are respec-tively: (0, 2), (0, 0), (2 tan(π/n), 0). The locus of points which have the same distancefrom the center and from the side is a parabola whose equation is x2 + (y− 2)2 = y2,that is

y =x2

4+ 1.

The arc of this parabola, which goes from (0, 1) to (a, a2/4 + 1), separates the righttriangle in two parts and the fraction of the area of the triangle that contains thecenter is the probability that we are looking for:

p =1

2 tan(π/n)

(a

2

(2 +

(a2

4+ 1

))−

∫ a

0

(x2

4+ 1

)dx

)=

a

4 tan(π/n)

(a2

12+ 1

).

a can be found by intersecting the parabola with the line which contains the hy-potenuse. Therefore a is the positive solution of the equation

a2

4+ 1 = 2− a

tan(π/n)

which yields a = 2 tan(π/(2n)). Finally, we find that

p =a

4 tan(π/n)

(a2

12+ 1

)=

1

3

(1− 1

4 cos4(π/(2n))

).

Note that, as one can expect, the limit as n goes to infinity is 1/4.

Page 9: Eserciziario Di Analisi Matematica 2

Mathematics Magazine, Vol. 81, April 2008

1782. Proposed by H. A. ShahAli, Iran.

Let N be a positive integer. Prove that there is a positive integer n such thatn2 + 3 is divisible by at least N distinct primes.

Solution proposed by G.R.A.20 Problem Solving Group, Roma, Italy.

It suffices to find an increasing sequence of positive numbers nNN>0 suchthat: n2

N +3 divides n2N+1+3 with quotient QN > 1 and gcd(n2

N +3, QN) = 1.Then, by induction, n2

1 + 3 > 3 has at least one prime factor and n2N+1 + 3

has at least one more prime factor than n2N + 3 (given by QN).

Let P (s) = s2 + s + 1 then P (s2) = P (s)P (s− 1) and for any integer s > 0,since P (s− 1) is odd,

gcd(2mP (s), P (s− 1)) = gcd(2m(P (s)− P (s− 1)), P (s− 1))

= gcd(2m+1s, P (s− 1)) = gcd(s, s2 − s + 1)

= gcd(s, 1) = 1.

Take nN = 2k2N−1+ 1 for k > 1 then n2

N + 3 = 4P (k2N−1),

n2N+1 + 3 = 4P (k2N

) = 4P (k2N−1

)P (k2N−1 − 1) = (n2N + 3)P (k2N−1 − 1)

and

gcd(n2N + 3, P (k2N−1 − 1)) = gcd(4P (k2N−1

), P (k2N−1 − 1)) = 1.

Page 10: Eserciziario Di Analisi Matematica 2

Mathematics Magazine, Vol. 81, April 2008

1791. Proposed by Mowaffaq Hajja, Jordan.

Let ABC be a triangle with circumcenter O, perimeter P , and area K. Provethat if

BC

P=

[OBC]

K=

1

3,

then ABC is equilateral.

Solution proposed by G.R.A.20 Problem Solving Group, Roma, Italy.

Let a = BC > 0, b = CA > 0, c = AB > 0, and p = P/2.Since 3[OBC] = K, then, by Heron’s formula,

9(R +

a

2

) (R− a

2

) (a

2

) (a

2

)= p(p− a)(p− b)(p− c)

Since R = abc/4K, then

9a4b2c2 = 64p2(p− a)2(p− b)2(p− c)2 + 36a4p(p− a)(p− b)(p− c).

Since a = BC = P/3 then p = 3a/2 and a = (b + c)/2. Therefore

4b2c2 = 9(3a− 2b)2(3a− 2c)2 + 3a2(3a− 2b)(3a− 2c),

and, after expanding and simplifying, we find

2bc = b2 + c2

that is b = c = a and we have proved that ABC is equilateral.

Page 11: Eserciziario Di Analisi Matematica 2

Mathematics Magazine, Vol. 81, February 2008

1786. Proposed by Marian Tetiva, Romania.

Let n ≥ 2 be a positive integer and let On = 1, 3, . . . , 2n − 1 be the set ofodd positive integers less than or equal to 2n− 1.

a. Prove that if m is a positive integer with 3 ≤ m ≤ n2 and m 6= n2 − 2,then m can be written as a sum of distinct elements from On.

b. Prove that n2 − 2 cannot be written as a sum of distinct elements of On.

Solution proposed by G.R.A.20 Problem Solving Group, Roma, Italy.

We will prove that an integer m ∈ [0, n2] can be written as a sum of distinctelements of On (possibly empty sum) iff m 6∈ 2, n2 − 2.

Since ∑k∈On

k = n2

then m ∈ [0, n2] can be written as a sum of distinct elements of On iff n2−mhas this property. Moreover 2, and therefore n2 − 2, cannot be written as asum of distinct elements of On because since 2 > 0 and 2 6∈ On we need atleast two elements of On. But if S ⊂ On with |S| ≥ 2 then∑

k∈On

k ≥ 1 + 3 > 2.

We will prove the rest of the statement by induction:For n = 2 the property holds trivially. Now we assume that the propertyholds for n ≥ 2, and we prove that it holds for n + 1. If

m ∈ [0, n2] \ n2 − 2, 2

then m can be written as a sum of distinct elements from On ⊂ On+1. If

m ∈ [2n + 1, (n + 1)2] \ (n + 1)2 − 2, 2n + 3

thenm− (2n + 1) ∈ [0, n2] \ n2 − 2, 2

and therefore m can be written as a sum of distinct elements from On ⊂ On+1

plus the odd number (2n + 1) ∈ On+1 \On. Now, since n ≥ 2 then

[0, n2]\n2−2, 2∪[2n+1, (n+1)2]\(n+1)2−2, 2n+3 = [0, (n+1)2]\(n+1)2−2, 2

and the property holds also for (n + 1).

Page 12: Eserciziario Di Analisi Matematica 2

The College Mathematics Journal, November 2008

887. Proposed by J.L. Diaz-Barrero (Spain).

Evaluate

limn→∞

n∏k=1

(1 +

k

n

)√n/k3

.

Solution proposed by G.R.A.20 Problem Solving Group, Roma, Italy.

We first transform the product in a sum by applying the logarithm

log

n∏k=1

(1 +

k

n

)√n/k3

=1

n

n∑k=1

(k

n

)−3/2

log

(1 +

k

n

).

Since such sum is actually a Riemann sum, its limit is the following integral∫ 1

0

x−3/2 log(1 + x) dx = −2[x−1/2 log(1 + x)]10 + 2

∫ 1

0

x−1/2

1 + xdx

= −2 log 2 + 2

∫ 1

0

2

1 + t2dt

= − log 4 + 4[arctan t)]10 = − log 4 + π

Therefore

limn→∞

n∏k=1

(1 +

k

n

)√n/k3

= exp(− log 4 + π) =eπ

4.

Page 13: Eserciziario Di Analisi Matematica 2

The College Mathematics Journal, November 2008

887. Proposed by J.L. Diaz-Barrero (Spain).

Evaluate2n+1∑k=1

2− zk

2 + zk

where z1, z2, . . . , z2n+1 are the (2n + 1)’st roots of unity, and z denotes the complexconjugate of z.

Solution proposed by G.R.A.20 Problem Solving Group, Roma, Italy.

Let N > 1 and let zk be one of the N -roots of unity. If a > 1 then |zk/a| < 1 and

a− zk

a + zk

=1− zk

a

1 + zk

a

= −zk

a+

a2 + 1

a2

1

1 + zk

a

= −zk

a+

a2 + 1

a2

∞∑m=0

(−zk

a

)m

.

Hence, since

1

N

N∑k=1

zmk = [N |m]

we have that

N∑k=1

a− zk

a + zk

=N∑

k=1

(−zk

a+

a2 + 1

a2

∞∑m=0

(−zk

a

)m)

= −1

a

N∑k=1

zk +a2 + 1

a2

∞∑m=0

(−1

a

)m N∑k=1

zmk

=a2 + 1

a2

∞∑m=0

(−1

a

)m

N [N |m]

= Na2 + 1

a2

∞∑j=0

(−1

a

)Nj

= Na2 + 1

a2

1

1−(− 1

a

)N = NaN + aN−2

aN − (−1)N.

Hence, for N = 2n + 1 and a = 2 we have that

2n+1∑k=1

2− zk

2 + zk

=10(2n + 1)4n−1

2 · 4n + 1.

Page 14: Eserciziario Di Analisi Matematica 2

The College Mathematics Journal, September 2008

885. Proposed by O. Furdui (USA).

Evaluate ∫ ∞0

∫ ∞0

log(1 + x2 + y2)

(1 + x2)(1 + y2)dx dy.

Solution proposed by G.R.A.20 Problem Solving Group, Roma, Italy.

We first use the polar coordinates

I =

∫ ∞0

∫ ∞0

log(1 + x2 + y2)

(1 + x2)(1 + y2)dx dy

=

∫ ∞ρ=0

∫ π/2

θ=0

log(1 + ρ2)

(1 + ρ2 cos2 θ)(1 + ρ2 sin2 θ)ρdρ dθ

=

∫ ∞ρ=0

log(1 + ρ2)

(2 + ρ2)

(∫ π/2

θ=0

1

1 + ρ2 cos2 θdθ +

∫ π/2

θ=0

1

1 + ρ2 sin2 θdθ

)ρdρ

= π

∫ ∞0

ρ log(1 + ρ2)

(2 + ρ2)√

1 + ρ2dρ.

Let u = 1/√

1 + ρ2. Then

I = = 4π

∫ 1

0

(log u)2

1 + u2du

= 4π limt→1+

∫ t

0

(log u)2

∞∑k=0

(−1)ku2k du

= 4π∞∑

k=0

(−1)k

∫ 1

0

u2k(log u)2 du

= 8π∞∑

k=0

(−1)k

(2k + 1)3= 8π · π3

32=

π4

4.

Page 15: Eserciziario Di Analisi Matematica 2

The College Mathematics Journal, September 2008

883. Proposed by B. Bradie (USA).

Evaluate

(a)

∫ 1

0

log(1 + x)

1 + x2dx and (b)

∫ 1

0

arctan x

1 + xdx

Solution proposed by G.R.A.20 Problem Solving Group, Roma, Italy.

As regards (a), let x = tan s then∫ 1

0

log(1 + x)

1 + x2dx =

∫ π/4

0

log(1 + tan s) ds

=

∫ π/4

0

log(1 + tan(π/4− s)) ds

=

∫ π/4

0

log

(1 +

1− tan s

1 + tan s

)ds

=π log 2

4−

∫ π/4

0

log (1 + tan s) ds.

Hence (note that the integral (a) is finite)∫ 1

0

log(1 + x)

1 + x2dx =

π log 2

8

As regards (b), by integrating by parts, we obtain∫ 1

0

arctan x

1 + xdx =

∫ 1

0

arctan x d (log(1 + x))

= [arctan x log(1 + x)]10 −∫ 1

0

log(1 + x) d (arctan x)

=π log 2

4−

∫ 1

0

log(1 + x)

1 + x2=

π log 2

8.

Page 16: Eserciziario Di Analisi Matematica 2

The College Mathematics Journal, September 2008

881. Proposed by K. McInturff (USA).

Prove that for every positive integer n

n−1∑k=0

(2k

k

)=

n∑k=1

2 sin(2kπ/3)√3

(2n

n + k

).

Solution proposed by G.R.A.20 Problem Solving Group, Roma, Italy.

We prove the identity by induction. It holds for n = 1.Assume that it holds for n > 1, then it suffices to prove that(

2n

n

)=

n∑k=0

(2k

k

)−

n−1∑k=0

(2k

k

)=

n+1∑k=1

(k

3

) (2n + 2

n + 1 + k

)−

n∑k=1

(k

3

) (2n

n + k

)=

∞∑k=1

(k

3

) ((2n + 2

n + 1 + k

)−

(2n

n + k

))

where (k/3) = 2 sin(2kπ/3)/√

3 is the Legendre symbol. Note that (k/3) is equal to0, 1, −1 whether k is equal to 0, 1, −1 mod 3. Since(

2n + 2

n + 1 + k

)=

(2n + 1

n + k

)+

(2n + 1

n + 1 + k

)=

(2n

n + k − 1

)+2

(2n

n + k

)+

(2n

n + k + 1

)then

∞∑k=1

(k

3

) ((2n + 2

n + 1 + k

)−

(2n

n + k

))=

∞∑k=1

(k

3

) ((2n

n + k − 1

)+

(2n

n + k

)+

(2n

n + k + 1

))=

(2n

n

)+

∞∑k=2

((k − 1

3

)+

(k

3

)+

(k + 1

3

)) (2n

n + k

)=

(2n

n

)because for any integer k (

k − 1

3

)+

(k

3

)+

(k + 1

3

)= 0

Page 17: Eserciziario Di Analisi Matematica 2

The College Mathematics Journal, March 2008

874. Proposed by M. S. Becker (USA).

Let f(x) = 1/(1 + x2) and let f (n)(x) denote its nth derivative. Show that

∞∑n=0

(f (4n)(1)

(4n)!+

f (2n)(1)

(2n)!

)= 1

Solution proposed by G.R.A.20 Problem Solving Group, Roma, Italy.

Let’s consider the expansion of the complex rational function f(z) = 1/(1 + z2) atz0 = 1

f(z) =∞∑

n=0

an(z − 1)n for z ∈ D = |z − 1| < |i− 1| =√

2

Let Ak =∑∞

n=0 a4n+k for k = 0, 1, 2, 3. Since an = f (n)(1)/n!, we have to prove that

∞∑n=0

(f (4n)(1)

(4n)!+

f (2n)(1)

(2n)!

)= A0 + (A0 + A2) = 2A0 + A2 = 1.

Since 0, 2, 1 + i ∈ D then

f(0) = A0 − A1 + A2 − A3 = 1

f(2) = A0 + A1 + A2 + A3 =1

5

f(1 + i) = A0 + iA1 − A2 − iA3 =1− 2i

5

and by solving this linear system of equations we find that

A0 =2

5A1 = −2

5A2 =

1

5A3 = 0

and finally we verify that 2A0 + A2 = 1.

Page 18: Eserciziario Di Analisi Matematica 2

The College Mathematics Journal, March 2008

873. Proposed by O. Furdui (USA).

Find

(a)∞∑

n=1

(−1)n−1 ln n

nand (b)

∞∑n=1

(−1)n−1 ln2 n

n

Solution proposed by G.R.A.20 Problem Solving Group, Roma, Italy.

The zeta function

ζ(s) =∞∑

n=1

1

ns

has a unique analytic continuation to C \ 1. At s = 1 it has a simple pole and itsexpansion there gives

ζ(s) =1

s− 1+

∞∑n=0

(−1)n

n!γn(s− 1)n

where γn is the n-th Stieltjes constant (γ0 = γ is the Eulero-Mascheroni’s constant).Therefore around 1 we can use the following approximations

ζ(s) =1

s− 1+ γ + o(1),

ζ ′(s) = − 1

(s− 1)2− γ1 + o(1),

ζ ′′(s) =2

(s− 1)3+ γ2 + o(1).

Moreover, for Re(s) > 0 and s 6= 1, the following identity holds

(1− 21−s)ζ(s) =∞∑

n=1

(−1)n−1

ns

(a) By deriving this identity with respect to s, we obtain

(1− 21−s)ζ ′(s) + 21−sζ(s) ln 2 = −∞∑

n=1

(−1)n−1 ln n

ns.

Since

21−s = exp(−(s−1) ln 2) = 1−(ln 2)(s−1)+(ln 2)(s−1)2/2−(ln 2)3(s−1)2/6+o(s−1)3

then, by using the above approximations of ζ(s) and its derivatives around s0 = 1,and by taking the limit as s → 1, we find that

∞∑n=1

(−1)n−1 ln2 n

n=

(ln 2)2

2− γ log 2 ≈ −0.15986890.

Page 19: Eserciziario Di Analisi Matematica 2

(b) By deriving the above identity two times with respect to s, we obtain

(1− 21−s)ζ ′′(s) + 2 · 21−sζ(s) ln 2− 21−s(ln 2)2ζ(s) =∞∑

n=1

(−1)n−1 ln2 n

ns.

and, in similar way as in case (a), we find that

∞∑n=1

(−1)n−1 ln2 n

n=

(ln 2)3

3− γ(log 2)2 − γ1(2 ln 2) ≈ −0.06537259.

2

Page 20: Eserciziario Di Analisi Matematica 2

The College Mathematics Journal, January 2008

870. Proposed by M. K. Azarian (USA).

Prove that∞∑i=0

(−1)i

∫ 1

0

16x3(1 + x8i)

(1 + x4)2i+2dx = π.

Solution proposed by G.R.A.20 Problem Solving Group, Roma, Italy.

Letting

f(x) =1

(1 + x4)2and g(x) =

x8

(1 + x4)2

we have that (note that |f(x)| < 1 and |g(x)| < 1 for x ∈ (0, 1])

∞∑i=0

(−1)i

∫ 1

0

16x3(1 + x8i)

(1 + x4)2i+2dx =

∫ 1

0

16x3

(1 + x4)2

∞∑i=0

(−1)i(f i(x) + gi(x)) dx

=

∫ 1

0

16x3

(1 + f(x))(1 + x4)2dx +

∫ 1

0

16x3

(1 + g(x))(1 + x4)2dx

= 4

∫ 1

0

4x3

(1 + x4)2 + 1dx + 4

∫ 1

0

4x3

(1 + x4)2 + x8dx

= 4[arctan(1 + x4)]10 + 4[arctan(1 + 2x4)]10= 4(arctan 2 + arctan 3)− 2π = 4(π − arctan 1)− 2π = π

where, in the last equality, we used the known fact that

arctan 1 + arctan 2 + arctan 3 = π.

Page 21: Eserciziario Di Analisi Matematica 2

The College Mathematics Journal, January 2008

869. Proposed by V. Radulescu (Romania).

Prove that there does not exist a positive twice-differentiable function f defined on[0, +∞) such that

f(x)f ′′(x) ≤ −1 for all x ≥ 0.

Solution proposed by G.R.A.20 Problem Solving Group, Roma, Italy.We will prove a more general result: if a > 0 then there is no positive twice-differentiable function f defined on [0, +∞) such that

f(x)f ′′(x) ≤ −a for all x ≥ 0.

Note that the bound −a can’t be replaced by 0 because f(x) = 1 + log(1 + x) is apositive twice-differentiable function defined on [0, +∞) such that

f(x)f ′′(x) = −1 + log(1 + x)

(1 + x)2≤ 0 for all x ≥ 0.

Proof. By contradiction, we assume that such function f exists.

1) f has no upper bound: if f(x) ≤ L for some L > 0 then for x ≥ 0

f ′′(x) ≤ − a

f(x)≤ − a

L

and by integrating this inequality two times over the interval [0, x] we obtain that

f(x) ≤ f(0) + f ′(0)x− a

2Lx2.

Since the RHS goes to −∞ as x → +∞ and the LHS is positive we have a contra-diction.

2) f ′ is positive: if there is x0 ≥ 0 such that f ′(x0) ≤ 0 then, since f is concave (f ′′(x)is negative by the hypothesis),

f(x) ≤ f(x0) + f ′(x0)(x− x0) ≤ f(x0) for x ≥ x0.

which implies that f(x) has max[0,x0] f(x) as upper bound, contradicting 1).

3) Final step: by 2) and 1) f is strictly increasing and f(x)→ +∞ as x→ +∞.By hypothesis, for x ≥ 0

D[(f ′(x))2] = 2f ′(x)f ′′(x) ≤ −2af ′(x)

f(x)= −2aD[log(f(x))]

and by integrating this inequality over the interval [0, x] we have

(f ′(x))2 − (f ′(0))2 ≤ −2a(log f(x)− log f(0)),

that is(f ′(x))2 ≤ (f ′(0))2 + 2a log f(0)− 2a log f(x)

Since the RHS goes to −∞ as x → +∞ and the LHS is positive we have a contra-diction.

Page 22: Eserciziario Di Analisi Matematica 2

The College Mathematics Journal, January 2008

868. Proposed by A. R. Miller (USA).

Evaluate the double infinite sum

∞∑m=0

∞∑n=0

1

Γ(m+n2

)Γ(m+n+12

),

where Γ(x) is the Gamma function.

Solution proposed by G.R.A.20 Problem Solving Group, Roma, Italy.

Since m + n = k for m, n ≥ 0 has k + 1 solutions then

S =∞∑

m=0

∞∑n=0

1

Γ(m+n2

)Γ(m+n+12

)=

∞∑k=0

k + 1

Γ(k2)Γ(k

2+ 1

2)

=∞∑

k=1

k + 1

Γ(k2)Γ(k

2+ 1

2)

where we dropped the first term because the Γ function has a (simple) pole at 0. Notethat we can alter the summation order because the terms are all non-negative. Sincethe duplication formula holds

Γ(z)Γ(z + 1/2) =√

π21−2zΓ(2z)

we have that

S =1√π

∞∑k=1

(k + 1)2k−1

Γ(k)=

1√π

∞∑k=1

(k + 1)2k−1

(k − 1)!

=1√π

∞∑k=0

(k + 2)2k

k!=

2√π

∞∑k=1

2k−1

(k − 1)!+

2√π

∞∑k=0

2k

k!

=4√π

∞∑k=0

2k

k!=

4e2

√π

.

Page 23: Eserciziario Di Analisi Matematica 2
Page 24: Eserciziario Di Analisi Matematica 2
Page 25: Eserciziario Di Analisi Matematica 2
Page 26: Eserciziario Di Analisi Matematica 2

Mathematics Magazine, Vol. 80, December 2007

1781. Proposed by Paul Bracken, USA.

Let γ be Euler’s constant and for positive integer n define

γn =n∑

k=1

1

k− log n and αn = 2n(γn − γ).

Prove that the sequence αn is monotonically increasing and bounded above.In addition determine lim

n→∞αn.

Solution proposed by G.R.A.20 Problem Solving Group, Roma, Italy.

It is well known that

Hn =n∑

k=1

1

k= γ + log n +

1

2n− 1

12n2+ o

(1

n2

).

therefore

αn = 2n(γn − γ) = 2n(Hn − log n− γ) = 1− 1

6n+ o

(1

n

)and it follows that the sequence αn is bounded and lim

n→∞αn = 1.

Let’s prove the last property: the sequence αn is increasing that is βn :=(αn+1 − αn)/2 > 0 for n ≥ 1. Since βn → 0 then

βn = limN→∞

(βn − βN) = limN→∞

N−1∑k=n

(βk − βk+1)

so it suffices to show that βn > βn+1 for n ≥ 1.Since βn = Hn + 1− [(n + 1) log(n + 1)− n log n]− γ then letting

f(x) =1

x− (x + 1) log(x + 1) + 2x log x− (x− 1) log(x− 1)

we have that

βn − βn+1 = −f(n + 1) =

∫ +∞

n+1

f ′(x) dx > 0

because limx→+∞

f(x) = 0 and for x > 1

f ′(x) = − 1

x2−log(x+1)+2 log x−log(x−1) =

(− 1

x2

)−log

(1 +

(− 1

x2

))> 0

(remember that t > log(1 + t) for t ∈ (−1, 0)).

Page 27: Eserciziario Di Analisi Matematica 2

Mathematics Magazine, Vol. 80, October 2007

1780. Proposed by Yiu Tung Poon, USA.

For positive integer n, define odd(2n) to be the number of odd digits in the(base-ten) expansion of 2n. Prove that

∞∑n=1

odd(2n)

2n=

1

9.

Solution proposed by G.R.A.20 Problem Solving Group, Roma, Italy.

For any k ≥ 1, 1/10k has a unique (base-2) representation (avoid a tail ofones)

1

10k=

∞∑j=1

aj

2jwith aj ∈ 0, 1.

So for any n ≥ 1

2n

10k=

n−1∑j=1

2n−jaj + an + α with α ∈ [0, 1)

and ⌊2n

10k

⌋mod 2 = an = oddk+1(2

n)

where oddk+1(n) is 1 if the (k + 1)-th digit of 2n is odd and 0 otherwise.Therefore we proved that for k ≥ 1 then

∞∑n=1

oddk+1(2n)

2n=

1

10k.

Hence (note that the series has non negative terms and it is convergent)

∞∑n=1

odd(2n)

2n=

∞∑n=1

1

2n

∞∑k=0

oddk+1(2n)

=∞∑

n=1

1

2n

∞∑k=1

oddk+1(2n)

=∞∑

k=1

∞∑n=1

oddk+1(2n)

2n

=∞∑

k=1

1

10k=

1

9.

Page 28: Eserciziario Di Analisi Matematica 2

Mathematics Magazine, Vol. 80, June 2007

1771. Proposed by Mowaffaq Hajja, Jordan.

Let P be a point inside a triangle ABC, and let AA′, BB′, and CC ′ be thecevians through P . Prove that if A′B′ = A′C ′ and BC ′ = CB′, then thetriangle ABC is isosceles.

Solution proposed by G.R.A.20 Problem Solving Group, Roma, Italy.

Let

x1 = BA′, x2 = CA′, y1 = CB′, y2 = AB′, z1 = AC ′, z2 = BC ′.

They are all positive numbers because the point P is inside the triangle.Since A′B′ = A′C ′ then

x22 + y2

1 − 2x2y1 cos(BCA) = x21 + z2

2 − 2x1z2 cos(ABC).

By eliminating the cosines we have

x22 + y2

1 − 2x2y1

(x2 + y2 − z2

2xy

)= x2

1 + z22 − 2x1z2

(x2 + z2 − y2

2xz

)where x = x1 + x2, y = y1 + y2, z = z1 + z2. Then

xyz(x22 + y2

1 − x21 − z2

2) + x2(yx1z2 − zx2y1) + (yx1z2 + zx2y1)(z2 − y2) = 0

We know that z2 = y1 and since by Ceva’s theorem x1y1z1 = x2y2z2 thenx2 = z1x1/y2. By eliminating z2 and x2, and by factoring we find

x1

y22

(z1 − y2)Q = 0

where

Q = y1y42 + 3y3

2y21 + y3

2z1y1 + 2y22y

31 + 2y2

2y21z1 + y2

2x21z1

+y22z

21y1 + 3y2y

21z

21 + 2y2z

21x

21 + 2y2z1y

31 + y2z

31y1 + x2

1z31 .

Since Q is positive it follows that z1 = y2. Since z2 = y1 then z = z1 + z2 =y1 + y2 = y and the triangle is isosceles.

Page 29: Eserciziario Di Analisi Matematica 2

Mathematics Magazine, Vol. 80, April 2007

1769. Proposed by Michel Bataille, France.

For positive integer n, let

Pn(x, y) =n∑

k=0

(2n + 1

2k + 1

)xn−k(x + y)k.

Find a closed form expression for the coefficient of xiyj when Pn is expanded.

Solution proposed by G.R.A.20 Problem Solving Group, Roma, Italy.

First note that

Pn(x, y) =n∑

k=0

(2n + 1

2k + 1

)xn−k

k∑j=0

(k

j

)xk−jyj =

n∑j=0

n∑k=j

(2n + 1

2k + 1

)(k

j

)xn−jyj.

Now we prove that

n∑k=j

(2n + 1

2k + 1

)(k

j

)= 4n−j

(2n− j

j

)and therefore for integers i, j ∈ [0, n] we have

[xiyj]Pn(x, y) =

4n−j

(2n−j

j

)if i = n− j

0 otherwise

Since

[zk](1 + z)2n+1 =

(2n + 1

k

)and [zk]

(z2)j

(1− z2)j+1=

0 if k is odd(

k/2j

)if k is even

then

[z2n](1 + z)2n+1(z2)j

(1− z2)j+1=

n∑k=0

(2n + 1

2n− 2k

)(k

j

)=

n∑k=j

(2n + 1

2k + 1

)(k

j

).

On the other hand

[z2n](1 + z)2n+1(z2)j

(1− z2)j+1= [z2n−j]

(1 + z)2n−jzj

(1− z)j+1=

2n−j∑k=0

(2n− j

k

)(2n− j − k

j

)

=

2n−j∑k=0

(2n− j

j

)(2n− 2j

k

)=

(2n− j

j

)4n−j.

because (2n− j

k

)(2n− j − k

j

)=

(2n− j

j

)(2n− 2j

k

).

Page 30: Eserciziario Di Analisi Matematica 2

Mathematics Magazine, Vol. 80, April 2007

1767. Proposed by Mowaffaq Hajja, Jordan.

Let G be the centroid of ∆ABC. Prove that if BAC = 60 and BGC = 120,then the triangle is equilateral.

Solution proposed by G.R.A.20 Problem Solving Group, Roma, Italy.

Let a, b, c be the sides and let ma, mb, mc be the medians of ∆ABC.Since Area(∆BAC) = 3Area(∆BGC) then

1

2bc sin(60) =

3

2

(2mb

3

) (2mc

3

)sin(120)

that is4mbmc = 3bc.

Moreover, it is well known that

4m2b = 2a2 + 2c2 − b2 and 4m2

c = 2a2 + 2b2 − c2.

By cosine rule applied to ∆BGC we have

a2 =

(2mb

3

)2

+

(2mc

3

)2

− 2 · 2mb

3· 2mc

3· cos(120)

then9a2 = (2a2 + 2c2 − b2) + (2a2 + 2b2 − c2) + 3bc

that is5a2 − (c2 + b2 + 3bc) = 0.

By cosine rule applied to ∆BAC we have

a2 = b2 + c2 − 2bc cos(60).

Therefore5(b2 + c2 − bc) − (c2 + b2 + 3bc) = 4(b − c)2 = 0

or b = c. Since a2 = b2 + c2 − bc then a = b = c and ∆ABC is equilateral.

Page 31: Eserciziario Di Analisi Matematica 2

Mathematics Magazine, Vol. 80, February 2007

1764. Proposed by Ovidiu Furdui, USA.

For positive integer n, let gn = 1 +1

2+ · · ·+ 1

n− log(n). Prove that

limn→∞

(gγ

n

γgn

)2n

=e

γ

where γ is the Euler-Mascheroni constant.

Solution proposed by G.R.A.20 Problem Solving Group, Roma, Italy.

Since

gn = Hn − log(n) = γ +1

2n+ o(1/n)

then

log

(gγ

n

γgn

)= γ log(gn)− gn log(γ)

= γ log(γ +1

2n+ o(1/n))− (γ +

1

2n+ o(1/n)) log(γ)

= γ log(γ) + γ log(1 +1

2nγ+ o(1/n))− (γ +

1

2n+ o(1/n)) log(γ)

=1− log(γ)

2n+ o(1/n).

Therefore(gγ

n

γgn

)2n

= exp

(2n log

(gγ

n

γgn

))= exp (1− log(γ) + o(1)) → e

γ.

Page 32: Eserciziario Di Analisi Matematica 2

Mathematics Magazine, Vol. 80, February 2007

1762. Proposed by Erwin Just, USA.

Let n be an integer with n ≥ 2. Prove that for any even integer k, there existodd primes p and q such that p + q = k (mod n).

Solution proposed by G.R.A.20 Problem Solving Group, Roma, Italy.

We have to prove that given two integers n ≥ 2 and m then we can find twoodd primes p and q and an integer x such that

p = (2m− q) + nx.

By Dirichlet’s Theorem, the linear progression x → (2m− q) + nx for x ∈ Ncontains infinite primes as soon as gcd((2m − q), n) = 1 therefore it sufficesto find a prime q such that 2m− q and n are relatively prime.If nα1

1 nα22 · · ·nαs

s is the prime factorization of n then the above condition isequivalent to the following:

gcd(2m− q, ni) = 1 for i = 1, . . . , s.

For any i = 1, . . . , s take an integer ri such that 0 < ri < ni and ri 6=2m (mod ni) and solve the system of congruences

q = 2m− r1 (mod n1)...q = 2m− rs (mod ns)

By the Chinese Remainder Theorem there exists a solution 0 < q0 < n1 · · ·ns

(note that gcd(ni, nj) = 1 for i 6= j). Hence y → q0 + (n1 · · ·ns)y fory ∈ N is a linear progression whose elements q solve the system and there-fore gcd(2m − q, ni) = 1 for i = 1, . . . , s. Moreover, since by constructiongcd(q0, n1 · · ·ns) = 1, then, again by Dirichlet’s Theorem, this progressioncontains infinite primes.

Page 33: Eserciziario Di Analisi Matematica 2

Mathematics Magazine, Vol. 80, February 2007

1761. Proposed by Steve Butler, USA.

For integer n ≥ 2 define the sets

A(n) = (k, j) : 1 ≤ k ≤ j ≤ n, k + j ≤ n, and gcd(k, j) = 1

B(n) = (k, j) : 1 ≤ k ≤ j ≤ n, k + j > n, and gcd(k, j) = 1.

Prove that A(n) and B(n) have the same cardinality.

Solution proposed by G.R.A.20 Problem Solving Group, Roma, Italy.

We denote by

[k, j] =

1 if gcd(k, j) = 10 otherwise

.

We first note that for n ≥ 2

|A(n)|+ |B(n)| =n∑

j=1

j∑k=1

[k, j] =n∑

j=1

ϕ(j).

Moreover for n > 2

|A(n)| − |A(n− 1)| = 1

2

n−1∑k=1

[k, n− k] =1

2

n−1∑k=1

[k, n] =ϕ(n)

2

because gcd(k, n− k) = gcd(k, n).Therefore, since A(2) = (ϕ(1) + ϕ(2))/2 = 1, we have that

|A(n)| = 1

2

n∑j=1

ϕ(j) =1

2(|A(n)|+ |B(n)|)

that is |A(n)| = |B(n)|.

Page 34: Eserciziario Di Analisi Matematica 2

The College Mathematics Journal, November 2007

861. Proposed by DiDomenico (USA).

Show that if P is an even perfect number, then there exist positive integers a < b < csuch that P = a + b + c and

(a + b)2 + (a + c)2 = (b + c)2 ;

that is, (a + b, a + c, b + c) is a Pythagorean triple.

Solution proposed by G.R.A.20 Problem Solving Group, Roma, Italy.

Any even perfect number has the form P = 2n(2n+1 − 1) for n ≥ 1. Let

a = 2n − 1 , b = 2n , c = P − a− b

then 0 < a < b and for n ≥ 1

c− b = 2n(2n+1 − 1)− (2n − 1)− 2 · 2n = 2(2n)2 − 4 · 2n + 1 > 2 · 2n(2n − 2) ≥ 0.

Moreover (a + b)2 + (a + c)2 = (b + c)2 is equivalent to aP = b(P − a − b), that isb(a + b) = P (b− a) and

b(a + b)− P (b− a) = 2n(2 · 2n − 1)− 2n(2n+1 − 1) = 0.

Page 35: Eserciziario Di Analisi Matematica 2

The College Mathematics Journal, May 2007

855. Proposed by Michel Bataille, France.

Show that for |x| < 1(∞∑

n=1

⌊n√2

⌋xn

)(∞∑

n=1

xbn√

3c

)=

(∞∑

n=1

⌊n√3

⌋xn

)(∞∑

n=1

xbn√

2c

)

where buc denotes the floor function.

Solution proposed by G.R.A.20 Problem Solving Group, Roma, Italy.

It suffices to prove that for |x| < 1(∞∑

n=1

⌊n

α

⌋xn

)=

x

1− x

(∞∑

n=1

xbnαc

)=

(∞∑

n=1

xn

)(∞∑

n=1

xbnαc

)

where α is a positive irrational number.All power series are clearly convergent for |x| < 1. Moreover the coefficient of xn ofthe product power series on the right is

∞∑k=1

[n− bkαc ≥ 1]

where [u] denotes the indicator function. Since n/α is never an integer then

[n− bkαc ≥ 1] = [bkαc ≤ n− 1] = [bkαc < n] = [kα < n] = [k < n/α] = [k ≤ bn/αc] .

Therefore∞∑

k=1

[n− bkαc ≥ 1] =∞∑

k=1

[k ≤ bn/αc] =⌊n

α

⌋that is the coefficient of xn of the power series on the left.

Page 36: Eserciziario Di Analisi Matematica 2

The College Mathematics Journal, May 2007

854. Proposed by Ovidiu Furdui, USA.

Prove that for every natural number n ≥ 3∞∑

k=1

Hk

k(k + 1)(k + 2) · · · (k + n)=

1

n!

(π2

6−

(1 +

1

22+ · · ·+ 1

(n− 1)2

))where Hk is the k-th harmonic number

∑kj=1 1/j.

Solution proposed by G.R.A.20 Problem Solving Group, Roma, Italy.

Since∑∞

k=1 1/k2 = π2/6, it suffices to prove that

F (n) := n!∞∑

k=1

Hk

k(k + 1)(k + 2) · · · (k + n)=∞∑

k=n

1

k2.

We first consider the case n = 1:

F (1) =∞∑

k=1

Hk

k(k + 1)=∞∑

j=1

1

j

∞∑k=j

(1

k− 1

k + 1

)=∞∑

j=1

1

j2.

Assume now that n > 1, then

F (n) = n!∞∑

k=1

Hk

n

(1

k(k + 1) · · · (k + n− 1)− 1

k(k + 1) · · · (k + n)

)= F (n− 1)− (n− 1)!

∞∑k=1

Hk

(k + 1) · · · (k + n).

Note that∞∑

k=1

Hk

(k + 1) · · · (k + n)=∞∑

j=1

1

j

∞∑k=j

1

(k + 1) · · · (k + n)

=∞∑

j=1

1

j(n− 1)

∞∑k=j

(1

(k + 1) · · · (k + n− 1)− 1

(k + 2) · · · (k + n)

)

=∞∑

j=1

1

j(n− 1)· 1

(j + 1) · · · (j + n− 1)

=1

(n− 1)

∞∑j=1

1

j(j + 1) · · · (j + n− 1)

=1

(n− 1)2

∞∑j=1

(1

j · · · (j + n− 2)− 1

(j + 1) · · · (j + n− 1)

)=

1

(n− 1)2(n− 1)!.

Page 37: Eserciziario Di Analisi Matematica 2

Therefore

F (n) = F (n− 1)− (n− 1)!

(n− 1)2(n− 1)!= F (n− 1)− 1

(n− 1)2

and the statement is proved by induction.

2

Page 38: Eserciziario Di Analisi Matematica 2

The College Mathematics Journal, May 2007

851. Proposed by Jose Luis Dıaz-Barrero, Spain.

Let a, b, and c be positive real numbers such that ab + bc + ca = 1. Prove that

a√

b2 + c2 + bc + b√

c2 + a2 + ca + c√

a2 + b2 + ab ≥√

3.

Solution proposed by G.R.A.20 Problem Solving Group, Roma, Italy.

The inequality is equivalent to

abf(c/b) + bcf(a/c) + caf(b/a) ≥√

3

where f(x) =√

x2 + x + 1.Since f(x) is convex on the real line and ab + bc + ca = 1 with ab, bc, and capositive real numbers then

abf(c/b)+bcf(a/c)+caf(b/a) ≥ f (ab(c/b) + bc(a/c) + ca(b/a)) = f(1) =√

3

Page 39: Eserciziario Di Analisi Matematica 2

The College Mathematics Journal, March 2007

847. Proposed by Juan-Bosco Romero Marquez, Spain.

Show that if r and R are the radii of the inscribed and circumscribed circles of∆ABC and s is its semiperimeter, then

3√

r2s ≤√

4Rr + r2

3≤ s

3.

Solution proposed by G.R.A.20 Problem Solving Group, Roma, Italy.

We will use Feuerbach’s equations.The first group involve the radii r1, r2, r3 of the ex-circles of ∆ABC

r1 + r2 + r3 = 4R + rr1r2 + r2r3 + r3r1 = s2

r1r2r3 = s2r.

By Symmetric Mean Inequality (AGM),

r1 + r2 + r3

3≥ (r1r2r3)

1/3

that is3√

r2s ≤√

4Rr + r2

3.

The second group involve the sides a1, a2, a3 of ∆ABC.a1 + a2 + a3 = 2sa1a2 + a2a3 + a3a1 = s2 + r(4R + r)a1a2a3 = 4sRr

.

By Symmetric Mean Inequality,

a1 + a2 + a3

3≥

(a1a2 + a2a3 + a3a1

3

)1/2

that is √4Rr + r2

3≤ s

3.

Page 40: Eserciziario Di Analisi Matematica 2

The College Mathematics Journal, March 2007

846. Proposed by Jose Luis Dıaz-Barrero, Spain.

Prove that for every positive integer n

1

n

n∑k=1

1(nk

)k ≥ (n + 1

2n

)n+12

.

Solution proposed by G.R.A.20 Problem Solving Group, Roma, Italy.

By AGM

1

n

n∑k=1

1(nk

)k ≥(

n∏k=1

1(nk

)k) 1

n

.

Therefore, it suffices to prove that

2n

n + 1≥

(n∏

k=1

(n

k

)k) 2

n(n+1)

,

Since

2n = 2 · 2n−1 = 2n∑

k=1

(n− 1

k − 1

)= 2

n∑k=1

k

n

(n

k

)=

2

n

n∑k=1

k

(n

k

)then, after taking the logarithm, the inequality becomes

log

(2n

n + 1

)= log

(2

n(n + 1)

n∑k=1

k

(n

k

))≥ 2

n(n + 1)

n∑k=1

k log

(n

k

)which holds because the logarithm is a concave function and

2

n(n + 1)

n∑k=1

k = 1.

Page 41: Eserciziario Di Analisi Matematica 2

The College Mathematics Journal, January 2007

844. Proposed by Ovidiu Furdui, USA.

Let

L = limn→∞

(n∑

k=1

ln k

k− ln2(n)

2

).

Prove that∞∑

n=1

∑nk=1

1k− γ − ln n

n= −γ2

2+

π2

12− L

where γ is Euler’s constant.

Solution proposed by G.R.A.20 Problem Solving Group, Roma, Italy.

The partial sum is

SN =N∑

n=1

∑nk=1

1k− γ − ln n

n=

N∑n=1

Hn

n− γHN −

N∑n=1

ln n

n

We note that

N∑n=1

Hn

n=

N∑n=1

1

n

n∑k=1

1

k=

1

2

(N∑

n=1

1

n

)2

+1

2

N∑n=1

1

n2=

H2N

2+

1

2

N∑n=1

1

n2.

Since γ = HN − ln N + O(1/N) then

SN =H2

N

2+

1

2

N∑n=1

1

n2− (HN − ln N)HN −

N∑n=1

ln n

n+ O(1/N)HN

= −(HN − ln N)2

2+

1

2

N∑n=1

1

n2−

(N∑

n=1

ln n

n− ln2 N

2

)+ O(1/N)HN .

Hence, by taking the limit we find

limN→∞

SN = −γ2

2+

π2

12− L.

Page 42: Eserciziario Di Analisi Matematica 2

The College Mathematics Journal, January 2007

841. Proposed Mohammad K. Azarian, USA.

Assume that the quadratic polynomial f(x) = ax2 + bx + c, a 6= 0, has two fixed pointsx1 and x2, x1 6= x2. If 1 and −1 are two fixed points of the function f(f(x)), but not off(x), then find the exact values of x1 and x2.

Solution proposed by G.R.A.20 Problem Solving Group, Roma, Italy.

The numbers x1 and x2 satisfy the following equations

f(x)− x = ax2 + (b− 1)x + c = 0

and

f(f(x))− x = a(ax2 + bx + c)2 + b(ax2 + bx + c) + c− x

= a3x4 + 2a2bx3 + C(a, b, c)x2 + D(a, b, c)x + ac2 + bc + c = 0

Since 1 and −1 are the other two solutions of the last equation then−2a2b

a3= x1 + x2 + 1− 1 = x1 + x2 = −b− 1

a

ac2 + bc + c

a3= x1 · x2 · 1 · (−1) = −x1 · x2 = − c

a

.

Therefore b = −1 and c = 0 or b = −1 and c = −a.If b = −1 and c = 0 then f(x) = ax2 − x and

f(f(1)) = f(a− 1) = a(a− 1)2 − (a− 1) = a3 − 2a2 + 1 = 1,

f(f(−1)) = f(a + 1) = a(a + 1)2 − (a + 1) = a3 + 2a2 − 1 = −1

that is a2(a− 2) = 0 and a2(a + 2) = 0 which do not hold if a 6= 0.If b = −1 and c = −a then f(x) = ax2 − x− a = a(x2 − 1)− x and

f(f(1)) = f(−1) = 1, f(f(−1)) = f(1) = −1.

By solvingf(x)− x = a(x2 − 1)− x− x = ax2 − 2x− a = 0

we find the two fixed points of f

x1 =1 +

√1 + a2

aand x2 =

1−√

1 + a2

a.

Page 43: Eserciziario Di Analisi Matematica 2
Page 44: Eserciziario Di Analisi Matematica 2
Page 45: Eserciziario Di Analisi Matematica 2
Page 46: Eserciziario Di Analisi Matematica 2

Mathematics Magazine, Vol. 79, December 2006

1760. Proposed by P. Ivady, Hungary.

Prove that for 0 < a < b <∞√a2 + b2

2+√

ab− a + b

2>

b− a

log(b)− log(a).

Solution proposed by G.R.A.20 Problem Solving Group, Roma, Italy.

We first divide by√

ab:√(a/b) + (b/a)

2−

( 4√

b/a− 4√

a/b)2

2>

√b/a−

√a/b

log(b/a).

Letting t = log(b/a) > 0, we can introduce the hyperbolic functions cosh(x) =(ex + e−x)/2 and sinh(x) = (ex − e−x)/2√

cosh(t)− 2 (sinh(t/4))2 >sinh(t/2)

(t/2)

or equivalently

cosh(t) >

(2 (sinh(t/4))2 +

sinh(t/2)

(t/2)

)2

.

We will prove this inequality by comparing the series expansions at 0 of the two sides(which converge for t > 0). The expansion of the left hand side gives

LHS = cosh(t) = 1 +∞∑

n=1

t2n

(2n)!.

The expansion of the right hand side gives

RHS =

(2 (sinh(t/4))2 +

sinh(t/2)

(t/2)

)2

=

((cosh(t/2)− 1) +

sinh(t/2)

(t/2)

)2

= (cosh(t/2))2 − 2 cosh(t/2) + 1 +

(sinh(t/2)

(t/2)

)2

+2 sinh(t/2) cosh(t/2)

(t/2)− 2 sinh(t/2)

(t/2)

=cosh(t) + 1

2− 2 cosh(t/2) + 1 + 2

cosh(t)− 1

t2+ 2

sinh(t)

t− 2 sinh(t/2)

(t/2)

= 1 +∞∑

n=1

t2n

(2n)!

[1

2− 2

22n+

2

(2n + 2)(2n + 1)+

2

2n + 1− 2

22n(2n + 1)

]

Page 47: Eserciziario Di Analisi Matematica 2

where we used the following identities to reduce the powers and the products of thehyperbolic functions

2 (sinh(x/2))2 = cosh(x)−1, 2 (cosh(x/2))2 = cosh(x)+1, sinh(2x) = 2 sinh(x) cosh(x).

Therefore the inequality is verified as soon as we show that for n ≥ 1

1 >1

2− 2

22n+

2

(2n + 2)(2n + 1)+

2

2n + 1− 2

22n(2n + 1)

that is1

4>

2n + 3

(2n + 2)(2n + 1)− 2n + 2

22n(2n + 1).

The last inequality holds for n = 1 and for n > 1 it easy to prove that

1

4>

2n + 3

(2n + 2)(2n + 1).

2

Page 48: Eserciziario Di Analisi Matematica 2

Mathematics Magazine, Vol. 79, December 2006

1759. Proposed by L. Cusick and M. Nogin, USA.

Let ∆ABC be an equilateral triangle with three external triangles ∆CDB, ∆AEC and

∆AFB. Assume that α = FAB = DCB, β = EAC = DBC, and γ = FBA = ECA.Prove that segments AD, BE, and CF are concurrent.

Solution proposed by G.R.A.20 Problem Solving Group, Roma, Italy.

Let P , Q, and R respectively the intersection between AD and CB, EB and AC,and CF and AB. It is easy to see that

|CP ||PB|

=tan β(tan α +

√3)

tan α(tan β +√

3).

Similarly

|BR||RA|

=tan α(tan γ +

√3)

tan γ(tan α +√

3)and

|AQ||QC|

=tan γ(tan β +

√3)

tan β(tan γ +√

3).

Therefore|CP ||PB|

· |BR||RA|

· |AQ||QC|

= 1

and concurrency follows from Ceva’s Theorem.

Page 49: Eserciziario Di Analisi Matematica 2

Mathematics Magazine, Vol. 79, December 2006

1758. Proposed by M. Goldenberg and M. Kaplan, USA.

Let Fn be the nth Fibonacci number, that is, F0 = 0, F1 = 1, and Fn = Fn−1 + Fn−2

for n ≥ 2. Prove that∞∏

n=2

F2n + 1

F2n − 1= 3.

Solution proposed by G.R.A.20 Problem Solving Group, Roma, Italy.

First we note that for N ≥ 2

SN := log

(N∏

n=2

F2n + 1

F2n − 1

)= 2

N∑n=2

arctanh(1/F2n)

where

arctanh(x) =1

2log

(1 + x

1− x

)for |x| < 1.

We have to show that limN→∞ SN = log 3. Since

arctanh(x)− arctanh(y) =1

2log

((1 + x)(1− y)

(1− x)(1 + y)

)=

1

2log

((1− xy) + (x− y)

(1− xy)− (x− y)

)= arctanh

(x− y

1− xy

)then

arctanh(1/F2n−1)−arctanh(1/F2n+1) = arctanh

(F2n+1 − F2n−1

F2n+1F2n−1 − 1

)= arctanh(1/F2n)

because F2n+1 − F2n−1 = F2n and F2n+1F2n−1 − 1 = F 22n. Therefore

SN = 2N∑

n=2

(arctanh(1/F2n−1)− arctanh(1/F2n+1)) = 2 (arctanh(1/F3)− arctanh(1/F2N+1))

and finallylim

N→∞SN = 2 arctanh(1/F3) = 2 arctanh(1/2) = log 3.

Page 50: Eserciziario Di Analisi Matematica 2

Mathematics Magazine, Vol. 79, December 2006

1756. Proposed by C. Moen and W. Wardlaw, USA.

For which non negative integers n do there exist nonnegative integers a and b such thatn! = 2a(2b − 1)?

Solution proposed by G.R.A.20 Problem Solving Group, Roma, Italy.

Any factorial n! is the product of a power of 2 by an odd number f(n). The questionis for which n, f(n) has the form 2b − 1 (note b has to be positive).By direct inspection we have that for n < 10 the answer is yes for n ∈ 0, 1, 2, 3, 4, 5:

0! = 1! = 20(21−1), 2! = 21(21−1), 3! = 21(22−1), 4! = 23(22−1), 5! = 23(24−1).

For n ≥ 10, if f(n) = 2b − 1 then b > 13 because f(n) is an increasing function(although not strictly), and

f(10) = 10!/28 = 14175 > 8191 = 213 − 1.

By a theorem of A. Schinzel (see On primitive prime factors of an − bn, Proc. Cam-bridge Philos. Soc. 58 (1962), 555-562)

for b ≥ 13 there is a prime p ≥ 2b + 1 such that p | 2b − 1.

Hence if n! = 2a(2b − 1) then p | n! which means that p | n and n ≥ p ≥ 2b + 1.So we have a bound for b :

b ≤ (n− 1)/2.

We can find also a bound for a : since a is the power of 2 in the factorization of n!then

a =∞∑

k=1

⌊ n

2k

⌋<

∞∑k=1

n

2k= n.

Therefore for n ≥ 10

n! = 2a(2b − 1) < 2a+b < 2n+(n−1)/2 < 23n/2

On the other hand, for n ≥ 10

n! > 3! · 4n−3 > 4n−2 = 22n−4

and we have a contradiction because

22n−4 < n! < 23n/2

means that 2n/2 < 16 which is evidently false for n ≥ 10.

Page 51: Eserciziario Di Analisi Matematica 2

Mathematics Magazine, Vol. 79, October 2006

1754. Proposed by M. Benzce, Romania.

Let a1, a2, . . . , an be positive real numbers. Prove that

n

√√√√ n∏k=1

ak ≤ log

1 + n

√√√√ n∏k=1

(eak − 1)

≤ 1

n

n∑k=1

ak.

Solution proposed by G.R.A.20 Problem Solving Group, Roma, Italy.

We introduce the following notations:

G (xk) = n

√√√√ n∏k=1

xk and A (xk) =1

n

n∑k=1

xk.

Note that for all positive real numbers x1, x2, . . . , xn then

log (G (ak)) = A (log (ak)) .

Letting f(x) = ex − 1, we have to show that

G (xk) ≤ f−1 (G (f(ak))) ≤ A (ak)

or, since f−1(x) = log(1 + x) is an increasing function,

f (G (ak)) ≤ G (f(ak)) ≤ f (A (ak)) .

The second one si equivalent to

A (log (f (ak))) = log (G (f (ak))) ≤ log (f (A (ak)))

which holds because the function log (f (x)) is concave in (0, +∞):

D2 (log (f (x))) = − ex

(ex − 1)2< 0 for x > 0.

The first one is equivalent to

log (f (exp (log (G (ak))))) ≤ log (G (f (exp (log (ak)))))

that islog (f (exp (A (log (ak))))) ≤ A (log (f (exp (log (ak)))))

which holds because the function log (f (exp (x))) is convex in R:

D2 (log (f (exp(x)))) =eex+x

(eex − (1 + ex)

)(eex − 1)2

> 0 for x ∈ R

(note that ey > 1 + y for any real y 6= 0).

Page 52: Eserciziario Di Analisi Matematica 2

Mathematics Magazine, Vol. 79, October 2006

1753. Proposed by J.C. George, New Mexico, USA.

Let n be a positive integer and let Sn be the set of all positive integers whose (base ten)digit sum is n. Determine the convergence or divergence of the series∑

k∈Sn

1

k.

Solution proposed by G.R.A.20 Problem Solving Group, Roma, Italy.

We will show that the series converge. Since the terms are positive we can rearrangethem and it suffices to prove that the sum is bounded.

First solution.Let Sn,d,p be the set of all positive integers whose digit sum is n, the number of digitsis d and the number of non-zero digits is p. Note that 1 ≤ p ≤ min(n, d) and

|Sn,d,p| ≤ 9p ·(

d − 1

p − 1

).

because there are 9 ways to choose one of the p non-zero digits and they have to beplaced one at the most significant position and the other p− 1 among the remainingd − 1 positions. Therefore∑

k∈Sn

1

k=

n∑p=1

∞∑d=p

∑k∈Sn,d,p

1

k≤

n∑p=1

∞∑d=p

|Sn,d,p|min(Sn,d,p)

≤n∑

p=1

∞∑d=p

9p ·(

d − 1

p − 1

)· 1

10d−1.

Since∞∑

d=p

(d − 1

p − 1

)xd−1 =

xp−1

(1 − x)p.

then ∑k∈Sn

1

k≤

n∑p=1

9p · 1/10p−1

(1 − 1/10)p=

n∑p=1

10 = 10n

Second solution.Let T1 be the set of all positive integers whose digits are different from 1 then by awell known result due to Kempner the series

∑k∈T1

1k

converge:

∑k∈T1

1

k=

∞∑d=1

∑k∈T1,d

1

k≤

∞∑d=1

|T1,d|min(T1,d)

≤∞∑

d=1

8 · 9d−1

10d−1= 8 · 1

1 − 9/10= 80

where T1,d is the set of the numbers of T1 with d digits.

Page 53: Eserciziario Di Analisi Matematica 2

Now we prove the statement by induction with respect to n. For n = 1

∑k∈S1

1

k=

∞∑d=1

1

10d−1=

1

1 − 1/10=

10

9.

For n > 1 ∑k∈Sn

1

k=

∑k∈Sn∩T1

1

k+

∑k∈Sn\T1

1

k≤ 80 +

∑k′∈Sn−1

1

k′

because any number k ∈ Sn \ T1 has a digit equal to 1 and therefore if we replacethat 1 with a 0 we obtain a nonzero number k′ ∈ Sn−1 which is less than k. Hence∑

k∈Sn

1

k≤ (n − 1)80 +

10

9.

2

Page 54: Eserciziario Di Analisi Matematica 2

Mathematics Magazine, Vol. 79, October 2006

1751. Proposed by I. Bluskov, Canada.

Let k1, k2, . . . , kn be integers with ki > 2, i = 1, 2, . . . , n, and let N =∑n

i=1

(ki

2

).

Prove that ∑1≤i<j≤n

(ki

2

)(kj

2

)+ 3

n∑1=1

(ki + 1

4

)=

(N

2

)

Solution proposed by G.R.A.20 Problem Solving Group, Roma, Italy.

We first note that for any k > 2 then

3

(k + 1

4

)=

1

2

((k

2

)2

−(

k

2

))

because

3

(k + 1

4

)=

(k + 1)k(k − 1)(k − 2)

8=

1

2

(k

2

)k2 − k − 2

2=

1

2

(k

2

)((k

2

)− 1

).

Therefore (N

2

)=

N2

2− N

2=

1

2

(n∑

i=1

(ki

2

))2

− 1

2

n∑i=1

(ki

2

)=

1

2

n∑1=1

(ki

2

)2

+∑

1≤i<j≤n

(ki

2

)(kj

2

)− 1

2

n∑i=1

(ki

2

)

=∑

1≤i<j≤n

(ki

2

)(kj

2

)+

1

2

n∑i=1

((ki

2

)2

−(

ki

2

))

=∑

1≤i<j≤n

(ki

2

)(kj

2

)+ 3

n∑1=1

(ki + 1

4

)

Page 55: Eserciziario Di Analisi Matematica 2

Mathematics Magazine, Vol. 79, April 2006

1744. Proposed by M. Goldenberg and M. Kaplan, The Ingenuity Project, BaltimorePolytechnic Institute, Baltimore, MD.

For real number x ≥ 1, define a1 = 2x and an+1 = a2n − 2, n = 1, 2, 3, . . .. Find a closed

form expression for

S(x) =∞∑

n=1

n∏k=1

1

ak

.

Solution proposed by G.R.A.20 Problem Solving Group, Roma, Italy.

We will show thatS(x) = x−

√x2 − 1 := z for x ≥ 1.

Since

x =1

2

(z +

1

z

)for 0 < z ≤ 1

then

an = z2n−1

+1

z2n−1

because a1 = z + 1/z, and

an+1 = a2n − 2 =

(z2n−1

+1

z2n−1

)2

− 2 = z2n

+ 2 +1

z2n − 2 = z2n

+1

z2n .

Moreover (z − 1

z

n∏k=1

ak = z2n − 1

z2n .

This identity holds for n = 1 and it can be easily verified by induction for n > 1(z − 1

z

n∏k=1

ak =

(z2n−1 − 1

z2n−1

)· an = z2n − 1

z2n .

Therefore

S(x) =∞∑

n=1

(z − 1

z

) /(z2n − 1

z2n

)= −

(z − 1

z

) ∞∑n=1

z2n

1− z2n+1

=1− z2

z·∞∑

n=1

z2n∞∑

k=0

(z2n+1

)k

=1− z2

z·∞∑

n=1

∞∑k=0

(z2)(2k+1)·2n−1

.

Since any positive integer m can be written in a unique way as the product of an oddnumber, 2k + 1, by a power of 2, 2n−1, then for 0 < z < 1

S(z) =1− z2

z·∞∑

m=1

(z2)m =1− z2

z· z2

1− z2= z = x−

√x2 − 1.

If z = 1 then x = 1, an = 2 for all n ≥ 1 and S(x) = 1 = z.

Page 56: Eserciziario Di Analisi Matematica 2

Mathematics Magazine, Vol. 79, February 2006

1739. Proposed by Emeric Deutsch, Polytechnic University, Brooklyn, NY.

An object moves in the plane, starting from the origin, and at each step moving one unitup, down, to the right, or to the left. Find the number of such paths that stay in thequadrant (x, y) : x, y ≥ 0, and consist of a total of n steps, exactly k of which arevertical (up or down).

Solution proposed by G.R.A.20 Problem Solving Group, Roma, Italy.

We first consider the one-dimensional problem: count the number of paths tn inx : x ≥ 0 of n steps, starting from the origin, and at each step moving one unit tothe right, or to the left.If the number of steps n is odd then the object’s position x is positive and we canchoose both directions:

tn+1 = 2 · tn if n is odd

If the number of steps n is even then the object’s position x can be positive and againwe can choose both directions, or it can be the origin and in this case we must moveto the right. Since the number of paths which arrive in the origin after n steps isprecisely the catalan number

Cn/2 =1

n/2 + 1

(n

n/2

)then

tn+1 = 2 · (tn − Cn/2) + 1 · Cn/2 = 2 · tn − Cn/2 if n is even.

Since t0 = 1 and t1 = 1 it suffices to verify that the formula

tn =

(n

bn/2c

)solves the two previous recurrences: if n is odd

tn+1 =

(n + 1

(n + 1)/2

)=

n + 1

(n + 1)/2·(

n

bn/2c

)= 2 · tn,

if n is even

tn+1 =

(n + 1

n/2

)=

(n + 1

n/2 + 1

)=

n + 1

n/2 + 1·(

n

n/2

)= 2 · tn − Cn/2.

Finally we note that a path in the quadrant can be decomposed in two paths: oneof k steps along the half-line x : x ≥ 0 and one of n − k steps along the half-liney : y ≥ 0. Therefore the number of paths is(

n

k

)tktn−k =

(n

k

)(k

bk/2c

)(n − k

b(n − k)/2c

).

Page 57: Eserciziario Di Analisi Matematica 2

The College Mathematics Journal, November 2006

838. Proposed by Arkady Alt, San Jose, CA.

Show that in any acute triangle ∆ABC with sides a, b and c, the followinginequality is true:

27 ≤ (a + b + c)2

(1

a2 + b2 − c2+

1

b2 + c2 − a2+

1

c2 + a2 − b2

)

Solution proposed by G.R.A.20 Problem Solving Group, Roma, Italy.

Since c2 = a2 + b2 − 2ab cos γ then

1

a2 + b2 − c2=

1

2ab cos γ=

tan γ

4A

where A is the triangle’s area. Simmetrically

1

b2 + c2 − a2=

tan α

4Aand

1

c2 + a2 − b2=

tan β

4A.

After transforming and rearranging the terms, the inequality becomes

√27 · A

s2≤ tan α + tan β + tan γ√

27

where s is the triangle’s semiperimeter. Now it suffices to prove that

A

s2≤ 1√

27and

√27 ≤ tan α + tan β + tan γ.

First inequality: by Heron’s formula A =√

s(s− a)(s− b)(s− c) and byapplying AGM inequality we obtain

A

s2=

√(1− a

s

) (1− b

s

) (1− c

s

)≤

√(1

3

(1− a

s+ 1− b

s+ 1− c

s

))3

=1√27

.

Page 58: Eserciziario Di Analisi Matematica 2

Second inequality: we note that

tan(α + β) =tan α + tan β

1− tan α tan β

therefore, since tan γ = − tan(α + β), we find that

tan α + tan β + tan γ = tan α tan β tan γ.

Since the triangle is acute then tan α, tan β and tan γ are positive and byapplying AGM inequality we obtain

tan α + tan β + tan γ = tan α tan β tan γ ≤(

tan α + tan β + tan γ

3

)3

which means that √27 ≤ tan α + tan β + tan γ.

2

Page 59: Eserciziario Di Analisi Matematica 2

The College Mathematics Journal, September 2006

835. Proposed by Juan-Bosco Romero Marquez, Universidad de Valladolid,Vallalodid, Spain.

Show that for any positive real numbers a, b, and c,

abc ≤ (a + b + c)3 − (a3 + b3 + c3)

24≤ 1

27(a + b + c)3.

Solution proposed by G.R.A.20 Problem Solving Group, Roma, Italy.

First inequality gives

(a + b + c)3 − (a3 + b3 + c3)

24≥ abc

3a2b + 3a2c + 3b2a + 3b2c + 3c2a + 3c2b + 6abc ≥ 24abc

a2b + a2c + b2a + b2c + c2a + c2b ≥ 6abc

a2b + a2c + b2a + b2c + c2a + c2b

6≥ abc

which holds by the AM-GM inequality.The second inequality gives

(a + b + c)3 − (a3 + b3 + c3)

24≤ 1

27(a + b + c)3

(a + b + c)3 − (a3 + b3 + c3)

8≤ 1

9(a + b + c)3

(a + b + c)3

9≤ a3 + b3 + c3(

a + b + c

3

)3

≤ a3 + b3 + c3

3

which holds by the Power Means inequality (or by the convexity of thefunction x3 for x ≥ 0).

Page 60: Eserciziario Di Analisi Matematica 2

The College Mathematics Journal, September 2006

834. Proposed by Jose Luis Dıaz-Barrero, Barcelona, Spain.

Let n be a positive integer. Prove that

1

2

(Fn

3

√Fn+1

Fn

+ Fn+13

√Fn+2

Fn+1

+ Fn+23

√Fn

Fn+2

)< Fn+2

where Fn is the nth Fibonacci number (F1 = 1, F2 = 1, Fn = Fn−1 + Fn−2).

Solution proposed by G.R.A.20 Problem Solving Group, Roma, Italy.

First Proof.

By A-G-M inequality

F 2/3n F

1/3n+1 ≤

2Fn + Fn+1

3, F

2/3n+1F

1/3n+2 <

2Fn+1 + Fn+2

3, F

2/3n+2F

1/3n <

2Fn+2 + Fn

3.

Note that in the last two inequalities the left terms are strictly less than theright terms because Fn+1 6= Fn+2 and Fn 6= Fn+2 for any positive integer n.Finally, since Fn + Fn+1 = Fn+2, we have that

1

2

(F 2/3

n F1/3n+1 + F

2/3n+1F

1/3n+2 + F

2/3n+2F

1/3n

)<

Fn + Fn+1 + Fn+2

2= Fn+2.

Second Proof.

We first show that

Fn3

√Fn+1

Fn

= Fn3

√1 +

Fn−1

Fn

≤ Fn +1

3Fn−1 for n ≥ 1

Fn+13

√Fn+2

Fn+1

= Fn+13

√1 +

Fn

Fn+1

≤ Fn+1 +1

3Fn for n ≥ 1

where we used the inequality 3√

1 + x ≤ 1 + 13x for x ≥ 0.

Page 61: Eserciziario Di Analisi Matematica 2

Moreover, since Fn+2 > 2Fn then

Fn+23

√Fn

Fn+2

≤ Fn+23

√Fn

2Fn

= Fn+23

√1

2<

4

5Fn+2 for n ≥ 1.

Now we sum the upper bounds and by the recurrence formula we obtain:

1

2(Fn +

1

3Fn−1 + Fn+1 +

1

3Fn +

4

5Fn+2) =

9

10Fn+2 +

1

6Fn+1.

So, it suffices to prove that

9

10Fn+2 +

1

6Fn+1 < Fn+2

that is5Fn+1 < 3Fn+2

which is true because

5Fn+1 = 5Fn−1 + 5Fn < 6Fn−1 + 6Fn = 3Fn + 3Fn+1 = 3Fn+2.

2

Page 62: Eserciziario Di Analisi Matematica 2

The College Mathematics Journal, September 2006

831. Proposed by Stanley Rabinowitz, MathPro Press, Chelmsford, MA.

In Problem 218 of this Journal (14 (1983) p.358), it was shown that

tan3π

11+ 4 sin

11=√

11

Prove the related identities

tanπ

11+ 4 sin

11=√

11

and

tan4π

11+ 4 sin

π

11=√

11.

Solution proposed by G.R.A.20 Problem Solving Group, Roma, Italy.

Let

S(z) := z + z3 + z4 + z5 + z9 and S ′(z) := z2 + z6 + z7 + z8 + z10

where z 6= 1 is a complex number not such that z11 = 1. Since S(z)+S ′(z) =−1 and S(z) · S ′(z) = 3 (see Problem 218) then the numbers S(z) and S ′(z)are the roots of the quadratic equation u2 + u + 3 = 0 and therefore

S(z)− S ′(z) = ±√

∆ = ±i√

11.

Let x = e2πi/11 then for any integer number k

i tankπ

11=

xk/2 − x−k/2

xk/2 + x−k/2=

xk − 1

xk + 1=

xk(1− x10k)

xk + 1

= xk − x2k + x3k − x4k + x5k − x6k + x7k − x8k + x9k − x10k

= S(xk)− S ′(xk)− 2(x4k − x7k)

Moreover, since x11 = 1,

2(x4k − x7k) = 2(x4k − x−4k) = 4i sin8kπ

11.

Page 63: Eserciziario Di Analisi Matematica 2

Therefore

i tankπ

11+ 4i sin

8kπ

11= S(xk)− S ′(xk)

and we found a general identity: for any integer number k not multiple of 11

tankπ

11+ 4 sin

8kπ

11= ±

√11

(when k is divisible by 11 the sum is trivially zero).Now, changing k we show all the requested identities:for k = 1

tanπ

11+ 4 sin

11= tan

π

11+ 4 sin

11=√

11,

for k = 3

tan3π

11+ 4 sin

24π

11= tan

11+ 4 sin

11=√

11,

for k = 4

tan4π

11+ 4 sin

32π

11= tan

11+ 4 sin

π

11=√

11.

Note that in these cases it was easy to choose the sign of ±√

11 because thevalue of the functions tan and sin were positive.This choice is not so trivial for k = 5:

tan5π

11+ 4 sin

40π

11= tan

11− 4 sin

11=√

11.

2

Page 64: Eserciziario Di Analisi Matematica 2

The College Mathematics Journal, Vol. 37, May 2006

827. Proposed by Syrous Marivani,Louisiana State University at Alexandria,Alexandria, LA.

Evaluate the sumsn∑

k=0

(n

k

)F4ak+b and

n∑k=0

(n

k

)L4ak+b

where Fk and Lk are elements of Fibonacci and Lucas sequences, respectively, aand b are integers, and n is a positive integer. (F1 = 1, F2 = 1, Fn = Fn−1+Fn−2

and L1 = 1, L2 = 3, Ln = Ln−1 + Ln−2)

Solution proposed by G.R.A.20 Problem Solving Group, Roma, Italy.

We will prove thatn∑

k=0

(n

k

)G4ak+b = G2an+b

(F4a

F2a

)n

where Gn is the General Fibonacci sequence: Gn = Gn−1 + Gn−2, but thestarting values of G0 and G1 can be specified. If G0 = 0 and G1 = 1 we havethe Fibonacci sequence, G0 = 2 and G1 = 1 gives the Lucas sequence.

Solving the characteristic polynomial z2 − z − 1 we have that

Gn = C1 · αn + C2 · βn

where α = (1 +√

5)/2, β = −1/α and the constants C1 and C2 are uniquelydetermined by the initial terms G0 and G1.

n∑k=0

(n

k

)G4ak+b = C1 · αb

n∑k=0

(n

k

)(α4a)k + C2 · βb

n∑k=0

(n

k

)(β4a)k

= C1 · αb(α4a + 1

)n+ C2 · βb

(β4a + 1

)n

= C1 · αb

(α2aF4a

F2a

)n

+ C2 · βb

(β2aF4a

F2a

)n

=(C1 · α2an+b + C2 · β2an+b

) (F4a

F2a

)n

= G2an+b

(F4a

F2a

)n

.

Page 65: Eserciziario Di Analisi Matematica 2

The identities

α4a + 1 =α2aF4a

F2a

and β4a + 1 =β2aF4a

F2a

can be easily verified: since β2 = α−2 and by the Binet’s formula

Fn = (αn − βn)/√

5

then

α4a + 1 = α2a · (α2a + α−2a) = α2a · α4a − α−4a

α2a − α−2a= α2a · α4a − β4a

α2a − β2a=

α2aF4a

F2a

,

and

β4a + 1 = β2a · (β2a + β−2a) = β2a · β4a − β−4a

β2a − β−2a= β2a · β4a − α4a

β2a − α2a=

β2aF4a

F2a

.

2

Page 66: Eserciziario Di Analisi Matematica 2

The College Mathematics Journal, Vol. 37, March 2006

824. Proposed by Jeremy Leach (student), Prescott, AZ.

Prove that the value of the sum

Sn :=n∑

j=1

n∑k=1

(−1)j+k 1

j!k!

(n− 1

j − 1

)(n− 1

k − 1

)(j + k)!

is independent of n

Solution proposed by G.R.A.20 Problem Solving Group, Roma, Italy.

We will show that the sum is always equal to

S1 := (−1)1+1 1

1!1!

(1− 1

1− 1

)(1− 1

1− 1

)(1 + 1)! = 2

We note that

Sn =n∑

j=1

n∑k=1

(−1)j+k

(n− 1

j − 1

)(n− 1

k − 1

)(j + k

k

)

=n∑

j=1

(−1)j

(n− 1

j − 1

) n∑k=1

(−1)k

(n− 1

k − 1

)(j + k

k

)Since (

j + k

k

)=

(j + k)k

k!=

(j + 1)k

k!= (−1)k · (−j − 1)k

k!= (−1)k ·

(−j − 1

k

)then we can apply the Vandermonde’s convolution formula to the internal sum

n∑k=1

(−1)k

(n− 1

k − 1

)(j + k

k

)=

n∑k=1

(n− 1

n− k

)(−j − 1

k

)=

(n− j − 2

n

)Therefore for n ≥ 2

Sn =n∑

j=1

(−1)j

(n− 1

j − 1

)(n− j − 2

n

)=

n∑j=n−1

(−1)j

(n− 1

j − 1

)(n− j − 2

n

)= (−1)n−1

(n− 1

n− 2

)(−1

n

)+ (−1)n

(n− 1

n− 1

)(−2

n

)= −(n− 1) + (n + 1) = 2

because for n ≥ 3 and 1 ≤ j ≤ n− 2 the binomial coefficient(

n−j−2n

)is zero.

Page 67: Eserciziario Di Analisi Matematica 2

The College Mathematics Journal, Vol. 37, March 2006

821. Proposed by Andrew Cusumano, Great Neck, NY.

Find

limn→∞

(∑n+2j=1 jj∑n+1j=1 jj

−∑n+1

j=1 jj∑nj=1 jj

)

Solution proposed by G.R.A.20 Problem Solving Group, Roma, Italy.

Let sn :=∑n

j=1 jj. Since for n > 1

nn < sn = sn−1 + nn = (n − 1) · (n − 1)n−1 + nn = (n − 1)n + n2 < 2 · nn

then sn/nn = O(1). We can refine this result

sn/nn = 1 +

sn−1

nn= 1 +

(n − 1)n−1

nn· O(1)

= 1 +1

n − 1·(

1 +1

n

)n

· O(1) = 1 + O(1/n)

and

sn/nn = 1 +

sn−1

nn= 1 +

(n − 1)n−1

nn· (1 + O(1/n))

= 1 +1

n − 1·(

1 − 1

n

)n

· (1 + O(1/n))

= 1 +1

en+ O(1/n2).

Moreover

sn+1/sn =(n + 1)n+1

nn·1 + 1

e(n+1)+ O(1/n2)

1 + 1en

+ O(1/n2)

= (n + 1) · (1 + 1/n)n ·(

1 +1

e(n + 1)+ O(1/n2)

)·(

1 − 1

en+ O(1/n2)

)= (n + 1) · e

(1 − 1

2n+ O(1/n2)

)·(1 + O(1/n2)

)= e

(n +

1

2+ O(1/n)

).

Therefore

sn+2

sn+1

− sn+1

sn

= e

((n + 1) +

1

2+ O(1/n)

)− e

(n +

1

2+ O(1/n)

)= e + O(1/n).

and the limit is e.

Page 68: Eserciziario Di Analisi Matematica 2

The College Mathematics Journal, Vol. 37, January 2006

819. Proposed by Michael Andreoli, Miami, FL.

For n ≥ 1, evaluate

∞∑k=1

1

k(k + 1)(k + 2) · · · (k + n)

Solution proposed by G.R.A.20 Problem Solving Group, Roma, Italy.

We will prove that

an :=∞∑

k=1

1

k(k + 1)(k + 2) · · · (k + n)=

1

n · n!.

The series converges by the comparison test: an ≤∑∞

k=1 1/k2 < +∞.Moreover

a1 =∞∑

k=1

1

k(k + 1)= lim

N→∞

N∑k=1

(1

k− 1

k + 1

)= lim

N→∞

(1− 1

N + 1

)= 1.

Since for n, k ≥ 1

1

k(k + 1)(k + 2) · · · (k + n)=

1

n·(

1

k · · · (k + n− 1)− 1

(k + 1) · · · (k + n)

)then for n > 1

an =1

(an−1 −

∞∑k=1

1

(k + 1) · · · (k + n)

)

=1

(an−1 −

∞∑k=2

1

k · · · (k + n− 1)

)

=1

n·(

an−1 −(

an−1 −1

n!

))=

1

n · n!

Page 69: Eserciziario Di Analisi Matematica 2
Page 70: Eserciziario Di Analisi Matematica 2
Page 71: Eserciziario Di Analisi Matematica 2
Page 72: Eserciziario Di Analisi Matematica 2
Page 73: Eserciziario Di Analisi Matematica 2

Mathematics Magazine, Vol. 78, December 2005

1734. Proposed by H. A. ShahAli, Teheran, Iran.

Let α be a fixed irrational number and let P be a polynomial with integercoefficients and with deg(P ) ≥ 1. Prove that there are infinitely many pairs(m, n) of integers such that P (m) = bnαc.

Solution proposed by G.R.A.20 Math Problems Group, Roma, Italy.

The equation P (m) = bnαc is equivalent to

P (m) ≤ nα < P (m) + 1.

We can assume that α > 0 (otherwise we change the sign of n) we have that

P (m)

α≤ n <

P (m)

α+

1

α.

If P (m) 6= 0 (there are a finite number of such m) then P (m)/α is irrationaland since n is integer the previous equation is equivalent to⌊

P (m)

α

⌋< n <

⌊P (m)

α

⌋+

P (m)

α

+

1

α

where x = x − bxc is the fractional part of x. These inequalities will besatisfied by at least an integer n if the length of the interval is greater than1 that is if

P (m)

α

+

1

α> 1.

If 0 < α < 1 then this condition is trivially satisfied for all m but for α > 1the inequality becomes harder: we need infinite integers m such that

P (m)

α

(1− 1

α, 1

).

This is provided by the following theorem due to H. Weyl where we takeQ(x) = P (x)/α:

Let Q(x) be a nonconstant polynomial with real coefficients such that atleast one of its nonconstant coefficients is irrational. Then the sequence offractional parts Q(m) is dense in [0, 1).

Page 74: Eserciziario Di Analisi Matematica 2

Mathematics Magazine, Vol. 78, October 2005

1729. Proposed by Brian T. Gill, Seattle Pacific University, Seattle, WA.

For positive integer k, let ck denote the product of the first k odd positiveintegers, and let c0 = 1. Prove that for each nonnegative integer n

n∑k=0

(−1)k

(n

k

)n!

k!2n−kck = cn.

Solution proposed by G.R.A.20 Math Problems Group, Roma, Italy.

First we note that

ck = 1 · 3 · · · (2k − 1) =(2k)!

2kk!.

Then the identity we are going to prove is equivalent to this onen∑

k=0

(n

k

)(−4)n−k

(2k

k

)= (−1)n

(2n

n

).

Since the binomial coefficient(2kk

)is the constant term of (x + 1/x)2k, then

the sum on the left side is the constant term of the rational function((x +

1

x

)2

− 4

)n

=n∑

k=0

(n

k

)· (−4)n−k ·

(x +

1

x

)2k

.

On the other hand((x +

1

x

)2

− 4

)n

=

(x2 +

1

x2+ 2 − 4

)n

=

((x − 1

x

)2)n

=

(x − 1

x

)2n

whose constant term (−1)n(2nn

)which is just the right side of the previous

equation.

We remark that if dk is the product of the first k even positive integers

dk = 2 · 4 · · · (2k) = 2kk!

then the analogous sum is identically zeron∑

k=0

(−1)k

(n

k

)n!

k!2n−kdk =

n∑k=0

(−1)k

(n

k

)n!2n = n!2n(1 − 1)n = 0.

Page 75: Eserciziario Di Analisi Matematica 2

Mathematics Magazine, Vol. 78, October 2005

1727. Proposed by Jody M. Lockhart and Willaim P. Wardlaw, U.S. NavalAcademy, Annapolis, MD.

Chain addition is a technique used in cryptography to extend a short sequenceof digits, called seed, to a longer sequence of pseudorandom digits. If the seedsequence of digits is a1, a2, . . . , an, then for positive integer k, an+k = ak +ak+1

mod 10, that is, an+k is the units digit in the sum ak + ak+1. Suppose that theseed sequence is 3, 9, 6, 4. Prove that the sequence is periodic and find, withoutthe use of calculator or computer, the number of digits in the sequence beforethe first repetition of 3, 9, 6, 4.

Solution proposed by G.R.A.20 Math Problems Group, Roma, Italy.

The sequence with seed 3, 9, 6, 4 can be generated in this way:

vk+1 = Mvk mod 10 for k ≥ 1 and v1 = [3, 9, 6, 4]t

where vk = [ak, ak+1, ak+2, ak+2]t and

M =

0 1 0 00 0 1 00 0 0 11 1 0 0

.

We will first prove thatM1560 = I mod 10

that is, by the Chinese Remainder Theorem,

M15 = I mod 2 and M312 = I mod 5.

Note that M4 = M + I mod 10 and if p is a prime then (A + I)p = A + Imod p. Using these two facts we get

M16 = (M4)4 = (M + I)4 = M4 + I = M + 2I = M mod 2

and since M is invertible mod 2 we have that M15 = I mod 2. Moreover,

M100 = (M4)25 = (M + I)25 = M25 + I = M(M4)6 + I

= M(M + I)6 + I = M(M + I)(M + I)5 + I = (M2 + M)(M5 + I) + I

= (M2 + M)(M(M4) + I) + I = (M2 + M)(M2 + M + I) + I

= M4 + 2M3 + 2M2 + M + I = 2(M3 + M2 + M + I) mod 5

Page 76: Eserciziario Di Analisi Matematica 2

and

M60 = (M4)15 = (M + I)5·3 = (M5 + I)3

= M15 + 3M10 + 3M5 + I

= M3(M + I)3 + 3M2(M + I)2 + 3M(M + I) + I

= M2(M4) + 3M(M4) + 3(M4) + M3 + 3(M4)

+6M3 + 3M2 + 3M2 + 3M + I

= 3M3 + 2M + 2I mod 5.

Therefore

M160 = 2(M3 + M2 + M + I) · (3M3 + 2M + 2I) = −M − I = −M4 mod 5

and since M is invertible mod 5 we have that M156 = −I mod 5 andM312 = I mod 5.Finally we prove that the period of the sequence is just 1560 = lcm(15, 312)by showing that

Mdv1 6= v1 mod 10 for any proper divisor d of 1560 = 23 · 3 · 5 · 13.

For d divisor of 520 = 8 · 5 · 13

M520v1 = M−5v1 = [0, 1, 0, 1]t 6= v1 = [1, 1, 0, 0]t mod 2.

For d divisor of 312 = 8 · 3 · 13

M312v1 = M−3v1 = [0, 1, 1, 1]t 6= v1 = [1, 1, 0, 0]t mod 2.

For d divisor of 120 = 8 · 3 · 5

M120v1 = (3M3 + 2M + 2I)2v1 = [0, 3, 2, 4]t 6= v1 = [3, 4, 1, 4]t mod 5.

For d divisor of 780 = 4 · 3 · 5 · 13

M780v1 = (M156)5v1 = −v1 = [2, 1, 4, 1]t 6= v1 = [3, 4, 1, 4]t mod 5.

Page 77: Eserciziario Di Analisi Matematica 2

Mathematics Magazine, Vol. 78, June 2005

1721. Proposed by Emerich Deutsch, Polytechnic University, Brooklyn, NY.

Let k and n be positive integers with k ≤ n. Find the number of permutations of1, 2, . . . , n in which 1, 2, . . . , k appears as a subsequence but 1, 2, . . . , k, k +1does not.

Solution proposed by G.R.A.20 Math Problems Group, Roma, Italy.

The number of permutations of 1, 2, . . . , n in which 1, 2, . . . , k appearsas a subsequence is determined by the number of different positions of thenumbers 1, 2, . . . , k, that is

(nk

), multiplied by the number of ways that the

remaining places can be filled, that is (n − k)!. Therefore for k < n therequired number of permutations is(

n

k

)· (n− k)!−

(n

k + 1

)· (n− (k + 1))! =

k · n!

(k + 1)!.

Of course for k = n there is only one permutation with such property.

Page 78: Eserciziario Di Analisi Matematica 2

Mathematics Magazine, Vol. 78, June 2005

1721. Proposed by Donald Knuth, Stanford University, Stanford, CA.

The Fibonacci graphs are defined by successively replacing the edge with maxi-mum label n by two edges n and n + 1, in series if n is even, and in parallel if nis odd. Prove that the Fibonacci graph with n edges has exactly Fn+1 spanningtrees, where F1 = F2 = 1 and Fn+1 = Fn+Fn−1. Show also that these spanningtrees can be listed in such a way that some edge k is replaced by k ± 1 as wepass from one tree to the next. For example, for n = 5 the eight spanning treescan be listed as 125, 124, 134, 135, 145, 245, 235, 234.

Solution proposed by G.R.A.20 Math Problems Group, Roma, Italy.

Let Gn be the nth Fibonacci graphs and denote by T (Gn) the set of all itsspanning trees. In order to count these spanning trees we consider first thetrees that contain the edge n and then the trees that do not contain the edgen. If we remove the edge n from a spanning tree of the first kind then itbecomes a spanning tree of the graph obtained by contracting the edge nin Gn that is Gn−1 if n is odd or Gn−2 if n is even. A spanning tree of thesecond kind is a spanning tree of Gn−2 plus the edge n − 1 if n is odd or itis a spanning tree of Gn−1 if n is even. Therefore in both cases for n > 2

|T (Gn)| = |T (Gn−1)| + |T (Gn−2)|

and since |T (G1)| = 1 and |T (G2)| = 2 we have that |T (Gn)| = Fn+1.Now we show a recursive algorithm that lists the trees of T (Gn) in such away that two consecutive trees differ only for some edge k that is replacedby k ± 1. Let T (G1) = 〈1〉 and T (G2) = 〈2, 1〉, and, by the remark we madeat the beginning, we are able to list T (Gn) for n > 2 in following way:

• take the trees of T (Gn−2) and add them the edge n − 1 if n is odd orthe edge n if n is even,

• take the trees of T (Gn−1) and add them the edge n if n is odd,

• reverse the order of the two list and merge them.

Below there are the lists for n = 3, 4, 5, 6:

T (G3) = 〈12, 13, 23〉

Page 79: Eserciziario Di Analisi Matematica 2

T (G4) = 〈14, 24, 23, 13, 12〉T (G5) = 〈234, 134, 124, 125, 135, 235, 245, 145〉T (G6) = 〈126, 136, 236, 246, 146, 145, 245, 235, 135, 125, 124, 134, 234〉.

For a generic n if

T (Gn−2) = 〈tn−21 , . . . , tn−2

Fn−1〉 and T (Gn−1) = 〈tn−1

1 , . . . , tn−1Fn

then the list of the spanning trees of Gn is

T (Gn) =

〈tn−2

Fn−1(n − 1), . . . , tn−2

1 (n − 1), tn−1Fn

n, . . . , tn−11 n, 〉 for n odd

〈tn−2Fn−1

n, . . . , tn−21 n, tn−1

Fn, . . . , tn−1

1 〉 for n even.

The required property holds because it holds for T (Gn−2), T (Gn−1) and thetrees in the middle are

tn−21 (n − 1) and tn−1

Fnn = tn−2

1 n for n oddtn−21 n and tn−1

Fn= tn−2

1 (n − 1) for n even.

2

Page 80: Eserciziario Di Analisi Matematica 2

Mathematics Magazine, Vol. 78, April 2005

1718. Proposed by David Callan, Madison, WI.

Let k, n be integers with 1 ≤ k ≤ n. Prove the identity.

k−1∑i=0

(k − 1

i

)(n− (k − 1)

k − i

)2k−i−1 =

k−1∑i=0

(k − i

i

)(n− i

k

).

Solution proposed by G.R.A.20 Math Problems Group, Roma, Italy.

We consider the polynomial

P (x) = (2 + x)k−1 · (1 + x)n−(k−1)

=k−1∑i=0

(k − 1

i

)2(k−1)−ixi ·

n−(k−1)∑j=0

(n− (k − 1)

j

)xj.

Then the coefficient of xk of P (x) is

k−1∑i=0

(k − 1

i

)2(k−1)−i ·

(n− (k − 1)

k − i

)which is the left side of the identity.On the other hand

P (x) = (1 + (1 + x))k−1 · (1 + x)n

(1 + x)k−1=

(1

1 + x+ 1

)k−1

· (1 + x)n

=k−1∑i=0

(k − 1

i

)1

(1 + x)i· (1 + x)n =

k−1∑i=0

(k − 1

i

)(1 + x)n−i

=k−1∑i=0

(k − 1

i

) n−i∑j=0

(n− i

j

)xj.

Therefore the coefficient of xk of P (x) is

k−1∑i=0

(k − 1

i

)(n− i

k

)which is the right side of the identity.

Page 81: Eserciziario Di Analisi Matematica 2

Mathematics Magazine, Vol. 78, April 2005

1716. Proposed by Cafe Dalat Problem Solving Group, Washington D.C.

Let k, n be integers with n ≥ 1 and 0 ≤ k ≤ n. Prove that there is an n × nmatrix A of 0s and 1s with per(A) = k. (Here per(A) denotes the permanentof A.)

Solution proposed by G.R.A.20 Math Problems Group, Roma, Italy.

The permanent of a n× n matrix A = [aij] is given by the formula

per(A) =∑

π

a1π(1)a2π(2) · · · anπ(n)

where the sum runs over all permutations π of 1, 2, . . . n.

We first define the following n× n matrix

An,n =

[1 1T

n−1

1n−1 In−1

]By induction, per(An,n) = n: per(A1,1) = per(1) = 1 and for n > 1,expanding the computation of the permanent along the last row, we havethat

per(An,n) = per

([1T

n−2 1In−2 0n−2

])+ per

([1 1T

n−2

1n−2 In−2

])= per (In−2) + per(An−1,n−1) = 1 + (n− 1) = n.

Now we define

An,0 = 0n,n and An,k =

[Ak,k 0k,n−k

0n−k,k Ik

]for 0 < k < n.

Therefore

per(An,0) = 0 and per(An,k) = per(Ak,k) · per(In−k) = k · 1 = k.

Page 82: Eserciziario Di Analisi Matematica 2

Mathematics Magazine, Vol. 78, February 2005

1713. Proposed by Shawn Hedman and David Rose, Florida Southern Col-lege, Lakeland, FL.

Prove that∞∑

n=4

n−2∑k=2

1(nk

) =3

2.

Solution proposed by G.R.A.20 Math Problems Group, Roma, Italy.

We will use the following expession for the binomial coefficient(n

k

)=

nk

k!

where nk = n(n− 1) · · · (n− k + 1).For n ≥ k we compute the first difference (with respect to n)

∆n

(1

(n− 1)k−1

)=

1

nk−1− 1

(n− 1)k−1=

(n− k + 1)− n

n(n− 1) · · · (n− k + 1)= −k − 1

nk.

Therefore, changing order of summation (the terms are non negative), weobtain that

∞∑n=4

n−2∑k=2

1(nk

) =∞∑

k=2

∞∑n=k+2

k!

nk

=∞∑

k=2

(− k!

k − 1

) ∞∑n=k+2

∆n

(1

(n− 1)k−1

)

=∞∑

k=2

(− k!

k − 1

) (− 1

(k + 1)k−1

)=

∞∑k=2

2

(k − 1)(k + 1)

=∞∑

k=2

(1

k − 1− 1

k + 1

)= −

∞∑k=2

∆k

(1

k+

1

k − 1

)=

1

2+ 1 =

3

2.

Page 83: Eserciziario Di Analisi Matematica 2

Mathematics Magazine, Vol. 78, February 2005

1712. Proposed by William P. Wardlaw, U.S. Naval Academy, Annapolis,MD.

For each integer m > 1, let Zm = 0, 1, . . . ,m − 1 be the ring of integersmodulo m, and let Z∗

m be the (multiplicative) group of units in Zm. Find thesum S(m) =

∑u∈Z∗

mu and the product P (m) =

∏u∈Z∗

mu of elements in Z∗

m.

Solution proposed by G.R.A.20 Math Problems Group, Roma, Italy.

Let m = ps11 ps2

2 · · · pstt the prime factorization of m. We note that

Z∗m = Zm \

t⋃i=1

n ∈ Zm : pi| gcd(n, m) .

By the inclusion-exclusion principle we have that

S(m) =

(m

2

)−

t∑i=1

pi

(mpi

2

)+

∑1≤i<j≤t

pipj

( mpipj

2

)−· · ·+(−1)tp1 · · · pt

( mp1···pt

2

).

Using the Mobius function µ(n), the Euler function ϕ(n) and the identities

ϕ(m) =∑d|m

µ(d)m

d=

∑d|m

µ(m

d)d and

∑d|m

µ(d) = 0 for m ≥ 1

then

S(m) =∑d|m

µ(d)d

(md

2

)=

1

2

∑d|m

µ(d)dm

d

(m

d− 1

)=

=m

2

∑d|m

µ(d)m

d−

∑d|m

µ(d)

=mϕ(m)

2.

Similarly for P (m):

P (m) =∏d|m

[(m

d

)!d

md

]µ(d)

=∏d|m

[d!

(m

d

)d]µ(m

d)

= mϕ(m)∏d|m

(d!

dd

)µ(md

)

.

Page 84: Eserciziario Di Analisi Matematica 2

The College Mathematics Journal, Vol. 36, November 2005

814. Proposed by Jose Luis Dıaz-Barrero, Universitat Politecnica de Cataluna,Barcelona, Spain.

Suppose that a, b, and c are positive real numbers such that a+b+c = ab+bc+ca.Find the minimum value of

a3

(b− c)2 + bc+

b3

(c− a)2 + ca+

c3

(a− b)2 + ab.

Solution proposed by G.R.A.20 Math Problems Group, Roma, Italy.

We will prove a that for all integers n ≥ 3 then

Fn(a, b, c) =an

(b− c)2 + bc+

bn

(c− a)2 + ca+

cn

(a− b)2 + ab=

∑cyc

an

(b− c)2 + bc

has minimum value 3 which is attained for a = b = c = 1. Note that it canbe proved that for n = 1 and n = 2 the minimum is attained for a = b = 2,c = 0 and its value is respectively 1 and 2.Letting

σ1 = a + b + c σ2 = ab + bc + ca σ3 = abc

we first note that since by a mean inequality σ21 ≥ 3σ2 and s = σ1 = σ2 > 0

then s = σ1 = σ2 ≥ 3. Moreover, for k ≥ 2, by Muirhead’s inequality((n, 0, 0) ≥ (n− 1, 1, 0)) then

2∑cyc

ak =∑sym

ak ≥∑sym

ak−1b =∑cyc

ak−1(b + c)

that is ∑cyc

ak ≥ 1

3(a + b + c) ·

∑cyc

ak−1 ≥∑cyc

ak−1 ≥∑cyc

a2.

Now we distinguish two cases: when n is odd and when n is even.If n = 2k + 1 with k ≥ 1 then, since the map 1/x is convex for x > 0,

Fn(a, b, c) =∑cyc

ak+1 ·(

b2 + c2 − bc

ak

)−1

∑cyc

ak+1

2

∑cyc

a(b2 + c2 − bc)

Page 85: Eserciziario Di Analisi Matematica 2

and by the previous inequality

Fn(a, b, c) ≥

∑cyc

a2

2

∑cyc

a(b2 + c2 − bc)=

(σ21 − 2σ2)

2

σ1σ2 − 6σ3

.

We finally verify that the last term is ≥ 3, that is

(s2 − 2s)2 − 3(s2 − 6σ3)

is ≥ 0: since by Schur’s inequality σ31 + 9σ3 ≥ 4σ1σ2 then we have that

(s4 − 6s3 + 9s2) + 2(s3 + 9σ3 − 4s2) ≥ s2(s− 3)2 ≥ 0.

If n = 2k with k ≥ 2 then, since the map 1/x is convex for x > 0,

Fn(a, b, c) =∑cyc

ak ·(

b2 + c2 − bc

ak

)−1

∑cyc

ak

2

∑cyc

(b2 + c2 − bc)

and by the previous inequality

Fn(a, b, c) ≥

∑cyc

a2

2

∑cyc

(b2 + c2 − bc)=

(σ21 − 2σ2)

2

2σ21 − 5σ2

.

We finally verify that the last term is ≥ 3, that is

(s2 − 2s)2 − 3(2s2 − 5s)

is ≥ 0: for s ≥ 3

s4 − 4s3 − 2s2 + 15s = (s− 3)(s2 − s− 5) ≥ 0

because s ≥ 3.

2

Page 86: Eserciziario Di Analisi Matematica 2

The College Mathematics Journal, Vol. 36, November 2005

812. Proposed by Ovidui Furdui, Western Michigan University, Kalamazoo,MI.

Calculate the limitlim

n→∞e

∑n+1k=1 sin(1/k) − e

∑nk=1 sin(1/k)

Solution proposed by G.R.A.20 Math Problems Group, Roma, Italy.

The function is (esin(1/(n+1)) − 1

)· eHn · e−

∑nk=1(1/k−sin(1/k)).

Since Hn = log(n) + γ + o(1), then the limit

limn→∞

(esin(1/(n+1)) − 1

)· eHn = lim

n→∞(1/(n + 1)) · elog(n)+γ = eγ.

Moreover since 0 ≤ x − sin(x) ≤ x3/3 for x ≥ 0 then the following sumconverges as n goes to infinity because it has non negative terms and it isbounded

0 < c :=∞∑

k=1

(1/k − sin(1/k)) ≤ 1

6

∞∑k=1

1

k3<

1

6

(1 +

∫ +∞

1

1

x3dx

)=

1

4.

Therefore the required limit exists: it is equal to

eγ−c ∈ (eγ−1/4, eγ) ⊂ (1, 2)

and it is approximately equal to 1.470079645.

Page 87: Eserciziario Di Analisi Matematica 2

The College Mathematics Journal, Vol. 36, September 2005

809. Proposed by Ovidui Furdui, Western Michigan University, Kalamazoo,MI.

Find

limn→∞

(2n∑

k=n+1

(2

2k√

2k − k√

k)− n

).

Solution proposed by G.R.A.20 Math Problems Group, Roma, Italy.

We first note that

limn→∞

(2n∑

k=n+1

(2

2k√

2k − k√

k)− n

)= lim

n→∞

2n∑k=n+1

(2

2k√

2k − k√

k − 1)

.

Since

|ex − 1− x| ≤∞∑

k=2

|x|k

k!≤ e|x|2 for x ∈ (−1, 1)

then∣∣∣∣ 2k√

2k − 1− log(2k)

2k

∣∣∣∣ ≤ e

∣∣∣∣ log(2k)

2k

∣∣∣∣2 and

∣∣∣∣ k√

k − 1− log(k)

k

∣∣∣∣ ≤ e

∣∣∣∣ log(k)

k

∣∣∣∣2and the absolute value of the difference∣∣∣∣∣

2n∑k=n+1

(2

2k√

2k − k√

k − 1)−

2n∑k=n+1

(2

(1 +

log(2k)

2k

)−(

1 +log(k)

k

)− 1

)∣∣∣∣∣is less than

2e2n∑

k=n+1

∣∣∣∣ log(2k)

2k

∣∣∣∣2 + e2n∑

k=n+1

∣∣∣∣ log(k)

k

∣∣∣∣2which goes to zero as n goes to infinity because

∞∑k=1

∣∣∣∣ log(k)

k

∣∣∣∣2 < +∞.

Page 88: Eserciziario Di Analisi Matematica 2

Therefore the problem is equivalent to find the limit as n goes to infinity of

2n∑k=n+1

(2

(1 +

log(2k)

2k

)−(

1 +log(k)

k

)− 1

)= log(2) ·

2n∑k=n+1

1

k

that is, since∑n

k=1 1/k = Hn = log(n) + γ + o(1),

log(2)·(H2n −Hn) = log(2)·(log(2n) + γ − log(n)− γ + o(1)) = (log(2))2+o(1).

So the required limit is equal to (log(2))2.

2

Page 89: Eserciziario Di Analisi Matematica 2

The College Mathematics Journal, Vol. 36, September 2005

807. Proposed by Angelo S. DiDomenico, Milford, MA.

Let a, b, c and d be positive integers such that gcd(a, c) = gcd(b, d) = 1 and b, dhave opposite parity.

(a) Find necessary and sufficient conditions under which bc = ab + ad + cd.(b) In part (a), let x = bc − ab, y = bc − cd, and z = bc − ad. Show that

(x, y, z) generates all primitive Pythagorean triples.

Solution proposed by G.R.A.20 Math Problems Group, Roma, Italy.

We prove that b−d = a and b+d = a are necessary and sufficient conditionsunder which bc = ab+ad+ cd. Clearly they are sufficient conditions. We canwrite bc = ab+ad+cd as a(b+d) = c(b−d). Since (a, c) = 1, then b−d = ahand b + d = ch for some h ∈ Z+. Otherwise we can write bc = ab + ad + cdas b(c − a) = d(c + a) and since (b, d) = 1, then c − a = dk and c + a = bkfor some k ∈ Z+. Comparing these last equalities, we get hk = 2 and sinceb− d ≡ 1 mod 2, then h = 1.

Now x = bc − ab = 2bd, y = bc − cd = ac = b2 − d2 and z = bc − ad =b(b + d) − (b − d)d = b2 + d2 and since b, d are coprime and have oppositeparity, then they generate all primitive Pythagorean triples.

Page 90: Eserciziario Di Analisi Matematica 2

The College Mathematics Journal, Vol. 36, May 2005

805. Proposed by Jose Luis Dıaz-Barrero, Universitat Politecnica de Cataluna,Barcelona, Spain.

Let n be a positive integer. Prove that

F 4nF 4

n+1 ≤ n3(F 81 + F 8

2 + . . . + F 8n)

where Fn is the nth Fibonacci number defined by F0 = 0, F1 = 1, and for n ≥ 2,Fn = Fn−1 + Fn−2.

Solution proposed by G.R.A.20 Math Problems Group, Roma, Italy.

By the Power Mean Inequality we have that(F 2

1 + F 22 + . . . + F 2

n

n

)1/2

≤(

F 81 + F 8

2 + . . . + F 8n

n

)1/8

.

Therefore

(F 21 + F 2

2 + . . . + F 2n)4 ≤ n3(F 8

1 + F 82 + . . . + F 8

n),

and since the following identity holds

F 21 + F 2

2 + . . . + F 2n = FnFn+1

thenF 4

nF 4n+1 ≤ n3(F 8

1 + F 82 + . . . + F 8

n).

Page 91: Eserciziario Di Analisi Matematica 2

The College Mathematics Journal, Vol. 36, May 2005

802. Proposed by William P. Wardlaw, U. S. Naval Academy, Annapolis, MD.

Let n and k be fixed positive integers and let Mn(Zk) be the algebra of all n×n matricesover the ring Zk of integers modulo k. Randomly choose matrices A, B ∈ Mn(Zk). Whatis the probability that AB = I?

Solution proposed by G.R.A.20 Math Problems Group, Roma, Italy.

The required probability is the product of the probability to choose in Mn(Zk) aninvertible matrix A and the probability to choose the corresponding unique inverse B

P =

(u(n, k)

kn2

)·(

1

kn2

)=

1

kn2 ·∏p|k

n∏j=1

(1− 1

pj

)where kn2

= |Mn(Zk)| and u(n, k) is the number of invertible matrices in Mn(Zk):

u(n, k) = kn2∏p|k

n∏j=1

(1− 1

pj

).

We now show that the formula for u(n, k) holds (see also problem AMM10767).A matrix A is invertible in Mn(Zk) iff det(A) is invertible in Zk and, by the ChineseRemainder Theorem, this happens iff det(A) 6≡ 0 mod p for each prime p whichdivides k. The construction of a generic invertible matrix in Mn(Zp) when p is primegoes as follows. We can select the first row in pn − 1 ways. The jth row have to belinearly independent with respect to the first j − 1 rows, therefore it can be selectedin a number of ways given by pn, the number of vectors in (Zp)

n, minus pj−1, thenumber of linear combinations of the first j − 1 rows. Hence

u(n, p) =n∏

j=1

(pn − pj−1

)= pn2

n∏j=1

(1− 1

pj

).

In order to determine a generic invertible matrix in Mn(Zpα) we can modify eachentry of an invertible matrix in Mn(Zp) in pα−1 ways. Therefore

u(n, pα) =(pα−1

)n2

· u(n, p).

Finally for k =∏T

t=1 pαtt , a generic invertible matrix in Mn(Zk) can be obtained by

choosing an invertible matrix in Mn(Zpαtt

) for each prime pt and solving the systemof congruences. Therefore, by the Chinese Remainder Theorem,

u(n, k) =T∏

t=1

u(n, pαtt ) =

T∏t=1

(pαt−1

t

)n2

· u(n, pt)

=T∏

t=1

(pαt−1

t

)n2

· pn2

t

n∏j=1

(1− 1

pjt

)= kn2

T∏t=1

n∏j=1

(1− 1

pjt

).

Page 92: Eserciziario Di Analisi Matematica 2

The College Mathematics Journal, Vol. 36, March 2005

800. Proposed by Hojoo Lee, Seoul National University, Seoul, Korea.

Let f(x) be a real cubic polynomial with positive leading coefficient such thatf(θ) ≤ θf ′(θ) ≤ 2f(θ) for some θ > 0. Suppose that both f(0) and f(θ) arenon-negative. Show that f(x) ≥ 0 for all x ≥ 0.

Solution proposed by G.R.A.20 Math Problems Group, Roma, Italy.

Let f(x) = ax3 + bx2 + cx + d with a > 0. Since f(θ) ≤ θf ′(θ) then

aθ3 + bθ2 + cθ + d ≤ θ(3aθ2 + 2bθ + c)

that isd ≤ 2aθ3 + bθ2

and this implies that (θ is positive)

b ≥ d

θ2− 2aθ.

Since θf ′(θ) ≤ 2f(θ) then

θ(3aθ2 + 2bθ + c) ≤ 2(aθ3 + bθ2 + cθ + d)

that isaθ3 ≤ cθ + 2d

and this implies that (θ is positive)

c ≥ aθ2 − 2d

θ.

Now, for x ≥ 0, using the two lower bounds established above, we have that

f(x) = ax3 + bx2 + cx + d ≥ ax3 +

(d

θ2− 2aθ

)x2 +

(aθ2 − 2d

θ

)x + d

≥ ax(x2 − 2θx + θ2

)+ d

((x

θ

)2

− 2(x

θ

)+ 1

)= ax (x− θ)2 + d

(x

θ− 1

)2

≥ 0.

where the last inequality holds because a > 0 and d = f(0) ≥ 0.

Page 93: Eserciziario Di Analisi Matematica 2

The College Mathematics Journal, Vol. 36, March 2005

797. Proposed by Jose Luis Dıaz-Barrero, Universitat Politecnica de Catalunya,Barcelona, Spain.

Let n be a positive integer. Prove that

n∑k=1

1(cos

(n− k)π

4n

)2 ≤4n

π≤

n∑k=1

1(cos

4n

)2

Solution proposed by G.R.A.20 Math Problems Group, Roma, Italy.

Consider the function f(x) = 1/ cos2x. Since it is increasing in the interval[0, π/4] then

n−1∑k=0

f(xk)(xk+1 − xk) ≤∫ π/4

0

f(x) dx ≤n−1∑k=0

f(xk+1)(xk+1 − xk)

where xk = (k/n)(π/4) for k = 1, . . . , n. Therefore

n−1∑k=0

1(cos

4n

)2

( π

4n

)≤ [tan x]π/4

0 ≤n−1∑k=0

1(cos

(k + 1)π

4n

)2

( π

4n

),

that is ( π

4n

) n∑k=1

1(cos

(n− k)π

4n

)2 ≤ 1 ≤( π

4n

) n∑k=1

1(cos

4n

)2

which is equivalent to the required inequality.

Page 94: Eserciziario Di Analisi Matematica 2

The College Mathematics Journal, Vol. 36, March 2005

796. Proposed by Andrew Cusamano, Great Neck, NY.

Let the Fibonacci sequence be defined as f1 = 1, f2 = 1, fn = fn−1+fn−2, n ≥ 3.

Let α = 1+√

52

.(a) Show that

1 +∞∑

k=1

(−1)k+1

fkfk+1

converges to α.(b) Let N ∈ Z+. Express each ak in terms of Fibonacci numbers so that theseries

1 +∞∑

k=1

ak

(fkfk+1)N

converges to αN .

Solution proposed by G.R.A.20 Math Problems Group, Roma, Italy.

(a) By the Cassini’s identity (−1)k+1 = −f 2k+1 + fk+2fk, we have that

1 +∞∑

k=1

(−1)k+1

fkfk+1

= 1 +∞∑

k=1

−f 2k+1 + fk+2fk

fkfk+1

= 1 +∞∑

k=1

(−fk+1

fk

+fk+2

fk+1

)= lim

n→∞1 +

(−1 + 2− 2 +

3

2+ · · · − fn+1

fn

+fn+2

fn+1

)= lim

n→∞

fn+2

fn+1

= α.

(b) Let ak = −f 2Nk+1 + fN

k+2fNk for k ≥ 1. Hence we find out that

1+∞∑

k=1

ak

(fkfk+1)N= 1+

∞∑k=1

(−(

fk+1

fk

)N

+

(fk+2

fk+1

)N)

= limn→∞

(fn+2

fn+1

)N

= αN .

Page 95: Eserciziario Di Analisi Matematica 2

The College Mathematics Journal, Vol. 35, November 2004

792. Proposed by Jose Luis Dıaz-Barrero, Universitat Politecnica de Catalunya,Barcelona, Spain.

Suppose that x1, x2, . . . , xn and α are positive real numbers. Prove that

1

n

n∑k=1

(xk + αk)2 ≥

(α(n+1)/2 +

n∏k=1

x1/nk

)2

.

Solution proposed by G.R.A.20 Math Problems Group, Roma, Italy.

Squaring each member, the inequality becomes

1

n

n∑k=1

x2k +

2

n

n∑k=1

xkαk +

1

n

n∑k=1

α2k ≥n∏

k=1

x2/nk + 2α(n+1)/2

n∏k=1

x1/nk + αn+1.

It holds because by AGM inequality we have that

1

n

n∑k=1

x2k ≥

(n∏

k=1

x2k

)1/n

=n∏

k=1

x2/nk ,

2

n

n∑k=1

xkαk ≥ 2

(n∏

k=1

xkαk

)1/n

= 2α(n+1)/2

n∏k=1

x1/nk ,

1

n

n∑k=1

α2k ≥

(n∏

k=1

α2k

)1/n

= αn+1.

Page 96: Eserciziario Di Analisi Matematica 2
Page 97: Eserciziario Di Analisi Matematica 2
Page 98: Eserciziario Di Analisi Matematica 2
Page 99: Eserciziario Di Analisi Matematica 2

Mathematics Magazine, Vol.77, December 2004

1709.Proposed by Mihaly Bencze, Sacele-Negyfalu, Romania.

Let x1, x2, . . . , x3n ≥ 0. Prove that

2n ·3n∏

k=1

1 + x2k

1 + xk

(1 +

3n∏k=1

x1/nk

)n

.

Solution proposed by G.R.A.20 Math Problems Group, Roma, Italy.

First we prove the two following facts:

1. Let x, y, z ≥ 0 then

1 + x2

1 + x· 1 + y2

1 + y· 1 + z2

1 + z≥ 1 + xyz

2.

The above inequality is equivalent to

(1+x2)(y−z)2+(1+y2)(z−x)2+(1+z2)(x−y)2+(xyz−1)(x−1)(y−1)(z−1) ≥ 0,

and therefore it holds trivially when 0 ≤ x, y, z ≤ 1 and when x, y, z ≥ 1.By symmetry we have two remaining cases to check: x ≥ 1 ≥ y ≥ z andx ≥ y ≥ 1 ≥ z. Letting f(x) = (1 + x2)/(1 + x) then xf(1/x) = f(x) and inthe first case:

f(x)f(y)f(z) = xf(1/x)f(y)f(z) ≥ x1 + yz/x

2=

x + yz

2≥ 1 + xyz

2

where the last inequality holds because (x− 1)(1− yz) ≥ 0.On the other hand, in the second case:

f(x)f(y)f(z) = xf(1/x)yf(1/y)f(z) ≥ xy1 + z/(xy)

2=

xy + z

2≥ 1 + xyz

2

where the last inequality holds because (z − 1)(1− xy) ≥ 0.

2. Let a0, a1, . . . , an−1 ≥ 0 then

n−1∏i=0

(1 + ai) ≥

1 +

(n−1∏i=0

ai

)1/nn

Let σk/(

nk

)be the symmetric mean of kth order of the sequence a0, a1, . . . , an−1

then by the AGM inequality

σk/

(n

k

)≥ (σn)k/n,

Page 100: Eserciziario Di Analisi Matematica 2

and therefore

n−1∏i=0

(1 + ai) =n∑

k=0

σk ≥n∑

k=0

(n

k

)(σn)k/n

=(1 + (σn)1/n

)n=

1 +

(n−1∏i=0

ai

)1/nn

.

Now we prove the main inequality using first 1. and then 2.

2n ·3n∏

k=1

1 + x2k

1 + xk

=n−1∏i=0

(2 ·

3∏j=1

1 + x23i+j

1 + x3i+j

)≥

n−1∏i=0

(1 +

3∏j=1

x3i+j

)

1 +

(n−1∏i=0

3∏j=1

x3i+j

)1/nn

=

(1 +

3n∏k=0

x1/nk

)n

.

It is interesting to note that the more general inequality

2n ·an∏

k=1

1 + x2k

1 + xk

(1 +

an∏k=1

x1/nk

)n

where a is a positive integer and x1, x2, . . . , xan ≥ 0 holds if and only ifa = 1, 2, 3 (for a ≥ 4, take xk = 2 and note that 2 · 5a < 3a + 6a). Thismotivates the choice of the number 3 in the statement.

2

Page 101: Eserciziario Di Analisi Matematica 2

Mathematics Magazine, Vol.77, December 2004

1708.Proposed by Stephen J. Herschkorn, Highland Park, NJ.

It is well known that the area of a square is half of the length of its diagonal.Show that if the area of a parallelogram is half the square of one of its diagonals,and if the area and each side have rational measure, then the parallelogram is asquare.

Solution proposed by G.R.A.20 Math Problems Group, Roma, Italy.

We can assume that the sides of the parallelogram have integer lengths a ≥b > 0 and that 0 < ϕ ≤ π/2 is the angle between these sides. Hence

A = ab sin(ϕ) and d2± = a2 + b2 ± 2ab cos(ϕ)

where A is the area and d− ≤ d+ are the lengths of its two diagonals. Since

2A = 2ab sin(ϕ) ≤ 2ab ≤ a2 + b2 ≤ a2 + b2 + 2ab cos(ϕ) = d2+,

if 2A = d2+ then a = b and ϕ = π/2 that is the parallelogram is a square.

Now assume that 2A = d2− then cos(ϕ) = x− sin(ϕ) where x is the rational

number (a2 + b2)/2ab ≥ 1. We have that

1 = cos2(ϕ) + sin2(ϕ) = (x− sin(ϕ))2 + sin2(ϕ) = x2 − 2x sin(ϕ) + 2 sin2(ϕ)

and2 sin2(ϕ)− 2x sin(ϕ) + (x2 − 1) = 0.

It follows that sin(ϕ) is equal to one of these numbers

1

2

(x±

√2− x2

)and since A is rational then sin(ϕ) is rational too and 2− x2 = y2 for somerational number 0 ≤ y ≤ 1. Therefore c = 2aby is a non-negative integersuch that

c2 = 2(2ab)2 − (a2 + b2)2 = (2ab)2 − (a2 − b2)2.

Moreover(a2 + b2)2 = (2ab)2 + (a2 − b2)2

and by multiplying these two equations we obtain that X = a2 − b2, Y =c(a2 +b2) and Z = 2ab is a non-negative solution of the diophantine equation

X4 + Y 2 = Z4.

By the following lemma, X = 0 or Y = 0. In the first case a = b, x = 1,y = 1, sin(ϕ) = 1 which means that the parallelogram is a square. In thesecond case c = 0, y = 0 and x2 = 2 contradicting the fact that x is rational.

Page 102: Eserciziario Di Analisi Matematica 2

Lemma. There are no positive integers X,Y, Z which solve the diophantineequation

X4 + Y 2 = Z4.

Proof. We will use the method of the infinite descent. Let X, Y, Z be positiveintegers which satisfy the equation and assume that Z is the minimum one.(X2, Y, Z2) is a pythagorean triple and we have two cases:

i) X2 = P 2−Q2, Y = 2PQ and Z2 = P 2 + Q2 for some integers P > Q > 0.Then (XZ)2 = (P 2 −Q2)(P 2 + Q2) = P 4 −Q4 that is

Q4 + (XZ)2 = P 4 < Z4

which contradicts the minimality of Z.

ii) X2 = 2PQ, Y = P 2−Q2 and Z2 = P 2 +Q2 for some integers P > Q > 0.Since 2PQ is a square and P and Q are relatively prime (otherwise X, Y, Zhave a common factor contradicting the minimality of Z) then P = 2U2,Q = V 2 and Z2 = 4U4 + V 4. (2U2, V 2, Z) is a pythagorean triple with2U2 = 2ST (Q is odd because P is even) and V 2 = S2−T 2 for some integersS > T > 0. Since ST is a square and S and T are relatively prime thenS = A2, T = B2 and V 2 = A4 −B4 that is

B4 + V 2 = A4 < Z4

which contradicts the minimality of Z.

2

Page 103: Eserciziario Di Analisi Matematica 2

Mathematics Magazine, Vol.77, December 2004

1706.Proposed by Steve Edwards and James Whitenton, Southern Polytech-nic State University, GA.

Let 0 < a, b < 1. Evaluate+∞∏

n=−∞

1 + b2n

1 + a2n .

Solution proposed by G.R.A.20 Math Problems Group, Roma, Italy.

Let

PN =n=+N∏n=−N

1 + b2n

1 + a2n = P+N · P−N

where

P+N =

n=N∏n=0

1 + b2n

1 + a2n =n=N∏n=0

1− b2n+1

1− b2n ·n=N∏n=0

1− a2n

1− a2n+1 =1− b2N+1

1− b· 1− a

1− a2N+1

and

P−N =n=N∏n=1

1 + 2n√b

1 + 2n√a

=n=N∏n=1

1− 2n−1√b

1− 2n√b·

n=N∏n=1

1− 2n√a

1− 2n−1√a

=1− b

1− 2N√b· 1− 2N√

a

1− a.

Hence

PN =1− b2N+1

1− a2N+1 ·1− 2N√

a

1− 2N√b.

Since 0 < a, b < 1 then log a, log b < 0 and

limN→+∞

PN = limN→+∞

1− 2N√a

1− 2N√b

= limN→+∞

elog a/2N − 1

elog b/2N − 1=

log a

log b.

Page 104: Eserciziario Di Analisi Matematica 2

Mathematics Magazine, Vol.77, October 2004

1705.Proposed by Michel Bataille, Rouen, France.

Let be n a positive integer. Find the minimum value of

(a− b)2n+1 + (b− c)2n+1 + (c− a)2n+1

(a− b)(b− c)(c− a)

for distinct real numbers a, b, c with bc + ca ≥ 1 + ab + c2.

Solution proposed by G.R.A.20 Math Problems Group, Roma, Italy.

We’ll show that the minimum is 4n−1. Letting x = a−b, y = b−c, z = c−athen x = −y− z and the problem is equivalent to find the minimum value of

F (y, z) =(y + z)2n+1 − z2n+1 − y2n+1

yz(y + z)

for y 6= 0, z 6= 0, y + z 6= 0 and yz ≥ 1.Since yz ≥ 1 and F (y, z) = F (−y,−z) we can assume that y, z > 0.

First proof.

F (y, z) =

2n∑k=1

(2n + 1

k

)y2n−kzk−1

y + z

Since yz ≥ 1 then

2n∑k=1

(2n + 1

k

)y2n−kzk−1 =

n∑k=1

(2n + 1

k

)(y2n−kzk−1 + yk−1z2n−k)

≥n∑

k=1

(2n + 1

k

)(y2(n−k)+1 + z2(n−k)+1).

Moreover for α ≥ 1, tα is convex for t ≥ 0 and

yα+1 + zα+1

y + z=

y

y + zyα +

z

y + zzα ≥

(y2 + z2

y + z

≥ 1

where in the last step we use the fact that yz ≥ 1. Hence

F (y, z) ≥n∑

k=1

(2n + 1

k

)y2(n−k)+1 + z2(n−k)+1

y + z

≥n∑

k=1

(2n + 1

k

)= (22n+1 − 2)/2 = 4n − 1 = F (1, 1).

Page 105: Eserciziario Di Analisi Matematica 2

Second proof.

F (y, z) =2n−1∑k=1

((2n

k

)− (−1)k

)y2n−kzk

We have that

2n−1∑k=1

((2n

k

)− (−1)k

)= 4n−1 and

2n−1∑k=1

k

((2n

k

)− (−1)k

)= n(4n−1).

Then by AGM inequality

F (y, z) ≥ (4n − 1)

(2n−1∏k=1

(y2n−kzk

)((2nk )−(−1)k)

)1/(4n−1)

≥ (4n − 1)(yn(4n−1)zn(4n−1)

)1/(4n−1)= (4n − 1)(yz)n

≥ 4n − 1 = F (1, 1)

where in the last step we use the fact that yz ≥ 1.

2

Page 106: Eserciziario Di Analisi Matematica 2

Mathematics Magazine, Vol.77, October 2004

1701.Proposed by Murray Klamkin, University of Alberta, Edmunton, AB.

Prove that for all positive numbers a, b, c, d,

a4b + b4c + c4d + d4a ≥ abcd(a + b + c + d).

Solution proposed by G.R.A.20 Math Problems Group, Roma, Italy.

We will prove the following more general result: let n ≥ 2 be an integer then

n∑k=1

ank · ak+1 ≥

(n∑

k=1

ak

(n∏

k=1

ak

)

for all positive numbers a1, a2, . . . , an, an+1 = a1.For n = 2 the inequality is trivial. Assume that n ≥ 3 and let

xk =1

ak

( ∏j 6=k,k+1

aj

)1/(n−2)

for k = 1, . . . , n.

Since the function f(x) = 1/xn−2 is convex in (0, +∞) then

n∑k=1

ank · ak+1(

n∑k=1

ak

(n∏

k=1

ak

) =

n∑k=1

ak · f(xk)(n∑

k=1

ak

) ≥ f

n∑

k=1

ak · xk(n∑

k=1

ak

)

=

n∑

k=1

ak

n∑k=1

( ∏j 6=k,k+1

aj

)1/(n−2)

n−2

n∑

k=1

ak

n∑k=1

( ∑j 6=k,k+1

aj

)/(n− 2)

n−2

= 1

where in the last step we applied the AGM inequality.

Page 107: Eserciziario Di Analisi Matematica 2

Mathematics Magazine, Vol.77, June 2004

1699.Proposed by Zhang Yun, First Middle School of Jinchung City, GanSu, China.

Let A1A2A3A4 be a nondegenerate tetrahedron, let hk, 1 ≤ k ≤ 4, be the lenghtof the altitude from Ak, and let r be the radius of the inscribed sphere. Provethat

h1

h1 + 3r+

h2

h2 + 3r+

h3

h3 + 3r+

h4

h4 + 3r≥ 16

7.

Solution proposed by G.R.A.20 Problems Group, Roma, Italy.

Let Sk be the area of the face opposite Ak, and let V be the volume of thetetrahedron. Since

1

3hkSk = V =

1

3r(S1 + S2 + S3 + S4) for 1 ≤ k ≤ 4

then4∑

k=1

r

hk

= 1.

Now we consider the function f(x) = 1/(1 + 3x) which satisfy by convexitythe following inequality

f(x) ≥ f ′(

1

4

) (x− 1

4

)+ f

(1

4

)∀x ∈ (−1

3, +∞).

So we have that

4∑k=1

f

(r

hk

)≥

4∑k=1

(f ′

(1

4

) (r

hk

− 1

4

)+ f

(1

4

))= 4f

(1

4

)=

16

7.

Page 108: Eserciziario Di Analisi Matematica 2

Mathematics Magazine, Vol.77, June 2004

1696. Proposed by Albert F. S. Wong, Temasek Polytechnic, Singapore.

For which positive integers k does the equation

x2k−1 + y2k = z2k+1

have a solution in positive integers x, y, and z?

Solution proposed by G.R.A.20 Problems Group, Roma, Italy.

We will show that for any integer k ≥ 1 the equation has a solution in positive integersx, y, and z of this form

x = 2α, y = 2β, z = 2γ

where α, β, and γ are positive integers such that

α(2k − 1) = β(2k) = γ(2k + 1)− 1.

These conditions are equivalent to find positive integers γ, m, and n such thatα =

γ(2k + 1)− 1

2k − 1= γ +

2γ − 1

2k − 1= γ + m

β =γ(2k + 1)− 1

2k= γ +

γ − 1

2k= γ + n

.

Therefore we have that m and n satisfy the following diophantine linear equation

(2k − 1)m− (4k)n = 1

which has infinite positive solutions for any integer k ≥ 1 because gcd(2k−1, 4k) = 1and the product of the coefficients is negative. Letting m = 2k− 1 and n = k− 1 weget the “simplest” solution and we have that

α = 2k2, β = 2k2 − k γ = 2k2 − 2k + 1.

Thus(2α)2k−1 + (2β)2k = 24k3−2k2

+ 24k3−2k2

= 24k3−2k2+1 = (2γ)2k+1.

Here are the numerical values of α, β, and γ for k = 1, 2, 3.

α β γk=1 2 1 1k=2 8 6 5k=3 18 15 13

Page 109: Eserciziario Di Analisi Matematica 2

Mathematics Magazine, Vol.77, April 2004

1693. Proposed by Erwin Just and Norman Schaumberger, Bronx Community College ofthe City of New York, Bronx, NY.

Let A = (x, y), B = (x2, y2) e C = (x3, y3) be the vertices of a non degenerate triangle.1) For how many pairs (x, y) is triangle ABC equilateral?2) If x or y is rational, can triangle ABC be equilateral?

Solution proposed by G.R.A.20 Problems Group, Roma, Italy.

The solution we present need two awful computations which we made by using Mathematica.However these algorithms have to be considered exact: they perform a finite number ofelementary operations between polynomials with integer coefficients.

The triangle ABC is equilateral iff |AB| = |BC| = |AC| that is

(x2 − x)2 + (y2 − y)2 = (x3 − x2)2 + (y3 − y2)2 = (x3 − x)2 + (y3 − y)2.

These equations give the following symmetric polynomial system:F (x, y) = (x6 + y6)− 3(x4 + y4) + 2(x3 + y3) = 0G(x, y) = 2(x5 + y5)− 3(x4 + y4) + (x2 + y2) = 0

Considering these polynomials with respect to the variable x then they have a commonroot iff the coefficent y is a root of the resultant Rx(F, G) which can be calculated usingMathematica: Rx(F, G) = 4y10(y − 1)10Q(y) where

P (y) = 32y10 + 32y9 − 40y8 − 64y7 − 10y6 + 26y5 + 5y4 − 76y3 − 175y2 − 162y − 54.

Since the triangle is non degenerate then y 6= 0, 1 and it suffices to consider P (y). We firstnote that if there is an equilateral triangle ABC with x or y is rational, by symmetry, wecan assume that y is rational. But Q(y) has no rational roots: one can easily check there isno roots among the finite set of fractions ±p/q with p a divisor of the last coefficient 54 andq a divisor of the first coefficient 32. So the answer to the second question is no.In order to find the number distinct real roots of P (y) we generate from P0(y) = P (y) and thederivative P1(y) = P ′(y) by the Euclidean Algorithm (changing the sign of the remainder) aSturm sequence of polynomials with decreasing degree (Mathematica si very helpful also forthese computations). Then we determine the number of sign changes of these polynomialsfor y that goes to −∞ and +∞. The difference of these two numbers is just the number ofdistinct real roots of P (y).

P0 P1 P2 P3 P4 P5 P6 P7 P8 P9 P10 ∆−∞ + − + − + + + − − − + 6+∞ + + + + + − + + − + + 4

Hence there are 6− 4 = 2 real roots. Here are the approximate values:

y1 = 1.48459 and y2 = −0.87791.

Page 110: Eserciziario Di Analisi Matematica 2

Finally, by symmetry, x can assume only these two values and therefore all the possiblesolutions (x, y) are:

(y1, y1) (y2, y2) (y1, y2) (y2, y1).

The first two pair do not work because when x = yF (x, x) = 2x3(x− 1)2(2x + 1) = 0G(x, x) = 2x3(x− 1)2(2 + x) = 0

which is solved by x = 0 and x = 1 (degenerate triangles). The two remaining symmetricpairs have to be solutions because the resultant Rx(F, G) is zero. So the answer to the firstquestion is: there are exactly two non degenerate equilateral triangles.

2

Page 111: Eserciziario Di Analisi Matematica 2

Mathematics Magazine, Vol.77, February 2004

1687. Proposed by Sung Soo Kim, Hanyang University, Ansan Kyunggi,Korea.

A two-player game starts with two sticks, one of length n and one of lengthn + 1, where n is a positive integer. Players alternate turns. A turn consists ofbreaking a stick into two sticks of positive integer lengths, or removing k sticksof length k for some positive integer k. The player who makes the last movewins. Which player can force a win?

Solution proposed by G.R.A.20 Problems Group, Roma, Italy.

The second player can always force a win using this strategy: whenever thereis a stick of positive even length then divide it in two sticks of odd lengths.Let E(m) and T (m) be respectively the number of sticks of even lengths andthe total number of sticks before the m-th move, then in order to prove thatthe stategy works it suffices to show that:

a) E(m) ≤ 1 when m is odd and E(m) ≤ 2 when m is even.b) T (m) is even when m is odd and T (m) is odd when m is even.

When the game start E(1) = 1 and T (1) = 2 so a) and b) hold. Now assumethat a) and b) are satisfied up to the m-th move. Here is a table which showsthe variations of E and T when a certain move is made

Move 1 2 3 4 5∆E +1 −1 +1 0 −k∆T +1 +1 +1 −k −k

where:1. Divide a positive even stick in two even sticks.2. Divide a positive even stick in two odd sticks.3. Divide an odd stick in two sticks one even and the other odd.4. Remove k sticks of length k with k odd.5. Remove k sticks of length k with k even.

Suppose first that it is second player turn: m is even and if E(m) is 1 or 2then he plays 2 and E(m+1) ≤ 1. Otherwise E(m) = 0 and since T (m) 6= 0he can play 3 or 4 then E(m+1) ≤ 1 in both cases. T (m+1) is even becauseT (m) is odd and he does not play 5.Finally assume that it is first player turn: m is odd and if T (m) = 0 thenthe game is finished and the second player wins. Otherwise, since he can notplay 5, then whatever move he chooses E(m + 1) ≤ 2 and T (m + 1) is odd.

Page 112: Eserciziario Di Analisi Matematica 2

The College Mathematics Journal, Vol. 35, November 2004

789. Proposed by Zengxiang Tong and Zhen Huang, Westerville, OH.

1. Show the convergence of the sequence given by a1 = 2√

2 and

an = 2n+1

2

√2n −

√4n − a2

n−1

for any n > 1, and find the limit limn→∞ an.2. Show the convergence of the sequence given by b1 = 2 and

bn = 2n2

√2n−1 −

√4n−1 − b2

n−1

for any n > 1, and find the limit limn→∞ bn.

Solution proposed by G.R.A.20 Math Problems Group, Roma, Italy.

For n ≥ 2 let An be the area of the regular 2n-polygon inscribed in thecircumference of radius 1. Then An = 2n−1 sin(π/2n−1).Morever, for n > 3 we have that cos(π/2n−2) ≥ 0 and

An = 2n−1

√1− cos(π/2n−2)

2

= 2n− 32

√1−

√1− sin2(π/2n−2)

= 2n− 32

√1−

√1− (An−1/2n−2)2

= 2n−1

2

√2n−2 −

√4n−2 − A2

n−1.

Since a1 = b2 = A3 = 2√

2 then an−2 = bn−1 = An and for n > 3. Thereforein both cases the limit is the area of the circle of radius 1, that is π.

Page 113: Eserciziario Di Analisi Matematica 2

The College Mathematics Journal, Vol. 35, September 2004

785. Proposed by Jose Luis Dıaz-Barrero, Univesitat Politecnica de Catalunya,Barcelona, Spain.

Prove that in any acute triangle ABC, with the usual notations, the followinginequality holds:

a2

√b2 + c2

cos(A/2) +b2

√c2 + a2

cos(B/2) +c2

√a2 + b2

cos(C/2) ≥ 9

2

√2 r

(r is the inradius of the triangle).

Solution proposed by G.R.A.20 Math Problems Group, Roma, Italy.

Let I be the left side of the inequality then by the AGM inequality

I ≥ 3

(a2

√b2 + c2

b2

√c2 + a2

c2

√a2 + b2

cos(A/2) cos(B/2) cos(C/2)

)1/3

.

Since the triangle ABC is acute then cos(A) ≥ 0 and by Carnot’s theorem

a√b2 + c2

=

√1− 2bc

b2 + c2cos(A) ≥

√1− cos(A) =

sin(A)√2 cos(A/2)

.

Hence by repeating the same argument for the other angles we obtain

I ≥ 3√2

(a b c sin(A) sin(B) sin(C))1/3 =3√2

(8 S3

abc

)1/3

= 3√

2S

(abc)1/3

where S = ab sin(C)/2 = bc sin(A)/2 = ca sin(B)/2 is the area of the triangle.Finally we apply AGM inequality again and we use S = r(a + b + c)/2

I ≥ 3√

2S

(abc)1/3≥ 3√

23S

a + b + c=

9

2

√2 r.

Page 114: Eserciziario Di Analisi Matematica 2

The College Mathematics Journal, Vol. 35, May 2004

780. Proposed by Jose Luis Dıaz-Barrero, Univesitat Politecnica de Catalunya,Barcelona, Spain.

Let n be a positive integer. Prove that(1

n

n∑k=1

1

F 2k

)n

≥ en−FnFn+1

where Fn is the nth Fibonacci number; that is, F0 = 0, F1 = 1, and for n ≥ 2,Fn = Fn−1 + Fn−2.

Solution proposed by G.R.A.20 Problems Group, Roma, Italy.

We present two solutions. In both of them we use the identity

n∑k=1

F 2k = FnFn+1.

1. Since 1/x is convex in (0, +∞) and ln x ≤ x− 1 for x > 0

1

n

n∑k=1

1

F 2k

≥ 1

1

n

n∑k=1

F 2k

=n

FnFn+1

= e− ln

(FnFn+1

n

)≥ e1−FnFn+1

n .

which is equivalent to the required inequality.

2. By the AGM inequality and since ln x ≤ x− 1 for x > 0 then(1

n

n∑k=1

1

F 2k

)n

≥n∏

k=1

1

F 2k

= e−∑n

k=1 ln(F 2k ) ≥ e−

∑nk=1(F 2

k−1) = en−FnFn+1 .

Page 115: Eserciziario Di Analisi Matematica 2

The College Mathematics Journal, Vol.35, March 2004

779. Proposed by Kedar Hardikar, San Jose, CA.

Let A be a positive definite linear operator on Rn (i.e., Ax ·x > 0 for all x ∈ Rn

with x 6= 0). Evaluate

I =

∫Rn

e−Ax·xdx

where dx denotes an infinitesimal element in Rn (i.e., dx = dx1dx2 . . . dxn).

Solution proposed by G.R.A.20 Problems Group, Roma, Italy.

First we consider the matrix A as the sum of the symmetric matrix As andthe antisymmetric Aa where

As =1

2(A + At), Aa =

1

2(A− At).

Since As is symmetric, then there will be an orthogonal matrix U such thatAs = U tDU , where

D =

λ1 0 . . . 00 λ2 . . . 0...

. . . . . ....

0 . . . 0 λn

with λk > 0 for k = 1, . . . , n.Since Aax · x = 0 for all x ∈ Rn, letting y = Ux, we have that

Ax · x = Asx · x = U tDUx · x = Dy · y and dy = | det(U)|dx = dx.

Therefore∫Rn

e−Ax·xdx =

∫Rn

e−Dy·ydy =

∫Rn

e−(λ1y21+···+λny2

n)dy1 · · · dyn

=n∏

k=1

1√λk

∫R

e−(λky2k)d

(√λkyk

)=

n∏k=1

√π√λk

=

√πn

det(As).

Page 116: Eserciziario Di Analisi Matematica 2

The College Mathematics Journal, Vol.35, March 2004

775. Proposed by John H. Webb, University of Cape Town, Rondebosch,South Africa

Let a, b and c be the lengths of the sides of a triangle. Show that

13

27≤ (a2 + b2 + c2)(a + b + c) + 4abc

(a + b + c)3<

1

2.

Solution proposed by G.R.A.20 Problems Group, Roma, Italy.

By the triangular inequality, we know that a + b− c > 0, b + c− a > 0 anda + c − b > 0. Now we apply the arithmetic-geometric mean inequality tothese three real positive numbers:

0 < (a + b− c)(b + c− a)(a + c− b) ≤(

a + b + c

3

)3

.

Dividing by −2(a + b + c)3 and then adding 12

we get

1

2− 1

54≤ 1

2− (a + b− c)(b + c− a)(a + c− b)

2(a + b + c)3<

1

2

and therefore the inequality becomes

13

27≤ (a + b + c)3 − (a + b− c)(b + c− a)(a + c− b)

2(a + b + c)3<

1

2

that is13

27≤ (a2 + b2 + c2)(a + b + c) + 4abc

(a + b + c)3<

1

2.

Page 117: Eserciziario Di Analisi Matematica 2

The College Mathematics Journal, Vol.35, March 2004

771. Proposed by Yi-chuan Pan, Jackson State University, Jackson, MS

Show that there are infinitely many rational numbers x such that both x2 + 5and x2 − 5 shall each be squares of rational numbers.

Solution proposed by G.R.A.20 Problems Group, Roma, Italy.

The following sequence gives infinitely many rational numbers with the re-quired property:

x0 = 41/12 and xn = (x4n−1 + 25)/(2xn−1

√x4

n−1 − 25) for n ≥ 1.

We define for n ≥ 1 pn = p4

n−1 + 25q4n−1

qn = 2pn−1qn−1rn−1sn−1

rn = 10p2n−1q

2n−1 + r2

n−1s2n−1

sn = 10p2n−1q

2n−1 − r2

n−1s2n−1

with p0 = 41, q0 = 12, r0 = 49 and s0 = 31. Now we show by induction that

r2n = p2

n + 5q2n and s2

n = p2n − 5q2

n for n ≥ 1.

These equalities are satisfied for n = 0 and if n ≥ 1 then

p2n ± 5q2

n = (p4n−1 + 25q4

n−1)2 ± 5(2pn−1qn−1rn−1sn−1)

2

= 100p4n−1q

4n−1 + (p4

n−1 − 25q4n−1)

2 ± 20(pn−1qn−1rn−1sn−1)2

= (10p2n−1q

2n−1)

2 + (r2n−1s

2n−1)

2 ± 20(pn−1qn−1rn−1sn−1)2

= (10p2n−1q

2n−1 ± r2

n−1s2n−1)

2

which is equal to r2n if the sign is + and it is equal to s2

n if the sign is −.Hence xn = pn/qn is a rational number and both

x2n + 5 = (p2

n + 5q2n)/q2

n = (rn/qn)2 and x2n − 5 = (p2

n − 5q2n)/q2

n = (sn/qn)2

are squares of rational numbers. Since the sequence pn is increasing, in orderto prove that the rational numbers xn are all different (and therefore infinite),it suffices to show that pn and qn are relatively prime. Note that pn is odd(because qn is even) and it is not a multiple of 5. Then

(pn, qn) = (pn, 2pn−1qn−1rn−1sn−1) = 1

if and only if(pn, pn−1) = (p4

n−1 + 25q2n−1, pn−1) = (q2

n−1, pn−1) = 1(pn, qn−1) = (p4

n−1 + 25q2n−1, qn−1) = (p4

n−1, qn−1) = 1(pn, r

2n−1s

2n−1) = (p4

n−1 + 25q4n−1, p

4n−1 − 25q4

n−1) = (p4n−1, q

4n−1) = 1

.

which are all true by the inductive hypothesis.

Page 118: Eserciziario Di Analisi Matematica 2

College Mathematics Journal, Vol.34, January 2004

769. Proposed by Wenchang Chu and Leontina Veliana Di Claudio, Univer-sita di Lecce, Lecce, Italy.

For a natural number n, show that

Ωn :=n∏

k=0

(2k)!(2k)!

k!(n + k)!= 2n.

Solution proposed by G.R.A.20 Problems Group, Roma, Italy.

First Proof: by induction on n. Ω0 = 0!/0! = 20 and if Ωn−1 = 2n−1 then

Ωn = 2n−1 · Ωn

Ωn−1

= 2n−1 · (2n)!(2n)!

n!(2n)!·

n−1∏k=0

((2k)!(2k)!

k!(n + k)!· k!(n − 1 + k)!

(2k)!(2k)!

)

= 2n · (2n − 1)!

(n − 1)!·

n−1∏k=0

1

(n + k)= 2n · (2n − 1)!

(n − 1)! ·∏n−1

k=0(n + k)= 2n

Second Proof: note that

Pn :=n∏

k=0

(2k)!(2k)! =n∏

k=1

(2k) ·

(n∏

k=1

(2k − 1)! ·n∏

k=0

(2k)!

)= 2n · n! ·

2n∏k=0

k!

and

Qn :=n∏

k=0

k!(n + k)! =n∏

k=0

k! ·n∏

k=0

(n + k)! =n∏

k=0

k! ·2n∏

k=n

k! = n! ·2n∏

k=0

k!

therefore Ωn = Pn/Qn = 2n.

Page 119: Eserciziario Di Analisi Matematica 2
Page 120: Eserciziario Di Analisi Matematica 2
Page 121: Eserciziario Di Analisi Matematica 2

Mathematics Magazine, Vol.76, December 2003

1684. Proposed by Ethan S. Brown, Massachusetts Istitute of Technology,Cambridge, MA., and Christopher J. Hillar, University of California, Berke-ley, CA.

Let S be the set of all words of length n in two letters, say a and b. Definean equivalence relation on S as follows: given a word w, the reverse of w, thecomplement of w (that is , change all as to bs and all bs to as) and the reverse ofthe complement are all equivalent to w. Find the number of equivalence classesof S that do not contain any palindromes.

Solution proposed by G.R.A.20 Problems Group, Roma, Italy.

Let G be the group transformations on the set of all words S: the identity i,the reverse r, the complement c and the reverse of the complement rc. Notethat r r = c c = i and c r = r c. Now [w] = w, r(w), c(w), r c(w) isthe equivalence class represented by w and its cardinality |[w]| can be equalto 2 or 4 because c(w) 6= w for all w ∈ S.If w ∈ P = w ∈ S : r(w) = w, the set of palindromes, then

[w] = w = r(w), c(w) = r c(w) ⊂ P and |[w]| = 2.

If w ∈ A = w ∈ S : r c(w) = w, the set of anti-palindromes, then

[w] = w = r c(w), r(w) = c(w) ⊂ A and |[w]| = 2.

Hence if w ∈ S \ (A ∪ P ) then [w] ⊂ S \ (A ∪ P ) and |[w]| = 4.The sets P , A and S\(A∪P ) give a partition of S therefore the total numberof equivalence classes N of S that do not contain any palindromes is equalto the number of equivalence classes in A, that is |A|/2, plus the number ofequivalence classes in S \ (A ∪ P ), that is (|S| − |A| − |P |)/4.Since |S| = 2n,

|P | =

2n/2 if n is even,2(n+1)/2 if n is odd

and |A| =

2n/2 if n is even,0 if n is odd

,

then we have that

N = |A|/2 + (|S| − |A| − |P |)/4 =

2n−2 if n is even,(2n − 2(n+1)/2)/4 if n is odd

.

Page 122: Eserciziario Di Analisi Matematica 2

Mathematics Magazine, Vol.76, December 2003

1682. Proposed by Paul Bracken, University of Texas, Edinburg, TX.

Let n be a positive integer. Prove that

n∑j=1

1(nj

) =n + 1

2n·

n−1∑j=0

2j

j + 1

Solution proposed by G.R.A.20 Problems Group, Roma, Italy.

We will prove the identity by induction on n. If n = 1 then

1∑j=1

1(1j

) = 1 =1 + 1

0∑j=0

2j

j + 1.

Now take n ≥ 2 and assume that the identity holds for smaller n.Note that for j = 1, . . . , n− 1

1(n

j+1

) +1(nj

) =

(nj

)+

(n

j+1

)(n

j+1

)·(

nj

) =

(n+1j+1

)(n

j+1

)·(

nj

) =

n+1j+1

·(

nj

)n

j+1·(

n−1j

)·(

nj

) =n + 1

n· 1(

n−1j

) .

Then summing over j and using the induction hypothesis we get

n−1∑j=1

1(n

j+1

) +n−1∑j=1

1(nj

) =n + 1

n−1∑j=1

1(n−1

j

) =n + 1

n· n

2n−1·

n−2∑j=0

2j

j + 1

that is

n∑j=2

1(nj

) +n−1∑j=1

1(nj

) = 2n∑

j=1

1(nj

) − 1

n− 1 =

n + 1

2n−1·

n−2∑j=0

2j

j + 1

and finally

n∑j=1

1(nj

) =n + 1

2n·

n−2∑j=0

2j

j + 1+

n + 1

2 · n=

n + 1

2n·

n−1∑j=0

2j

j + 1.

Page 123: Eserciziario Di Analisi Matematica 2

Mathematics Magazine, Vol. 76, December 2003

1681. Proposed by Mihai Manea, Princeton University, NJ.

Let p be a prime number. Prove that the polynomial

xp−1 + 2xp−2 + 3xp−3 + · · ·+ (p− 1)x + p

is irreducible in Z[x].

Solution proposed by G.R.A.20 Problems Group, Roma, Italy.

We will use the following variation of Eisenstein’s criterion: let f(x) = xn +an−1x

n−1 · · · + a1x + a0 ∈ Z[x] with n ≥ 2 and assume that f(x) has nointeger roots. If there exists a prime number p such that p divides all ai fori = 0, . . . , n− 2 and p2 does not divide a0, then f(x) is irreducible in Z[x].Proof: assume that f(x) is the product of two polynomials in Z[x] say bsx

s +· · · + b1x + b0 and cn−sx

n−s + · · · + c1x + c0 where 2 ≤ s ≤ n − 2 becausef(x) has no integer roots. By hypothesis p divides a0 = b0c0 but not bothof b0 and c0, so we can suppose that p | b0 and p - c0. Since f is monic thenbs = cn−s = 1 and therefore not all the bj are divisible by p . Let k be thesmallest index such that p - bk. Note that 1 ≤ k ≤ n − 2 and therefore pdivides ak = bkc0 + bk−1c1 + · · ·+ b0ck and every term on the right side exceptbkc0 which yields a contradiction. In our case

f(x) = xp−1 + 2xp−2 + 3xp−3 + · · ·+ (p− 1)x + p

=

p∑k=1

xk − 1

x− 1=

∑pk=0 xk − 1− p

x− 1=

xp+1 − 1− (p + 1)(x− 1)

(x− 1)2.

Letting x = y + 1 we obtain a new polynomial

f(y) = f(y + 1) =

∑p+1k=0

(p+1k

)yk − (p + 1)y − 1

y2

= yp−1 + (p + 1)yp−2 +

(p + 1

p− 1

)yp−3 + · · ·+

(p + 1

3

)y +

(p + 1

2

).

Now f(y) satisfies the criterion’s hypothesis just for the prime p and thereforef(x) = f(x− 1) is irreducible in Z[x].

Page 124: Eserciziario Di Analisi Matematica 2

Mathematics Magazine, Vol.76, October 2003

1679. Proposed by Jose Luis Dıaz-Barreo and Juan Jose Egozcue, UnivesitatPolitecnica de Catalunya, Barcelona, Spain.

Let fk denote the kth Fibonacci number, that is, f0 = 0, f1 = 1, and fk+2 =fk+1 + fk, k ≥ 0. Prove that for any positive integer n,

n∑k=1

(n

k

)log(f f2n

k ) ≤ (2n − 1)n∑

k=1

(n

k

)log(f fk

k ).

Solution proposed by G.R.A.20 Problems Group, Roma, Italy.

The inequality can be written in this way

1

2n − 1

n∑k=1

(n

k

)log(fk) ≤

1

f2n

n∑k=1

(n

k

)fk log(fk).

We will use the following two identities that can be easily proved by induction

n∑k=1

(n

k

)= 2n − 1 and

n∑k=1

(n

k

)fk = f2n.

Since the function log(x) is concave then

1

2n − 1

n∑k=1

(n

k

)log(fk) ≤ log

(1

2n − 1

n∑k=1

(n

k

)fk

)= log

(f2n

2n − 1

).

On the other hand, the function − log(x) is convex and therefore

1

f2n

n∑k=1

(n

k

)fk log(fk) =

1

f2n

n∑k=1

(n

k

)fk

(− log

(1

fk

))

≥ − log

(1

f2n

n∑k=1

(n

k

))= log

(f2n

2n − 1

).

Hence

1

2n − 1

n∑k=1

(n

k

)log(fk) ≤ log

(f2n

2n − 1

)≤ 1

f2n

n∑k=1

(n

k

)fk log(fk).

Page 125: Eserciziario Di Analisi Matematica 2

Mathematics Magazine, Vol.76, June 2003

1674. Proposed by H. A. Shah Ali, Teheran, Iran.

Given that 0 ≤ x1 ≤ x2 ≤ · · · ≤ xn, and xn+1 = x1, prove thatn∑

k=1

xk − xk+1

1 + xkxk+1

≥ 0.

Solution proposed by G.R.A.20 Problems Group, Roma, Italy.

The inequality can be written in this way

n−1∑k=1

xk+1 − xk

1 + xk+1xk

≤ xn − x1

1 + xnx1

.

Since tan(α− β) =tan α− tan β

1 + tan α tan β, then

n−1∑k=1

tan(arctan xk+1 − arctan xk) ≤ tan(arctan xn − arctan x1),

and letting βk = arctan xk+1 − arctan xk ≥ 0 the inequality becomes

n−1∑k=1

tan βk ≤ tan

(n−1∑k=1

βk

)with β1 + · · ·+ βn−1 < π

2. Now we prove it by induction on n ≥ 2.

If n = 2 then, since 0 ≤ β1 < π2− β2 ≤ π

2, we have that

0 ≤ tan β1 tan β2 < tan(π

2− β2) tan β2 = 1

and therefore

tan(β1 + β2) =tan β1 + tan β2

1− tan β1 tan β2

≥ tan β1 + tan β2.

Finally, for n > 2

n−1∑k=1

tan βk =n−2∑k=1

tan βk + tan βn−1 ≤ tan

(n−2∑k=1

βk

)+ tan βn−1

≤ tan

(n−2∑k=1

βk + βn−1

)≤ tan

(n−1∑k=1

βk

).

Page 126: Eserciziario Di Analisi Matematica 2

Mathematics Magazine, Vol.76, February 2003

1662. Proposed by Erwin Just (Emeritus) and Norman Schaumberger (Emer-itus), Bronx Community College, Bronx, NY.

Let xk, 1 ≤ k ≤ n, be positive real numbers with∑n

k=1 x2k−1k ≤ n.

Prove that∑n

k=1(2k − 1)xk ≤ n2.

Solution proposed by G.R.A.20 Problems Group, Roma, Italy.

The function f(x) = x2k−1 is convex for k ≥ 1 and x > 0 and therefore itsgraphic stays above the line y = (2k− 1)(x− 1) + 1 which is tangent to f atthe point (1, 1):

(2k − 1)(x− 1) + 1 ≤ x2k−1 ∀x > 0.

Now letting x = xk and summing for k = 1, . . . , n, we obtain

n∑k=1

[(2k − 1)(xk − 1) + 1] ≤n∑

k=1

x2k−1k

that isn∑

k=1

(2k − 1)xk ≤n∑

k=1

x2k−1k +

n∑k=1

(2k − 1)−n∑

k=1

1.

Since

n =n∑

k=1

1 and n2 =n∑

k=1

(2k − 1)

then the inequality becomes

n∑k=1

(2k − 1)xk ≤n∑

k=1

x2k−1k + n2 − n.

By hypothesis∑n

k=1 x2k−1k ≤ n and therefore

n∑k=1

(2k − 1)xk ≤n∑

k=1

x2k−1k + n2 − n ≤ n2.

Page 127: Eserciziario Di Analisi Matematica 2

College Mathematics Journal, Vol.34, November 2003

761. Proposed by Charles L. Cooper and Michael Scott McClendon, Uni-versity of Central Oklahoma, Edmond, OK.

Let C(a, b) denote the circle centered at point a that passes through point b,and let D(a, b) denote the closed disk with boundary C(a, b). Suppose p, q, andw are three distinct points and that x is a point on the line segment connectingp and q.

(a) Must it be the case that C(x, w) is contained in D(p, w) ∪D(q, w)?(b) Must it be the case that D(x, w) is contained in D(p, w) ∪D(q, w)?

Solution proposed by G.R.A.20 Problems Group, Roma, Italy.

We will show that the following inclusion holds

D(x, w) ⊂ D(p, w) ∪D(q, w),

therefore the answer is always yes.Let’s consider our problem in the complex plane C. We can assume withoutlack of generality that p, q and x are on the real axis with p ≤ x ≤ q. Nowit suffices to take z ∈ D(x, w) and prove that z ∈ D(p, w) ∪D(q, w).The point z belongs to D(x, w) if and only if |z − x|2 ≤ |w − x|2:

|z|2 − 2Re(x z) + |x|2 ≤ |w|2 − 2Re(x w) + |x|2

that is|z|2 ≤ |w|2 + 2x Re(z − w).

We distinguish two cases. If Re(z − w) < 0 then, since x ≥ p,

|z|2 ≤ |w|2 + 2x Re(z − w) ≤ |w|2 + 2p Re(z − w)

and this means that z ∈ D(p, w).On the other hand, if Re(z − w) ≥ 0 then, since x ≤ q ,

|z|2 ≤ |w|2 + 2x Re(z − w) ≤ |w|2 + 2q Re(z − w)

that is z ∈ D(q, w).

Page 128: Eserciziario Di Analisi Matematica 2

College Mathematics Journal, Vol.34, September 2003

757. Proposed by W. V. Grounds, SUNY New Paltz, New Paltz, NY.

For x and y integers, let,

f(x, y) = 4x4 + y4 and g(x, y) = 4x4 − y4

Find all primes assumed by either f(x, y) or g(x, y).

Solution proposed by G.R.A.20 Problems Group, Roma, Italy.

The polynomial f(x, y) can be factored over Z in this way:

f(x, y) = 4x4 + y4 = (2x2 + 2xy + y2)(2x2 − 2xy + y2)

If f(x, y) is a prime number then the smaller factor 2x2−2xy+y2 is necessarlyequal to 1

2x2 − 2xy + y2 = x2 + (x− y)2 = 1

and, since x and y are positive integers, this equation is solved only for x = 1and x − y = 0, that is x = 1 and y = 1. Therefore the polynomial f(x, y)can assume only one prime number: f(1, 1) = 5.The polynomial g(x, y) can be factored over Z in this way:

g(x, y) = 4x4 − y4 = (2x2 + y2)(2x2 − y2)

If g(x, y) is a prime number then the smaller factor 2x2 − y2 is necessarlyequal to 1. Hence y2 = 2x2 − 1 and

g(x, y) = (2x2 + y2)(2x2 − y2) = 4x2 − 1 = (2x + 1)(2x− 1).

Again, assuming that g(x, y) is prime, we have that the smaller factor 2x−1is equal to 1, that is x = 1 and y = 1. Therefore the polynomial g(x, y) canassume only one prime number: g(1, 1) = 3.

Page 129: Eserciziario Di Analisi Matematica 2

College Mathematics Journal, Vol.34, September 2003

756. Proposed by Michel Bataille, Rouen, France

Prove that6∑

k=1

sin(

kπ7

)sin(

(3k+7)π21

) = 0.

Solution proposed by G.R.A.20 Problems Group, Roma, Italy.

The sum can be written in this way

6∑k=1

sin(kπ7

)

sin(kπ7

+ π3)

=6∑

k=1

sin(kπ7

)12sin(kπ

7) +

√3

2cos(kπ

7)

= 26∑

k=1

tan(kπ7

)

tan(kπ7

) +√

3.

Since tan(π − x) = − tan(x), then tan(

(7−k)π7

)= − tan(kπ

7) and

23∑

k=1

(tan(kπ

7)

tan(kπ7

) +√

3+

tan(kπ7

)

tan(kπ7

)−√

3

)= 4

3∑k=1

tan2(kπ7

)

tan2(kπ7

)− 3

Letting tk = tan2(kπ7

), the sum becomes

43∑

k=1

tktk − 3

= 4 · 9(t1 + t2 + t3)− 6(t1t2 + t1t3 + t2t3) + 3(t1t2t3)

(t1 − 3)(t2 − 3)(t3 − 3).

By De Moivre’s formula, it is easy to show that

tan(7θ) = tan θ · tan6 θ − 21 tan4 θ + 35 tan2 θ − 7

7 tan6 θ − 35 tan4 θ + 21 tan2 θ − 1.

Since tan(7θ) = 0 when θ = kπ7

for k = 1 . . . 6, then t1, t2 and t3 are the rootsof the polynomial

x3 − 21x2 + 35x− 7.

Therefore

t1 + t2 + t3 = 21 , t1t2 + t1t3 + t2t3 = 35 , t1t2t3 = 7

and6∑

k=1

sin(

kπ7

)sin(

(3k+7)π21

) = 4 · 9(21)− 6(35) + 3(7)

(t1 − 3)(t2 − 3)(t3 − 3)= 0.

Page 130: Eserciziario Di Analisi Matematica 2

Junior problems

J163. Let a, b, c be nonzero real numbers such that ab+ bc+ ca ≥ 0. Prove that

ab

a2 + b2+

bc

b2 + c2+

ca

c2 + a2≥ −1

2.

Proposed by Titu Andreescu, University of Texas at Dallas, USA

Solution by Ercole Suppa, Teramo, ItalyWe have∑

cyc

ab

a2 + b2=∑cyc

(ab

a2 + b2+

1

2

)− 3

2=∑cyc

(a+ b)2

2 (a2 + b2)− 3

2

≥∑cyc

(a+ b)2

2 (a2 + b2 + c2)− 3

2=

2(a2 + b2 + c2

)+ 2(ab+ bc+ ca)

2 (a2 + b2 + c2)− 3

2

= 1 +ab+ bc+ ca

a2 + b2 + c2− 3

2=ab+ bc+ ca

a2 + b2 + c2− 1

2≥ −1

2

where in the last step we have used the fact that ab+ bc+ ca ≥ 0.

Also solved by Arkady Alt, San Jose, California, USA; Daniel Lasaosa, Universidad Publicade Navarra, Spain; Perfetti Paolo, Dipartimento di Matematica, Universita degli studi di TorVergata Roma, Italy; Prithwijit De, HBCSE, India; Andrea Ligori, Universita di Roma “TorVergata”, Italy; Piriyathumwong P., Bangkok, Thailand.

Mathematical Reflections 4 (2010) 1

Page 131: Eserciziario Di Analisi Matematica 2

J164. If x and y are positive real numbers such that(x+√x2 + 1

)(y +

√y2 + 1

)= 2011, find the

minimum possible value of x+ y.

Proposed by Neculai Stanciu, “George Emil Palade”, Buzau, Romania

First solution by Michel Bataille, France The required minimum value is 2010√2011

.

Write x = sinh(a) and y = sinh(b) where a = ln(x+√x2 + 1) > 0 and b = ln(y+

√y2 + 1) > 0.

From the hypothesis, we have a+ b = ln(2011) and using a known formula,

x+ y = sinh(a) + sinh(b) = 2 sinh

(a+ b

2

)cosh

(a− b

2

)≥ 2 sinh

(a+ b

2

)= 2 sinh(ln(

√2011)

where the inequality follows from cosh(t) ≥ 1 for all t and sinh(u) > 0 for u > 0.Since 2 sinh(ln(

√2011) =

√2011− 1√

2011= 2010√

2011, we obtain

x+ y ≥ 2010√2011

.

Clearly equality holds when a = b (since cosh(0) = 1), that is, when x = y. The result follows.

Second solution by the authors

Let z = x +√x2 + 1. We have z > 0 and (1) x = z2−1

2z . From hypothesis y +√y2 + 1 = 2011

z ,

we get (2) y = 20112−z22·2011·z . From (1) and (2),

x+ y =z2 − 1

2z+

20112 − z2

2 · 2011 · z=

2010

2 · 2011

(z +

2011

z

)≥ 2010

2011

√z · 2011

z.

The equality occurs for z 2011z or equivalently z2 = 2011. Then from (1) and

(2)

we obtain

x = y =2010

2√

2011=

1005√2011

.

So min(x+ y) = 2010√2011

.

Also solved by Arkady Alt, San Jose, California, USA; Francisco Javier Garcia Capitan, Spain;Ercole Suppa, Teramo, Italy; Daniel Lasaosa, Universidad Publica de Navarra, Spain; PerfettiPaolo, Dipartimento di Matematica, Universita degli studi di Tor Vergata Roma, Italy.

Mathematical Reflections 4 (2010) 2

Page 132: Eserciziario Di Analisi Matematica 2

J165. Find all triples (x, y, z) of integers satisfying the system of equations(x2 + 1

)(y2 + 1

)+ z2

10 = 2010

(x+ y)(xy − 1) + 14z = 1985.

Proposed by Titu Andreescu, University of Texas at Dallas, USA

Solution by Arkady Alt, San Jose, California, USA

Note that z = 10k for some integer k becausez2

10= 2010−

(x2 + 1

) (y2 + 1

)is an integer. Let

p = x+ y and q = xy − 1. Then(x2 + 1

) (y2 + 1

)= x2y2 + x2 + y2 + 1 = (xy − 1)2 + (x+ y)2 = p2 + q2

and the system becomesp2 + q2 + 10k2 = 2010pq + 140k = 1985

⇐⇒p2 + q2 = 2010− 10k2

pq = 1985− 140k(1)

Since (p− q)2 = 2010− 10k2 − 2 (1985− 140k) = −10 (k − 14)2 then only k = 14 can provide

solvability to (1). And for k = 14, (1) becomes

p2 + q2 = 50pq = 25

⇐⇒ p = q = 5.

Hence,

x+ y = 5xy = 4

⇐⇒x = 4y = 1

or

x = 1y = 4

and triples (5, 1, 140) , (1, 5, 140) are all

integer solutions of the original system in integers.

Also solved by Daniel Lasaosa, Universidad Publica de Navarra, Spain; Piriyathumwong P.,Bangkok, Thailand.

Mathematical Reflections 4 (2010) 3

Page 133: Eserciziario Di Analisi Matematica 2

J166. Let P be a point inside triangle ABC and let da, db, dc be the distances from point P to thesides of the triangle. Prove that

K

dadbdc≥ s

Rr

where K is the area of the pedal triangle of P and s,R, r are the semiperimeter, circumradius,and inradius of triangle ABC.

Proposed by Andrei Razvan Baleanu, “George Cosbuc”, Motru, Romania

Remark: The problem contains a typ and the inequality that needs to be proven is

K

dadbdc≥ s

2Rr.

Many readers have solved the correct inequality.

First solution by Daniel Lasaosa, Universidad Publica de Navarra, Spain The proposed in-equality is not true, since if P = I is the incenter of equilateral triangle ABC, then K = rs

4 isone quarter the area of ABC, while da = db = dc = r, and the proposed inequality would beequivalent to R ≥ 4r = 2R, absurd. We show that the correct inequality that always holds is

2K

dadbdc=

s

Rr.

Now, denoting by PA, PB, PC the respective projections of P on sides BC,CA,AB, we have∠CPAP = ∠CPBP = 90, or ∠PAPPB = 180 − C, and the area of PPAPB is

dadb sinC

2=dadbdc

4R

c

dc.

Adding the analogous expressions for the areas of PPBPC and PPCPA, we find

K =dadbdc

4R

(a

da+

b

db+

c

dc

),

or the proposed inequality is equivalent to

a

da+

b

db+

c

dc≥ 2s

r.

Now, 1x is convex for positive x because d2

dx2

(1x

)= 2

x3 > 0, or by Jensen’s inequality,

a

da+

b

db+

c

dc≥ (a+ b+ c)

a+ b+ c

ada + bdb + cdc=

4s2

S=

2s

r,

where 2S = ada + bdb + cdc = 2rs is twice the area of ABC because ada is twice the area ofBPC, and similarly for its cyclic permutations, and equality is reached iff da = db = dc, ie iff

P is the incenter of ABC, in which case we easily find K = r2(sinA+sinB+sinC)2 = r2s

2R .

Mathematical Reflections 4 (2010) 4

Page 134: Eserciziario Di Analisi Matematica 2

Second solution by G.R.A.20 Math Problems Group, Roma, Italy Since

4KR = cdadb + adbdc + bdcda and 2sr = ada + bdb + cdc,

it follows that the inequality becomes(a

da+

b

db+

c

dc

)· (ada + bdb + cdc) ≥ (a+ b+ c)2

which holds by Cauchy-Schwarz.

Also solved by Arkady Alt, San Jose, California, USA; Michel Bataille, France; Ercole Suppa,Teramo, Italy.

Mathematical Reflections 4 (2010) 5

Page 135: Eserciziario Di Analisi Matematica 2

J167. Let a, b, c be real numbers greater than 1 such that

b+ c

a2 − 1+

c+ a

b2 − 1+

a+ b

c2 − 1≥ 1.

Prove that (bc+ 1

a2 − 1

)2

+

(ca+ 1

b2 − 1

)2

+

(ab+ 1

c2 − 1

)2

≥ 10

3.

Proposed by Titu Andreescu, University of Texas at Dallas, USA

First solution by Prithwijit De, HBCSE, India

Observe that (bc+ 1

a2 − 1

)2

−(b+ c

a2 − 1

)2

=(b2 − 1)(c2 − 1)

(a2 − 1)2;(

ca+ 1

b2 − 1

)2

−(c+ a

b2 − 1

)2

=(c2 − 1)(a2 − 1)

(b2 − 1)2;(

ab+ 1

c2 − 1

)2

−(a+ b

c2 − 1

)2

=(a2 − 1)(b2 − 1)

(c2 − 1)2.

Therefore∑(

bc+ 1

a2 − 1

)2

=∑(

b+ c

a2 − 1

)2

+∑ (b2 − 1)(c2 − 1)

(a2 − 1)2.. . . (1)

Now observe that∑(

b+ c

a2 − 1

)2

(∑ b+ c

a2 − 1

)2

3≥ 1

3. . . (2)

and by A.M-G.M inequality we get

∑ (b2 − 1)(c2 − 1)

(a2 − 1)2≥ 3 3

√(a2 − 1)2(b2 − 1)2(c2 − 1)2

(a2 − 1)2(b2 − 1)2(c2 − 1)2= 3. . . . (3)

By virtue of (1),(2) and (3) we obtain∑(

bc+ 1

a2 − 1

)2

≥ 3 +1

3=

10

3.

Second solution by Perfetti Paolo, Dipartimento di Matematica, Universita degli studi di TorVergata Roma, Italy

We observe that bc + 1 = b + c + (b − 1)(c − 1) and then (bc + 1)2 = (b + c)2 + 2(b + c)(b −1)(c− 1) + (b− 1)2(c− 1)2. By power–means–inequality and the constraint on a, b, c we have

∑cyc

(b+ c)2

(a2 − 1)2≥ 1

3

(∑cyc

b+ c

a2 − 1

)2

≥ 1

3

thus the inequality becomes∑cyc

(2

(b+ c)(b− 1)(c− 1)

(a2 − 1)2+

(b− 1)2(c− 1)2

(a2 − 1)2

)≥ 3

Mathematical Reflections 4 (2010) 6

Page 136: Eserciziario Di Analisi Matematica 2

or ∑cyc

(b− 1)(c− 1)(bc+ b+ c+ 1)

(a2 − 1)2≥ 3

Since the inequality in the statement is symmetric, we can set a ≥ b ≥ c. Then we observe that((b− 1)(c− 1)

(a− 1)2,(c− 1)(a− 1)

(b− 1)2,(a− 1)(b− 1)

(c− 1)2

)and (

bc+ b+ c+ 1

(a+ 1)2,ca+ c+ a+ 1

(b+ 1)2,ab+ a+ b+ 1

(c+ 1)2

)are equally sorted. This allows us to employ Chebyshev–inequality∑

cyc

(b− 1)(c− 1)(bc+ b+ c+ 1)

(a2 − 1)2≥ 1

3

∑cyc

(b− 1)(c− 1)

(a− 1)2·∑cyc

bc+ b+ c+ 1

(a+ 1)2

Moreover by AGM we have ∑cyc

(b− 1)(c− 1)

(a− 1)2≥ 3

and ∑cyc

bc+ b+ c+ 1

(a+ 1)2=∑cyc

(b+ 1)(c+ 1)

(a+ 1)2≥ 3

Bringing together the last three inequalities we obtain∑cyc

(b− 1)(c− 1)(bc+ b+ c+ 1)

(a2 − 1)2≥ 1

33 · 3 = 3

and we are done.

Also solved by Daniel Lasaosa, Universidad Publica de Navarra, Spain; Prithwijit De, HBCSE,India.

Mathematical Reflections 4 (2010) 7

Page 137: Eserciziario Di Analisi Matematica 2

J168. Let n be a positive integer. Find the least positive integer a such that the systemx1 + x2 + · · ·+ xn = a

x21 + x22 + · · ·+ x2n = a

has no integer solutions.

Proposed by Dorin Andrica, “Babes-Bolyai University”, Cluj-Napoca, Romania

Solution by Lorenzo Pascali, Universita di Roma La Sapienza, Italy

First, we notice that if xi 6= 0, 1 for an integer component xi then x2i > xi and we have acontradiction

a = x21 + x22 + · · ·+ x2n > x1 + x2 + · · ·+ xn = a.

Hence any component xi is 0 or 1 and the system has integer solutions for a = 1, . . . , n: takex1 = · · · = xa = 1 and xa+1 = · · · = xn = 0. Therefore the least positive integer a such thatthe system has no integer solutions is n+ 1:

x1 + x2 + · · ·+ xn ≤ x21 + x22 + · · ·+ x2n ≤ n < a = n+ 1.

Also solved by Arkady Alt, San Jose, California, USA; Daniel Lasaosa, Universidad Publica deNavarra, Spain; Ercole Suppa, Teramo, Italy.

Mathematical Reflections 4 (2010) 8

Page 138: Eserciziario Di Analisi Matematica 2

Senior problems

S163. (a) Prove that for each positive integer n there is a unique positive integer an such that

(1 +√

5)n =√an +

√an + 4n.

(b) When n is even, prove that an is divisible by 5 · 4n−1 and find the quotient.

Proposed by Dorin Andrica, “Babes-Bolyai University”, Cluj-Napoca, Romania

First solution by G. C. Greubel, Newport News, VA

Let 2α = 1 +√

5. With this we have

2nαn =√an +

√an + 4n. (1)

Squaring both sides leads to

4n(α2n − 1) = 2an + 2√an(an + 4n). (2)

Subtracting 2an from both sides and squaring the resulting value leads to

[4n(α2n − 1)− 2an]2 = 4an(an + 4n). (3)

This is reduced to

an = 4n−1(α2n − 1

αn

)2

= 4n−1 (αn − (−1)nβn)2

= 4n−1(α2n + β2n − 2

)= 4n−1(L2n − 2) (4)

where Lm is the mth Lucas number. Hence it has been shown that an is a positive integer andis given by

an = 4n−1(L2n − 2).

B) If n is an even value, say n = 2m, then

a2m = 42m−1(L4m − 2)

= 42m−1 · 5F 22n

= 5 · 4m−1 · (2mF2m)2 . (5)

From this relation it is shown that a2m is divisible by 5 · 4m−1 and has the quotient value(2mF2m)2.

Mathematical Reflections 4 (2010) 9

Page 139: Eserciziario Di Analisi Matematica 2

Second solution by the authors

(a) Let (1 +√

5)n = xn + yn√

5, where xn, yn are positive integers, n = 1, 2, . . . Then

(1−√

5)n = xn − yn√

5, n = 1, 2, . . . ,

hencex2n − 5y2n = (−4)n, n = 1, 2, . . . (1)

If n is even, consider an = x2n − 4n and we have

√an +

√an + 4n =

√x2n − 4n +

√x2n =

√5y2n +

√x2n

= yn√

5 + xn = (1 +√

5)n.

If n is odd, consider an = 5y2n − 4n and we have

√an +

√an + 4n =

√5y2n − 4n +

√5y2n =

√x2n +

√5y2n

= xn + yn√

5 = (1 +√

5)n.

(b) If n is even, then we have an = x2n − 4n = 5y2n, where

yn =1

2√

5[(1 +

√5)n − (1−

√5)n]

=2n

2√

5

[(1 +√

5

2

)n

(1−√

5

2

)n]= 2n−1Fn,

where Fn is the nth Fibonacci number. In this case we get an = 5 · 4n−1F 2n , hence 5 · 4n−1|an

and the quotient is F 2n .

Also solved by Arkady Alt, San Jose, California, USA; Daniel Lasaosa, Universidad Publicade Navarra, Spain.

Mathematical Reflections 4 (2010) 10

Page 140: Eserciziario Di Analisi Matematica 2

S164. Let ABCD be a cyclic quadrilateral whose diagonals are perpendicular to each other. Fora point P on its circumscribed circle denote by `P the line tangent to the circle at P. LetU = `A ∩ `B, V = `B ∩ `C ,W = `C ∩ `D,K = `D ∩ `A. Prove that UVWK is a cyclicquadrilateral.

Proposed by Ivan Borsenco, Massachusetts Institute of Technology, USA

Solution by Michel Bataille, France

Let U1, V1,W1,K1 be the midpoints of AB,BC,CD,DA, respectively. The Varignon paral-lelogram U1V1W1K1 of the quadrilateral ABCD is a rectangle (because AC ⊥ BD), henceU1, V1,W1,K1 lie on a circle γ centered at the centre of the rectangle. Note that O,U1, U arecollinear (on the perpendicular bisector of AB) and that AB is the polar of U with respectto Γ. Similar results hold for V1,W1,K1 and it follows that the inverses of U1, V1,W1,K1 inthe circle Γ are U, V,W,K, respectively, so that U, V,W,K all lie on the inverse of the circleγ. Since U, V,W,K clearly cannot be collinear, the inverse of γ is a circle and so UVWK is acyclic quadrilateral.

A

B

C

D

U1

V1 W1

K1

O

U

V

W

K

Also solved by Ercole Suppa, Teramo, Italy; Daniel Lasaosa, Universidad Publica de Navarra,Spain; Prithwijit De, HBCSE, India.

Mathematical Reflections 4 (2010) 11

Page 141: Eserciziario Di Analisi Matematica 2

S165. Let I be the incenter of triangle ABC. Prove that

AI ·BI · CI ≥ 8r3,

where r is the inradius of triangle ABC.

Proposed by Dorin Andrica, “Babes-Bolyai University”, Cluj-Napoca, Romania

Solution by Piriyathumwong P.,Bangkok, ThailandSince

AI =r

sin A2

, BI =r

sin B2

, CI =r

sin C2

,

the inequality above is equivalent to sin A2 sin B

2 sin C2 ≤

18 ,which is immediately true because

of the two well-known facts below:

r

R= 4 sin

A

2sin

B

2sin

C

2, R ≥ 2r

,where R is the circumradius of triangle ABC.

Also solved by Arkady Alt, San Jose, California, USA; Michel Bataille, France; Scott H. Brown,Auburn University Montgomery, USA; Ercole Suppa, Teramo, Italy; Daniel Lasaosa, Univer-sidad Publica de Navarra, Spain; G.R.A.20 Math Problems Group, Roma, Italy.

Mathematical Reflections 4 (2010) 12

Page 142: Eserciziario Di Analisi Matematica 2

S166. If a1, a2, . . . , ak ∈ (0, 1), and k, n are integers such that k > n ≥ 1, prove that the followinginequality holds

mina1(1− a2)n, a2(1− a3)n, . . . , ak(1− a1)n ≤nn

(n+ 1)n+1.

Proposed by Marin Bancos, North University of Baia Mare, Romania

First solution by Arkady Alt, San Jose, California, USA

Let M = min a1 (1− a2)n , a2 (1− a3)n , . . . , ak (1− a1)n and for any function f (x, y) let

k∑cyc

f (a1, a2) = f (a1, a2) + f (a2, a3) + · · ·+ f (ak, a1) .

Since for any x, y ∈ (0, 1) by the AM–GM inequality

n+1

√nx (1− y)n ≤ nx+ n− ny

n+ 1=n (x− y) + n

n+ 1.

Then

n+1√M = min

n+1

√a1 (1− a2)n, n+1

√a2 (1− a3)n, . . . , n+1

√ak (1− a1)n

≤ 1

k

k∑cyc

n+1

√a1 (1− a2)n

=1

k n+1√n

k∑cyc

n+1

√na1 (1− a2)n

≤ 1

k (n+ 1) n+1√n

k∑cyc

(n (a1 − a2) + n)

=nk

k (n+ 1) n+1√n

=n

(n+ 1) n+1√n

=n+1√nn

(n+ 1).

Then M ≤ nn

(n+ 1)n+1 .

Second solution by the author

“Reductio ad absurdam”

Let’s suppose that the inequality doesn’t hold.

Therefore

a1(1− a2)n >nn

(n+ 1)n+1

Mathematical Reflections 4 (2010) 13

Page 143: Eserciziario Di Analisi Matematica 2

a2(1− a3)n >nn

(n+ 1)n+1

.....

ak(1− a1)n >nn

(n+ 1)n+1

Multiplying these relations up, we get

a1 · a2 · ... · ak · (1− a1)n · (1− a2)n · ... · (1− ak)n >

[nn

(n+ 1)n+1

]k(∗)

But, for a ∈ (0, 1), we have

a(1− a)n ≤ nn

(n+ 1)n+1

Let’s prove this inequality.

a·(1−a)n =1

n·n·a·(1−a)n

AM−GM≤ 1

na+n times

(1− a) + · · ·+ (1− a)

n+ 1

n+1

=1

n·(

n

n+ 1

)n+1

=nn

(n+ 1)n+1

The equality holds for: na = 1− a ⇔ a = 1n+1 ∈ (0, 1)

Using the proved inequality for a1, a2, . . . , ak, we get:

a1(1− a1)n ≤nn

(n+ 1)n+1

a2(1− a2)n ≤nn

(n+ 1)n+1

.....

ak(1− ak)n ≤ nn

(n+ 1)n+1

Multiplying these relations up, we get

a1 · a2 · · · · · ak · (1− a1)n · (1− a2)n · · · · · (1− ak)n ≤[

nn

(n+ 1)n+1

]kThis inequality contradicts (∗), which follows from the initial assumption. Therefore, thatassumption is false.

Also solved by Michel Bataille, France; Daniel Lasaosa, Universidad Publica de Navarra, Spain;Perfetti Paolo, Dipartimento di Matematica, Universita degli studi di Tor Vergata Roma, Italy.

Mathematical Reflections 4 (2010) 14

Page 144: Eserciziario Di Analisi Matematica 2

S167. Let Ia be the excenter corresnponding to the side BC of triangle ABC. Denote by A′, B′, C ′

the tangency points of the excircle of center Ia with the sides BC,CA,AB, respectively. Provethat the circumcircles of triangles AIaA

′, BIaB′, CIaC

′ have a common point, different fromIa, situated on the line GaIa, where Ga is the centroid of triangle A′B′C ′.

Proposed by Dorin Andrica, “Babes-Bolyai University”, Cluj-Napoca, Romania

First solution by Michel Bataille, France

A

BC

I1

A'

C'

B'G1

B1 C1

A1

For typographical reasons, Ia and Ga are denoted by I1 and G1 on the figure above.

Let γ be the excircle. Since IaA′ = IaC

′ an BA′ = BC ′, the line IaB is the perpendicularbisector of A′C ′ and intersects A′C ′ in its midpoint B1 . Since A′C ′ is the polar of B withrespect to γ, the inversion in the circle γ exchanges B1 and B. Since B′ is invariant underthis inversion, the circumcircle of ∆IaBB

′ inverts into the median B′B1 of triangle A′B′C ′.Similarly, the circumcircles of ∆IaAA

′,∆IaCC′ invert into the medians A′A1, C

′C1. As aresult, the three circumcircles all pass through Ia and through the inverse of Ga (because Ga

lies on the three medians A′A1, B′B1, C

′C1). The second result follows from the fact that theinverse of Ga is on the line through Ia and Ga.

Second solution by Daniel Lasaosa, Universidad Publica de Navarra, Spain Let D′ be themidpoint of B′C ′. Now AB′ ⊥ IaB

′, while AIa ⊥ B′C ′ where B′D′ = C ′D′ by symmetryaround the internal bisector of angle A. Thus, triangles AB′D′ and B′IaD

′ are similar, henceB′D′ · C ′D′ = B′D′2 = IaD

′ · AD′, and the power of D′ with respect to the circumcircles ofA′B′C ′ and AIaA

′ is the same, or D′ lies on the radical axis of both circles, which is medianA′D′.

Mathematical Reflections 4 (2010) 15

Page 145: Eserciziario Di Analisi Matematica 2

Let E′ be the midpoint of C ′A′. BA′ ⊥ IaA′, while BIa ⊥ A′C ′ where A′E′ = C ′E′ by

symmetry around the external bisector of angle B. Thus, triangles BE′A′ and A′E′Ia aresimilar, hence A′E′ · C ′E′ = A′E′2 = BE′ · IaE′, and median B′E′ is the radical axis of thecircumcircles of A′B′C ′ and BIaB

′. Similarly, median C ′F ′ (F ′ is the midpoint of A′B′) is theradical axis of the circumcircles of A′B′C ′ and CIaC

′.

Clearly, the point Ga where the medians A′D′, B′E′ and C ′F ′ meet, has the same powerwith respect to the four circumcircles; consider now the second point P where IaGa meets thecircumcircle of AIaA

′. Since IaGa is the radical axis of the circumcircles of AIaA′ and BIaB

because Ia, Ga have the same power with respect to both, then P also has the same power withrespect to both, but since it is on the circumcircle of AIaA

′, it is also on the circumcircle ofBIaB

′. Similarly, it is also on the circumcircle of CIAC′. The conclusion follows.

Mathematical Reflections 4 (2010) 16

Page 146: Eserciziario Di Analisi Matematica 2

S168. Let a0 ≥ 2 and an+1 = a2n − an + 1, n ≥ 0. Prove that

loga0(an − 1) loga1(an − 1) · · · logan−1(an − 1) ≥ nn,

for all n ≥ 1.

Proposed by Titu Andreescu, University of Texas at Dallas, USA

Solution by Perfetti Paolo, Dipartimento di Matematica, Universita degli studi di Tor VergataRoma, Italy

Proof Induction

loga0(an+1 − 1) loga1(an+1 − 1) · · · logan−1(an+1 − 1) logan(an+1 − 1) ≥ (n+ 1)n+1

an+1 − 1 = an(an − 1) ≥ (an − 1)2 =⇒ logx(an+1 − 1) ≥ 2 logx(an − 1)

This implies

loga0(an+1 − 1) loga1(an+1 − 1) · · · logan−1(an+1 − 1) logan(an+1 − 1) ≥

2n(loga0(an − 1) loga1(an − 1) · · · logan−1

(an − 1))

logan(an+1 − 1) ≥2nnn logan(an+1 − 1) = 2nnn logan(an(an − 1)) =(2n)n(1 + logan(an − 1)) ≥ (n+ 1)n+1

(1)

For n ≥ 4 we have (2n)n ≥ (n+ 1)n+1. Indeed

2n ≥ (n+ 1)e ≥ (n+ 1)

(1 +

1

n

)n

, ∀ n ≥ 4

We need to show yet the validity of our inequality for n = 1, 2, 3.

For n = 1 the inequality is

loga0(a0(a0 − 1)) = 1 + loga0(a0 − 1) ≥ 1

being a0 − 1 ≥ 1.

For n = 2 we haveloga0(a2 − 1) loga1(a2 − 1) ≥ 4

orloga0

(a1(a1 − 1)

)loga1

(a1(a1 − 1)

)≥ 4

namelyloga0

(a1a0(a0 − 1)

)loga1

(a1a0(a0 − 1)

)≥ 4 (2)

We rewrite (2) as(1 + loga0 a1 + loga0(a0 − 1)

) (1 + loga1 a0 + loga1(a0 − 1)

)≥ 4

Mathematical Reflections 4 (2010) 17

Page 147: Eserciziario Di Analisi Matematica 2

which is implied by, use again a0 − 1 ≥ 1,(1 + loga0 a1

) (1 + loga1 a0

)≥ 4

and this holds true since it may be written as (1 + x)(1 + 1/x) ≥ 4, and x+ 1/x ≥ 2, x > 0.

The last integer still remaining is n = 3 and by (1) we need to show

63(1 + lna3(a3 − 1)) ≥ 44

The first step is: lna3(a3 − 1) increases with a3 ≥ 7. To prove this let’s write loga3(a3 − 1) =ln(a3−1)

ln a3so that

d

da3

ln(a3 − 1)

ln a3=

1

ln a3

(1

a3 − 1− ln(a3 − 1)

a3 ln a3

)> 0 for a3 ≥ 7 (3)

The monotonicity of lna3(a3− 1) for a3 ≥ 7 implies that it is greater than or equal to ln7 6 andthis in turn implies that it suffices to show

63(1 + ln7 6) ≥ 44

which evidently holds true and we are done.

Also solved by Michel Bataille, France; Daniel Lasaosa, Universidad Publica de Navarra, Spain;Prithwijit De, HBCSE, India; Lorenzo Pascali, Universita di Roma “La Sapienza”, Roma,Italy; Piriyathumwong P., Bangkok, Thailand.

Mathematical Reflections 4 (2010) 18

Page 148: Eserciziario Di Analisi Matematica 2

Undergraduate problems

U163. Find the minimum of f(x, y, z) = x2 +y2 +z2−xy−yz−zx over all triples (x, y, z) of positiveintegers for which 2010 divides f(x, y, z).

Proposed by Titu Andreescu, University of Texas at Dallas, USA

Solution by Daniel Lasaosa, Universidad Publica de Navarra, SpainIf wlog z is odd and x, y are even, then z2 is the only odd term, f(x, y, z) is odd, hence not amultiple of 2010, while if wlog x, y are odd and z is even, then x2, y2, xy are the only odd terms,f(x, y, z) is again odd. Therefore, x, y, z have the same parity, and we may define u = x−y

2 ,v = y−z

2 , or

3s2 + d2 = 4(u2 + v2 + uv) = (x− y)2 + (y − z)2 + (x− y)(y − z) = f(x, y, z),

where s = u+ v and d = u− v, and if 2010 divides f(x, y, z), then 4020 = 22 · 3 · 5 · 67 dividesf(x, y, z). Now, any perfect square leaves a remainder equal to −1, 0, 1 modulus 5, hence ifd, s are not both multiples of 5, then 3s2 + d2 cannot be a multiple of 5, hence 52 divides3s2 + d2 = f(x, y, z), and 20100 divides f(x, y, z). Defining s′ = s

5 and d′ = d5 , we find that

3s′2 + d′2 =f(x, y, z)

25=

20100k

25= 804k.

But taking s′ = 16, d′ = 6, we find 3s′2 +d′2 = 768+36 = 804, or f(x, y, z) ≥ 25 ·804 = 20100,with equality for example for s = 80 and d = 30, ie u = 55 and v = 25, or f(z+160, z+50, z) =20100 for all positive integer z as it is easily checked by direct calculation.

Note: We have restricted ourselves to positive values of f(x, y, z), since clearly f(x, x, x) = 0is a multiple of 2010 for all positive integer x, making the problem trivial.

Mathematical Reflections 4 (2010) 19

Page 149: Eserciziario Di Analisi Matematica 2

U164. Prove that ϕ(

22010! − 1)

ends in at least 499 zeros.

Proposed by Dorin Andrica, “Babes-Bolyai University”, Cluj-Napoca, Romania

Solution by G.R.A.20 Math Problems Group, Roma, ItalyWe will prove that ϕ(22010!− 1) ends with 501 zeros by showing that it is divisible by 5501 and2501.

Since∞∑k=1

⌊2010

5k

⌋= 501 and

∞∑k=1

⌊2010

2k

⌋= 2002,

it follows that 2010! = 4 · 5501 · a = 22002 · b for some positive integers a and b.

By Euler’s Theorem

22010! − 1 = 24·5501·a − 1 = (2a)ϕ(5

502)− 1 ≡ 0 (mod 5502)

which implies that 5501 divides ϕ(22010! − 1).

Now we show that 2k divides ϕ(22k·b − 1) for all k ≥ 1. For k = 1 we have that 22·b − 1 is odd

and 2 divides ϕ(22·b − 1). Moreover, since gcd(22k−1·b − 1, 22

k−1·b + 1) = 1,

ϕ(22k·b − 1) = ϕ(22

k−1·b − 1) · ϕ(22k−1·b + 1)

and 2k−1 divides ϕ(22k−1·b − 1) by inductive hypothesis and 2 divides ϕ(22

k−1·b + 1) because

22k−1·b + 1 is odd. Hence 22002 divides ϕ(22010! − 1).

Also solved by Daniel Lasaosa, Universidad Publica de Navarra, Spain; Neacsu Adrian, Pitesti,Romania.

Mathematical Reflections 4 (2010) 20

Page 150: Eserciziario Di Analisi Matematica 2

U165. Let G = A1, A2, . . . , Am ⊂Mn(R) such that (G, ·) is a group. Prove that Tr(A1 +A2 + · · ·+Am) is an integers divisible by m.

Proposed by Mihai Piticari, “Dragos Voda” National College, Campulung Moldovenesc,Romania

Solution by Michel Bataille, France

Let B = A1+A2+ · · ·+Am. Since (G, ·) is a group, for any fixed j ∈ 1, 2, . . . ,m the mappingA 7→ AAj is a bijection from G onto G. It follows that BAj = B, and since this is true forj = 1, 2, . . . ,m, we have

B2 = B(A1 + +A2 + · · ·+Am) = BA1 +BA2 + · · ·+BAm = mB.

Now, the matrix C = 1m B is idempotent since

C2 =1

m2B2 =

1

m2(mB) = C,

hence Tr(C) = rank(C). Thus, Tr(B) = m · (rank(C)), a multiple of m.

Also solved by Moubinool Omarjee, Paris, France; Carlo Pagano, Universita di Roma “TorVergata”, Roma, Italy

Mathematical Reflections 4 (2010) 21

Page 151: Eserciziario Di Analisi Matematica 2

U166. Find all functions f : [0,∞)→ [0,∞) such that

(a) f is multiplicative

(b) limx→∞ f(x) exists, is finite, and different from 0.

Proposed by Mihai Piticari, “Dragos Voda” National College, Campulung Moldovenesc,Romania

First solution by Emanuele Natale, Universita di Roma “Tor Vergata”, Roma, Italy Letlimt→∞ f(t) = c 6= 0 and let a > 0, then

c = limt→∞

f(at) = limt→∞

f(a)f(t) = f(a) limt→∞

f(t) = f(a)c

which implies that f(a) = 1. Hence f is identically equal to 1 in (0,+∞), whereas f(0)can assume any nonnegative real number. It’s trivial to check that such functions verify theassumptions.

Second solution by Daniel Lasaosa, Universidad Publica de Navarra, Spain Since f is multi-plicative, for any non-negative real x we have

f(x)f(1) = f(x · 1) = f(x), f(x) (f(1)− 1) = 0.

If f(1) 6= 1, then f(x) = 0 for all non-negative real x, in contradiction with condition (b),hence f(1) = 1.

Since f(xy) = f(x)f(y) for all non-negative reals x, y, after trivial induction we find that, forany positive integer n and any non-negative integer x, we have f(xn) = (f(x))n. Take anyy > 1, and assume that f(y) = a > 1. Then,

limx→∞

f(x) = limn→∞

f(yn) = limn→∞

an =∞,

in contradiction with condition (b). Similarly, if f(y) = a < 1, we find limx→∞ f(x) = 0, againin contradiction with condition (b), or f(y) = 1 for all y > 1, and limx→∞ f(x) = 1, finite andnonzero.

For any 0 < z < 1, y = 1z > 1, or f(z) = f(y)f(z) = f(yz) = f(1) = 1. We conclude that

f(x) = 1 for all positive real x. Now, f(0) = f(02) = (f(0))2, and either f(0) = 0 or f(0) = 1.In the second case, f(x) is trivially multiplicative since f(xy) = 1 = 12 = f(x)f(y) for allnon-negative reals x, y, while in the first case, if wlog y = 0, we find for any non-negative realx that f(x ·0) = f(0) = 0 = f(x) ·0 = f(x)f(0). There are therefore two functions that satisfysimultaneously both conditions, f(x) = 1 for any non-negative real x, and f(x) = 1 for allpositive x and f(0) = 0.

Mathematical Reflections 4 (2010) 22

Page 152: Eserciziario Di Analisi Matematica 2

U167. Let f : [0, 1]→ R be a continuously differentiable function such that f(1) = 0. Prove that∣∣∣∣∫ 1

0xf(x)dx

∣∣∣∣ ≤ 1

6maxx∈[0,1]

|f ′(x)|.

Proposed by Duong Viet Thong, National Economics University, Ha Noi, Vietnam

Solution by Arkady Alt, San Jose, California, USA

Using integration by parts we obtain∫ 10 xf (x) dx =

(x2

2· f (x)

)1

0

−∫ 10

x2f ′ (x)

2dx =

−1

2

∫ 10 x

2f ′ (x) dx. Since by condition f (x) is continuously differentiable then

M := maxx∈[0,1]

|f ′ (x)| and, therefore,∣∣∣∫ 1

0 xf (x) dx∣∣∣ =

∣∣∣∣−1

2

∫ 10 x

2f ′ (x) dx

∣∣∣∣ =

1

2

∣∣∣∫ 10 x

2f ′ (x) dx∣∣∣ ≤ 1

2

∫ 10 x

2 |f ′ (x)| dx ≤ M

2

∫ 10 x

2dx =1

6maxx∈[0,1]

|f ′ (x)| ..

Also solved by Michel Bataille, France; Daniel Lasaosa, Universidad Publica de Navarra, Spain;Carlo Pagano, Universita di Roma “Tor Vergata”, Roma, Italy; Perfetti Paolo, Dipartimentodi Matematica, Universita degli studi di Tor Vergata Roma, Italy.

Mathematical Reflections 4 (2010) 23

Page 153: Eserciziario Di Analisi Matematica 2

U168. Let f : [a, b]→ R be a twice differentiable function on (a, b) and letmaxx∈[a,b] |f ′′(x)| = M. Prove that∣∣∣∣∫ b

af(x)dx− f

(a+ b

2

)(b− a)

∣∣∣∣ ≤ (b− a)3

24M.

Proposed by Duong Viet Thong, National Economics University, Ha Noi, Vietnam

Solution by Michel Bataille, FranceLet ∆ =

∫ ba f(x) dx− f

(a+b2

)(b− a). From the easily verified result

∫ ba

(x− a+b

2

)dx = 0, we

deduce

∆ =

∫ b

a

(f(x)− f

(a+ b

2

)−(x− a+ b

2

)f ′(a+ b

2

))dx.

Now, for any x ∈ [a, b], we have

f(x) = f

(a+ b

2

)+

(x− a+ b

2

)f ′(a+ b

2

)+

1

2

(x− a+ b

2

)2

f ′′(θ)

for some real θ between x and a+b2 , hence∣∣∣∣f(x)− f

(a+ b

2

)−(x− a+ b

2

)f ′(a+ b

2

)∣∣∣∣ ≤ M

2

(x− a+ b

2

)2

.

It follows that

|∆| ≤ M

2

∫ b

a

(x− a+ b

2

)2

dx =M

6

((b− a

2

)3

−(a− b

2

)3)

=(b− a)3

24M.

Also solved by Arkady Alt, San Jose, California, USA; Daniel Lasaosa, Universidad Publica deNavarra, Spain; G.R.A.20 Math Problems Group, Roma, Italy; Perfetti Paolo, Dipartimentodi Matematica, Universita degli studi di Tor Vergata Roma, Italy.

Mathematical Reflections 4 (2010) 24

Page 154: Eserciziario Di Analisi Matematica 2

Olympiad problems

O163. Prove that the equationx3 + y3

x− y= 2010

is not solvable in positive integers.

Proposed by Titu Andreescu, University of Texas at Dallas, USA and Dorin Andrica,“Babes-Bolyai University”, Cluj-Napoca, Romania

First solution by Daniel Lasaosa, Universidad Publica de Navarra, Spain Assume that x, yhave opposite parity, then x3 + y3 is odd, and 2010 is odd, contradiction, hence x, y have the

same parity and x − y is even. If x, y are both odd, x2 − xy + y2 = x3+y3

x+y is odd, hence 2divides x + y with multiplicity a + 1, where a is the multiplicity with which 2 divides x − y.Now, (x + y) + (x − y) = 2x is divided by 2 with multiplicity 1, hence a = 1, or if x, y areboth odd, x− y is divisible by 2 but not by 4, and x+ y is divisible by 4 but not by 8. If x, yare both even, and both are divided by 2 with different multiplicity, then x+ y and x− y areboth divided by the lowest of both multiplicities, hence xy(x+y)

x−y = 2010− x2 + y2 is a multiple

of 4, or since x2, y2 are also multiples of 4, 2010 is a multiple of 4, absurd. Therefore, x, y aredivisible by 2 with the same multiplicity a, and x+y, x−y are divisible by 2 with multiplicityat least a+ 1, hence x− y is divisible by 2 with multiplicity at least 3a ≥ 3.

It is well known that any perfect cube leaves remainder −1, 0, 1 modulus 9, or since x3 + y3 isa multiple of 3 because 3 divides 2010, then x3 + y3 is a multiple of 9, and x− y is a multipleof 3 because 2010 is not a multiple of 9. If x, y are not multiples of 3, since x − y dividesxy(x + y), then x + y must be a multiple of 3, or 2x = (x + y) + (x − y) is a multiple of 3,contradiction, hence x, y are multiples of 3, hence x3 + y3 is a multiple of 27, and x− y mustbe a multiple of 9.

Since 452 = 2025 > 2010 = x3+y3

x−y > x2, we have x ≤ 44, or since y ≥ 1, then x− y ≤ 43 must

be an even multiple of 9, divisible by 2 if x, y are both odd, or divisible by at least 23 = 8if x, y are both even. We conclude that x − y = 18, and x, y are both odd. Writing x as afunction of y, the proposed equation becomes

y3

27+ y2 + 18y = 562, , z3 + 9z2 + 54z = 562,

where we have defined z = y3 because y is clearly divisible by 3. Note now that the LHS

increases strictly with z, and if z = 5 then z3 + 9z2 + 54z = 125 + 225 + 270 = 620 > 562,while if z = 4 then z3 + 9z2 + 54z = 64 + 144 + 216 = 424 < 562. It follows that no positiveintegral solutions exist for the proposed equation.

Second solution by the authors

Assume that the equation is solvable in positive integers. It is clear that x > y. We can write

2010 =x3 + y3

x− y>x3 − y3

x− y= x2 + xy + y2 = (x− y)2 + 3xy > (x− y)2,

Mathematical Reflections 4 (2010) 25

Page 155: Eserciziario Di Analisi Matematica 2

and get x− y <√

2010. It follows x− y ≤ 44.

On the other hand, we have

2010 =x3 + y3

x− y≥ x3 + y3

44=

3

44(x3 + y3

3) ≥ 3

44(x+ y

3)3,

hence we obtain x+ y ≤ 96.

The equation is equivalent to

(x+ y)(x2 − xy + y2) = 2 · 3 · 5 · 67 · (x− y).

If x+y is divisible by 67, then necessarily x+y = 67, since x+y ≤ 96 and 67 is a prime number.In this case we get x2− xy+ y2 = 30(x− y) and x+ y = 67, hence (x+ y)2− 3xy = 30(x− y).That is 672 = 30(x−y)+3x(67−x), equation with no integer solutions, since 67 is not divisibleby 3.

If x2 − xy + y2 is divisible by 67, then x2 − xy + y2 = 67k for some positive integer k. Theequation is equivalent to

k(x+ y) = 30(x− y),

that is (30− k)x = (30 + k)y. It follows y = 30−k30+kx, hence

x2[(30 + k)2 − (30− k)(30 + k) + (30− k)2] = 67k(30 + k)2.

and we getx2(3k2 + 302) = 67k(30 + k)2.

It is clear that k is divisible by 3, hence we have k = 3a for some positive integer 1 ≤ a ≤ 9.Then

x2(3a2 + 100) = 67 · a · (a+ 10)2.

Because x2 can’t be divisible by 67 it follows that 3a2 + 100 is divisible by 67. Replacinga = 1, 2, ..., 9 is easy to see that 3a2 + 100 has no this property, hence the equation is notsolvable.

Also solved by Raul A. Simon, Chile.

Mathematical Reflections 4 (2010) 26

Page 156: Eserciziario Di Analisi Matematica 2

O164. Let ABC be a triangle and let A1 be a point on the side BC. Starting with A1 constructreflections in one of the angle bisectors of triangle such that the next point lies on the otherside of the triangle. The process is done in one direction: either clockwise or counterclockwise.Thus at the first step we construct an isosceles triangle A1CB1 with point B1 lying on AC.At the second step we construct an isosceles triangle B1AC1 with point C1 on AB. In fact weget a sequence of points A1, B1, C1, A2, . . . .

(a) Prove that the process terminates in six steps, that is A1 ≡ A3

(b) Prove that A1, A2, B1, B2, C1, C2 lie on the same circle.

Proposed by Ivan Borsenco, Massachusetts Institute of Technology, USA

Solution by Michel Bataille, France

A

BCA

1

I

B1C1

A2

B2C2

Let RMN denote the reflection in the line MN and let I be the incentre of ∆ABC.(a) As the product of three opposite isometries (reversing the orientation), the isometry R =RBI RAI RCI is opposite as well and since R(I) = I, R must be a reflection in a line `. SinceR(A1) = A2, ` must be the perpendicular bisector of the line segment A1A2 in the general casewhen A1 6= A2 (and IA1 if A1 = A2). Thus, R = R` and RBI RAI RCI RBI RAI RCI =R` R` = Id where Id denotes the identity of the plane. As a result, A3 = Id(A1) = A1.(b) Since ` is the the perpendicular bisector of A1A2, we have IB1 = IA1 = IA2 = IC1.Similarly, if `′ denote the perpendicular bisector of B1B2, we have R`′ = RCI RBI RAI andso IC2 = IB2 = IB1.In conclusion, IA1 = IA2 = IB1 = IB2 = IC1 = IC2 and the six points A1, A2, B1, B2, C1, C2

all lie on a circle with centre I.

Also solved by Daniel Lasaosa, Universidad Publica de Navarra, Spain; Raul A. Simon, Chile.

Mathematical Reflections 4 (2010) 27

Page 157: Eserciziario Di Analisi Matematica 2

O165. Let R and r be the circumradius and the inradius of a triangle ABC with the lengths of sidesa, b, c. Prove that

2− 2∑cyc

(a

b+ c

)2

≤ r

R.

Proposed by Dorin Andrica, “Babes-Bolyai University”, Cluj-Napoca, Romania

Solution by Arkady Alt, San Jose, California, USA

Note that 2− 2∑cyc

(a

b+ c

)2

≤ r

R⇐⇒ 6− 2

∑cyc

(a

b+ c

)2

≤ 4 +r

R⇐⇒

2

(3−

∑cyc

(a

b+ c

)2)≤ 4 +

r

R⇐⇒ 2

∑cyc

(b+ c)2 − a2

(b+ c)2≤ 4 +

r

R.

Since cosA+ cosB + cosC = 1 +r

Rand

1

(b+ c)2≤ 1

4bcthen

(b+ c)2 − a2

2bc= 1 + cosA

2∑cyc

(b+ c)2 − a2

(b+ c)2≤∑cyc

(b+ c)2 − a2

2bc=∑cyc

(1 + cosA) = 4 +r

R.

Remark.

Let la, lb, lc be angle bisectors of a triangle ABC.Noting that(b+ c)2 − a2

(b+ c)2=al2aabc

we can

rewrite original inequality in such form 2∑cyc

al2aabc≤ 4 +

r

R⇐⇒ 2

∑cyc

al2a4Rrs

≤ 4 +r

R⇐⇒

al2a + bl2b + cl2ca+ b+ c

≤ r (4R+ r) .

Also solved by Daniel Lasaosa, Universidad Publica de Navarra, Spain; Michel Bataille, France.

Mathematical Reflections 4 (2010) 28

Page 158: Eserciziario Di Analisi Matematica 2

O166. The incircle σ of triangle ABC with incenter I is tangent to sides BC and AC at points A1

and B1, respectively. Points A2 and B2 are diametrically opposite to A1 and B1 in σ. Let A3

and B3 be the intersection points of AA2 with BC and BB2 with AC, respectively. Let Mbe the midpoint of side AC and let N be the midpoint of A1A3. Line MI meets BB1 in Tand line AT meets BC in P. Let Q ∈ (BC), R be the intersection of lines AB and QB1 andNR ∩AC = S. Prove that [AS] = 2[SM ] if and only if [BP ] = [PQ].

Proposed by Andrei Razvan Baleanu, “George Cosbuc”, Motru, Romania

Solution by Daniel Lasaosa, Universidad Publica de Navarra, SpainConsider the parallel to BC through A2, clearly the incircle of ABC is the excircle of thetriangle formed by this line, AB and AC, and this triangle is homothetic to ABC, with centerof homothety in A, or A3 = AA2 ∩BC is the contact point of the excircle of ABC tangent tosegment BC and to lines AB and AC. It is well known that the contact points of the incircleand the excircle on a given triangle side are symmetric with respect to its midpoint, so N isalso the midpoint of BC. Note that AS = 2SM iff 3AS = 2AM = AC, or AS

SC = 12 . Therefore,

since CNNB = 1, and R,S,N are collinear, by Menelaus’ theorem, AS = 2SM iff BR = 2AR.

Now, it is well known that AB1 = b+c−a2 and CB1 = a+b−c

2 , or AB1B1C

= b+c−aa+b−c , and again by

Menelaus’ theorem, and using that Q,B1, R are collinear, AS = 2SM iff BQCQ = 2(b+c−a)

a+b−c , or

equivalently, iff BQ = 2a(b+c−a)3b+c−a .

Using exact trillinear coordinates, M ≡(ha2 , 0,

hc2

), where ha, hb, hc are the lengths of the

respective altitudes from A,B,C, while I ≡ (r, r, r), r being the inradius. It follows that lineMI has equation aα+(c−a)β−cγ = 0. Using again exact trillinear coordinates, B ≡ (0, hb, 0),while B1 ≡

(a+b−c

2 sinC, 0, b+c−a2 sinA

), or line BB1 has equation a(b+c−a)α = c(a+b−c)γ,

or point T has (non-exact) trillinear coordinates T ≡ (c(a+ b− c), 2ac, a(b+ c− a)). SinceA ≡ (1, 0, 0) in non-exact trillinear coordinates, line AT has equation 2cγ = (b + c − a)β, orpoint P satisfies

b ·BPc · CP

=BP sinB

CP sinC=γ

β=b+ c− a

2c,

or CPBP = 2b

b+c−a , yielding BCBP = 3b+c−a

b+c−a , and BP = a(b+c−a)3b+c−a . It clearly follows that BQ = 2BP ,

and equivalently BP = PQ, iff AS = 2SM .

Mathematical Reflections 4 (2010) 29

Page 159: Eserciziario Di Analisi Matematica 2

O167. Prove that in any convex quadrilateral ABCD,

cosA−B

4+ cos

B − C4

+ cosC −D

4+ cos

D −A4

≥ 2 +1

2(sinA+ sinB + sinC + sinD).

Proposed by Titu Andreescu, University of Texas at Dallas, USA

Solution by Daniel Lasaosa, Universidad Publica de Navarra, SpainWe can write

2 + sinA+ sinB = 2 + 2 sinA+B

2cos

A−B2

≤ 2 + 2 cosA−B

2= 4 cos2

A−B4

≤ 4 cosA−B

4,

with equality iff A+B = 180 and simultaneously A = B, ie, equality holds iff A = B = 90.Adding the cyclic permutations of both sides of this resulting inequality, we obtain the proposedinequality with both sides multiplied by 4. The conclusion follows, equality holds in theproposed inequality iff ABCD is a rectangle.

Mathematical Reflections 4 (2010) 30

Page 160: Eserciziario Di Analisi Matematica 2

O168. Given a convex polygon A1A2 . . . An, n ≥ 4, denote by Ri the radius of the circumcircle oftriangle Ai−1AiAi+1, where i = 2, 3, . . . , n and An+1 is the vertex A1. Given that R2 = R3 =· · · = Rn, prove that the polygon A1A2 . . . An is cyclic.

Proposed by Nairi Sedrakyan, Armenia

Solution by Raul A. Simon, ChileThat the polygon A1A2 . . . An is convex means that ∠Ai−1AiAi+1 is obtuse - if you take thesmallest of the two angles formed at the vertex. Therefore, the circumcenter Oi of ∠Ai−1AiAi+1

is exterior to it. In fact, all circumcenters Oi lie in a zone Z that is exterior to all trianglesAi−1AiAi+1 ( where i = 2, 3, . . . , n and An+1 is the vertex A1) and interior to the polygonA1A2 . . . An. We have then

O2A1 = O2A2 = O2A3 = R,

andO3A2 = O3A3 = O3A4 = R,

etc. We see that O2 is the intersection of two arcs of circle of radius R, centered at A2 andA3; and that O3 is determined in exactly the same way. Therefore, since O2 and O3 lie onthe same side of A2A3, O2 and O3 must coincide. Repeating this reasoning, we find that allcircumcenters must coincide in a unique circumcenter O common to all vertices. The circleO(O,R) is the circumcircle of the polygon; therefore, the latter is cyclic.

Mathematical Reflections 4 (2010) 31

Page 161: Eserciziario Di Analisi Matematica 2

Junior problems

J169. If x, y, z > 0 and x+ y + z = 1, find the maximum of

E(x, y, z) =xy

x+ y+

yz

y + z+

zx

z + x.

Proposed by Dorin Andrica, Babes-Bolyai University, Cluj-Napoca, Romania

Solution by Ercole Suppa, Teramo, Italy

By the AM–GM inequality,

E(x, y, z) ≤(x+y)2

4

x+ y+

(y+z)2

4

y + z+

(z+x)2

4

z + x=

=x+ y

4+y + z

4+z + x

4=

=x+ y + z

2=

1

2

Therefore the maximum of E(x, y, z) is 12 , occurring for example if

x = y = z =1

3

Also solved by Arkady Alt, San Jose, California, USA; Michel Bataille, France; Daniel Lasaosa,Universidad Publica de Navarra, Spain; Perfetti Paolo, Dipartimento di Matematica, Universitadegli studi di Tor Vergata Roma, Italy; Sayan Mukherjee; Lorenzo Pascali, Universita di Roma“La Sapienza”, Roma, Italy.

Mathematical Reflections 5 (2010) 1

Page 162: Eserciziario Di Analisi Matematica 2

J170. In the interior of a regular pentagon ABCDE consider the point M such that triangle MDEis equilateral. Find the angles of triangle AMB.

Proposed by Catalin Barbu, Vasile Alecsandri College, Bacau, Romania

Solution by Lorenzo Pascali, Universita di Roma “La Sapienza”, Roma, Italy

We have that∠MEA = ∠AED − ∠MED = 108 − 60 = 48.

Morever, AE = EM implies that

∠EAM = ∠EMA =1

2(180 − 48) = 66.

By symmetry

∠ABM = ∠MBC =1

2∠ABC = 54.

Finally

∠MAB = ∠EAB −∠DCM = 108− 66 = 42, ∠AMB = 180−∠MAB −∠ABM = 84.

Also solved by Daniel Lasaosa, Universidad Publica de Navarra, Spain; Sayan Mukherjee; RaulA. Simon, Chile; Ercole Suppa, Teramo, Italy; Vicente Vicario Garcia, Huelva, Spain.

Mathematical Reflections 5 (2010) 2

Page 163: Eserciziario Di Analisi Matematica 2

J171. If different letters represent different digits, could the addition

AXXXU

BXXV

CXXY

+DEXXZ

−−−−−−−−XXXXX

be correct?

Proposed by Titu Andreescu, University of Texas at Dallas, USA

Solution by Francisco Javier Garcia Capitan, Spain

We assume that leading digits A, B, C, D and X cannot be 0.

Now, A and D are distinct numbers, so A+D > 3. In addition, the sum in the fourth column(we refer the columns from right to left) has carrying, thus we have in fact X > 3.

The sum in the first column has carry as well, because if not, we have 4X ≡ X mod 10 inthe second column, and this is impossible. Call R the carrying of the sum in the first column.We’ll have R + 4X = 10R +X adding the second column, thus X = 3R. Since X > 3, it canbe X = 6, R = 2 or X = 9, R = 3, but the latter is impossible, since U + V + X + Y <=6 + 7 + 8 + 9 = 30 and X = 9, R = 3 implies U + V + Y + Z = 39.

If we must have X = 6, R = 2, which implies U + V + Y + Z = 26 and some of

(1)

B + C + E = 8

A+D = 5, (2)

B + C + E = 18

A+D = 4.

If we take E = 0 and the remaining digits such that B,C = 1, 7, A,D = 2, 3 andU, V, Y, Z = 4, 5, 8, 9 then (1) holds and we have a solution of the problem.

2 6 6 6 41 6 6 57 6 6 8

+ 3 0 6 6 9− − − − − − −

6 6 6 6 6

Note that the conditions (2) are impossible since they imply that

(U + V + Y + Z) +X + (B + C + E) + (A+D)

=26 + 6 + 18 + 4 = 54 6= 45,

which is the sums of the ten digits.

Also solved by Daniel Lasaosa, Universidad Publica de Navarra, Spain; Ercole Suppa, Teramo,Italy; Andrea Ligori, Universita di Roma “Tor Vergata”, Roma, Italy.

Mathematical Reflections 5 (2010) 3

Page 164: Eserciziario Di Analisi Matematica 2

J172. Let P be a point situated in the interior of an equilateral triangle ABC and let A′, B′, C ′ bethe intersectionsof lines AP,BP,CP with sides BC,CA,AB, respectively. Find P such that

A′B2 +B′C2 + C ′A2 = AB′2 +BC ′2 + CA′2.

Proposed by Catalin Barbu, Vasile Alecsandri College, Bacau, Romania

Solution by Sayan Mukherjee

Let us denote the areas of 4BPC,4APB,4APC as x, y, z respectively. we have,

BA′

A′C=4AA′B4AA′C

=4BPA′

4CPA′=4AA′B −4BPA′

4AA′C −4CPA′=4APB4APC

=y

z.

Let AB = BC = CA = a, then BA′ =ay

y + z; and A′C =

az

y + z. Similarly we obtain four other

relations.

Substituting these values in our first relation we obtain,

y2

(y + z)2+

z2

(z + x)2+

x2

(x+ y)2=

z2

(y + z)2+

x2

(z + x)2+

y2

(x+ y)2;

Which, on transposition, is equivalent to

y2 − z2

(y + z)2+z2 − x2

(z + x)2+x2 − y2

(x+ y)2= 0.

This relation can be simplified to∑cyc

x− yx+ y

= 0, which rewrites into

(x− y)(z − x)(z − y)

(x+ y)(y + z)(z + x)= 0.

Mathematical Reflections 5 (2010) 4

Page 165: Eserciziario Di Analisi Matematica 2

Hence the first relation actually holds true if and only if x = y or y = z or z = x. So, if P issuch that at least two of the triangles 4APB,4BPC,4CPA have the same area, then thefirst relation holds good. So, P must lie on any of the three medians of 4ABC.

Also solved by Michel Bataille, France; Daniel Lasaosa, Universidad Publica de Navarra,Spain; Francisco Javier Garcia Capitan, Spain; Raul A. Simon, Chile; Ercole Suppa, Teramo,Italy;Vicente Vicario Garcia, Huelva, Spain.

Mathematical Reflections 5 (2010) 5

Page 166: Eserciziario Di Analisi Matematica 2

J173. Let a and b be rational numbers such that

|a| ≤ 47

|a2 − 3ab2|and |b| ≤ 52

|b2 − 3a2|.

Prove that a2 + b2 ≤ 17.

Proposed by Titu Andreescu, University of Texas at Dallas, USA

First solution by Michel Bataille, France

Let X = a3 − 3ab2 and Y = 3a2b − b3. From the hypothesis, we have |X| ≤ 47 and |Y | ≤ 52.Observing that X + iY = (a+ ib)3 (easily checked), we deduce that

a2 + b2 = |a+ ib|2 =∣∣(a+ ib)3

∣∣ 23 =(

(X2 + Y 2)12

) 23

= (X2 + Y 2)13 ≤ (472 + 522)

13 .

Since 472 + 522 = 4913 = 173, it follows that a2 + b2 ≤ 17.

Second solution by Daniel Lasaosa, Universidad Publica de Navarra, Spain

Note that a = b = 0 clearly satisfies a2 + b2 < 17, therefore a2 − 3b2 and b2 − 3a2 arenonzero rationals, since otherwise 3 would be the square of a rational, absurd. Define x = a2,y = b2, hence x(x− 3y)2 ≤ 472 and y(3x− y)2 ≤ 522. Adding these two inequalities results in(x + y)3 ≤ 472 + 522 = 173, or x + y = a2 + b2 ≤ 17. Note that equality is reached wheneverequality holds simultaneously on both inequalities given as condition in the problem statement,for example when a = ±1 and b = ±4.

Mathematical Reflections 5 (2010) 6

Page 167: Eserciziario Di Analisi Matematica 2

J174. The incircle of triangle ABC touches sides BC,CA,AB at D,E, F , respectively. Let K be apoint on side BC and let M be the point on the line segment AK such that AM = AE = AF .Denote by L and N the incenters of triangles ABK and ACK, respectively. Prove that K isthe foot of the altitude from A if and only if DLMN is a square.

Proposed by Bogdan Enescu, B.P.Hasdeu College, Buzau, Romania

Solution by Ercole Suppa, Teramo, Italy

We begin by proving two lemmas.

Lemma 1. The points D, K lie on the circle with diameter LN .

Proof. Suppose wlog that c < b. Let I, U , V be the incenter of ABC and the points where thecircles (L), (N) touch the side BC; let r, r1, r2 be the inradii of the circles (I), (L), (N), asshown in figure.

A

B C

I

K

F

E

D

N

L

VU

Denote a = BC, b = CA, c = AB, m = BK, n = KC, x = AK.

Since L and N are the incenters of 4ABK and 4ACK we have

∠LKN = ∠LKA+ ∠AKN =1

2(∠BKA+ ∠AKC) = 90

In order to prove that ∠LDN = 90 we will show that

LD2 +DN2 = LN2 (1)

The Pythagora’s theorem yields LD2 = r21 + UD2, ND2 = r22 +DV 2 and

LN2 = UV 2 + (r1 − r2)2 = UD2 +DV 2 − 2UD ·DV + r21 + r22 − 2r1r2

Mathematical Reflections 5 (2010) 7

Page 168: Eserciziario Di Analisi Matematica 2

Therefore to establish (3) it is enough to show that UD ·DV = r1r2.

We clearly have

UD = BD −BU =a+ c− b

2− m+ c− x

2=a+ x− b−m

2(2)

DV = DC − CV =a+ b− c

2− m+ b− x

2=a+ x− c− n

2(3)

From (2) and (3), plugging n = a−m into the expression, we obtain

UD ·DV =(x+ a− b−m)(x− c+m)

4(4)

From the similar triangles 4BUL ∼ 4BDI, 4CV N ∼ 4CDI, it follows that

LU : ID = BU : BD ⇒ r1 = r · c+m− xa+ c− b

(5)

NV : ID = CV : CD ⇒ r2 = r · b+ n− xa+ b− c

(6)

From (5),(6) using the well known formula

r2 =(b+ c− a)(a+ c− b)(a+ b− c)

4(a+ b+ c)

and plugging n = a−m into the expression, we obtain

r1r2 =(b+ c− a)(a+ b−m− x)(c+m− x)

4(a+ b+ c)(7)

Using (4) and (7) we have

UD ·DV − r1r2 =ax2 − ac2 + a2m− b2m+ c2m− am2

2(a+ b+ c)(8)

From the Stewart’s theorem we get

x2 =mb2 + (a−m)c2 − am(a−m)

a

Finally, plugging x2 into (8), after a boring calculation, we have

UD ·DV − r1r2 =

(c2 − am

)(m+ n− a)

2(a+ b+ c)= 0

Thus LD2 +DN2 = LN2 and the lemma is proved.

Lemma 2. If M is the second intersection point of AK with the circle γ circumscribed toDKNL, then DM ⊥ LN and AM = AE = AF .

Mathematical Reflections 5 (2010) 8

Page 169: Eserciziario Di Analisi Matematica 2

A

B CK

F

E

D

N

L

VU Y

X

M

F'

K'

Proof. Let the incircle of triangle ABK touches side AB at F ′. According to Lemma 1 thecenter of γ is the midpoint of LN , so the point M lies on the external tangent to the circles(L), (N). Therefore, because of simmetry of the figure, we have DM ⊥ LN and

AM = AF ′ − UD = AF ′ − (BD −BU)

=c+ x−m

2− a+ c− b

2+c+m− x

2

=b+ c− a

2= AF

The lemma is proved.

Considering now the original problem, from Lemma 1 and Lemma 2 it follows that

• DLMN is cyclic;

• ∠LDN = ∠LMN = 90;

• DM ⊥ LN .

A

B CK

F

E

D

N

L

M

Mathematical Reflections 5 (2010) 9

Page 170: Eserciziario Di Analisi Matematica 2

Therefore DLMN is a square if and only if MD is a diameter of the circumcircle of DLMN ,i.e. ∠MKD = 90 (i.e. AK ⊥ BC).

Also solved by Michel Bataille, France; Daniel Lasaosa, Universidad Publica de Navarra, Spain;Sayan Mukherjee.

Mathematical Reflections 5 (2010) 10

Page 171: Eserciziario Di Analisi Matematica 2

Senior problems

S169. Let k > 1 be an odd integer such that ak + bk = ck + dk for some positive integers a, b, c, d.Prove that ak+bk

a+b is not a prime.

Proposed by Ivan Borsenco, Massachusetts Institute of Technology, USA

Solution by Daniel Lasaosa, Universidad Publica de Navarra, Spain

Note first that we need to assume that a, b 6= c, d, since otherwise we may take k = 3,

a = c = 2, b = d = 1, and ak+bk

a+b = 3 is prime. Moreover, as long as a, b = c, d, the

condition ak + bk = ck + dk in the problem is irrelevant, so I will assume that a, b 6= c, d.Note also that, if a = b > 1, then ak+bk

a+b = ak−1 cannot be a prime because k ≥ 3, whereasa = b = 1 is only possible if c = d = 1. We may then assume wlog that a ≥ b + 1 ≥ 2, hencefor any odd k ≥ 3,

ak + bk − (a+ b)2 ≥ a3 + b3 − a2 − b2 − 2ab = (a+ b)(a2 − ab+ b2 − a− b) =

= (a+ b)(a− b− 1)2 + b(a+ b)(a− 2) + (a+ b)(a− b− 1) ≥ 0,

with equality iff k = 3, a = 2 and b = 1, possible only if c, d = 1, 2.

We may therefore assume that a, b 6= c, d, where moreover wlog a > b and c ≥ d, and

by the previous result, ak+bk

a+b > a + b, and similarly ck+dk

c+d > c + d. Note that, if a = c, then

bk = dk, or b = d, in contradiction with our assumption, and similarly if b = d, hence eithera > c ≥ d > b, or c > a > b > d.

In the first case, note that

(a− c)(ak−1 + cak−2 + · · ·+ ck−1

)= (d− b)

(bk−1 + dbk−2 + · · ·+ dk−1

),

and since aicj > bidj because a > b and c ≥ d, we have a − c < d − b, or a + b < c + d.Therefore, ak+bk

a+b > ck+dk

c+d , and if ak+bk

a+b is prime, it cannot divide ck+dk

c+d , hence it divides c+ d,

or ak + bk ≤ (a+ b)(c+ d) < (c+ d)2 < ck + dk, absurd.

In the second case, we obtain similarly that a+ b > c+ d, and if ak+bk

a+b divides c+ d, we have

ak + bk ≤ (a + b)(c + d) < (a + b)2, contradiction, hence ak+bk

a+b divides ck+dk

c+d , or c + d dividesa+ b, and since a+ b > c+ d, then a+ b ≥ 2(c+ d) > 2c > a+ b, absurd.

The conclusion follows.

Mathematical Reflections 5 (2010) 11

Page 172: Eserciziario Di Analisi Matematica 2

S170. Consider n(n ≥ 6) circles of radius r < 1 that are pairwise tangent and all tangent to a circleof radius 1. Find r.

Proposed by Catalin Barbu, Vasile Alecsandri College, Bacau, Romania

Solution by Daniel Lasaosa, Universidad Publica de Navarra, Spain

We will assume that the problem statement means that n circles or radius r are all tangent toa given circle of radius 1, and each circle of radius r is tangent to another two circles of radiusr, since it is well known that at most four circles can be tangent to each other, and at mostthree of them would have the same radius, since either one of the four circles contains the otherthree inside them, or one of them is in the gap formed by the other three.

With this assumption, clearly all centers of the circles with radii r are at a distance 1 + r fromthe center of the circle with radius 1, and at a distance 2r from their closest neighbours ofradius r, and by cyclic symmetry, these n centers are at the vertices of a regular n-gon, withsidelength 2r and circumradius 1 + r, hence considering the isosceles triangle with equal sidesof length 1 + r and sidelength 2r for the other side, where the different angle is 360

n , we have

r = (1 + r) sin 180

n , or equivalently

r =sin 180

n

1− sin 180

n

.

Also solved by Raul A. Simon, Chile.

Mathematical Reflections 5 (2010) 12

Page 173: Eserciziario Di Analisi Matematica 2

S171. Prove that if the polynomial P ∈ R[X] has n distinct real zeros, then for any α ∈ R thepolynomial Q(X) = αXP (X) + P ′(X) has at least n− 1 distinct real zeros.

Proposed by Dorin Andrica, Babes-Bolyai University, Cluj-Napoca, Romania

Solution by Carlo Pagano, Universita di Roma “Tor Vergata”, Roma, Italy

Let be a1 < a2 < · · · < an be the real roots of P and

P (x) = Q(x)n∏j=1

(x− aj)

where Q ∈ R[x] has no real roots. Therefore

R(ai) = αaiP (ai) + P ′(ai) = P ′(ai) = Q(ai)∏j 6=i

(ai − aj)

and it’s clear that in the last product has n− i− 1 negative factors. Since Q(x) has constantsign, it follows that R(ai)R(ai+1) < 0 for i = 1, . . . , n− 1, and by Intermediate Value Theoremthere exists at least a zero of R in (ai, ai+1). This means that R has at least n− 1 distinct realzeros.

Also solved by Arkady Alt, San Jose, California, USA; Michel Bataille, France; Daniel Lasaosa,Universidad Publica de Navarra, Spain; Vicente Vicario Garcia, Huelva, Spain.

Mathematical Reflections 5 (2010) 13

Page 174: Eserciziario Di Analisi Matematica 2

S172. Let a, b, c be positive real numbers. Prove that∑cyc

a2b2(b− c)a+ b

≥ 0.

Proposed by Titu Andreescu, University of Texas at Dallas, USA

Solution by Perfetti Paolo, Dipartimento di Matematica, Universita degli studi di Tor VergataRoma, Italy

Clearing the denominators, the inequality is equivalent to∑cyc

(a4c2b+ c4b3) ≥∑cyc

2a3b2c

but this follows from the AM–GM (a4c2b+ c4b3)/2 ≥ a3b2c

Also solved by Arkady Alt, San Jose, California, USA; Michel Bataille, France; Daniel Lasaosa,Universidad Publica de Navarra, Spain; Sayan Mukherjee; Ercole Suppa, Teramo, Italy.

Mathematical Reflections 5 (2010) 14

Page 175: Eserciziario Di Analisi Matematica 2

S173. Let

fn(x, y, z) =(x− y)zn+2 + (y − z)xn+2 + (z − x)yn+2

(x− y)(y − z)(x− z).

Prove that fn(x, y, z) can be written as a sum of monomials of degree n and find fn(1, 1, 1) forall positive integers n.

Proposed by Ivan Borsenco, Massachusetts Institute of Technology, USA

Solution by Michel Bataille, France

It is easily verified that

1

(x− y)(x− z)+

1

(y − x)(y − z)+

1

(z − y)(z − x)= 0.

This allows one to transform fn(x, y, z) as follows:

fn(x, y, z) =xn+2

(x− y)(x− z)+

yn+2

(y − x)(y − z)+

zn+2

(z − y)(z − x)

=−xn+2

(y − x)(y − z)+

−xn+2

(z − y)(z − x)+

yn+2

(y − x)(y − z)+

zn+2

(z − y)(z − x)

=yn+2 − xn+2

(y − x)(y − z)+

zn+2 − xn+2

(z − y)(z − x)

=yn+1 + ynx+ · · ·+ xny + xn+1

(y − z)+zn+1 + znx+ · · ·+ xnz + xn+1

(z − y)

=(yn+1 − zn+1) + x(yn − zn) + · · ·+ xn(y − z)

y − z,

and finally

fn(x, y, z) = (yn+yn−1z+· · ·+yzn−1+zn)+x(yn−1+yn−2z+· · ·+yzn−2+zn−1)+· · ·+xn−1(y+z)+xn.

Thus, fn(x, y, z) is the sum of all monomials xaybzc over all triples (a, b, c) of nonnegativeintegers satisfying a+ b+ c = n. This answers the first part of the question.As for the second part, fn(1, 1, 1) is just the total number of monomials in the above sum, that

is, (n+ 1) + n+ · · ·+ 2 + 1 = (n+1)(n+2)2 .

Also solved by Daniel Lasaosa, Universidad Publica de Navarra, Spain; Moubinool Omarjee ,Paris France; Perfetti Paolo, Dipartimento di Matematica, Universita degli studi di Tor VergataRoma, Italy.

Mathematical Reflections 5 (2010) 15

Page 176: Eserciziario Di Analisi Matematica 2

S174. Prove that for each positive integer k the equation

x31 + x32 + · · ·+ x3k + x2k+1 = x4k+2

has infinitely many solutions in positive integers with x1 < x2 < · · · < xk+1.

Proposed by Dorin Andrica, Babes-Bolyai University, Cluj-Napoca, Romania

First solution by Arkady Alt, San Jose, California, USA

Since

13 + 23 + · · ·+ k3 =k2 (k + 1)2

4= (1 + 2 + · · ·+ k)2

then by substitution

x1 = x, x2 = 2x, . . . , xk = kx, xk+1 = (1 + 2 + · · ·+ k) y2, xk+2 = (1 + 2 + · · ·+ k) y

in original equation, we obtain(13 + 23 + · · ·+ k3

)x3 + (1 + 2 + · · ·+ k)2 y4 = (1 + 2 + · · ·+ k)4 y4 ⇐⇒ x3 = ay4

where a = (1 + 2 + · · ·+ k)2 − 1 =(k − 1) (k + 2)

(k2 + k + 2

)4

. Since x = a7n4, y = a5n3 for

any positive integer n , then x3 = ay4. Hence

x1 = a7n4, x2 = 2a7n4, . . . , xk = ka7n4, xk+1 = (1 + 2 + · · ·+ k) a10n6, xk+2 = (1 + 2 + · · ·+ k) a5n3

for any positive integer n is a solution to the original equation and obviously, x1 < x2 < · · · <xk+1.

Second solution by the authors

For any positive integer n we have the well-known identity:

13 + 23 + · · ·+ n3 + (n+ 1)3 + · · ·+ (n+ k)3 =

((n+ k)(n+ k + 1)

2

)2

,

that is (n(n+ 1)

2

)2

+ (n+ 1)3 + · · ·+ (n+ k)3 =

((n+ k)(n+ k + 1)

2

)2

.

Consider the positive integers n such that the triangular number tn+k = (n+k)(n+k+1)2 is a per-

fect squares. There are infinitely many such integers since the relation tn+k = u2 is equivalentto the Pell’s equation (2n+ 2k+ 1)2− 2u2 = 1. The fundamental solution to this Pell equationis (3, 2), hence all these integers are given by the sequence (ns), where

2ns + 2k + 1 + us√

2 = (3 + 2√

2)s,

for s big enough such that ns ≥ 1.

Mathematical Reflections 5 (2010) 16

Page 177: Eserciziario Di Analisi Matematica 2

We can take

x1 = ns + 1, · · · , xk = ns + k, xk+1 =ns(ns + 1)

2, xk+2 = us.

It is clear that for s big enough we have ns ≥ 1 and n(n+1)2 > n + k, hence we get an infinite

family of solutions.

Also solved by Daniel Lasaosa, Universidad Publica de Navarra, Spain.

Mathematical Reflections 5 (2010) 17

Page 178: Eserciziario Di Analisi Matematica 2

Undergraduate problems

U169. Sequences (xn)n≥1 and (yn)n≥1 are defined by x1 = 2, y1 = 1, and xn+1 = x2n + 1, yn+1 = xnynfor all n. Prove that for all n ≥ 1

xnyn

<651

250.

Proposed by Dorin Andrica, Babes-Bolyai University, Cluj-Napoca, Romania

Solution by Andrea Ligori, Universita di Roma “Tor Vergata”, Roma, Italy

The inequality holds for 1 ≤ n ≤ 4 because x1/y1 = 2, x2/y2 = 5/2, x3/y3 = 13/5, andx4/y4 = 677/260,. Moreover, for n ≥ 5

xnyn

=x2n−1 + 1

xn−1yn−1=xn−1yn−1

+1

yn=x4y4

+

n∑k=5

1

yk<

677

260+

∞∑k=5

1

yk.

Hence it suffices to show that

∞∑k=4

1

yk≤ 651

250− 677

260=

1

6500.

We have that for n ≥ 1

xn ≥ x2n−1 ≥ x4n−2 ≥ x8n−3 ≥ · · · ≥ 22n−1

and,

yn = xn−1yn−1 ≥ 22n−2

yn−1 ≥ 22n−2+2n−3

yn−2 ≥ · · · ≥ 22n−2+2n−3+···+1 = 22

n−1−1.

This means that yn ≥ 22n−1−1 ≥ 23n = 8n for n ≥ 5, and

∞∑k=5

1

yk≤∞∑k=5

1

8k=

1

7 · 84=

1

28672<

1

6500.

Also solved by Arkady Alt, San Jose, California, USA; Daniel Lasaosa, Universidad Publicade Navarra, Spain.

Mathematical Reflections 5 (2010) 18

Page 179: Eserciziario Di Analisi Matematica 2

U170. Sequences (xn)n≥1 , (yn)n≥1 are defined as follows: x1 = α, y1 = β, with |α| 6= |β| 6= 0, and

xn+1 = max (xn − yn, xn + yn) ,

yn+1 = min (xn − yn, xn + yn) ,

for all n ≥ 1. Prove thatlimn→∞

xn = limn→∞

yn =∞.

Proposed by Bogdan Enescu, B.P.Hasdeu College, Buzau, Romania

Solution by Michel Bataille, France

It is easily seen that (yn) has no limit (finite or not) if α = 0, β = 1. We will show the followingresults: limn→∞ xn = limn→∞ yn = ∞ if α 6= 0 and |α| 6= |β|, limn→∞ xn = limn→∞ yn = 0 ifα = β = 0, and otherwise, limn→∞ xn =∞ and (yn) has no limit either finite or infinite.For all real numbers a, b, we have max(a, b) = 1

2(a+b+ |a−b|) and min(a, b) = 12(a+b−|a−b|).

It follows that for all n ≥ 1,

xn+1 = xn + |yn|, yn+1 = xn − |yn| (1).

First, suppose |α| < |β| that is, −|β| < α < |β|. Then, using (1), x2 = α+ |β|, y2 = α− |β| <0, x3 = 2|β|, y3 = 2α so that x4 = 2(|β| + |α|), y4 = 2(|β| − |α|), x5 = 4|β|, y5 = 4|α|. Aneasy induction yields x2n = 2n−1(|β| + |α|), y2n = 2n−1(|β| − |α|), x2n+1 = 2n|β|, y2n+1 =2n|α| for all integer n ≥ 2. Thus, limn→∞ xn = ∞ (since limn→∞ 2n = ∞ and |β| > 0)and limn→∞ yn = ∞ if α 6= 0 while (yn) has no limit (finite or not) if α = 0 (since thenlimn→∞ y2n =∞ and limn→∞ y2n+1 = 0).If α > |β|, with the help of (1) again, we obtain

x2n−1 = 2n−1α, y2n−1 = 2n−1|β|, x2n = 2n−1(|β|+ α), y2n = 2n−1(α− |β|)

for all n ≥ 2 and limn→∞ xn = limn→∞ yn =∞ follows.If α < −|β|, similarly,

x2n+1 = 2n|α|, y2n+1 = 2n|β|, x2n+2 = 2n(|α|+ |β|), y2n+2 = 2n(|α| − |β|)

for all n ≥ 2 and limn→∞ xn = limn→∞ yn =∞ again.Lastly, in the same way it is readily seen that if α and β are equal or opposite nonzero realnumbers, then limn→∞ xn = ∞ while (yn) has no limit (finite or not). Also, we clearly havelimn→∞ xn = limn→∞ yn = 0 if α = β = 0 and the proof is complete.

Also solved by Arkady Alt, San Jose, California, USA; Daniel Lasaosa, Universidad Publicade Navarra, Spain; Perfetti Paolo, Dipartimento di Matematica, Universita degli studi di TorVergata Roma, Italy; Andrea Ligori, Universita di Roma “Tor Vergata”, Roma, Italy.

Mathematical Reflections 5 (2010) 19

Page 180: Eserciziario Di Analisi Matematica 2

U171. Let A be a matrix of order n such that A10 = On. Prove that

1

4A4 +

1

2A3 +

1

2A2 +A+ In

is invertible.

Proposed by Titu Andreescu, University of Texas at Dallas, USA

Solution by Arkady Alt, San Jose, California, USA

Let f (A) = −1

4A3 − 1

2A3 − 1

2A− In and B = Af (A) . Since for any two polynomial

P (x) and Q (x) and any matrix A holds P (A)Q (A) = Q (A)P (A) then B10 = (Af (A))10 =

A10f (A)10 = 0n.Hence,

(1

4A4 +

1

2A3 +

1

2A2 +A+ In

)(In +B +B2 + · · ·+B9

)=

(In −B)(In +B +B2 + · · ·+B9

)= In −B10 = In.Thus, In +B +B2 + · · ·+B9 is

inverse matrix for1

4A4 +

1

2A3 +

1

2A2 +A+ In.

Also solved by Michel Bataille, France; Daniel Lasaosa, Universidad Publica de Navarra, Spain;Moubinool Omarjee , Paris France; Andrea Ligori, Universita di Roma “Tor Vergata”, Roma,Italy.

Mathematical Reflections 5 (2010) 20

Page 181: Eserciziario Di Analisi Matematica 2

U172. Let f : R → R be a strictly increasing invertible function such that for all x ∈ R, f(x) +f−1(x) = ex − 1 for all x ∈ R. Prove that f has at most one fixed point.

Proposed by Samin Riasat, University of Dhaka, Bangladesh

Solution by Emanuele Natale, Universita di Roma “Tor Vergata”, Roma, Italy

If x0 is a fixed point then f(x0) = f−1(x0) = x0 and

f(x0) + f−1(x0) = 2x0 = ex0 − 1

which has two solutions: 0 and a some c > 0. So, if f has more than one fixed point thenit has just these two fixed points: 0 and c. Since f is a strictly increasing invertible functionthen f−1 is strictly increasing too.

Take x < 0 then f(x) < f(0) = 0, f−1(x) < f−1(0) = 0. Moreover f(x) < x otherwisef(x) > x and x > f−1(x). In both cases

ex − 1 = f(x) + f−1(x) < x+ 0 = x

which is a contradiction because for x < 0 we have that x < ex − 1.

Therefore c is the only possible fixed point.

Also solved by Michel Bataille, France; Daniel Lasaosa, Universidad Publica de Navarra,Spain.

Mathematical Reflections 5 (2010) 21

Page 182: Eserciziario Di Analisi Matematica 2

U173. Let θ be a real number. Prove that

n−1∑k=0

sin(2kπn − θ

)3 + 2 cos

(2kπn − θ

) =(−1)nn sin(nθ)

5F 2n + 4(−1)n sin2

(nθ2

) ,where Fn denotes the nth Fibonacci number.

Proposed by Javier Buitrago, Universidad National de Colombia, Colombia

Solution by G. C. Greubel, Newport News, USA

Let

Pn =sin(2kπn − θ

)3 + 2 cos

(2kπn − θ

) . (1)

It is readily clear that

Pn =1

2

d

[ln

(3 + 2 cos

(2kπ

n− θ))]

. (2)

From this we have

n−1∑k=0

Pn =1

2

d

n−1∑k=0

[ln

(3 + 2 cos

(2kπ

n− θ))]

=1

2

d

dθln Sn , (3)

where

Sn =n−1∏k=0

[3 + 2 cos

(2kπ

n− θ)]

. (4)

Taking n = 1 we have

S1 =

0∏k=0

[3 + 2 cos (2kπ − θ)]

= 3 + 2 cos θ

= 5− 4 sin2

2

). (5)

In the case when n = 2 we have

S2 =1∏

k=0

[3 + 2 cos

(2kπ

2− θ)]

= (3 + 2 cos θ)(3 + 2 cos(π − θ)= 9− 4 cos2 θ

= 5 + 4 sin2 θ. (6)

Mathematical Reflections 5 (2010) 22

Page 183: Eserciziario Di Analisi Matematica 2

The two cases of n given here have the general form

Sn = 5F 2n + 4(−1)n sin2

(nθ

2

). (7)

Further values of n lead to the same result. Now using (7) in (4) leads to

n−1∑k=0

Pn =1

2

d

dθln

5F 2

n + 4(−1)n sin2

(nθ

2

)=

1

2Sn· ddθ

[5F 2

n + 4(−1)n sin2

(nθ

2

)]=

1

2Sn[2(−1)nn sin(nθ)]

=(−1)nn sin(nθ)

Sn. (8)

By combining the components of this result leads to the expression

n−1∑k=0

sin(2kπn − θ

)3 + 2 cos

(2kπn − θ

) =(−1)n · n · sin(nθ)

5F 2n + 4(−1)n sin2

(nθ2

) . (9)

Also solved by Michel Bataille, France; Daniel Lasaosa, Universidad Publica de Navarra,Spain.

Mathematical Reflections 5 (2010) 23

Page 184: Eserciziario Di Analisi Matematica 2

U174. Let p be a prime. A linear recurrence of degree n in Fp is a sequence akk≥0 in Fp satisfyinga relation of the form

ai+n = cn−1ai+n−1 + · · ·+ c1ai+1 + c0ai for all i ≥ 0,

where c0, c1, . . . , cn−1 ∈ Fp and c0 6= 0.

(a) What is the maximal possible period of a linear recurrence of degree n in Fp?(b) How many distinct linear recurrences of degree n have this maximal period?

Proposed by Holden Lee, Massachusetts Institute of Technology

Solution by Holden Lee, Massachusetts Institute of Technology

(a) pn−1. There are only pn possibilities for n consecutive terms of the sequence. If (0, . . . , 0)appears then nothing else can appear. (Pf. of existence below)

(b) Suppose a recurrence has period pn − 1. Note that by Vandermonde’s determinant thefunctions fα(t) = αt where α ∈ F×pn are a basis for the space of functions Z/(pn − 1)Z→Fpn . Then the general term can be written as

ak = b1αk1 + · · ·+ bmα

km.

for some m, distinct αi ∈ F×pn , and b1, . . . , bm ∈ F×pn . By plugging into the recurrence, weget that all the αi must be roots of the characteristic polynomial. If αi has degree di then

αpdi−1i = 1. Thus in order for the period to be pn − 1 the characteristic polynomial must

be irreducible (so the roots have degree n). The roots have the same order; the commonorder is the least N so that the characteristic polynomial divides xN −1. Again using thefact the fα are linearly independent we can show that the period has to be this commonorder and not less. Hence the characteristic polynomial has to divide the cyclotomicpolynomial φpn−1(x). Each factor of it in Fp has degree n, so there are ϕ(pn − 1)/nvalid polynomials. For each of these we can choose a0, . . . , an−1 in pn − 1 different ways.(Note a nonzero sequence cannot satisfy two recurrences corresponding to two different

irreducible characteristic polynomials.) Answer: (pn−1)ϕ(pn−1)n .

Mathematical Reflections 5 (2010) 24

Page 185: Eserciziario Di Analisi Matematica 2

Olympiad problems

O169. Let a, b, c, d be real numbers such that

(a2 + 1)(b2 + 1)(c2 + 1)(d2 + 1) = 16.

Prove that−3 ≤ ab+ bc+ cd+ da+ ac+ bd− abcd ≤ 5.

Proposed by Titu Andreescu, University of Texas at Dallas, USA, and Gabriel Dospinescu,Ecole Normale Superieure, France

Solution by Michel Bataille, France

Consider the complex number Z = (1 + ia)(1 + ib)(1 + ic)(1 + id). An easy calculation yields:

Re(Z) = 1− (ab+ bc+ cd+ da+ ac+ bd) + abcd and |Z|2 = (a2 + 1)(b2 + 1)(c2 + 1)(d2 + 1).

Now, the hypothesis gives |Z| = 4 so that the inequality |Re(Z)| ≤ |Z| writes as

|(ab+ bc+ cd+ da+ ac+ bd− abcd)− 1| ≤ 4

that is,−3 ≤ ab+ bc+ cd+ da+ ac+ bd− abcd ≤ 5,

as required.

Also solved by Daniel Lasaosa, Universidad Publica de Navarra, Spain; Sayan Mukherjee;Pedro H. O. Pantoja, Natal-RN, Brazil.

Mathematical Reflections 5 (2010) 25

Page 186: Eserciziario Di Analisi Matematica 2

O170. Let a and b be positive integers such that a does not divide b and b does not divide a. Provethat there is an integer x such that 1 < x ≤ a and both a and b divide xφ(b)+1 − x, where φ isEuler’s totient function.

Proposed by Vahgan Aslanyan, Yerevan, Armenia

Solution by Sayan Mukherjee

We have, x(xϕ(b) − 1

)is divisible by x and when gcd(b, x) = 1; applying Euler’s theorem this

is also divisible by b.

So, letting x =a

gcd(a, b)we get, using gcd

(a

gcd(a, b), b

)= 1, that

a

gcd(a, b)

[(a

gcd(a, b)

)ϕ(b)− 1

]≡ 0 mod

(a

gcd(a, b)· b)≡ 0 mod lcm(a, b).

Therefore the required x isa

gcd(a, b)since this also satisfies 1 < x ≤ a.

Also solved by Daniel Lasaosa, Universidad Publica de Navarra, Spain; Tigran Hakobyan,Armenia.

Mathematical Reflections 5 (2010) 26

Page 187: Eserciziario Di Analisi Matematica 2

O171. Prove that in any convex quadrilateral ABCD,

sin

(A

3+ 60

)+ sin

(B

3+ 60

)+ sin

(C

3+ 60

)+ sin

(D

3+ 60

)≥ 1

3(8 + sinA+ sinB + sinC + sinD).

Proposed by Titu Andreescu, University of Texas at Dallas, USA

Solution by Perfetti Paolo, Dipartimento di Matematica, Universita degli studi di Tor VergataRoma, Italy

We employ the theory of Schur–concave functions, see A.W.Marshall, I.Olkin, Inequalities:Theory of Majorization and Its Applications, Academic Press, 1979.

Let’s recall few notions about the theory as exposed in chapters 3 and 4 of the cited book.

For any x = (x1, x2, . . . , xn) ∈ Rn let x[1] ≥ x[2] ≥ . . . ≥ x[n] denote the components of x inthe decreasing order. We write

x ≺ y if

k∑i=1

x[i] ≤k∑i=1

y[i] k = 1, 2, . . . , n− 1,

n∑i=1

x[i] =

n∑i=1

y[i]

When x ≺ y we say that x is majorized by y or that y majorizes x. A typical example is( 1

n,

1

n, . . . ,

1

n

)≺( 1

n− 1,

1

n− 1, . . . ,

1

n, 0)≺ (

1

2,1

2, 0, . . . , 0) ≺ (1, 0, . . . , 0)

Two other examples are (2s

4,2s

4,2s

4,2s

4

)≺(a1, a2, a3, a4

)where (a1, . . . , a3) are the sides of a quadrilateral and(π

2,π

2,π

2,π

2

)≺ (A,B,C,D) ≺ (π, π, 0, 0)

where A,B,C,D, are the angles of a quadrilateral.

Definition A function ϕ : D ⊆ Rn → R is said to be “Schur–convex” (concave) if for anyx ∈ D, y ∈ D

x ≺ y =⇒ ϕ(x) ≤ ϕ(y) (ϕ(x) ≥ ϕ(y))

Now let D be a subset of Rn such that: i) x ∈ D =⇒ x′ ∈ D if the components of x′ arepermutated respect to the components of x. ii) D is convex and has non–empty interior.We have the theorem (the proof is at page 57 of [1]; see also the remark [A.4.a]).

Mathematical Reflections 5 (2010) 27

Page 188: Eserciziario Di Analisi Matematica 2

Theorem Let D be the set x ∈ R4 : 0 ≤ x1 ≤ a, 0 ≤ x2 ≤ a, 0 ≤ x3 ≤ a, 0 ≤ x4 ≤ a andlet ϕ : D → R be a continuously differentiable function. Necessary and sufficient condition forϕ to be Schur–convex (concave) are

1) ϕ is symmetric

2) (x1 − x2)(ϕx1 − ϕx2) ≥ 0 (≤ 0) for any x ∈ D.

To apply the theorem to our inequality we define D = x ∈ R4, 0 ≤ xi ≤ π and ϕ : D → R,

ϕ(A,B,C,D) = sin

(A+ π

3

)+ sin

(B + π

3

)+ sin

(C + π

3

)+ sin

(D + π

3

)+

−1

3(8 + sinA+ sinB + sinC + sinD)

and we show that (A − B)(ϕA − ϕB) ≥ 0 if A ≥ B namely the function ϕ is Schur–convex.This implies that

ϕ(A,B,C,D) ≥ ϕ(π/2, π/2, π/2, π/2) = 0

and the result is achieved.

We have to prove that ϕA − ϕB ≥ 0 if A ≥ B.

ϕA − ϕB = cosA+ π

3− cos

B + π

3− (cosA− cosB)

and it’s nonnegative since the function cosx is strictly decreasing between x = 0 and x = πand A−B > A+π

3 − B+π3 = A−B

3

Also solved by Daniel Lasaosa, Universidad Publica de Navarra, Spain; Sayan Mukherjee.

Mathematical Reflections 5 (2010) 28

Page 189: Eserciziario Di Analisi Matematica 2

O172. Prove that if a 7 × 7 square board is covered by 38 dominoes such that each domino coversexactly two squares of the board, then it is possible to remove one domino after which theremaining 37 cover the board.

Proposed by Nairi Sedrakyan, Yerevan, Armenia

Solution by Daniel Lasaosa, Universidad Publica de Navarra, Spain

Consider a graph with 49 vertices, representing each vertex one of the squares in the board,where two vertices are joined by and edge iff their corresponding squares are covered by thesame domino. Assume that the board can be covered by 38 dominoes so that no matter whichdomino is removed, at least one square becomes uncovered. Therefore, no edge in the graphmay join two vertices, such that both of them are ends of other edges. Consider two vertices V1and V2, joined by an edge. Clearly, one of them (wlog V2) cannot have any other edges. If V1has other vertices joined to it through edges, these must also have no other edges connectingthem to further vertices in the graph, or all points that may be reached from V1 through edgesof the graph form a tree with root V1 and leaves V2, V3, . . . , no other vertices involved. Clearly,no vertex can be joined to more than 4 other vertices, otherwise by Dirichlet’s principle, twodominoes would cover the same two squares, and one of them could be removed leaving all49 squares covered. Therefore, the graph may be decomposed in disjoint subgraphs, eachone of them with k vertices, such that k − 1 of them are joined by an edge to the remainingvertex, no other vertices or edges are present in each subgraph, ie, denoting nk the number ofsubgraphs with k vertices (where clearly k = 2, 3, 4, 5) we have 2n2 +3n3 +4n4 +5n5 = 49 andn2 + 2n3 + 3n4 + 4n5 = 38. Assume that n5 = 4− d where d ≥ 0, hence n2 + n3 + n4 = 7 + d,whereas n2 + 2n3 + 3n4 = 22 + 4d > 3n2 + 3n3 + 3n4, absurd, hence n5 ≥ 5. Now, on theboard, a graph with k = 5 would represent a cross-shaped pentamino with one square joinedto its four neighbours, hence no more than 2 squares on each side of the board, and none ofits corners, can be covered by a domino involved in each one of these n5 subgraphs. It followsthat the n5 subgraphs cover no more than 33 squares, ie n5 ≤ 6.

Assume that n5 = 5, then 2n2 + 3n3 + 4n4 = 24 and n2 + 2n3 + 3n4 = 18. Note now that

0 = 3 · 24− 4 · 18 = 2n2 + n3,

and since n2, n3 ≥ 0, we have n2 = n3 = 0, n4 = 6. Note however that each corned must becovered by one of the n4 subgraphs because it cannot be covered by one of the n5 subgraphs,and each n4 subgraph is a T-shaped tetramino, hence each one of the four corners of the boardmust be covered by one of the two squares forming the horizontal bar of the T, leaving one”prisoner” square, limiting on one side with the side of the board, and on two sides with sidesof this T-shaped tetramino. This square must therefore be covered, next to each corner square,by another one of the n4 subgraphs, hence n4 ≥ 8, contradiction. Therefore n5 = 6.

Assume finally that n5 = 6, then 2n2 + 3n3 + 4n4 = 19 and n2 + 2n3 + 3n4 = 14, or

1 = 3 · 19− 4 · 14 = 2n2 + n3,

or since n2, n3 ≥ 0, it follows that n2 = 0, n3 = 1, and n4 = 4. As in the case of n5 = 5, itfollows that one of the four corners of the board must be covered by the n3 subgraph, while

Mathematical Reflections 5 (2010) 29

Page 190: Eserciziario Di Analisi Matematica 2

the other three must be covered by n4 subgraphs, yielding n4 ≥ 6 (two of the n4 subgraphsmust be placed on or close to each corner), contradiction again. The conclusion follows.

It is possible however to cover the board with n5 = 3 pentaminoes, n4 = 7 tetraminoes andn3 = 2 trominoes (leaving n2 = 0), or the board can be covered by 37 dominoes, such that nodomino can be removed without leaving uncovered squares.

Also solved by G.R.A.20 Math Problems Group, Roma, Italy.

Mathematical Reflections 5 (2010) 30

Page 191: Eserciziario Di Analisi Matematica 2

O173. Find all triples (x, y, z) of integers such that

x3 + y3 + z3

3− xyz = 2010 max 3

√x− y, 3

√y − z, 3

√z − x.

Proposed by Titu Andreescu, University of Texas at Dallas, USA and Gabriel Dospinescu,Ecole Normale Superieure, France

Solution by Daniel Lasaosa, Universidad Publica de Navarra, Spain

Denote u = 3√x− y, v = 3

√y − z and w = 3

√z − x, hence clearly u3 + v3 + w3 = 0, which is

known to have integral solutions only if at least one of u, v, w is zero, while

x3 + y3 + z3

3− xyz =

(x+ y + z)(x2 + y2 + z2 − xy − yz − zx)

3=

=(x+ y + z)(u6 + v6 + w6)

6,

(x+ y + z)(u6 + v6 + w6) = 12060u,

where we have assume wlog by cyclic symmetry in the variables that u = maxu, v, w. Notealso that u ≥ 0, with equality iff u = v = w = 0, since if u ≤ 0, then 0 = u3 + v3 + w3 ≤ 0,equality must hold, hence u = v = w = 0. We then have two possible cases:

• If u = 0, then u = v = w = 0, or x = y = z. Note that any x = y = z results in bothsides of the proposed equation being zero, hence (x, y, z) = (r, r, r) is a solution for anyinteger r.

• If u > 0, and since one of v, w is necessarily zero, while v3 + w3 = −u3, then the otherone necessarily equals −u, ie (u, v, w) is some permutation of (u, 0,−u), yielding

(x+ y + z)u5 = 6030 = 2 · 32 · 5 · 67,

or since the RHS is not divisible by any fifth power, u = 1, and x+ y + z = 6030, wherex = y + 1. Now, since one of v, w is zero, either z = y or z = x, yielding respectively3y+ 1 = 6030 or 3y+ 2 = 6030, impossible since 6030 is a multiple of 3, but 3y+ 1, 3y+ 1are not.

It follows that (x, y, z) is an integral solution of the proposed equation if x = y = z = r forsome integer r.

Mathematical Reflections 5 (2010) 31

Page 192: Eserciziario Di Analisi Matematica 2

O174. The point O is considered inside of the convex quadrilateral ABCD of area S. Supposethat K,L,M,N are interior points of the sides AB,BC,CD,DA, respectively. If OKBL andOMDN are parallelograms of areas S1 and S2, respectively, prove that

(a)√S1 +

√S2 < 1.25

√S;

(b)√S1 +

√S2 < C0

√S, where C0 = max0<α<π

4

sin(2α+π4)

cosα .

Proposed by Nairi Sedrakyan, Yerevan, Armenia

Solution by the authors

Without loss of generality we can suppose that points O and D are not on different sides ofthe line AC. Let SABC = a, SACD = b, and SOAC = x. We have SOKB = SOBL = SKLB = S1

2and

SOKBSOAB

· SOBLSOBC

=KB

AB· BLBC

=SKBLSABC

,

frpm which we find S1 = 2SOAB ·SOBCa . Similarly, we obtain S2 = 2SOAD·SOCD

b ; hence

√S1 +

√S2 ≤

SOAB + SOBC√2a

+SOAD + SOCD√

2b=a+ x√

2a+b− x√

2b=

√a+√b√

2−√a−√b√

2abx.

If a ≥ b, then√S1 +

√S2 ≤

√a+√b√

2≤√a+ b =

√S. If a < b, then the point O can not be

outside the parallelogram ABCE, thus x ≤ a, so that

√S1 +

√S2 ≤

√a+√b√

2−√a−√b√

2aba =

b+√

2ab− a√2n

.

Let ab = tan2 α, where α

[0, π4

]. Then b+

√2ab−a√2b

:√a+ b =

sin(2α+π4 )

cosα ≤ C0. Consequently√S1+√S2 ≤ b+

√2ab−a√2b

≤ C0

√S. When α = π

4 ,sin(2α+π/4)

cosα = 1, that is C0 ≥ 1. Thus in all cases√S1 +

√S2 ≤ C0

√S. If the quadrilateral satisfies the following conditions: AB = BC,AD =

CD, SABCtanα0where C0 =

sin(2α0+π4 )

cosα0and ABCO is a parallelogram, then

√S1 +

√S2 = C0

√S.

a) To prove the inequality it is sufficient to prove that when 0 ≤ α ≤ π4 , sin

(2α+ π

4

)<

1.25 cosα. Indeed, let φ ∈[0, π4

]and cosφ = 4

5 , then, if 0 ≤ α < φ, then sin(2α + φ) ≤ 1 =54 cosφ. And if φ ≤ α ≤ π

4 , then tanφ = 34 >√

2− 1 = tan π8 ; hence φ > π

8 and

sin(

2α+π

4

)≤ sin

(2φ+

π

4

)=

√2

2· 31

25<

√2

2· 5

4≤ 1.25 cosα.

Remark. Using derivatives it is possible to prove that

tanα0 =3

√√2 + 1− 3

√√2− 1 = 0.59 . . . while C0 = 1.11 . . . .

Mathematical Reflections 5 (2010) 32

Page 193: Eserciziario Di Analisi Matematica 2

Junior problems

J175. Let a, b ∈ (0, π2 ) such that sin2 a+ cos 2b ≥ 12 sec a and sin2 b+ cos 2a ≥ 1

2 sec b. Prove that

cos6 a+ cos6 b ≥ 12.

Proposed by Titu Andreescu, University of Texas at Dallas, USA

Solution by Prithwijit De, HBCSE, India

We will use the following well-known trigonometric identities

(a) sin2 x = 1− cos2 x,

(b) cos 2x = 2 cos2 x− 1,

(c) secx =1

cosx.

The inequalities can be written as

2 cos2 b cos a− cos3 a ≥ 12

(1)

and2 cos2 a cos b− cos3 b ≥ 1

2. (2)

The signs of the inequalities are preserved because cosx is positive when x ∈(

0,π

2

). Now by

squaring both sides of (1) and (2) and adding them we get

cos6 a+ cos6 b ≥ 12

.

Also solved by Arkady Alt, San Jose, California, USA; Daniel Lasaosa, Universidad Publicade Navarra, Spain; Perfetti Paolo, Dipartimento di Matematica, Universita degli studi di TorVergata Roma, Italy; Tigran Hakobyan, Armenia.

Mathematical Reflections 6 (2010) 1

Page 194: Eserciziario Di Analisi Matematica 2

J176. Solve in positive real numbers the system of equationsx1 + x2 + · · ·+ xn = 11x1

+ 1x2

+ · · ·+ 1xn

+ 1x1x2···xn

= n3 + 1.

Proposed by Neculai Stanciu, George Emil Palade Secondary School, Buzau, Romania

Solution by Tigran Hakobyan, Armenia

We have1x1

+1x2

+ · · ·+ 1xn≥ n2

x1 + x2 + · · ·+ xn= n2

and1

x1x2 · · ·xn≥ 1(

x1+x2+···+xnn

)n = nn.

Thus,n3 + 1 ≥ nn + n2

which implies that n ≤ 2. If n = 1 we get a contradiction. For n = 2 we getx1 + x+ 2 = 11x1

+ 1x2

+ 1x1x2

= 9

which is (n, x1, x2) ∈(

2, 13 ,

23

),(2, 2

3 ,13

).

Also solved by Arkady Alt, San Jose, California, USA; Daniel Lasaosa, Universidad Publicade Navarra, Spain; Perfetti Paolo, Dipartimento di Matematica, Universita degli studi di TorVergata Roma, Italy; Lorenzo Pascali, Universita di Roma “La Sapienza”, Roma, Italy.

Mathematical Reflections 6 (2010) 2

Page 195: Eserciziario Di Analisi Matematica 2

J177. Let x, y, z be nonnegative real numbers such that ax + by + cz ≤ 3abc for some positive realnumbers a, b, c. Prove that√

x+ y

2+

√y + z

2+

√z + x

2+ 4√xyz ≤ 1

4(abc+ 5a+ 5b+ 5c).

Proposed by Titu Andreescu, University of Texas at Dallas, USA

Solution by the author

From the given condition,

3a ≥ ax

bc+y

c+z

b

3b ≥ x

c+by

ca+z

a

3c ≥ x

b+y

a+cz

ab.

Then

3(a+ b+ c) ≥ x+ y

c+y + z

a+z + x

b+(ax

bc+by

ca+cz

ab

)hence

abc+ 5(a+ b+ c) ≥(x+ y

c+ 2c

)+(y + z

a+ 2a

)+(z + x

b+ 2b

)+(abc+

ax

bc+by

ca+cz

ab

)≥ 2√

2(x+ y) + 2√

(y + z) + 2√

(z + x) + 4 4√xyz

and the conclusion follows. The equality holds if and only if x+y = 2c2, y+z = 2a2, z+x = 2b2

and axbc = by

ca = czab = abc. This implies b2c2 = c2a2 = 2c2, c2a2 + a2b2 = 2a2, a2b2 + b2c2 = 2b2,

that is b2 + a2 = c2 + b2 = a2 + c2 = 2, implying a = b = c = 1 and x = y = z = 1. Ifa = b = c = x = y = z = 1, the equality, as well as the condition of the problem, hold

Mathematical Reflections 6 (2010) 3

Page 196: Eserciziario Di Analisi Matematica 2

J178. Find the sequences of integers (an)n≥0 and (bn)n≥0 such that

(2 +√

5)n = an + bn1 +√

52

for each n ≥ 0.

Proposed by Dorin Andrica, Babes-Bolyai University, Cluj-Napoca, Romania

First solution by Arkady Alt, San Jose, California, USA

Let

pn =

(2 +√

5)n

+(2−√

5)n

2and

qn =

(2 +√

5)n − (2−√5

)n2√

5

for n = 1, 2, . . . . Then(2 +√

5)n

= pn + qn√

5, n = 0, 1, 2, . . . and both obtained sequencessatisfy the same recurrence

xn+1 = 4xn + xn−1, n ∈ N (1)

with initial conditions p0 = 1, p1 = 2, q0 = 0, q1 = 1. It is clear that (pn)n≥0 and (qn)n≥0 aresequences of nonnegative integers and since

an + bn1 +√

52

= pn + qn√

5 ⇐⇒ an +bn2

+bn2

√5 = pn + qn

√5

⇐⇒

an +

bn2

= pnbn2

= qn

⇐⇒an = pn − qnbn = 2qn

, n ∈ N ∪ 0

we have that (an)n≥0 and (bn)n≥0 are sequences of integers and can be defined independentlyby recurrence (1) with initial conditions a0 = 1, a1 = 1, b0 = 0, b1 = 2. In explicit form

bn =

(2 +√

5)n − (2−√5

)n√

5, an =

(√5− 1

) (2 +√

5)n

+(√

5 + 1) (

2−√

5)n

2√

5.

Second solution by Daniel Lasaosa, Universidad Publica de Navarra, Spain

Using Newton’s binomial formula, exchanging√

5 by −√

5 in the first term results in exchanging√5 by −

√5 in the second term, i.e., (2+

√5)n+(2−

√5)n = 2an+bn and (2+

√5)n−(2−

√5)n =√

5bn, yielding

bn =(2 +

√5)n − (2−

√5)n√

5, an =

(√

5− 1)(2 +√

5)n + (√

5 + 1)(2−√

5)n

2√

5.

Mathematical Reflections 6 (2010) 4

Page 197: Eserciziario Di Analisi Matematica 2

The fact that an, bn are integers for all n may be easily proved considering that they aresolutions of the recursive equations an = 4an−1 + an−2 and bn = 4bn−1 + bn−2, with initialconditions a0 = a1 = 1 and b0 = 0, b1 = 2.

Also solved by Ivan Dinkov Gerganov, P. R. Slaveikov Secondary School, Bulgaria; LorenzoPascali, Universita di Roma “La Sapienza”, Roma, Italy; Tigran Hakobyan, Armenia.

Mathematical Reflections 6 (2010) 5

Page 198: Eserciziario Di Analisi Matematica 2

J179. Solve in real numbers the system of equations(x+ y)(y3 − z3) = 3(z − x)(z3 + x3)(y + z)(z3 − x3) = 3(x− y)(x3 + y3)(z + x)(x3 − y3) = 3(y − z)(y3 + z3)

Proposed by Titu Andreescu, University of Texas at Dallas, USA

Solution by Lorenzo Pascali, Universita di Roma “La Sapienza”, Roma, Italy

Without loss of generality assume that x = 0, then it is clear that y = z = 0. Because thegiven system is symmetric we can assume that x, y, z 6= 0. Assume that x = y, then x = z ory = −z. If we assume that y = −z then the first equation becomes 4x4 = 6x4, contradiction.Now x 6= y 6= z 6= 0 and after multiplying all three equations we get

3(x2 − xy + y2)3(y2 − yz + z2)3(z2 − zx+ x2)(x2 + xy + y2)(y2 + yz + z2)(z2 + zx+ x2)

= 1

which can be written as(1− 2xy

(x− y)2 + 3xy

)(1− 2yz

(y − z)2 + 3yz

)(1− 2zx

(z − x)2 + 3zx

)=

127.

It is not difficult to see that each factor of the LHS is greater than 13 which leads to a contra-

diction. So the only possible solution is x = y = z.

Also solved by Arkady Alt, San Jose, California, USA; Daniel Lasaosa, Universidad Publica deNavarra, Spain; Tigran Hakobyan, Armenia.

Mathematical Reflections 6 (2010) 6

Page 199: Eserciziario Di Analisi Matematica 2

J180. Let a, b, c, d be distinct real numbers such that

13√a− b

+1

3√b− c

+1

3√c− d

+1

3√d− a

6= 0.

Prove that 3√a− b+ 3

√b− c+ 3

√c− d+ 3

√d− a 6= 0.

Proposed by Dorin Andrica, Babes-Bolyai University, Cluj-Napoca, Romania

Solution by Anthony Erb Lugo

Let 3√a− b, 3

√b− c, 3

√c− d, 3

√d− a by w, x, y, z, respectively and let S = w + x + y + z. So

that,

w3 + x3 + y3 + z3 = 0(∗) and1w

+1x

+1y

+1z6= 0(∗∗)

Since a, b, c and d are distinct, we can conclude that w, x, y, z 6= 0, and thus,

wxy + wxz + wyz + xyz = wxyz

(1w

+1x

+1y

+1z

)6= 0(∗ ∗ ∗)

Furthermore, using (*), we have that,

S3 = 6(wxy + wxz + wyz + xyz) + 3(w2 + x2 + y2 + z2)SS(S2 − 3(w2 + x2 + y2 + z2)) = 6(wxy + wxz + wyz + xyz)

Which implies that S 6= 0, otherwise the right hand side would be 0 which is a contradictionof (∗ ∗ ∗), and we’re done.

Also solved by Arkady Alt, San Jose, California, USA; Daniel Lasaosa, Universidad Publicade Navarra, Spain; Lorenzo Pascali, Universita di Roma “La Sapienza”, Roma, Italy; PerfettiPaolo, Dipartimento di Matematica, Universita degli studi di Tor Vergata Roma, Italy; TigranHakobyan, Armenia; Daniel Campos Salas, Costa Rica.

Mathematical Reflections 6 (2010) 7

Page 200: Eserciziario Di Analisi Matematica 2

Senior problems

S175. Let p be a prime. Find all integers a1, . . . , an such that a1 + · · ·+ an = p2− p and all solutionsto the equation pxn + a1x

n−1 + · · ·+ an = 0 are nonzero integers.

Proposed by Titu Andreescu, University of Texas at Dallas, USA and Dorin Andrica,Babes-Bolyai University, Cluj-Napoca, Romania

Solution by Daniel Lasaosa, Universidad Publica de Navarra, Spain

Let −r1,−r2, . . . ,−rn be the n nonzero integral roots of the equation given in the problemstatement, then the equation rewrites as p(x+ r1)(x+ r2) . . . (x+ rn) = 0, or all its coefficientsare nonzero integral multiples of p. In particular, ak is p times the sum of all possible productsof k of the ri’s, hence p + a1 + a2 + · · · + an = p(1 + r1)(1 + r2) . . . (1 + rn) = p2, yielding(1 + r1)(1 + r2) . . . (1 + rn) = p, where each one of the 1 + ri are integers other than 1, ie, n− 1of them must be equal to −1, and the remaining one must be p when n is odd and −p when nis even; in other words, n− 1 of the ri’s are equal to −2, and the other one is p− 1 when n isodd and −p− 1 when n is even. Now, there are

(n−1k−1

)products of k of the ri such that exactly

one of them is not −2, and(n−1k

)products of k of the ri such that all of them are −2, ie, when

n is odd,

akp

=(n− 1k − 1

)(−2)k−1(p− 1) +

(n− 1k

)(−2)k =

(n− 1)!(n− k)!k!

(−2)k−1(pk + k − 2n),

and when n is even,

akp

=(n− 1k − 1

)(−2)k−1(−p− 1) +

(n− 1k

)(−2)k = −(n− 1)!(2n+ pk − k)

(n− k)!k!(−2)k−1.

Note now that, when n is odd,

n∑k=1

akp

=n−1∑k−1=0

(p+ 1)(n− 1)!

(n− k)!(k − 1)!(−2)k−1 +

n∑k=0

n!(n− k)!k!

(−2)k − 1 =

= (p+ 1)(1− 2)n−1 + (1− 2)n − 1 = p− 1,

while when n is even,

n∑k=1

akp

= −(p− 1)n−1∑k−1=

(n− 1)!(n− k)!(k − 1)!

(−2)k−1 +n∑k=0

n!(n− k)!k!

(−2)k − 1 =

= −(p− 1)(1− 2)n−1 + (1− 2)n − 1 = p− 1.

There can be no other solutions.

Also solved by Anthony Erb Lugo; Tigran Hakobyan, Armenia.

Mathematical Reflections 6 (2010) 8

Page 201: Eserciziario Di Analisi Matematica 2

S176. Let ABC be a triangle and let AA1, BB1, CC1 be cevians intersecting at P . Denote byKa = KAB1C1 , Kb = KBC1A1 , Kc = KCA1B1 . Prove that KA1B1C1 is a root of the equation

x3 + (Ka +Kb +Kc)x2 − 4KaKbKc = 0.

Proposed by Ivan Borsenco, Massachusetts Institute of Technology, USA

First solution by Arkady Alt, San Jose, California, USA

Without loss of generality assume that area of triangle ABC is 1. Let pa, pb, pb be the baricentriccoordinates of P, that is pa, pb, pc > 0, pa + pb + pc = 1 and

pa−→PA+ pb

−−→PB + pc

−−→PC = 0.

SinceAC1

BC1=pbpa

and[ACC1][BCC1]

=AC1

BC1then

[ACC1] =pb

pa + pb[ABC] =

pbpa + pb

.

Also,[AB1C1][CB1C1]

=AB1

CB1=pcpa

yields

[AB1C1][ACC1]

=pc

pc + pa.

Hence, Ka = [AB1C1] =pbpc

(pa + pb) (pc + pa). Similarly,

Kb =pcpa

(pb + pc) (pa + pb), Kc =

papb(pc + pa) (pb + pc)

.

Let K = [A1B1C1] , then

K = [ABC]− ([AB1C1] + [BC1A1] + [CA1B1])

= 1− (Ka +Kb +Kc) = 1−∑cyc

pbpc(pa + pb) (pc + pa)

=2papbpc

(pa + pb) (pb + pc) (pc + pa).

On the other hand, since

Ka

K=pb + pc

2pa,Kb

K=pc + pa

2pb,Kc

K=pa + pb

2pc

thenKaKbKc

K3=

(pa + pb) (pb + pc) (pc + pa)8papbpc

=1

4K.

Mathematical Reflections 6 (2010) 9

Page 202: Eserciziario Di Analisi Matematica 2

Thus, K2 = 4KaKbKc and, therefore,

K = 1− (Ka +Kb +Kc) ⇐⇒ K3 = K2 − (Ka +Kb +Kc)K2

⇐⇒ K3 + (Ka +Kb +Kc)K2 − 4KaKbKc = 0.

Second solution by Daniel Campos Salas, Costa Rica

Let x, y, z be the areas of the triangles BPC, CPA, APB, respectively. It’s easy to prove thatAB1B1C

= zx and AC1

C1B= y

x . Therefore we have that

Ka =12AB1 ·AC1 sinA =

12· zACx+ z

· yABx+ y

sinA =yz(x+ y + z)(x+ y)(x+ z)

.

The expressions for Kb and Kc are obtained analogously. It follows that KA1B1C1 equals

(x+ y + z)(

1− yz

(x+ y)(x+ z)− zx

(y + z)(y + x)− xy

(z + x)(z + y)

)=

2xyz(x+ y + z)(x+ y)(y + z)(z + x)

.

Finally, note that

K3A1B1C1

+KA1B1C1(Ka +Kb +Kc)= K2

A1B1C1(KA1B1C1 +Ka +Kb +Kc)

=(

2xyz(x+ y + z)(x+ y)(y + z)(z + x)

))2

(x+ y + z)

=4(xyz)2(x+ y + z)3

((x+ y)(y + z)(z + x))2

= 4KaKbKc,

which proves that KA1B1C1 is a root of the given polynomial.

Also solved by Daniel Lasaosa, Universidad Publica de Navarra, Spain.

Mathematical Reflections 6 (2010) 10

Page 203: Eserciziario Di Analisi Matematica 2

S177. Prove that in any acute triangle ABC,

sinA

2+ sin

B

2+ sin

C

2≥ 5R+ 2r

4R.

Proposed by Titu Andreescu, University of Texas at Dallas, USA

First solution by Arkady Alt, San Jose, California, USA

Since r = 4R sinA

2sin

B

2sin

C

2, the inequality can be rewritten as

sinA

2+ sin

B

2+ sin

C

2− 2 sin

A

2sin

B

2sin

C

2≥ 5

4. (A)

Inequality (A) is an immediate corollary from more general inequality represented by thefollowing theorem

Theorem. Let k be any real number such that k ≥ k∗ where

k∗ =4

2√

2−√

2−√

2 + 3≈ 1.2835.

Then for any α, β, γ ∈(

0,π

4

], such that α+ β + γ =

π

2the following inequality holds

sinα+ sinβ + sin γ − k sinα sinβ sin γ ≥ 12− k8

. (M)

Proof. Assuming, due symmetry, that α ≤ β ≤ γ and, denoting ϕ = α + β we obtainγ =

π

2− ϕ, β = ϕ − α, where the new variables α and ϕ satisfy the inequalities 0 < α ≤

ϕ− α ≤ π

2− ϕ ≤ π

4or equivalently

π

4≤ ϕ ≤ π

32ϕ− π

2≤ α ≤ ϕ

2

. (1)

Since

sinα+ sinβ + sin γ − k sinα sinβ sin γ = 2 sinα+ β

2cos

α− β2

+ sin γ (1− k sinα sinβ)

= 2 sinϕ

2cos(ϕ

2− α

)+ cosϕ (1− k sinα sin (ϕ− α))

then inequality (M) can be equivalently rewritten as

2 sinϕ

2cos(ϕ

2− α

)+ cosϕ (1− k sinα sin (ϕ− α)) ≥ 12− k

8(2)

where variables α and ϕ are subject to the system (1).

Mathematical Reflections 6 (2010) 11

Page 204: Eserciziario Di Analisi Matematica 2

Leth (α) = 2 sin

ϕ

2cos(ϕ

2− α

)+ cosϕ (1− k sinα sin (ϕ− α))

for any fixed ϕ ∈[π

4,π

3

]and k∗ >

2√3

= 1. 154 7. We will prove that h (α) is decreasing on[2ϕ− π

2,ϕ

2

]. Indeed,

h′ (α) = 2 sin(ϕ

2− α

)(sin

ϕ

2− k cosϕ cos

(ϕ2− α

))≤ 0

on[2ϕ− π

2,ϕ

2

]since sin

(ϕ2− α

)≥ 0, k∗ >

2√3

and

sinϕ

2− k cosϕ cos

(ϕ2− α

)≤ sin

ϕ

2− k cosϕ cos

ϕ

2

≤ sinπ

6− k cos

π

3cos

π

6=

12− k

2·√

32

≤ 12−√

32· k∗

2<

12−√

32· 1√

3= 0.

Thus, h (α) ≥ h(ϕ

2

)= 2 sin

ϕ

2+ cosϕ

(1− k sin2 ϕ

2

)and it remains to prove the inequality

2 sinϕ

2+ cosϕ

(1− k sin2 ϕ

2

)≥ 12− k

8. (3)

Let t = sinϕ

2then sin

π

8≤ t ≤ 1

2and (3) is equivalent to

2t+(1− 2t2

) (1− kt2

)≥ 12− k

8⇐⇒ 16kt4 − (8k + 16) t2 + 16t− (4− k) ≥ 0

⇐⇒ (1− 2t)2(k (2t+ 1)2 − 4

)≥ 0

because k (2t+ 1)2 ≥ k∗

(2 sin

π

8+ 1)2

= 4. Since equality in (2) occurs if and only if α =ϕ

2and ϕ =

π

3⇐⇒ α =

π

6and ϕ =

π

3then in (M) equality occurs if and only if α = β = γ =

π

6.

In particular for k = 2 and k =43

, replacing (α, β, γ) in (M) with(A

2,B

2,C

2

), for any acute

triangle ABC we, respectively, obtain inequality (A) and inequality

sinA

2+ sin

B

2+ sin

C

2− 4

3sin

A

2sin

B

2sin

C

2≥ 4

3. (G)

(The last one is an inequality due to J. Garfuncel, given in [RAGI] without proof in a privatecommunication.)

Remark.

Mathematical Reflections 6 (2010) 12

Page 205: Eserciziario Di Analisi Matematica 2

The original inequality immediately follows from (G). Indeed,

sinA

2+ sin

B

2+ sin

C

2− 2 sin

A

2sin

B

2sin

C

2= sin

A

2+ sin

B

2+ sin

C

2− 4

3sin

A

2sin

B

2sin

C

2

− 23

sinA

2sin

B

2sin

C

2

≥ 43− 2

3sin

A

2sin

B

2sin

C

2≥ 4

3− 2

3· 1

8

=54.

[RAGI]. Mitrinovic D.S., Pecaric J. E. , Volenec V. Recent Advances, Geometric Inequality,p.269, inequality 5.10.

Second solution by Daniel Lasaosa, Universidad Publica de Navarra, Spain

Assume wlog that C ≥ B ≥ A, and denote A+B4 = α, B−A

4 = δ. It is well known thatr = 4R sin A

2 sin B2 sin C

2 , or the proposed problem is equivalent to showing that

sinA

2+ sin

B

2+ sin

C

2− 2 sin

A

2sin

B

2sin

C

2≥ 5

4.

Assume that C is known such that the LHS is minimum, or for that value of C, define

f(x, y) = sinx+ sin y − 2 sinx sin y sinC

2,

where x+ y = A+B2 = 90 − C

2 is fixed. Now,

f

(A

2,B

2

)− f(α, α) = f(α− δ, α+ δ)− f(α, α) = 2 sinα(cos δ − 1) + 2 sin2 δ sin

C

2=

= 4 sin2 δ

2

(2 cos2 δ

2sin

C

2− sinα

).

Now, if δ > 0, and since δ = B−A4 < B

4 < 45, we have sin2 δ2 > 0, 2 cos2 δ

2 > 2 cos2(45) > 1.Moreover, C

2 > A+B4 , since equality would only hold iff A = B = C, which is not true because

δ > 0. Thus, f(A2 ,

B2

)≥ f

(A+B

4 , A+B4

), with equality iff A = B = 90− C

2 . It therefore sufficesto show that, for all 90 > C ≥ 60, we have

2u+ (1− 2u2)− 2u2(1− 2u2) ≥ 54.

where we have defined u = sin(45 − C

4

), and therefore sin C

2 = cos(2(45 − C

4

))= 1 − 2u2.

After some algebra, this last inequality is equivalent to (2u − 1)2(4u2 + 8u + 1) ≥ 0. Since90 > C ≥ 60, we have 45

2 < 45 − C4 ≤ 30, or u > 0. The conclusion follows, equality

holds iff u = 12 , ie iff 45 − C

4 = 30, or C = 60. We conclude that equality holds iff ABC isequilateral.

Third solution by Neculai Stanciu, George Emil Palade, Romania

Mathematical Reflections 6 (2010) 13

Page 206: Eserciziario Di Analisi Matematica 2

We have

sinA

2+ sin

B

2+ sin

C

2≥ 5R+ 2r

4R⇐⇒ 2

∑sin

A

2≥ 1 +

r

R+

32

⇐⇒ 2 cosA+B

2+ 2 cos

B + C

2+ 2 cos

C +A

2≥ cosA+ cosB + cosC + 3 cos

A+B + C

3

which is true from Popoviciu’s Inequality for the concave function cosx on the interval(0, π2

).

Also solved by Daniel Campos Salas, Costa Rica

Mathematical Reflections 6 (2010) 14

Page 207: Eserciziario Di Analisi Matematica 2

S178. Prove that there are sequences (xk)k≥1 and (yk)k≥1 of positive rational numbers such that forall positive integers n and k,

(xk + yk√

5)n = Fkn−1 + Fkn1 +√

52

,

where (Fm)m≥1 is the Fibonacci sequence.

Proposed by Dorin Andrica, Babes-Bolyai University, Cluj-Napoca, Romania

Solution by G.R.A.20 Problem Solving Group, Roma, Italy

Take xk = 12Lk and yk = 1

2Fk (see problem J178) where Ln is the n-th Lucas number. Since

Fk =1√5

(1 +√

52

)k−

(1−√

52

)k and Lk =

(1 +√

52

)k+

(1−√

52

)k

it follows that

(xk + yk√

5)n =

(1 +√

52

)kn= Fkn−1 + Fkn

(1 +√

52

).

Also solved by Arkady Alt, San Jose, California, USA; Daniel Lasaosa, Universidad Publicade Navarra, Spain; Daniel Campos Salas, Costa Rica.

Mathematical Reflections 6 (2010) 15

Page 208: Eserciziario Di Analisi Matematica 2

S179. Find all positive integers a and b for which (a2+1)2

ab−1 is a positive integer.

Proposed by Valcho Milchev, Petko Rachov Slaveikov Secondary School, Bulgaria

Solution by Daniel Campos Salas, Costa Rica

Note ab − 1 divides (ab − 1)(2a2 + ab + 1) = (a2 + ab)2 − (a2 + 1)2. This implies that ab − 1divides a2(a + b)2, but ab− 1 and a are coprime, from where we conclude that ab− 1 divides(a+ b)2. Let k = (a+b)2

ab−1 . Note that it is not possible for a, b to be equal, since it would imply4a2

a2−1= 4 + 4

a2−1to be an integer, and 4 has no divisor of the form a2 − 1. The equation can

also be written in the forma2 − (k − 2)ab+ b2 + k = 0. (1)

Note that for every pair of integers (a, b) satisfying the equation k = (a+b)2

ab−1 , the pair(a, a

2+kb

)and its permutations are also positive integer solutions (it’s easy to prove that a2+k

b is aninteger). Iterating this process we find that every solution is part of a family of solutions. Let(a0, b0) be a solution for any of these families with the minimal sum. We may assume withoutloss of generality that b0 > a0. This implies that a2

0+kb0≥ b0 since

(a0,

a20+kb0

)and

(a20+kb0

, a0

)are also solutions to the equation. It follows that

k ≥ b20 − a20 = (b0 − a0)(a0 + b0) ≥ a0 + b0.

This implies that (a0+b0)2

a0b0−1 = k ≥ a0 +b0, or equivalently, 2 ≥ (a0−1)(b0−1). Note that a0 < 3,

since a0 < b0. If a0 = 1, it follows that (b0+1)2

b0−1 is an integer. This implies that 4b0−1 , so it

follows that b0 can be equal to 2, 3 or 5. If a0 = 2, it follows that (b0+2)2

2b0−1 is an integer. Thisimplies that 25

2b0−1 , or equivalently, b0 can equal 3 or 13, since b0 > a0 = 2.

For (a0, b0) = (1, 2) it follows that k = 9. This initial solution generates the family of solutionsgiven by the pairs, (cn, cn+1), n ≥ 0, which satisfy, c0 = 1, c1 = 2, cn+2 = 7cn+1 − cn, wherethe recursion holds by (1). The explicit expression for cn is given by the formula

cn =√

5− 12√

5

(7 + 3

√5

2

)n+√

5 + 12√

5

(7− 3

√5

2

)n

=1√5

(1 +√

52

)4n−1

− 1√5

(1−√

52

)4n−1

= F4n−1,

where Fn is the n-th term of the Fibonacci sequence defined by F0 = 0, F1 = 1 and Fn+2 =Fn+1 + Fn (abusing of the definition we set F−1 = 1).

Mathematical Reflections 6 (2010) 16

Page 209: Eserciziario Di Analisi Matematica 2

For (a0, b0) = (1, 3) it follows that k = 8. This initial solution generates the family of solutionsgiven by the pairs, (dn, dn+1), n ≥ 0, which satisfy, d0 = 1, d1 = 2, dn+2 = 6dn+1 − dn, wherethe recursion holds by (1). The explicit expression for dn is given by the formula

dn =12

(3 + 2

√2)n

+12

(3− 2

√2)n

=12

((√

2 + 1)2n + (√

2− 1)2n).

For (a0, b0) = (1, 5) it follows that k = 9. This initial solution generates the family of solutionsgiven by the pairs, (en, en+1), n ≥ 0, which satisfy, e0 = 1, e1 = 2, en+2 = 7en+1 − en, wherethe recursion holds by (1). The explicit expression for en is given by the formula

en =√

5 + 12√

5

(7 + 3

√5

2

)n+√

5− 12√

5

(7− 3

√5

2

)n

=1√5

(1 +√

52

)4n+1

− 1√5

(1−√

52

)4n+1

= F4n+1.

We may interpret this solution as an extension of the first one for negative indexes sinceF4n+1 = F−4n−1.

For (a0, b0) = (2, 3) it follows that k = 5. This initial solution generates the family of solutionsgiven by the pairs, (fn, fn+1), n ≥ 0, which satisfy, f0 = 1, f1 = 2, fn+2 = 3dn+1 − fn, wherethe recursion holds by (1). The explicit expression for fn is given by the formula

fn =

(3 +√

52

)n+

(3−√

52

)n

=

(1 +√

52

)2n

+

(1−√

52

)2n

= L2n,

where Ln is the n-th term of the Lucas sequence defined by L0 = 2, L1 = 1 and Ln+2 =Ln+1 + Ln.

Therefore, we conclude that all the possible pairs of positive integers are of the form (F4n−1, F4n+3),(L2n, L2n+2) (for all integers n) and (dn, dn+1) (defined above), and its permutations, and we’redone.

Also solved by Tigran Hakobyan, Armenia

Mathematical Reflections 6 (2010) 17

Page 210: Eserciziario Di Analisi Matematica 2

S180. Solve in nonzero real numbers the system of equationsx4 − y4 = 121x−122y

4xy

x4 + 14x2y2 + y4 = 122x+121yx2+y2

.

Proposed by Titu Andreescu, University of Texas at Dallas, USA

Solution by Daniel Lasaosa, Universidad Publica de Navarra, Spain

Note first that x4 + 14x2y2 + y4 = 4(x2 + y2)2 − 3(x2 − y2)2 = s4 − s2d2 + d4, where we havedefined x + y = s and x − y = d, while x4 − y4 = sd(x2 + y2) = sd(s2+d2)

2 , 4xy = s2 − d2,x2 + y2 = s2+d2

2 . Therefore, the system may be rewritten assd(s2 + d2)(s2 − d2) = 243d− s,

(s4 − s2d2 + d4)(s2 + d2) = 243s+ d.

We can then obtain

(243d− s)(243s+ d) = sd(s2 + d2)(s2 − d2)(243s+ d) = (s4 − s2d2 + d4)(s2 + d2)(243d− s).

Since s2 +d2 > 0 (otherwise x = y = 0, in contradiction with the problem statement), it followsthat

243s2d(s2 − d2) + sd2(s2 − d2) = 243d(s4 − s2d2 + d4)− s(s4 − s2d2 + d4),

which after simplification yields s5 = 243d5, or s = 3d. Substitution in both equations yieldsd6 = d, or since x 6= y (if x = y 6= 0 the LHS of the first equation would be zero, but the RHSwould not), we find that d5 = 1, ie s = 3 and d = 1 for x = 2, y = 1. These values can beclearly shown to satisfy the system by plugging them into the given equations, and no othersolutions exist.

Second solution by Arkady Alt, San Jose, California, USA

Since (x4 + 14x2y2 + y4

) (x2 + y2

)= 122x+ 121y, 4xy

(x4 − y4

)= 121x− 122y

and x, y 6= 0 then(x4 + 14x2y2 + y4

) (x2 + y2

)(x− y)− 4xy

(x4 − y4

)(x+ y)

= (122x+ 121y) (x− y)− (121x− 122y) (x+ y)

⇐⇒(x2 + y2

) ((x4 + 14x2y2 + y4

)(x− y)− 4xy

(x2 − y2

)(x+ y)

)= x2 + y2 ⇐⇒

(x4 + 14x2y2 + y4

)(x− y)− 4xy

(x2 − y2

)(x+ y)

= 1 ⇐⇒ (x− y)5 = 1 ⇐⇒ x− y = 1.

Let t = x+ y then

x2 − y2 = t, x2 + y2 =t2 + 1

2, 4xy = t2 − 1, y =

t− 12

, 121x− 122y = 121− t− 12

Mathematical Reflections 6 (2010) 18

Page 211: Eserciziario Di Analisi Matematica 2

and the equation x4 − y4 =121x− 122y

4xybecomes

t(t4 − 1

)2

= 121− t− 12⇐⇒ t

(t4 − 1

)+ t− 1 = 242 ⇐⇒ t5 = 243 ⇐⇒ t = 3.

Hence, x− y = 1x+ y = 3

⇐⇒ x = 2, y = 1.

Mathematical Reflections 6 (2010) 19

Page 212: Eserciziario Di Analisi Matematica 2

Undergraduate problems

U175. What is the maximum number of points of intersection that can appear after drawing in aplane l lines, c circles, and e ellipses?

Proposed by Dorin Andrica, Babes-Bolyai University, Cluj-Napoca, Romania

Solution by Andrea Ligori and Emanuele Natale, Universita di Roma “Tor Vergata”, Roma,Italy

We note that

• the intersection of two circles yields 2 points, so the contribution is 2(c2

);

• the intersection of a line with a circle or an ellipse yields 2 points, so the contribution is2l(c+ e);

• the intersection of an ellipse with a circle or another ellipse yields 4 points, so the contri-bution is 4

(e2

)+ 4ec;

• the intersection of two incindent lines yields 1 point so the contribution is(l2

).

Therefore the final formula is

2(c

2

)+ 2l(c+ e) + 4

(e

2

)+ 4ec+

(l

2

).

It is easy to find a configuration of l lines, c circles, and e ellipses with such a number ofintersection points.

Also solved by Daniel Lasaosa, Universidad Publica de Navarra, Spain.

Mathematical Reflections 6 (2010) 20

Page 213: Eserciziario Di Analisi Matematica 2

U176. In the space, consider the set of points (a, b, c) where a, b, c ∈ 0, 1, 2. Find the maximumnumber of non-collinear points contained in the set.

Proposed by Ivan Borsenco, Massachusetts Institute of Technology, USA

Solution by Daniel Lasaosa, Universidad Publica de Navarra, Spain

It is possible to have 16 non-collinear points, taking for example the following ones:

(0, 0, 0), (0, 1, 0), (1, 0, 0), (1, 2, 0), (2, 1, 0), (2, 2, 0), (0, 0, 1), (0, 2, 1),

(2, 0, 1), (2, 2, 1), (0, 1, 2), (0, 2, 2), (1, 0, 2), (1, 2, 2), (2, 0, 2), (2, 1, 2).

Consider a set of non-collinear points of the form (a, b, c) where a, b, c ∈ 0, 1, 2. Note firstthat for each pair of fixed values (b, c), there may be at most two values of a such that (a, b, c)is in the set, otherwise if points with a = 0, 1, 2 are in the set, they would be collinear. Thesame reasoning applies to all pairs of fixed values (c, a) and (a, b). It follows that, for eachfixed value of c, there are at most six pairs of values (a, b) such that (a, b, c) is in the set, andsuch that no three of them have the same value of a or of b. Assume now that, for a givenvalue of c and two distinct values b1, b2 of b, the same two values a1, a2 of a exist such that(a, b, c) is in the set. Calling b3 the third possible value of b, note that neither (a1, b3, c) nor(a2, b3, c) may be in the set, ie, at most five points with the aforementioned value of c are inthe set. We conclude that, if there are six points with the same value of c, then the pairs ofvalues of a such that (a, b, c) is in the set as b takes its three possible values, must be distinctand therefore must be (0, 1), (1, 2), (0, 2), each pair corresponding to each one of the possiblevalues of b. Finally, if (0, 1, c) and (1, 1, c) are in the set, note that (0, 0, c) cannot be in the set,since neither (0, 2, c) nor (2, 2, c) could be on the set, and similarly for (2, 1, c) by symmetry,ie, if there are six points which have the same value of c, they must be either (0, 0, c), (0, 1, c),(1, 0, c), (1, 2, c), (2, 1, c) and (2, 2, c), or (0, 1, c), (0, 2, c), (1, 0, c), (1, 2, c), (2, 0, c) and (2, 1, c).

Assume that it is possible to have 17 non-collinear points. By the previous arguments, exactlytwo of the planes with constant value of c must contain exactly 6 points, and the remainingplane must contain exactly 5 points. Moreover, both plains containing exactly 6 points musthave a different combination out of the two mentioned above, since if (a, b, c) belongs in theset for two distinct values of c, and six distinct pairs of values (a, b), then (a, b, c) cannot bein the set for each one of this six pairs of values and the third value of c. It follows that, theplane containing exactly 5 points, cannot have points at (0, 1, c), (1, 0, c), (1, 2, c) or (2, 1, c),since they would be collinear with the corresponding points of the other two planes. Thereforethe plane containing exactly 5 points contains at least points (0, 0, c), (1, 1, c) and (2, 2, c),which are collinear, contradiction. There cannot be more than 16 non-collinear points, butwe have produced an example of such 16 non-collinear points, hence the maximum number ofnon-collinear points is 16.

Also solved by Andrea Ligori, Universita di Roma “Tor Vergata”, Roma, Italy.

Mathematical Reflections 6 (2010) 21

Page 214: Eserciziario Di Analisi Matematica 2

U177. Let a1, a2, . . . , an and b1, b2, . . . , bn be integers greater than 1. Prove that there are infinitely

many primes p such that p divides bp−1aii − 1 for all i = 1, 2, . . . , n.

Proposed by Gabriel Dospinescu, Ecole Normale Superieure, France

Solution by the author

This is a very simple (but rather surprising) application of the following beautiful result.

Theorem 1.1. (Nagell) Let f1, f2, . . . , fn be nonconstant polynomials with integer coefficients.Then for infinitely many primes p one can find integers x1, x2, . . . , xn such that p divides fi(xi)for all i.

For the proof, we refer to T. Andreescu, G. Dospinescu, Problems from the Book. Now,consider the polynomials fi(X) = Xai − bi and add to their collection the polynomials ϕai ,where ϕn is the n−th cyclotomic polynomial. Pick a prime p > b1b2 · · · bn + a1a2 · · · an as inthe theorem and xi, yi such that p divides fi(xi) and ϕai(yi). As p does not divide any of theais, it is a classical property of cyclotomic polynomials that we must have p ≡ 1 (mod ai).Moreover, p clearly does not divide xi, since it does not divide bi. Since xai

i ≡ bi (mod p) and

ai ≡ 1 (mod p), Fermats little theorem implies that p divides bp−1aii and we are done.

Mathematical Reflections 6 (2010) 22

Page 215: Eserciziario Di Analisi Matematica 2

U178. Let k be a fixed positive integer and let S(j)n =

(nj

)+(nj+k

)+(

nj+2k

)+ · · · , j = 0, 1, . . . , k − 1.

Prove that(S(0)n + S(1)

n cos2πk

+ · · ·+ S(k−1)n cos

2(k − 1)πk

)2

+(S(1)n sin

2πk

+ S(2)n sin

4πk

+ · · ·+ S(k−1)n sin

2(k − 1)πk

)2

=(

2 cosπ

k

)2n.

Proposed by Dorin Andrica, Babes-Bolyai University, Cluj-Napoca, Romania

Solution by Arkady Alt, San Jose, California, USA

Let Z+ = N ∪ 0 and Dj = j +mk | m ∈ Z+ and j +mk ≤ n . Then S(j)n =

∑p

(np

)and

k−1⋃j=0

Dj = 0, 1, 2, . . . , n . Let

a =k−1∑j=0

S(j)n cos

2jπk

b =k−1∑j=0

S(j)n sin

2jπk

ε = cos2πk

+ i sin2πk·k−1∑j=0

S(j)n

(cos

2πk

+ i sin2πk

)j.

Then εk = 1 and

a+ ib =k−1∑j=0

S(j)n cos

2jπk

+ ik−1∑j=0

S(j)n cos

2jπk

=k−1∑j=0

S(j)n

(cos

2jπk

+ i sin2jπk

)

=k−1∑j=0

S(j)n εj =

k−1∑j=0

∑p∈Dj

(np

)εp

=∑p∈∪Dj

(np

)εp =

n∑p=1

(np

)εp = (1 + ε)n

=(

1 + cos2πk

+ i sin2πk

)n=(

2 cosπ

k

(cos

π

k+ i sin

π

k

))n=(

2 cosπ

k

)n (cos

π

k+ i sin

π

k

)n.

Mathematical Reflections 6 (2010) 23

Page 216: Eserciziario Di Analisi Matematica 2

Hence,

|a+ ib| =∣∣∣(2 cos

π

k

)n (cos

π

k+ i sin

π

k

)n∣∣∣ =∣∣∣(2 cos

π

k

)n∣∣∣ ∣∣∣(cosπ

k+ i sin

π

k

)n∣∣∣=∣∣∣(2 cos

π

k

)∣∣∣n ∣∣∣(cosπ

k+ i sin

π

k

)∣∣∣n =∣∣∣(2 cos

π

k

)∣∣∣n .Therefore, a2 +b2 =

(2 cos

π

k

)2n.

Also solved by Daniel Lasaosa, Universidad Publica de Navarra, Spain; Daniel Campos Salas,Costa Rica.

Mathematical Reflections 6 (2010) 24

Page 217: Eserciziario Di Analisi Matematica 2

U179. Let f : [0,∞] → R be a continuous function such that f(0) = 0 and f(2x) ≤ f(x) + x for allx ≥ 0. Prove that f(x) < x for all x ∈ [0,∞].

Proposed by Samin Riasat, University of Dhaka, Bangladesh

Solution by Perfetti Paolo, Dipartimento di Matematica, Universita degli studi di Tor VergataRoma, Italy

Note thatf(x) ≤ f

(x2

)+x

2≤ f

(x4

)+x

4+x

2≤ f

(x8

)+x

8+x

4+x

2. . .

and after n steps we have

f(x) ≤ f( x

2n)

+n∑k=1

x

2n

The limit n→∞ yields

f(x) ≤ limn→∞

(f( x

2n)

+n∑k=1

x

2n

)= lim

n→∞f( x

2n)

+ limn→∞

n∑k=1

x

2n= 0 + x = x

where we have used the continuity of f(x) and f(0) = 0 for writing

limn→∞

f( x

2n)

= 0

and we are done.

Also solved by Arkady Alt, San Jose, California, USA; Daniel Lasaosa, Universidad Publica deNavarra, Spain; John Mangual, UCSB; Emanuele Natale, Universita di Roma “Tor Vergata”,Roma, Italy; Tigran Hakobyan, Armenia; Daniel Campos Salas, Costa Rica.

Mathematical Reflections 6 (2010) 25

Page 218: Eserciziario Di Analisi Matematica 2

U180. Let a1, . . . , ak, b1, . . . , bk, n1, . . . , nk be positive real numbers and a = a1 + · · · + ak, b = b1 +· · ·+ bk, n = n1 + · · ·+ nk, k ≥ 2. Prove that∫ 1

0(a1 + b1x)n1 · · · (ak + bkx)nkdx ≤ (a+ b)n+1 − an+1

(n+ 1)b.

Proposed by Dorin Andrica, Babes-Bolyai University, Cluj-Napoca, Romania

Solution by Perfetti Paolo, Dipartimento di Matematica, Universita degli studi di Tor VergataRoma, Italy

By concavity of the logarithm we have

lnk∏j=1

(aj + bkx)nj = nk∑j=1

njn

ln(aj + bjx) ≤ n ln

k∑j=1

njn

(aj + bjx)

Moreover the monotonicity (increasing) of the logarithm yields

n ln

k∑j=1

njn

(aj + bjx)

≤ n ln

k∑j=1

(aj + bjx)

= ln(a+ bx)n

Now exponentiating and integrating we have∫ 1

0(a1 + b1x)n1 · · · (ak + bkx)nkdx ≤

∫ 1

0(a+ bx)ndx =

(a+ b)n+1 − an+1

(n+ 1)b

concluding the proof.

Also solved by Arkady Alt, San Jose, California, USA; Daniel Lasaosa, Universidad Publicade Navarra, Spain; Daniel Campos Salas, Costa Rica.

Mathematical Reflections 6 (2010) 26

Page 219: Eserciziario Di Analisi Matematica 2

Olympiad problems

O175. Find all pairs (x, y) of positive integers such that x3 − y3 = 2010(x2 + y2).

Proposed by Titu Andreescu, University of Texas at Dallas, USA

First solution by the author

Write x = du, y = dv with d ≥ 1 and u, v relatively prime positive integers. The equationbecomes d(u3 − v3) = 2010(u2 + v2). Thus u2 + uv + v2 divides 2010(u2 + v2) and since it isrelatively prime to u2 + v2, we deduce that u2 + uv + v2 divides 2010 = 2 · 3 · 5 · 67. We claimthat A = u2 + uv + v2 actually divides 67. It is immediate that A is odd (if not, u, v must beboth even). Next, it is easy to see that if 5 divides A, then 5 divides both u, v, a contradiction.Finally, if 3 divides A, we must have u ≡ v mod 3 and so u3 − v3 = (u − v)A is a multipleof 9. Thus 2010(u2 + v2) is a multiple of 9, which is not the case. Thus A is a divisor of 67and since u, v ≥ 1, we deduce that u2 + uv + v2 = 67. Clearly u ≥ v, then 67 ≥ 3v2, thusv = 4. Considering each case, we deduce that v = 2, u = 7 and so d = 318. Hence there is onesolution, x = 7d, y = 2d with d = 318.

Also solved by Tigran Hakobyan, Armenia.

Mathematical Reflections 6 (2010) 27

Page 220: Eserciziario Di Analisi Matematica 2

O176. Let P (n) be the following statement: for all positive real numbers x1, x2, . . . , xn such thatx1 + x2 + · · ·+ xn = n,

x2√x1 + 2x3

+x3√

x2 + 2x4+ · · ·+ x1√

xn + 2x2≥ n√

3.

Prove that P (n) is true for n ≤ 4 and false for n ≥ 9.

Proposed by Gabriel Dospinescu, Ecole Normale Superieure, France

First solution by the author

Let S(x1, x2, . . . , xn) be the left hand side of the inequality. Using Holder’s inequality, weobtain

S2(x2(x1 + 2x3) + · · ·+ x1(xn + 2x2)) ≥ (x1 + x2 + · · ·+ xn)3 = n3.

On the other hand, we have

x2(x1 + 2x3) + · · ·+ x1(xn + 2x2) = 3(x1x2 + x2x3 + · · ·+ xnx1).

Using the fact thatx1x2 + x2x3 + · · ·+ xnx1 ≤ n

whenever x1 + x2 + · · ·+ xn = n and n ≤ 4. The last fact follows from the fact that

ab+ bc+ ca ≤ (a+ b+ c)2

3

and

ab+ bc+ cd+ da = (a+ c)(b+ d) ≤ (a+ b+ c+ d)2

4.

The conclusion follows easily for n ≤ 4. Chosing x1, x2, x3, x4 close to n4 and the other variables

equal and close to 0, one easily obtains that the expression is smaller than n√3

for n ≥ 9. Theconclusion follows.

Second solution by Arkady Alt, San Jose, California, USA

Let n ≤ 4.Then, applying consequentially AM-GM and Cauchy inequalities, we obtain∑ncyc

x2√3 (x1 + 2x3)

≥∑n

cyc

2x2

3 + (x1 + 2x3)= 2

∑ncyc

x22

3x2 + (x1x2 + 2x2x3)≥

2 (∑n

k=1 xk)2

3∑n

k=1 xk +∑n

cyc (x1x2 + 2x2x3)=

2n2

3n+ 3∑n

cyc x1x2.

Thus,∑n

cyc

x2√x1 + 2x3

≥ 2√3· n2

n+∑n

cyc x1x2.

For n = 3, since x1 + x2 + x3 = 3 we have∑n

cyc x1x2 ≤(x1 + x2 + x3)2

3= 3.

Mathematical Reflections 6 (2010) 28

Page 221: Eserciziario Di Analisi Matematica 2

Then2√3· n2

n+∑n

cyc x1x2=

2√3· 9

3 +∑n

cyc x1x2≥ 2√

3· 9

6=√

3 =3√3.

If n = 4 then x1 + x2 + x3 + x4 = 4 and∑n

cyc x1x2 = (x1 + x3) (x2 + x4) ≤((x1 + x3) + (x2 + x4)

2

)2

= 4.Therefore,2√3· n2

n+∑n

cyc x1x2=

2√3· 16

4 +∑n

cyc x1x2≥ 2√

3· 16

4 + 4=

4√3.

Let n ≥ 9 and let xk =n

2k, k = 1, 2, ..., n.Then

L.H.S. =∑n−2

k=1

xk+1√xk + 2xk+2

+xn√

xn−1 + 2x1+

x1√xn + 2x2

=∑n−2

k=1

n

2k+1√n

2k+ 2 · n

2k+2

+

n

2n√n

2n−1+ 2 · n

2

+

n

2√n

2n+ 2 · n

4

=∑n−2

k=1

√n√

2k+2 + 2k+1+

√n√

2n+1 + 22n+

√n√

12n−2

+ 2.

Since∑n−2

k=1

√n√

2k+2 + 2k+1=√n∑n−2

k=1

1√3 · 2k+1

=√n

3∑n−2

k=1

1

2√

2k−1<

12

√n

3· 1

1− 1√2

=12

√n

3·√

2√2− 1

=√n

6(√

2 + 1),

√n√

2n+1 + 22n<

√n

2√

2n

and√n√

12n−2

+ 2<

√n

2then L.H.S. <

√n

3

(√2 + 1√

2+√

32√

2n+√

3√2

).

Moreover, since n ≥ 9 we obtain

1 +1√2

+√

32√

2n+√

3√2<

√2 + 1√

2+√

3

2√

29+√

3√2< 1 +

1 +√

3√2

+1√29

= 2. 976 < 3,

and , therefore, L.H.S. <√

3n <n√3.

So, P (n) is false for n ≥ 9.

Mathematical Reflections 6 (2010) 29

Page 222: Eserciziario Di Analisi Matematica 2

O177. Let P be point situated in the interior of a circle. Two variable perpendicular lines throughP intersect the circle at A and B. Find the locus of the midpoint of the segment AB.

Proposed by Dorin Andrica, Babes-Bolyai University, Cluj-Napoca, Romania

First solution by G.R.A.20 Problem Solving Group, Roma, Italy

We can assume, without loss of generality, that P = t ∈ [0, 1] and the circle C = |z| = 1.Let A = z = x+ iy ∈ C then B = w = si(z − P ) + P ∈ C with some s > 0. Hence

1 = |w|2 = (t− sy)2 + s2(x− t)2 (1)

The midpoint of the segment AB is given by M = (A+B)/2.

Now we verify that|M − P/2| =

√2− |P |2/2.

In fact, by (1),

(2|M − P/2|)2 = (x− sy)2 + (s(x− t) + y)2 = x2 + y2 + 1− t2 = 2− t2.

Hence the required locus is a circle with center P/2 and radius√

2− |P |2/2.

In the general setting, if the circle C has center at P0 and radius R then the locus is a circlewith center (P0 + P )/2 and radius

√2R2 − |P − P0|2/2.

Second solution by the author

Let ABCD be a quadrilateral and let M and N be the midpoints of sides AB and CD,respectively. Using the Median Theorem it is easy to prove that the following relation holds :

AC2 +BD2 +BC2 +DA2 = AB2 + CD2 + 4MN2.

Let M be the midpoint of the segment AB and let N be the midpoint of the segment OP ,where O is the center of the given circle. Applying the relation above in the quadrilateralABPO we obtain

AP 2 +R2 +BP 2 +R2 = AB2 +OP 2 + 4MN2.

It is clear that AP 2 +BP 2 = AB2, hence we get

4MN2 = 2R2 −OP 2,

that isNM =

12

√2R2 −OP 2.

Since the point N is fixed, it follows that the desired locus is the circle of center N and radius12

√2R2 −OP 2.

Also solved by Daniel Lasaosa, Universidad Publica de Navarra, Spain; Daniel Campos Salas,Costa Rica; Tomas Calderon Gomez, Costa Rica.

Mathematical Reflections 6 (2010) 30

Page 223: Eserciziario Di Analisi Matematica 2

O178. Let m and n be positive integers. Prove that for each odd positive integer b there are infinitelymany primes p such that pn ≡ 1 (mod b)m implies bm−1 | n.

Proposed by Vahagn Aslanyan, Yerevan, Armenia

Solution by the author

Let b = pα11 pα2

2 · · · pαkk be the canonical factorization of b. Because b is odd pi > 2 for i =

1, 2, . . . , k. Let P = p1p2 · · · pk. Consider the sysmte of conguences (∗)x ≡ pi + 1 (mod p2i ), i =

1, 2, . . . , k. By the Chinese Remainder Theorem the system (∗) has solution. Let that solutionbe x0. We have x0 ≡ pi+1 (mod p2

i ) for all i. If x ≡ x0 (mod P 2), then x is solution of system(∗).Claim. If x ≡ x0 (mod P 2), then from condition xn ≡ 1 (mod bm) it follows that bm−1 | n.Proof. Suppose p ∈ P is a prime and a is a positive integer. Let vp(a) be the degree of p inthe canonical factorization of a. For each i, 1 ≤ i ≤ k, pi | x0 − 1. We know that pi is odd,therefore by a well knwon lemma vpi(x

n − 1) = vpi(x − 1) + vpi(n). But x ≡ x0 ≡ pi + 1(mod p2

i ), so vpi(x− 1) = 1. Hence vpi(n) = vpi(xn − 1)− 1 ≥ mαi − 1 ≥ (m− 1)αi (because

xn ≡ 1 (mod bm)). So p(m−1)αi

i | n. It is true for all i, and gcd(pi, pj) = 1 whenever i 6= j,

therefore bm−1 =∏ki=1 p

(m−1)αi

i | n. Now from Dirichlet’s theorem there are infinitely manyprimes p such that p ≡ x0 (mod P 2).

Also solved by Tigran Hakobyan, Armenia.

Mathematical Reflections 6 (2010) 31

Page 224: Eserciziario Di Analisi Matematica 2

O179. Prove that any convex quadrilateral can be dissected into n ≥ 6 cyclic quadrilaterals.

Proposed by Dorin Andrica, Babes-Bolyai University, Cluj-Napoca, Romania

Solution by Daniel Lasaosa, Universidad Publica de Navarra, Spain

Any convex quadrilateral is dissected into two triangles by either of its diagonals; any concavequadrilateral is dissected into two triangles by exactly one of its diagonals; any crossed quadri-lateral is already formed by two triangles joined at one vertex, and where two of the sides ofeach triangle are on the straight line containing two of the sides of the other.

In triangle ABC, let I be the incenter and D,E, F the points where the incircle touchesrespectively sides BC,CA,AB. Clearly, ABC my be dissected into three cyclic quadrilateralsAEIF , BFID, CDIE.

In triangle ABC, wlog acute at C, consider the circumcenter O, and take a point O′ on theperpendicular bisector of AB that is closer to AB than O. The circle with center O′ throughA,B leaves C outside, hence it must intersect the interior of segments AC,BC at E,D, orABDE is cyclic.

We may then proceed as follows: write n = 3 + 3u + v, where u ≥ 1 is an integer andv ∈ 0, 1, 2. Dissect (any) quadrilateral ABCD in two triangles, then dissect one of theminto three cyclic quadrilaterals. If v 6= 0, dissect the other triangle into one cyclic quadrilateraland one triangle, and if v = 2, dissect again this latter triangle into one cyclic quadrilateral andone triangle. After this procedure, we have dissected the original quadrilateral into 3+v cyclicquadrilaterals (3, 4, 5 respectively for v = 0, 1, 2) and one triangle. Dissect now this triangleinto u triangles (for example dividing one of its sides in u equal parts and joining each pointof division with the opposite vertex), and dissect now each one of these u triangles into threecyclic quadrilaterals. We have thus dissected the original quadrilateral into 3 + v + 3u = ncyclic quadrilaterals.

Also solved by Daniel Campos Salas, Costa Rica.

Mathematical Reflections 6 (2010) 32

Page 225: Eserciziario Di Analisi Matematica 2

O180. Let p be a prime. Prove that each positive integer n ≥ p, p2 divides(n+pp

)2 − (n+2p2p

)−(n+p2p

).

Proposed by Dorin Andrica, Babes-Bolyai University, Cluj-Napoca, Romania

First solution by G.R.A.20 Problem Solving Group, Roma, Italy

Since by Wolstenholme Theorem, for any prime p > 2,(ap

bp

)≡(a

b

)(mod p2), and

p−1∑k=1

1k≡ 0 (mod p),

it is easy to verify by induction with respect to n = bn/pc · p+ [n]p ≥ 0 that for any prime p

(n+ ap

bp

)≡(bn/pc+ a

b

)1 + bp

[n]p∑k=1

1k

(mod p2).

Hence by letting x = bn/pc and y =∑[n]p

k=11k we obtain(

n+ p

p

)2

−(n+ 2p

2p

)−(n+ p

2p

)≡ (x+ 1)2(1 + py)2 −

((x+ 2

2

)+(x+ 1

2

))(1 + 2py)

≡ p2y2(x+ 1)2 ≡ 0 (mod p2).

Second solution by Campos Salas, Costa Rica

We’ll prove it first for p = 2. It’s easy to verify that the expression equals 4(n+3

4

), and it’s

divisible by 4.

Suppose p ≥ 3. Note that the greatest power of p that divides (p!)2 and (2p)! is p2. Thefollowing congruences will be taken modulo p2. It’s well-known that

(2pp

)≡ 2. This implies

that (2p)!p2≡ 2(p!)2

p2, or equivalently p2

(p!)2≡ 2p2

(2p)! . Therefore, the problem is equivalent to provethat p4 divides

p∏i=1

(n+ i)

(p∏i=1

(n+ p+ i) +p∏i=1

(n− p+ i)− 2p∏i=1

(n+ i)

).

It would suffice to prove that p3 divides

p∏i=1

(n+ p+ i) +p∏i=1

(n− p+ i)− 2p∏i=1

(n+ i).

Consider the expression (∏pi=1(n+ x+ i)−

∏pi=1(n+ i)) modulo x3 (this can be done since

p ≥ 3). Note that

Mathematical Reflections 6 (2010) 33

Page 226: Eserciziario Di Analisi Matematica 2

p∏i=1

(n+ x+ i)−p∏i=1

(n+ i) ≡p∏i=1

(n+ i)(x2s2 − xs1

),

where sk represents the sum of all the possible products of k different terms of the set 1n+1 , ...,

1n+p.

Therefore, modulo p3 we have that

p∏i=1

(n+ p+ i) +p∏i=1

(n− p+ i)− 2p∏i=1

(n+ i) ≡ 2p2s2

p∏i=1

(n+ i).

We’re left to prove that p divides s2∏pi=1(n+ i). It’s clear that this expression, modulo p, is

congruent to s′2p!, where s′2 represents the sum of all the possible products of 2 different termsof the set 1

1 , ...,1p. If a product doesn’t have 1

p as one of its factors, then is divisible by p

when multiplied by p!. Then, it is enough to prove that p divides 11 + 1

2 + ... + 1p−1 which is

true by grouping the k-th term and the (p− k)-th term, and we’re done.

Third solution by the author

According to the Li-Jen-Shu fromula, for any positive integers n and p we have

p∑k=0

(p

k

)2(n+ 2p− k2p

)=(n+ p

p

)2

.

It followsp−1∑k=1

(p

k

)2(n+ 2p− k2p

)=(n+ p

p

)2

−(n+ 2p

2p

)−(n+ p

2p

).

Considering p a prime, we have(pk

)≡ 0 (mod p), hence

(pk

)2 ≡ 0 (mod p2). From the aboverelation, we obtain

p−1∑k=1

(p

k

)2(n+ 2p− k2p

)≡ 0(modp2),

hence p2 divides (n+ p

p

)2

−(n+ 2p

2p

)−(n+ p

2p

),

and we are done.

Also solved by Daniel Lasaosa, Universidad Publica de Navarra, Spain.

Mathematical Reflections 6 (2010) 34

Page 227: Eserciziario Di Analisi Matematica 2

Laboratorio di Matematica

Foglio n.1 - 15 ottobre 2010

Esercizio 1. Siano A e B matrici n × n a coefficienti reali. Dimostrare che se A e simmetrica edefinita non negativa e AB + BA = 0 allora AB = BA = 0.

Esercizio 2. Siano A1, A2, . . . , An i vertici di un poligono regolare di n lati inscritto in una lacirconferenza C di raggio unitario. Dimostrare che per ogni P ∈ C

1

n

n∑

k=1

|PAk|2

e un numero intero e determinarne il valore.

Esercizio 3. Un poligono e formato da n rettangoli 3 × 2 uniti lungo una parte del lato lungoe disposti in modo alternato. Tale poligono deve essere ricoperto con tessere rettangolari 2 × 1 o1 × 2. Qui sotto e rappresentato un esempio di ricoprimento per n = 5:

Sia an il numero di tali ricoprimenti. Dimostrare che 3n ≤ an < 4n per ogni n intero positivo.

Esercizio 4. Sia x un numero reale non negativo. Calcolare la somma della serie

∞∑

n=1

(−1)⌊2n

x⌋

2n.

Esercizio 5. Dimostrare o confutare che per ogni intero positivo k esiste una potenza di due chein base dieci ha almeno k cifre uguali a 9.

Page 228: Eserciziario Di Analisi Matematica 2

Laboratorio di Matematica

Foglio n.1 - 15 ottobre 2010

Esercizio 1. Siano A e B matrici n × n a coefficienti reali. Dimostrare che se A e simmetrica edefinita non negativa e AB + BA = 0 allora AB = BA = 0.

Soluzione.

Dato che A e simmetrica, A e diagonalizzabile e tutti i suoi autovalori sono reali.Inoltre, se Av = λv allora AB + BA = 0 implica che

A(Bv) = −B(Av) = −λ(Bv)

Siccome A e definita non negativa, λ ≥ 0 e

0 ≤ 〈A(Bv), Bv〉 = −λ||Bv||2 ≤ 0.

Cosı λ||Bv||2=0, ossia λ = 0 oppure Bv = 0Sia v1, v2, · · · , vn una base di autovettori che diagonalizza A con λ1, λ2, · · · , λn i relativi auto-valori allora

A(Bvi) = −λi(Bvi) = 0

e dunque AB = 0 e BA = −AB = 0.Si noti che le ipotesi del problema non implicano che A = 0 o B = 0. Ad esempio

A =

[

Ik 00 0n−k

]

e B =

[

0k 00 In−k

]

con 1 < k < n soddisfano le ipotesi del problema.

Esercizio 2. Siano A1, A2, . . . , An i vertici di un poligono regolare di n lati inscritto in una lacirconferenza C di raggio unitario. Dimostrare che per ogni P ∈ C

1

n

n∑

k=1

|PAk|2

e un numero intero e determinarne il valore.

Soluzione.

Possiamo supporre senza perdere di generalita che i vertici del poligono siano

Ak = eikα con α = 2π/n e k = 1, . . . , n.

Sia P = eiθ ∈ C allora

|PAk|2 = (eiθ − eikα)(e−iθ − e−ikα) = 2 − 2Re(eiθ−ikα).

Quindi

1

n

n∑

k=1

|PAk|2 = 2 − 2

nRe

(

eiθ

n∑

k=1

e−ikα

)

= 2 − 2

nRe

(

eiθ1 − e−inα

1 − e−iα

)

= 2.

Page 229: Eserciziario Di Analisi Matematica 2

Esercizio 3. Un poligono e formato da n rettangoli 3 × 2 uniti lungo una parte del lato lungoe disposti in modo alternato. Tale poligono deve essere ricoperto con tessere rettangolari 2 × 1 o1 × 2. Qui sotto e rappresentato un esempio di ricoprimento per n = 5:

Sia an il numero di tali ricoprimenti. Dimostrare che 3n ≤ an < 4n per ogni n intero positivo.

Soluzione.

Gli an ricoprimenti del poligono di ordine n possono essere divisi in due categorie:

• i tn ricoprimenti che non contengono tessere che attraversano la linea dicongiunzione tra i primi n − 1 rettangoli e l’ultimo;

• i restanti sn ricoprimenti dove tale linea viene attraversata da almeno unatessera. In questo caso una parte del poligono ha un ricoprimento forzato.

tn sn

Dato che un singolo rettangolo ha 3 ricoprimenti diversi allora tn = 3an−1 per n ≥ 2.Inoltre nei ricoprimenti del secondo tipo ogni ricoprimenti della parte non forzata puo essere estesaunivocamente a un ricoprimento del poligono formato da n − 1 rettangoli e quindi sn ≤ an−1 pern ≥ 2. Infine siccome a1 = 3 e per n ≥ 2

3an−1 = tn ≤ an = tn + sn ≤ 4an−1

si deduce facilmente per induzione che per ogni n intero positivo

3n ≤ an ≤ 3 · 4n−1 < 4n.

Con un ragionamento simile si puo dedurre che la successione (an)n≥1 soddisfa la ricorrenza lineare

an+1 = 4an − 2an−1

e quindi

an =(2 +

√2)n+1 + (2 −

√2)n+1

4.

Esercizio 4. Sia x un numero reale non negativo. Calcolare la somma della serie

∞∑

n=1

(−1)⌊2n

x⌋

2n.

Soluzione.

Sia (ak)k≥1 la rappresentazione diadica di x, la parte frazionaria di x, allora

x = ⌊x⌋ + x = ⌊x⌋ +

∞∑

k=1

ak

2k.

2

Page 230: Eserciziario Di Analisi Matematica 2

Quindi per n ≥ 1 si ha che

2nx = (2n⌊x⌋ + 2n−1a1 + · · · + 2an−1) + an +∞∑

k=1

an−k

2k.

Siccome il numero tra parentesi e pari e la somma della serie e un numero non negativo minore di1 (ogni ai ∈ 0, 1 e non sono definitivamente uguali a 1) allora

(−1)⌊2n

x⌋ = (−1)an = 1 − 2an

e quindi∞∑

n=1

(−1)⌊2n

x⌋

2n=

∞∑

n=1

1 − 2an

2n= 1 − 2

∞∑

n=1

an

2n= 1 − 2x.

Esercizio 5. Dimostrare o confutare che per ogni intero positivo k esiste una potenza di due chein base dieci ha almeno k cifre uguali a 9.

Soluzione.

Dimostriamo un risultato piu generale: per ogni numero intero N ≥ 1 esiste un numero interom ≥ 0 tale che la rappresentazione decimale di 2m

inizia con N .

Basta far vedere che esistono degli interi a, m ≥ 0 tali che

m log10 2 − a ∈ [log10(N), log10(N + 1)).

Siccome log10 2 e un numero irrazionale, per il teorema di Weyl, la successione (n log10 2)n≥1 edensa in [0, 1) e dunque esiste un intero n0 ≥ 1 tale che

0 < n0 log10 2 <1

2(log10(N + 1) − log10(N)) =

1

2log10(1 + 1/N).

Allora e possibile trovare un numero intero k0 ≥ 1 tale che

k0n0 log10 2 ∈ [log10(N), log10(N + 1))

e basta porre m = k0n0 e a = k0⌊n0 log10 2⌋.

Una soluzione alternativa. Il numero 2 e un generatore di Z∗5n e quindi esiste an tale che

2an ≡ −1 (mod 5n)

(si puo prendere an = ϕ(5n)/2 = 2 · 5n−1). Sia m = 2k + a2k allora

2m ≡ 0 ≡ −22k (mod 22k) e 2m ≡ −22k (mod 52k)

e per il Teorema Cinese dei Resti 2m ≡ −22k (mod 102k). Cosı

2m ≡ −22k = 102k − 22k ≡ (10k − 1)10k + (10k − 4k) (mod 102k).

Questo significa che le k cifre decimali che precedono le ultime k sono tutte uguali a 9.

3

Page 231: Eserciziario Di Analisi Matematica 2

Laboratorio di MatematicaFoglio n.2 - 19 novembre 2010

Esercizio 1. Siano n e k interi tali che 1 ≤ k ≤ n. Qual e il numero medio dei cicli di lunghezzak di una permutazione dei numeri 1, 2, . . . , n.

Esercizio 2. Per a > 1, si consideri la serie

S(a) =∞∑

n=0

1an!

,

(i) dimostrare che S(a) e irrazionale per ogni intero a > 1;

(ii) esiste un numero reale a > 1 tale che S(a) e razionale?

Esercizio 3. Sia f ∈ C2(R). Provare che∫ +∞

−∞|f ′′(x)|2 dx ≥ 12

|b− a|3·(f(a)− 2f

(a+ b

2

)+ f(b)

)2

per ogni scelta dei numeri reali distinti a e b.

Esercizio 4. Dati tre numeri interi distinti a, b e c dimostrare o confutare che esiste un polinomioP (x) ∈ Z[x] che permuta ciclicamente tali numeri ossia che:

P (a) = b, P (b) = c e P (c) = a.

Esercizio 5. Sia π(n) la funzione che per ogni intero n ≥ 1 indica il numero di primi nell’intervallo[1, n]. Esistono infiniti numeri interi n ≥ 2 tali che π(n) divide n?

Page 232: Eserciziario Di Analisi Matematica 2

Laboratorio di Matematica

Foglio n.2 - 19 novembre 2010

Esercizio 1. Siano n e k interi tali che 1 ≤ k ≤ n. Qual e il numero medio dei cicli di lunghezzak di una permutazione dei numeri 1, 2, . . . , n.

Soluzione. Sia π una permutazione di Sn. Indichiamo con Cyck(π) l’insieme costituito dainumeri i ∈ 1, 2, . . . , n che appartengono a un ciclo di lunghezza k di π.Il numero medio µn(k) dei cicli di lunghezza k di una permutazione di Sn e dato da

µn(k) =1

n!

π∈Sn

|Cyck(π)|

k=

1

k n!

π∈Sn

n∑

k=1

[i ∈ Cyck(π)] =1

k n!

n∑

k=1

π∈Sn

[i ∈ Cyck(π)]

dove [Q] vale 1 se la proposizione Q e vera e 0 altrimenti. La somma∑

π∈Sn

[i ∈ Cyck(π)]

enumera il numero di permutazioni che hanno il numero i contenuto in un ciclo di lunghezza k ede uguale al prodotto di

(

n−1

k−1

)

· (k−1)! (numero di modi di completare il ciclo) per (n−k)! (numerodi modi di completare la permutazione), ossia (n − 1)!. Cosı

µn(k) =1

k n!

n∑

k=1

(n − 1)! =1

k.

Esercizio 2. Per a > 1, si consideri la serie

S(a) =∞∑

n=0

1

an!,

(i) dimostrare che S(a) e irrazionale per ogni intero a > 1;

(ii) esiste un numero reale a > 1 tale che S(a) e razionale?

Soluzione.

(i) Il numero S(a), espresso in base a, ha un sviluppo infinito costituito da cifre 0 e 1 dove ladistanza tra l’n-simo 1 e il precedente e uguale a n! − (n − 1)!. Dato che tale distanza diventaarbitrariamente grande, lo sviluppo non e periodico e quindi S(a) non e razionale.Una dimostrazione piu diretta e la seguente.Supponiamo per assurdo che S(a) = p/q con p e q interi positivi. Allora per ogni n0 ≥ 1

t := q · an0!

∞∑

n=n0+1

1

an!= p · an0! − q · an0!

n0∑

n=0

1

an!= intero positivo ≥ 1.

D’altra parte, siccome n! − n0! ≥ n − n0, si ha che per n0 sufficientemente grande

t =q

an0·n0!

∞∑

n=n0+1

1

an!−n0!≤

q

an0·n0!

∞∑

n=n0+1

1

an−n0

=q

(a − 1)an0·n0!< 1

in contraddizione con la disequazione precedente.(ii) La serie di potenze

∞∑

n=0

xn!,

dominata dalla serie geometrica di ragione x, converge per |x| < 1 a una funzione continua F (x).Per il Teorema dei Valori Intermedi, per a > 1, S(a) = F (1/a) assume tutti i valori nell’intervallo(F (0), F (1)) = (0, +∞). In particolare esistono infiniti valori di a > 1 per cui S(a) e razionale.

Page 233: Eserciziario Di Analisi Matematica 2

Esercizio 3. Sia f ∈ C2(R). Provare che

∫ +∞

−∞

|f ′′(x)|2

dx ≥12

|b − a|3·

(

f(a) − 2f

(

a + b

2

)

+ f(b)

)2

per ogni scelta dei numeri reali distinti a e b.

Soluzione.

Possiamo supporre che a < b. Sia g ∈ C2(R) allora per la disuguaglianza di Cauchy-Schwarz

∫ b

a

|f ′′(x)|2

dx

∫ b

a

|g(x)|2

dx ≥

(

∫ b

a

f(x)g(x) dx

)2

.

Sia c = (a + b)/2 e posto

g(x) =

x − a per x ∈ [a, c]b − x per x ∈ [c, b]

,

si ha che∫ b

a

|g(x)|2

dx =(b − a)3

12.

Inoltre

∫ b

a

f ′′(x)g(x) dx =

∫ b

a

g(x) d(f ′(x)) = [g(x)f ′(x)]b

a−

∫ c

a

f ′(x)g′(x) dx −

∫ b

c

f ′(x)g′(x) dx

= 0 − f(c) + f(a) + f(b) − f(c) = f(a) − 2f(c) + f(b).

Infine,∫ +∞

−∞

|f ′′(x)|2

dx ≥

∫ b

a

|f ′′(x)|2

dx ≥12

|b − a|3· (f(a) − 2f(c) + f(b))

2.

Esercizio 4. Dati tre numeri interi distinti a, b e c dimostrare o confutare che esiste un polinomioP (x) ∈ Z[x] che permuta ciclicamente tali numeri ossia che:

P (a) = b, P (b) = c e P (c) = a.

Soluzione. Se P (x) ∈ Z[x] e P (a) = b allora esiste un polinomio Q(x) ∈ Z[x] tale che

P (x) − b = (x − a)Q(x).

Quindi

m1 :=c − b

b − a=

P (x) − b

x − a

x=b

= Q(b) ∈ Z.

In modo simile si ottiene che

m2 :=a − c

c − b∈ Z e m3 :=

b − a

a − c∈ Z.

Dato che m1 ·m2 ·m3 = 1 se ne deduce che |m1| = |m2| = |m3| = 1. Cosı |c−b| = |b−a| = |a−c| > 0ovvero a, b, c sono tre punti distinti di una retta ognuno equidistante dagli altri due. Tale condizionenon puo mai essere soddisfatta e dunque abbiamo una contraddizione.

2

Page 234: Eserciziario Di Analisi Matematica 2

Esercizio 5. Sia π(n) la funzione che per ogni intero n ≥ 1 indica il numero di primi nell’intervallo[1, n]. Esistono infiniti numeri interi n ≥ 2 tali che π(n) divide n?

Soluzione.

Sia a ≥ 2 un intero e consideriamo l’insieme

Sa = k ≥ 1 : π(a · k) ≥ k.

L’insieme Sa non e vuoto perche 1 ∈ Sa. Inoltre, siccome π(n) ≈ n/ logn, si ha che

limk→∞

π(a · k)

a · k= 0

e cosı, definitivamente, π(a · k)/(a · k) < 1/a ossia π(a · k) < k. Quindi l’insieme Sa e finito.Sia ka := max Sa, allora π(a · ka) ≥ ka. Se π(a · ka) > ka allora

π(a · (ka + 1)) ≥ π(a · ka) ≥ ka + 1

contro la massimalita di ka (si noti che la funzione π(n) e non decrescente ed e a valori interi).Quindi π(a · ka) = ka e posto na := a · ka, si ha che

a · π(na) = a · π(a · ka) = a · ka = na,

ossia π(na) divide na. Dato che na ≥ a, per l’arbitrarieta della scelta di a, possiamo concludereche esistono infiniti na.

3

Page 235: Eserciziario Di Analisi Matematica 2

Laboratorio di MatematicaFoglio n.3 - 14 gennaio 2010

Esercizio 1. Se si scelgono a caso n punti in un intervallo di lunghezza L, qual e il valore mediodella somma delle distanze di tutte le possibili coppie di punti?

Esercizio 2. Sia f ∈ C1([a, b]). Dimostrare che esistono due punti a < x1 < x2 < b tali che

f ′(x1) · f ′(x2) =(

f(b)− f(a)b− a

)2

.

Esercizio 3. Determinare una formula chiusa per la somma multipla∑A⊂1,...,n

A6=∅

∑B⊂1,...,n

B 6=∅

∑x∈A∪B

x.

Esercizio 4. Sia n = 2010100. Determinare il numero di elementi dell’insieme

Sn =d ∈ N : 1 ≤ d ≤ n, d | n2, d - n

.

Esercizio 5. Sia p un numero primo. Dimostrare che per ogni intero n ≥ 0 si ha che

Lpn ≡ Ln (mod p)

dove Ln e l’n-simo numero di Lucas definito dalla ricorsione

L0 = 2, L1 = 1 e Ln = Ln−1 + Ln−2 per n ≥ 2.

Page 236: Eserciziario Di Analisi Matematica 2

1. Elementi di analisi funzionaleEsercizi

http://www.ciram.unibo.it/~barozzi/MI2/PDF/MI2-Cap.1-Ese.pdf

1.1. Spazi vettoriali

1.2. Spazi vettoriali normati

1.2-1. Dimostrare la diseguaglianza triangolare in Cn relativamente alla norma ‖ · ‖1

(↑ esempio 1).Suggerimento Si ha ‖x + y‖1 =

∑nk=1 |xk + yk| ≤ . . . .

Soluzione. Si ha

‖x + y‖1 =n∑

k=1

|xk + yk| ≤n∑

k=1

(|xk| + |yk|) =

=n∑

k=1

|xk| +n∑

k=1

|yk| = ‖x‖1 + ‖y‖1.

1.2-2. Dimostrare la diseguaglianza triangolare in Cn relativamente alla norma ‖ · ‖∞

(↑ esempio 1).Suggerimento Si parta dal fatto che ‖x + y‖∞ = |xk + yk| per un certo indice k.Soluzione. Si ha

‖x + y‖∞ = |xk + yk| ≤ |xk| + |yk| ≤ maxk

|xk| + maxk

|yk| = ‖x‖∞ + ‖y‖∞.

1.2-3. Per ogni x ∈ Rn mostrare che

limp→∞

‖x‖p = ‖x‖∞ = max1≤k≤n

|xk|.

Suggerimento Se x = 0 non c’e niente da dimostrare. Altrimenti, supposto,per fissare le idee, che la componente di valore assoluto massimo sia la prima, cioe|x1| = ‖x‖∞, si puo scrivere

( n∑k=1

|xk|p)1/p

= |x1|(1 +

n∑k=2

∣∣∣xk

x1

∣∣∣p)1/p

,

dove l’ultima quantita entro parentesi tonde e compresa tra 1 e n.Soluzione. Proseguendo nel ragionamento iniziato nel suggerimento, si ha

1 ≤(1 +

n∑k=2

∣∣∣xk

x1

∣∣∣p)1/p

≤ n1/p,

dove l’ultima quantita tende a 1 per p → ∞.

1.2-4. Verificare che sullo spazio C[a, b] delle funzioni reali continue sull’intervallo[a, b] le norme di indici 1 e ∞ sono effettivamente tali (↑ esempio 2).

Page 237: Eserciziario Di Analisi Matematica 2

2 Capitolo 1. Elementi di analisi funzionale ©c 88-08-07923-6

Suggerimento Si tratta di verificare (per entrambe le norme) la diseguaglianzatriangolare: per la seconda norma utilizzare il teorema di Weierstrass sull’esistenzadel massimo di una funzione continua su un compatto.Soluzione. Si ha

‖f + g‖1 =∫ b

a

|f(x) + g(x)| dx ≤∫ b

a

|f(x)| dx +∫ b

a

|g(x)| dx = ‖f‖1 + ‖g‖1.

Sia poi x ∈ [a, b] un punto tale ‖f + g‖∞ = |f(x) + g(x)|. Si ha

‖f + g‖∞ = |f(x) + g(x)| ≤ |f(x)| + |g(x)| ≤ maxx

|f(x)| + maxx

|g(x)| =

= ‖f‖∞ + ‖g‖∞.

1.2-5. Verificare che per le funzioni costanti sull’intervallo [a, b], f(x) = c, si ha

‖ f ‖1 = (b − a) ‖ f ‖∞.

Soluzione. Si ha infatti

‖ f ‖1 =∫ b

a

|c| dx =∫ b

a

‖ f ‖∞ dx = (b − a) ‖ f ‖∞.

1.2-6. Dimostrare che le norme ‖ · ‖1 e ‖ · ‖∞ sono equivalenti sullo spazio vettorialedelle funzioni polinomiali di grado ≤ 1 sull’intervallo [a, b] (in accordo con la Propo-sizione 1.2-1), calcolando due costanti che consentano di maggiorare una norma conl’altra.

Suggerimento Sia p( · ) un polinomio di grado ≤ 1; posto A := |p(a)|, B := |p(b)|,si cominci con l’osservare che si ha ‖ p ‖∞ = maxA, B,

‖ p ‖1 =∫ b

a

|p(x)| dx =b − a

2(A + B)

se p(a) · p(b) ≥ 0 (in tal caso il grafico di |p| e un segmento), mentre

‖ p ‖1 =∫ b

a

|p(x)| dx =b − a

2A2 + B2

A + B

se p(a) ·p(b) < 0 (in tal caso il grafico di |p| e una spezzata composta da due segmentiugualmente inclinati sull’asse delle ascisse).

aa bb

AA

BB

Soluzione. Con riferimento alla figura di sinistra, e chiaro che si tratta dell’area di untrapezio con basi A e B e altezza b−a. Nel caso della figura di destra, la similitudinetra i due triangoli evidenziati implica che le basi degli stessi triangoli valgono

A

(A + B)(b − a),

B

(A + B)(b − a).

Si ha poi A + B ≤ 2‖ p ‖∞, e per A + B > 0 (in caso contrario p e identicamentenullo e non c’e niente da dimostrare) si ha A2 + B2 ≤ (A + B)2, quindi

Page 238: Eserciziario Di Analisi Matematica 2

©c 88-08-07923-6 Esercizi 3

A2 + B2

A + B≤ A + B.

f1(x, y) = x + y

f2(x, y) = x2 + y2

x+y

1

1

1

2

In conclusione:

‖ p ‖1 ≤ b − a

22 ‖ p ‖∞ = (b − a) ‖ p ‖∞.

Vale il segno di uguaglianza per i polinomi costanti.

1.2-7. Utilizzando le funzioni polinomiali fornite al termine dell’esempio 1.2-2, di-mostrare che le norme ‖ · ‖1 e ‖ · ‖∞ non sono equivalenti sullo spazio vettoriale dellefunzioni polinomiali (senza limitazioni sul grado) sull’intervallo [a, b].Soluzione. Si tratta delle funzioni fn(x) = xn, considerate sull’intervallo [0, 1]. Peresse abbiamo trovato ‖ fn ‖1 = 1/(n+1), ‖ fn ‖∞ = 1. Dunque non esiste una costanteC > 0 tale da aversi ‖ p ‖∞ ≤ C ‖ p ‖1 per ogni funzione polinomiale p.

1.2-8. Sia V = C(1)[a, b] lo spazio delle funzioni continue su [a, b] assieme alla derivataprima; si considerino le norme

‖ f ‖1 := maxa≤x≤b

|f(x)| + maxa≤x≤b

|f ′(x)|,

‖ f ‖2 := |f(a)| + maxa≤x≤b

|f ′(x)|.

Dimostrare che esse sono effettivamente due norme e che sono equivalenti.

Suggerimento Si puo osservare (teorema del valor medio) che

f(x) = f(a) + (x − a) f ′(ξ),

con ξ ∈ [a, b], da cui

|f(x)| ≤ |f(a)| + (b − a) · maxa≤x≤b

|f ′(x)|.

Soluzione. Proseguendo nel ragionamento iniziaro nel suggerimento, si ha

maxx

|f(x)| ≤ |f(a)| + (b − a) · maxx

|f ′(x)|,

da cui

Page 239: Eserciziario Di Analisi Matematica 2

4 Capitolo 1. Elementi di analisi funzionale ©c 88-08-07923-6

‖ f ‖1 ≤ |f(a)| + (b − a) · maxx

|f ′(x)| + maxx

|f ′(x)| =

= |f(a)| + (b − a + 1) maxx

|f ′(x)| <

< (b − a + 1)[|f(a)| + max

x|f ′(x)|

]= (b − a + 1) ‖ f ‖2.

La disuguaglianza ‖ f ‖2 ≤ ‖ f ‖1 e evidente.

1.2-9. Sia V = C[a, b], w una funzione continua e positiva su [a, b]. Dimostrare chela quantita

‖ f ‖w := maxa≤x≤b

|w(x) f(x)|

e una norma e dire se essa e equivalente alla norma del massimo.

Suggerimento Si ha 0 < minx w(x) ≤ w(x) ≤ maxx w(x); sfruttare l’identitaf(x) = w(x) f(w)/w(x).Soluzione. Si ha

‖ f ‖∞ = maxx

|f(x)| = maxx

∣∣∣w(x)f(x)w(x)

∣∣∣ ≤ maxx

∣∣∣w(x)f(x)

minx w(x)

∣∣∣ =

=1

minx w(x)max

x|w(x) f(x)| =

1minx w(x)

‖ f ‖w.

Inversamente

‖ f ‖w = maxx

|w(x) f(x)| ≤ maxx

∣∣( maxx

w(x))f(x)

∣∣ =

= maxx

w(x) · maxx

|f(x)| = maxx

w(x) · ‖ f ‖∞.

1.2-10. Siano ‖ · ‖1 e ‖ · ‖2 due norme equivalenti su V . Sia (xn)n∈N una successionein V e x ∈ V . Verificare che(

‖xn − x‖1 → 0)

⇐⇒(‖xn − x‖2 → 0

).

Soluzione. Infatti per due opportune costanti positive c1 e c2 si ha

‖xn − x‖1 ≤ c2‖xn − x‖2, ‖xn − x‖2 ≤ c1‖xn − x‖1.

1.2-11. Se V e uno s.v.n. reale o complesso, una trasformazione lineare f : V →R (o rispettivamente f : V → C) si chiama semplicemente un funzionale lineare.Si consideri lo spazio C(1)[0, 1] munito della norma del massimo. Verificare che ilfunzionale lineare x( · ) → x′(0) non e limitato, dunque (↑ Proposizione 1.2-2) non econtinuo.

Suggerimento Scegliere, ad esempio, xn(t) = sin nt.Soluzione. Per le funzioni suggerite si ha x′

n(t) = n cos nt, quindi x′n(0) = n, mentre

‖xn ‖∞ = 1 per ogni n.

1.2-12. Si consideri lo spazio C[0, 1] munito della norma del massimo. Controllareche i funzionali lineari x( · ) →

∫ 1

0x(t) dt, e x( · ) → x(t0), per ogni fissato t0 ∈ [0, 1],

sono continui.Soluzione. Si ha∣∣∣

∫ 1

0

x(t) dt∣∣∣ ≤

∫ 1

0

|x(t)| dt ≤∫ 1

0

‖x ‖∞ dt = ‖x ‖∞.

Si ha poi |x(t0)| ≤ maxx |x(t)| = ‖x ‖∞.

1.2-13. Si considerino gli spazi V = C(1)[0, 1] e W = C[0, 1] muniti entrambi dellanorma del massimo. Controllare che l’operatore lineare x( · ) → x′( · ) non e continuoda V a W (considerare, ad esempio, le funzioni xn(t) := (sin nt)/

√n ).

Soluzione. Per le funzioni suggerite si ha ‖xn ‖∞ = 1/√

n, ‖x′n ‖∞ =

√n.

Page 240: Eserciziario Di Analisi Matematica 2

©c 88-08-07923-6 Esercizi 5

1.2-14. Si consideri lo spazio V = C[0, 1] munito della norma del massimo. Con-trollare che l’operatore lineare di V in se che ad x( · ) ∈ V associa la funzionet →

∫ t

0x(s) ds, t ∈ [0, 1], e continuo.

Soluzione. Posto X(t) :=∫ t

0x(s) ds si ha infatti

‖X ‖∞ = maxt

∣∣∣∫ t

0

x(s) ds∣∣∣ ≤ max

t

∫ t

0

|x(s)| ds =∫ 1

0

|x(s)| ds =

= ‖x ‖1 ≤ 1 · ‖x ‖∞ = ‖x ‖∞.

Dunque l’operatore lineare in esame e continuo anche da C[0, 1] munito della normadi indice 1 allo stesso spazio munito della norma del massimo.

1.2-15. Si consideri lo spazio C[0, 1] munito della norma del massimo (norma diindice ∞). Controllare se sono chiusi in tale spazio gli insiemi (alcuni dei quali sonosottospazi) costituiti dalle funzioni x( · ) tali che:

a) x(0) = 0;b) x(0) = x(1).c) x e non negativa;d) l’integrale di x su [0, 1] e nullo;e) l’integrale di x su [0, 1] e non negativo;f) x e derivabile in 1/2 e x′(1/2) = 0;g) x e costante su [0, 1].

Suggerimento Per f) si consideri, ad esempio, la successione di funzioni

xn(t) :=

√1n2

+(t − 1

2

)2

.

Soluzione. Si tratta di verificare se, data una successione (xn) convergente uni-formemente ad una funzione limite x, dal fatto che tutte le xn verificano una dellecondizioni dalla a) alla g) segue (o meno) che la stessa condizione e verificata dallafunzione limite x. La risposta e affermativa tranne nel caso f); infatti la funzionelimite della successione suggerita e |x − 1/2| che non e derivabile nel punto 1/2.

Si consideri, alternativamente, la successione

xn(x) :=1n

sin(n(t − 1/2)

)− t;

essa converge uniformemente alla funzione t → −t, derivabile nel punto 1/2 conderivata uguale a −1, pur essendo x′

n(1/2) = 0 per ogni n.

1.2-16. Stesso problema del precedente esercizio per gli insiemi costituiti dalle fun-zioni x( · ) tali che:

a) x e un polinomio di grado ≤ 2;b) x e un polinomio di grado esattamente 2;c) x e un polinomio.

Suggerimento Si tenga presente che la convergenza uniforme implica la convergenzapuntuale. Scelti tre valori distinti della variabile indipendente, ad esempio t = 0,t = 1/2, t = 1, se xn(t) := ant2 + bnt + cn e una successione di polinomi di grado ≤ 2convergente uniformemente su [0, 1], dalla convergenza delle tre successioni

n → xn(0) = cn,

n → xn(1/2) = an/4 + bn/2 + cn,

n → xn(1) = an + bn + cn,

dedurre la convergenza delle successioni n → an, n → bn (oltre a quella della succes-sione n → cn).

Page 241: Eserciziario Di Analisi Matematica 2

6 Capitolo 1. Elementi di analisi funzionale ©c 88-08-07923-6

Per c) si considerino i polinomi di Taylor della funzione seno.

Soluzione. Proseguendo secondo le linee fornite dal suggerimento, si osserva che ilsistema ottenuto si scrive

0 0 11/4 1/2 11 1 1

an

bn

cn

=

xn(0)

xn(1/2)xn(1)

,

la cui soluzione e data da an

bn

cn

=

2 −4 2−3 4 11 0 0

xn(0)

xn(1/2)xn(1)

.

Dunque la convergenza delle successioni(xn(0)

),(xn(1/2)

),(xn(1)

)implica la con-

vergenza delle successioni (an), (bn) e (cn). Dunque la risposta alla domanda a) eaffermativa.Al contrario la risposta alla domanda b) e negativa: la successione dei polinomin → x2/n converge uniformemente a 0 sull’intervallo [0, 1] (di fatto su ogni intervallocompatto).Lo stesso per la domanda c). la successione dei polinomi di Taylor della funzioneseno (in breve: la serie di Taylor) converge uniformemente a sinx su ogni intervallocompatto, e la funzione limite non e polinomiale. Si tenga presente che se Tn(x) e ilpolinomio di Taylor relativo alla funzione seno (punto iniziale x = 0) per il relativoresto si ha l’espressione secondo Lagrange

rn(x) =xn+1

(n + 1)!f (n+1)(ξ) =⇒ |rn(x)| ≤ |x|n+1

(n + 1)!,

in quanto le derivate successive della funzione seno sono del tipo ± sin x, ± cos x.

1.2-17. Sia limn→∞ xn = x nello s.v.n. V ; dimostrare che limn→∞ ‖xn‖ = ‖x‖.Suggerimento Utilizzare la diseguaglianza (3′).Soluzione. Si ha infatti

∣∣‖xn‖ − ‖x‖∣∣ ≤ ‖xn − x‖.

1.2-18. Sia limn→∞ xn = x nello s.v.n. V ; dimostrare che esiste una costante C percui ‖xn‖ < C, ∀n. A parole: ogni successione convergente e limitata in norma.Soluzione. Il precedente esercizio ci assicura che la successione n → ‖xn‖ convergea ‖x‖. Scelto ε > 0 esiste nε tale che per ogni n > nε si ha

‖xn‖ ∈ [ ‖x ‖ − ε, ‖x ‖ + ε ].

Al di fuori dell’intervallo appena considerato restano dunque, al piu, le norme deiprimi nε elementi. bastera prendere

a := min‖x1‖, . . . , ‖xnε‖, ‖x‖ − ε, b := max‖x1‖, . . . , ‖xnε‖, ‖x‖ + εper avere un intervallo [a, b] che contiene tutte le norme.

1.2-19. Sia limn→∞ xn = x nello s.v.n. complesso V , limn→∞ an = a in C; dimostrareche limn→∞ anxn = ax.

Suggerimento Da anxn − ax = anxn − anx + anx − ax segue ... .Soluzione. . . . segue

‖ anxn − ax ‖ ≤ ‖ anxn − anx ‖ + ‖ anx − ax ‖ == |an| ‖xn − x ‖ + |an − a| ‖x ‖;

la successione (|an|) e limitata in quanto convergente, dunque il secondo membrotende a 0 per n → ∞.

Page 242: Eserciziario Di Analisi Matematica 2

©c 88-08-07923-6 Esercizi 7

1.3. Spazi vettoriali con prodotto scalare

1.3-1. Verificare che ponendo in Cn (come in R

n) (x |y) :=∑n

k=1 xkyk, avremmointrodotto una definizione scorretta.

Suggerimento Si consideri il vettore avente tutte le componenti uguali all’unitaimmaginaria i.Soluzione. Per il vettore in questione si avrebbe infatti (x |x) = −n.

1.3-2. Dimostrare che vale il segno di uguaglianza nella diseguaglianza di Cauchy-Schwarz se e solo se i vettori x e y sono linearmente dipendenti. Se x e y sono vettorinon nulli, verificare che vale il segno di uguaglianza nella diseguaglianza triangolarese e solo se x = ty, con t > 0.Soluzione. Con riferimento alla dimostrazione della Proposizione 1.3-2, e chiaro chavale il segno di = nella diseguaglianza di Cauchy-Schwarz se si ha 0 = ‖αx+y‖2, conα = −(y |x)/‖x ‖2, dunque se αx + y = 0, cioe i due vettori dati sono linearmentedipendenti. Inversamente, sa x = λy, allora (x |y) = λ ‖y ‖2, da cui

|(x |y)| = |λ| ‖y ‖ ‖y ‖ = ‖x ‖ ‖y ‖.Perche valga il segno di = nella diseguaglianza triangolare occorre che in tutti i pas-saggi della dimostrazione i segni ≤ siano sostituiti da =. Questo richiede che valgail segno di = nella diseguaglianza di Cauchy-Schwarz e dunque deve essere x = λy.Deve poi essere Re(x |y) = |(x |y)|, cioe Re(λ) = |λ|, dunque λ > 0.

1.3-3. Dimostrare che in uno s.v. con prodotto scalare il prodotto scalare stesso euna funzione continua, nel senso che se

xn → x, yn → y

(vale a dire ‖xn − x‖ → 0, ‖yn − y‖ → 0 per n → +∞), allora (xn |yn) → (x |y).

Suggerimento Si utilizzi la diseguaglianza di Cauchy-Schwarz.Soluzione. Si ha

(xn |yn) − (x |y) = (xn |yn) − (xn |y) + (xn |y) − (x |y) == (xn |yn − y) + (xn − x |y).

Dalla diseguaglianza triangolare e da quella di Cauchy-Schwarz segue allora

|(xn |yn) − (x |y)| ≤ |(xn |yn − y)| + |(xn − x |y)| ≤≤ ‖xn‖ ‖yn − y‖ + ‖xn − x‖ ‖y‖.

Il risultato segue allora dalle ipotesi, tenendo conto del fatto che la successione n →‖xn‖ e limitata in quanto convergente (v. esercizio 1.2-18).

1.3-4. Dedurre dalle identita (9) che, se la norma ‖ · ‖ sullo spazio vettoriale Ve hilbertiana, cioe indotta dal prodotto scalare ( · | · ), allora tale prodotto scalare eunivocamente individuato dalla norma mediante l’uguaglianza

(x |y) =14

[‖x + y‖2 − ‖x − y‖2

]se V e reale, dall’uguaglianza

(x |y) =14

[‖x + y‖2 − ‖x − y‖2 + i ‖x + iy‖2 − i ‖x − iy‖2

]se V e complesso.Soluzione. Basta sviluppare i secondi membri. Si ha, ad esempio,

14

[‖x + y‖2 − ‖x − y‖2

]=

14

[ (x + y |x + y) − (x − y |x − y)] =

=14

[ 2(x |y) + 2(x |y)] = (x |y).

Page 243: Eserciziario Di Analisi Matematica 2

8 Capitolo 1. Elementi di analisi funzionale ©c 88-08-07923-6

1.3-5. Dimostrare che le norme ‖ · ‖1 e ‖ · ‖∞ su Rn, n ≥ 2 (↑ esempio 1.2-1) non

verificano l’identita del parallelogramma, e dunque non sono norme hilbertiane.

Suggerimento Prendere x = e1 = (1, 0, 0, . . . , 0), y = e2 = (0, 1, 0, . . . , 0).Soluzione. Per i vettori in questione si ha infatti

‖ e1 ‖1 = ‖ e2 ‖1 = 1, ‖ e1 + e2‖1 = ‖ e1 − e2‖1 = 2;

‖ e1 ‖∞ = ‖ e2 ‖∞ = ‖ e1 + e2‖∞ = ‖ e1 − e2‖∞ = 1.

1.3-6. Dimostrare che le norme ‖ · ‖1 e ‖ · ‖∞ su C[0, 1] (↑ esempio 1.2-2) nonverificano l’identita del parallelogramma, e dunque non sono norme hilbertiane.

Suggerimento Prendere, ad esempio, f(x) = x, g(x) = 1 − x.Soluzione. Sia ha f(x) + g(x) = 1, f(x) − g(x) = 2x − 1, da cui facilmente

‖ f ‖1 = ‖ g ‖1 = 1, , ‖ f + g ‖1 = 1, ‖ f − g ‖1 = 1/2;

‖ f ‖∞ = ‖ g ‖∞ = ‖ f + g ‖∞ = ‖ f − g ‖∞ = 1.

1.3-7. Sia V uno spazio vettoriale con prodotto scalare ( · | · ). Verificare che se V ereale, allora(

‖u + v‖2 = ‖u‖2 + ‖v‖2)

⇐⇒((u|v) = 0

),

mentre se V e complesso, allora(‖u + v‖2 = ‖u‖2 + ‖v‖2

)⇐⇒

(Re(u|v) = 0

).

Costruire due vettori non nulli di Cn per cui valga l’uguaglianza

‖u + v‖2 = ‖u‖2 + ‖v‖2

ma (u|v) = 0 (s’intende di utilizzare il prodotto scalare canonico e la relativa norma).Soluzione. Si ha infatti

‖u + v‖2 = (u + v|u + v) = ‖u‖2 + ‖v‖2 + (u|v) + (v|u),

dove la somma degli ultimi due addendi vale 2(Re(u|v) se si tratta di uno spazio

complesso, e semplicemente 2(u|v) se si tratta di uno spazio reale. Per i vettoriu = (i, i, . . . , i) e v = (1, 1, . . . , 1) si ha Re(u|v) = 0 ma (u|v) = 0.

1.3-8. Utilizzando le stesse funzioni dell’esempio 1.2-16, si verifichi che lo spazioC

([−1, 1], R

)non e completo rispetto alla norma indotta dal prodotto scalare (f |g) :=

=∫ b

af(x)g(x) dx.

Suggerimento Si tenga presente che, se |f(x)| ≤ 1, allora |f(x)|2 ≤ |f(x)|.Soluzione. Si tratta di dimostrare che la successione di funzioni (fn) consideratanell’esempio citato e di Cauchy rispetto alla norma di indice 2. Ora si ha (con isimboli dell’esempio)

‖fn − fm‖22 = ‖(fn − f) + (f − fm)‖2

2 ≤(‖fn − f‖2 + ‖fm − f‖2

)2 ≤≤ 2

(‖fn − f‖2

2 + ‖fm − f‖22

)=

= 2(∫ 1

−1

|fn(x) − f(x)|2 dx +∫ 1

−1

|f(x) − fm(x)|2 dx

)≤

= 2(∫ 1

−1

|fn(x) − f(x)| dx +∫ 1

−1

|f(x) − fm(x)| dx

)=

2n

+2m

.

Abbiamo sfruttato il fatto che |fn(x) − f(x)| ≤ 1 e |fm(x) − f(x)| ≤ 1, dunque lefunzioni ai primi membri sono ≥ dei rispettivi quadrati. Si e utilizzata inoltre ladisuguaglianza (A + B)2 ≤ 2(A2 + B2), dove A, B ≥ 0.

Page 244: Eserciziario Di Analisi Matematica 2

©c 88-08-07923-6 Esercizi 9

1.3-9. Utilizzando la diseguaglianza di Cauchy-Schwarz nello spazio C([a, b]; C

)rela-

tivamente alle funzioni x → |f(x)| e x → 1, dove f e una qualsivoglia funzione dellospazio in esame, dimostrare la diseguaglianza ‖f‖1 ≤

√b − a ‖f‖2.

Soluzione. Si ha infatti

‖ f ‖1 =∫ b

a

|f(x)| dx =∫ b

a

|f(x)| · 1 dx ≤ ‖ f ‖2 ‖ 1 ‖2 =√

b − a ‖ f ‖2.

1.4. Proiezioni ortogonali

Esercizi proposti

1.4-P.1. Sia V = Rn munito del prodotto scalare canonico, V1 il sottospazio di V di

dimensione 1 generato dal vettore v1 := (1, 1, . . . , 1). Un vettore x = (x1, x2, . . . , xn)e ortogonale a v1 (dunque appartiene a V ⊥

1 ) se e solo se∑n

k=1 xk = 0.Verificare che la proiezione ortogonale di x su V1 e data dal vettore con tutte lecomponenti uguali alla media aritmetica delle componenti di x stesso.

1.4-P.2. Sia V = R8 munito del prodotto scalare canonico e V2 sia il sottospazio di

V di dimensione 2 generato dai vettori

v1 = ( 1, 1, . . . , 1︸ ︷︷ ︸8 componenti

), v2 = (1, 2, 3, . . . , 8).

V2 e costituito da tutti (e soltanto) i vettori di R8 le cui componenti sono in “pro-

gressione aritmetica”: xk − xk−1 = costante, per k = 2, 3, . . . , 8.Verificare che il procedimento di Gram-Schmidt, applicato ai vettori dati, produce

i vettori

z1 = v1, z2 = v2 − (9/2)v1 = (1 − 9/2, 2 − 9/2, . . . , 8 − 9/2)

dove 9/2 e la media aritmetica delle componenti di v2.

1 2 3 4 5 6 7 8

2

4

6

8

1 2 3 4 5 6 7 8

-3-2-1

123

A sinistra i vettori v1 e v2, a destra i vettori z1 e z2.

La proiezione ortogonale di x = (x1, x2, . . . , xn) su V2 si scrive dunque

x · z1

‖z1‖2z1 +

x · z2

‖z2‖2z2 =

∑8k=1 xk

8z1 +

∑8k=1 k xk

42z2

42 essendo il quadrato della norma di z2.

1.4-P.3. Generalizzare i risultati del precedente esercizio al caso dello spazio Rn, con

n qualunque ≥ 2. Ora abbiamo

v1 = ( 1, 1, . . . , 1︸ ︷︷ ︸n componenti

), v2 = (1, 2, 3, . . . , n),

da cui segue

z1 = v1, z2 = (1 − m, 2 − m, . . . , n − m)

Page 245: Eserciziario Di Analisi Matematica 2

10 Capitolo 1. Elementi di analisi funzionale ©c 88-08-07923-6

dove m = (n + 1)/2 e la media ritmetica delle componenti di v2. Il quadrato dellanorma di z1 vale n, mentre il quadrato della norma di z2 vale n(n2 − 1)/12. Pereffettuare quest’ultimo calcolo occorre ricordare la formula (v. PCAM, pag. 54)

n∑k=1

k2 =n(n + 1)(2n + 1)

6.

1.4-P.4. Consideriamo i polinomi p0(x) = 1, p1(x) = x, p2(x) = x2 nello spaziodelle funzioni continue sull’intervallo [0, 2], munito del consueto prodotto scalare(f |g) =

∫ 2

0f(x)g(x) dx.

Applicare ad essi il procedimento di Gram-Schmidt (Prop. 1.4-3), ottenendo i polinomiq0(x), q1(x), q2(x). Si ottiene una base ortogonale del sottospazio P2 costituito daipolinomi di grado ≤ 2. Utilizzando tale base, calcolare il polinomio di secondo gradoche meglio approssima la funzione f(x) = x3 nello spazio considerato.Suggerimento Utilizzare la Proposizione 1.3-4.)

Soluzione. Si trova

q0(x) = 1, q1(x) = x − 1, q1(x) = x2 − 2x + 2/3.

Il polinomio di migliore approssimazione e p(x) = c0 q0(x) + c1 q1(x) + c2 q2(x) conck = (f |qk)/(qk|qk), k = 0, 1, 2, quindi

c0 = 2, c1 = 18/5, c2 = 3,

da cui finalmente p(x) = 3x2 − 12/5 x + 2/5.

Page 246: Eserciziario Di Analisi Matematica 2

2. Elementi di teoria dell’integrazioneEsercizi

http://www.ciram.unibo.it/~barozzi/MI2/PDF/MI2-Cap.2-Ese.pdf

2.1. Richiami sull’integrale di Riemann

2.2. La misura di Lebesgue

2.2-1. Ritocchiamo la definizione di pluri-intervallo, convenendo di considerare sol-tanto unioni finite di intervalli: P =

⋃Nk=1 Ik, con N naturale positivo ad arbitrio. In

conseguenza di tale scelta, definiamo una “misura esterna”

me(E) := infP

m(P ),

intendendo che l’estremo inferiore venga considerato al variare di P tra i pluri-intervalli che “coprono” E.

Una tale misura esterna viene spesso associata ai nomi di G. Peano (1858-1932)e C. Jordan (1838-1922). Se E = [0, 1] ∩ Q (↑ esempio 2.2-1), si trova

me(E) = me

([0, 1] \ E

)= me

([0, 1]

)= 1,

dunque E non e “misurabile” nel senso di Peano-Jordan, in quanto non vale l’analogadell’identita (6).

Soluzione. Se P =⋃N

k=1 Ik, con N ∈ N∗, possiamo riscrivere P nella forma P ==

⋃N ′

k=1 I ′k, con N ′ ≤ N , dove gli intervalli I ′k sono a due a due disgiunti. Si tengapresente che se l’intersezione tra due intervalli non e vuota, la loro unione e ancoraun intervallo.

Ora tanto E (insieme dei razionali dell’intervallo [0, 1]) quanto [0, 1] \ E (insiemedegli irrazionali dell’intervallo [0, 1]) sono densi nell’intervallo [0, 1], quindi sono con-tenuti in un plurintervallo del tipo P =

⋃N ′

k=1 I ′k solo se [0, 1] e contenuto in uno degliintervalli I ′k. Questo implica che m(P ) ≥ 1. D’altra parte l’intervallo [0, 1], di misura1, contiene tanto E quanto [0, 1] \ E, e questo prova che, per entrambi gli insiemi, lamisura esterna vale 1.

2.2-2. Vogliamo definire l’insieme ternario di Cantor per sottrazioni successive a par-tire dall’intervallo I = [0, 1]. Sia P1 l’insieme che si ottiene togliendo da I l’intervalloaperto concentrico di ampiezza 1/3:

P1 = I \ (1/3, 2/3) = [0, 1/3] ∪ [2/3, 1].

Da ciascuno dei due intervalli di cui si compone P1 togliamo l’intervallo aperto con-centrico di ampiezza pari ad 1/3 dell’ampiezza dello stesso intervallo. Dunque P2 el’unione di 4 intervalli chiusi di lunghezza 1/9, quindi m(P2) = 4/9 = (2/3)2.Proseguiamo sempre col medesimo criterio, togliendo da ciascuno dei 2n intervalli dicui si compone Pn l’intervallo aperto concentrico di ampiezza pari a 1/3 dell’intervallostesso. Si trova, per ogni n, m(Pn) = (2/3)n.

Page 247: Eserciziario Di Analisi Matematica 2

2 Capitolo 2. Elementi di teoria dell’integrazione ©c 88-08-07923-6

0 1

Sia P l’intersezione di tutti i pluri-intervalli Pn: P :=⋂

n≥1 Pn. Verificare che P edi misura nulla, anzi e tale anche rispetto alla “misura di Peano-Jordan” consideratanel precedente esercizio.

Soluzione. Chiaramente P ⊂ Pn per ogni n, dove Pn e un pluri-intevallo unione di2n intervalli disgiunti, di misura (2/3)n. Ma limn→∞(2/3)n = 0, quindi P puo esserericoperta da un pluri-intervallo (unione di un numero finito di intervalli) di misuraarbitrariamente piccola. Cio corrisponde al fatto che la misura degli intervalli chesono stati rimossi vale

13

+29

+427

+881

+ . . . =13

(1 +

23

+22

32+

23

33+ . . .

)=

=13

∑n≥0

(23

)n

=13

11 − 2/3

= 1

2.2-3. Verificare che l’insieme di Cantor del precedente esercizio puo essere identi-ficato con l’insieme dei numeri reali dell’intervallo [0, 1] che possono essere espressiin base tre senza utilizzare la cifra uno. Ad esempio: 1 = (0.2)3, 2/3 = (0.2)3,1/3 = (0.02)3.

Soluzione. Consideriamo la rappresentazione ternaria (= in base tre) dei numeridell’intervallo [0, 1]. Poiche 1/3 si scrive (0.1)3 e 2/3 si scrive (0.2)3, vediamo che alprimo passo, quando rimuoviamo il terzo intermedio dell’intervallo [0, 1], vengono toltii numeri nella cui rappresentazione ternaria compare la cifra 1 nella prima posizionedelle parte frazionaria. Se si tiene presente che 1/3 (punto che non viene rimosso) siscrive anche (0.02)3 possiamo dire che vengono rimossi tutti i numeri che presentano1 nella prima posizione della parte frazionaria.

Al secondo passo vengono rimossi tutti i punti che presentano la cifra 1 nellaseconda posizione della parte frazionaria e cosı via.

In generale, i secondi estremi degli intervalli che costituiscono il pluri-intervalloPn hanno rappresentazioni ternarie limitate in cui l’ultima cifra vale 1 mentre le(eventuali) cifre precedenti valgono 0 oppure 2. Ciascuna di tali rappresentazioniequivale a quella che si ottiene srivendo 0 al posto di 1, seguito da una successione dicifre tutte uguali a 2. Ad esempio il numero 7/9 (secondo estremo del terzo intervallodi P2) si scrive

79

=23

+19

= (0.21)3 = (0.2022222 . . .)3 = (0.202)3.

Ci si puo chiede quali punti restino nell’insieme di Cantor, oltre agli estremi degliintervalli che costituiscono ciascun Pn. Un esempio e fornito dal numero 1/4, la cuirappresentazione ternaria e (0.02)3. Per convincersene basta scrive 4 in base 3, cioe4 = (11)3, ed iniziare la divisione di 1 per 4 facendo i calcoli in base 3. Dopo duepassi si sono ottenute le cifre 0 e 2 e nuovamente il resto 1:

Page 248: Eserciziario Di Analisi Matematica 2

©c 88-08-07923-6 Esercizi 3

1 1110 0.02100221

Di fatto l’insieme di Cantor, pur essendo di misura nulla, ha la stessa cardinalitadell’intervallo [0, 1], cioe puo essere posto in corrispondenza biunivoca con tale inter-vallo. Per convincersene basta considerare la corrispondenza che, ad ogni allineamentoternario costituito soltanto dalle cifre 0 e 2, associa l’allineamento binario ottenutoscrivendo 1 al posto di 2. Ad esempio, al numero

(0.0202020202 . . .)3 = (0.02)3 = 1/4

viene associato il numero

(0.0101010101 . . .)2 = (0.01)2 = 1/3.

Poiche l’insieme di Cantor contiene tutti (e soltanto) i numeri che hanno rap-presentazioni ternarie costituite da una parte intera nulla e dalle cifre 0 e 2, si otten-gono come corrispondenti tutti gli allineamenti binari con parte intera nulla, dunquetutti i punti dell’intervallo [0, 1].

Se ϕ e la corrispondenza appena considerata, che trasforma l’insieme di CantorC sull’intervallo [0, 1], possiamo prolungarla in una funzione dell’intervallo [0, 1] su sestesso convenendo che ϕ sia costante su ciascuno degli intervalli che costituiscono ilcomplementare dell’insieme C, e precisamente sia uguale al valore che ϕ assume incorrispondenza degli estremi dello stesso intervallo.

Ad esempio, poiche 1/3 e 2/3 hanno le rappresentazioni (0.022222 . . .)3 e (0.2)3, acui corrispondono la rappresentazioni binarie (0.0111111 . . .)2 e (0.1)2 rispettivamente,e queste rappresentano entrambe 1/2, allora la funzione ϕ varra 1/2 nell’intervallo[1/3, 2/3].

La funzione cosı costruita coincide con quella che viene introdotta e studiatanell’esempio 2.3-8, dove viene chiamata V . Il suo grafico e la “scala del diavolo”.

2.2-4. Dimostrare che, in R2, ogni segmento con i lati paralleli agli assi coordinati edi misura nulla.

Soluzione. Sia L > 0 la lunghezza del segmento in esame; dato ε > 0 ad arbitrio,possiamo coprire il segmento con un unico intervallo bidimensionale (un rettangolo)di dimensioni L e ε/L.

2.2-5. Si consideri in R2 un segmento non parallelo agli assi coordinati; con riferi-mento alla figura 2.2-7, si verifichi che la misura del pluri-intervallo (= pluri-rettan-golo) che ricopre il segmento vale (L2 sinα cos α)/n, dove L e la lunghezza del seg-mento, α la misura dell’angolo che esso forma con l’asse delle ascisse, n il numero diparti uguali in cui lo stesso segmento e stato suddiviso.

α

Figura 2.2-7.

La misura del pluri-intervallo

in figura si puo rendere

arbitrariamente piccola

prendendo convenientemente

grande il numero degli

intervalli (= rettangoli) che lo

compongono.

Page 249: Eserciziario Di Analisi Matematica 2

4 Capitolo 2. Elementi di teoria dell’integrazione ©c 88-08-07923-6

Se ne deduca che il segmento in questione e di misura nulla in R2. Si dimostri cheogni retta ha misura nulla in R2.Soluzione. Ciascuno dei rettangoli in figura ha area

L

ncos α

L

nsinα

quindi il plurirettangolo mostrato ha misura (L2 sinα cos α)/n → 0 per n → ∞.Quanto al caso di una retta nel piano, possiamo pensarla come unione di un’infi-

nita numerabile di segmenti di lunghezza L (peraltro arbitraria) e applicare il risultatocontenuto nell’esercizio seguente.

2.2-6. Dimostrare che l’unione di un’infinita numerabile di insiemi di misura nulla eancora di misura nulla.Soluzione. Sia E = ∪n∈N∗En , dove ciascuno degli insieme En ha misura nulla. Sceltoε > 0, copriamo En con un plurintervallo Pn, unione di un’infinita numerabile diintervalli In,k di misura complessiva ≤ ε/2n. Allora

⋃∞n=1

( ⋃∞k=1 In,k

)e una famiglia

numerabile di intervalli di misura complessiva non superiore a ε.

2.2-7. Sia f : E → R una funzione misurabile, e siano f+j e f−

j due successionicrescenti di funzioni semplici non negative, convergenti q.o. a f+ e f− rispettivamente.Verificare che f+

j + f−j e una successione crescente di funzioni semplici convergente

q.o. a |f |; se quest’ultima e sommabile, dalle relazioni∫E

f+j (x) dx ≤

∫E

(f+

j (x) + f−j (x)

)dx ≤

∫E

|f(x)| dx,

∫E

f−j (x) dx ≤

∫E

(f+

j (x) + f−j (x)

)dx ≤

∫E

|f(x)| dx

dedurre la sommabilita di f+ e f− e quindi la sommabilita di f .

Soluzione. Ciascuna funzione f+j + f−

j e semplice in quanto somma di funzionisemplci; la successione j → f+

j + f−j e crescente in quanto somma di successioni

crescenti. Per ottenere la sommabilita di f+ e f− si puo ricorrere al teorema di B.Levi.

2.2-8. Sia f : R+ → R una funzione sommabile; dimostrare che se esiste il limitelimx→+∞ f(x), esso e necessariamente uguale a 0.Suggerimento Sia, per assurdo, limx→+∞ f(x) = λ > 0; allora per tutti gli xabbastanza grandi, diciamo per x > xλ, si ha f(x) > λ/2. Per gli stessi x avremmo∫ x

0

f(t) dt =∫ xλ

0

f(t) dt +∫ x

f(t) dt >

∫ xλ

0

f(t) dt +λ

2(x − xλ),

quindi per x → +∞ . . . .

Soluzione. Proseguendo nel ragionamento iniziato nel suggerimento, nelle ipotesiammesse avremmo limx→+∞

∫ x

0f(t) dt = +∞, in quanto il prodotto λ/2 (x − xλ)

tende a +∞, contro l’ipotesi di sommabilita della funzione f . Un ragionamento deltutto analogo vale se limx→+∞ f(x) = λ < 0.

2.2-9. Si consideri la funzione caratteristica del pluri-intervallo di misura finita (↓esercizio 3.2-6) P :=

⋃n≥1[n − 1/n2, n] ; se ne concluda che una funzione sommabile

su R+ puo essere priva di limite all’infinito.

Soluzione. La sommabilita della funzione in esame, sia f , equivale alla convergenzadella serie armonica generalizzata∑

n≥1

1n2

,

Page 250: Eserciziario Di Analisi Matematica 2

©c 88-08-07923-6 Esercizi 5

verificata in PCAM, Esempio 6.2-2, ed anche nell’esercizio citato, dove si dimostrache la somma delle serie vale π2/6.

10 2 3 4

1

Chiaramente la funzione in esame e priva di limite per x → ∞: comunque si fissi unvalore x, esistono valori di x > x tali f(x) = 0, ed altri tali che f(x) = 1.

2.3. L’integrale di Lebesgue

2.3-1.P. Dedurre dal teorema della convergenza dominata di Lebesgue il seguenteenunciato:

Sia f(x, t) una funzione definita per x ∈ E ⊂ Rn , t ∈ I intervallo di R. Suppo-niamo f derivabile parzialmente rispetto a t per ogni t e si abbia

|ft(x, t)| ≤ g(x) q.o. in E

con g sommabile su E. Allorad

dt

∫E

f(x, t) dx =∫

E

ft(x, t) dx

Soluzione. Fissiamo t ∈ I e scegliamo una successione infinitesima hn tale chet + hn ∈ I. Posto

fn(x) :=f(x, t + hn) − f(x, t)

hn,

la tesi significa che

limn→∞

∫E

fn(x) dx =∫

E

limn→∞

fn(x) dx.

Si puo scrivere

f(x, t + hn) − f(x, t)hn

=∂f(x, t)

∂t

∣∣∣t=τ

per un opportuno τ , dunque per q.o. x si ha |fn(x)! ≤ g(x). A questo punto possiamoapplicare il teorema della convergenza dominata.Nota. Applicando il teorema appena dimostrato si ha una dimestrazione diretta dellaProposizione 6.2-5, relativa alla derivata della trasformata di Fourier.

2.4. Spazi di funzioni sommabili

2.4-1. Verificare che il predicato “x e irrazionale”, e verificato q.o. su R. Altrettantoper il predicato “x non e intero”.

Soluzione. Infatti il primo predicato e falso in corrispondenza dei numeri razionali,il cui insieme, in quanto numerabile, e di misura nulla. Lo stesso vale per l’insiemedegli interi in cui e falso il secondo predicato.

2.4-2. Verificare che q.o. su R si ha limn→∞ e−n|x| = 0, e precisamente ... .

Page 251: Eserciziario Di Analisi Matematica 2

6 Capitolo 2. Elementi di teoria dell’integrazione ©c 88-08-07923-6

Soluzione. L’unico punto in cui il limite in questione non e 0 e l’origine.

2.4-3. Sia f : R+ → R la funzione f(x) = 1/x (funzione definita q.o. su R+);procedendo come nell’esempio 2.3-4, verificare che

f /∈ L1[0, 1], f /∈ L1[1,+∞),

dove abbiamo indicato semplicemente con f la restrizione della stessa funzione agliintervalli [0, 1] e rispettivamente [1,+∞).

Soluzione. Infatti le successioni

n →∫ 1

1/n

1x

dx = log n, n →∫ n

1

1x

dx = log n

sono divergenti.

2.4-4. Sia fε : R+ → R la funzione fε(x) = 1/x1+ε, ε > 0. Verificare che

fε /∈ L1[0, 1], fε ∈ L1[1,+∞).

Ad esempio x → 1/x2 e sommabile su [1,∞), non e sommabile su [0, 1].

Soluzione. Una primitiva di fε si scrive

Fε(x) = −1ε

1xε

,

quindi∫ 1

1/n

fε(x) dx = −1ε

[1 − nε

]→ +∞ per n → ∞,

∫ n

1

fε(x) dx = −1ε

[ 1nε

− 1]→ 1

εper n → ∞.

2.4-5. Sia f−ε : R+ → R la funzione f−ε(x) = 1/x1−ε, 0 < ε < 1. Verificare che

f−ε ∈ L1[0, 1], f−ε /∈ L1[1,+∞).

Ad esempio x → 1/√

x non e sommabile su [1,∞), mentre e sommabile su [0, 1].

Soluzione. Una primitiva di f−ε si scrive F−ε(x) = xε/ε, quindi∫ 1

1/n

f−ε(x) dx =1ε

[1 − 1

]→ 1

εper n → ∞,

∫ n

1

f−ε(x) dx =1ε

[nε − 1

]→ +∞ per n → ∞.

funzione degli errori 2.4-6. Verificare che la funzione degli errori

erf(x) :=2√π

∫ x

0

e−t2 dt

tende a 1 per x → +∞ (↑ esempio 2.3-6). Dallo sviluppo in serie della funzioneintegranda, dedurre il seguente sviluppo (utile per x “piccolo”):

erf(x) =2√π

∞∑0

(−1)n x2n+1

(2n + 1)n!=

2√π

(x − x3

3+

x5

10− x7

42+

x9

216+ . . .

)

Soluzione. Sappiamo dall’esempio citato che∫ ∞0

e−t2 dt =√

π/2. Dallo sviluppo

e−t2 =∞∑0

(−1)n t2n

n!

segue il risultato integrando termine a termine sull’intervallo [0, x].

Page 252: Eserciziario Di Analisi Matematica 2

©c 88-08-07923-6 Esercizi 7

2.4-7. Partendo dalla uguaglianza

erf(x) = (2/√

π)[ ∫ ∞

0

e−t2 dt −∫ ∞

x

e−t2 dt]

= 1 − (2/√

π)∫ ∞

x

e−t2 dt,

dedurre lo sviluppo asintotico per x “grande”:

erf(x) = 1 − e−x2

√π

( 1x− 1

2 · x3+

1 · 322 · x5

− 1 · 3 · 523 · x7

+ . . .).

Suggerimento Applicare per k = 0, 2, 4, . . . l’identita (integrazione per parti)∫ ∞

x

e−t2

tkdt =

e−x2

2xk+1− k + 1

2

∫ ∞

x

e−t2

tk+2dt.

0.5 1 1.5 2

1

0.5 1 1.5 2 2.5 3

1

1

2 4

3 5

3

5

7

9

Figura 2.4-3. A sinistra: grafico della funzione degli errori e di alcune somme parziali del suo

sviluppo in serie (il numero a fianco della curva indica il grado della somma corrispondente);

a destra alcuni termini dello sviluppo asintotico della stessa funzione (il numero a fianco

della curva indica quanti termini sono stati presi in considerazione).

Soluzione. Per interpretare correttamente l’integrazione per parti suggerita occorrescrivere la funzione integranda a primo membro come

−2t e−t2

−2tk+1

e considerare −2t e−t2 come la derivata di e−t2 . Dopodiche, applicando l’uguaglianzascritta per k = 0, 2, 4, . . ., i termini integrati forniscono uno dopo l’altro gli addendientro parentesi tonde.

Page 253: Eserciziario Di Analisi Matematica 2

3. Serie di FourierEsercizi

http://eulero.ing.unibo.it/~barozzi/MI2/PDF/MI2-Cap.3-Ese.pdf

3.1. Polinomi di Fourier

3.2. Serie di Fourier: convergenza puntuale

3.2-1. Si dimostri che la serie (v. esempio 3.2-2)∞∑

n=0

1(2n + 1)2

cos((2n + 1)x

)= cos x +

132

cos 3x +152

cos 5x + . . . ,

e totalmente, dunque uniformemente, convergente su R.Soluzione. Si ha∥∥∥ cos

((2n + 1)x

)(2n + 1)2

∥∥∥∞

=1

(2n + 1)2

e la serie che ha come termine generale l’ultima quantita scritta converge in quanto1

(2n + 1)2∼ 1

4n2.

3.2-2. Data la funzione sommabile f(x) := 1/√

x, 0 < x ≤ 1 (v. esempio 3.2-3), sidetermini una funzione g costante a tratti, minorante di f e tale che∫ 1

0

|f(x) − g(x)| dx =∫ 1

0

[f(x) − g(x)] dx =

=∫ 1

0

f(x) dx −n∑

k=1

ck(xk − xk−1) < ε

(↑ Prop. 3.2-1), procedendo nel modo che segue. Si determini x1 > 0 in modo taleche ∫ x1

0

1√x

dx < ε/2;

(si determini esplicitamente l’integrale a primo membro). Scelto cosı x1, si suddividal’intervallo [x1, 1] in n−1 parti uguali (con n abbastanza grande) e si ponga g(x) := 0,per 0 < x < x1,

g(x) := inf f(x) | xk−1 < x < xk = 1/√

xk

per xk−1 < x < xk, k = 2, 3, . . . , n.Soluzione. Si ha∫ x1

0

1√x

dx = 2√

x1,

dunque basta scegliere x1 < ε2/16 per soddisfare la prima condizione posta.

Page 254: Eserciziario Di Analisi Matematica 2

2 Capitolo 3. Serie di Fourier ©c 88-08-07923-6

Si ha poi, per ogni k = 2, 3, . . . , n,∫ xk

xk−1

[f(x) − g(x)] dx =∫ xk

xk−1

( 1√x− 1√

xk

)dx =

= 2[√

xk −√xk−1] −

xk − xk−1√xk

=

= 2xk − xk−1√xk + √

xk−1− xk − xk−1√

xk=

= (xk − xk−1)√

xk −√xk−1√

xk(√

xk + √xk−1)

≤ (xk − xk−1)√

xk −√xk−1

2=

1 − x1

n − 1

√xk −√

xk−1

2,

quindin∑

k=2

∫ xk

xk−1

[f(x) − g(x)] dx ≤ 1 − x1

n − 1

n∑k=2

(√xk −√

xk−1

)=

1 − x1

n − 1(1 −√

x1),

dove l’ultima quantita tende a 0 per n → ∞.

3.2-3. Dimostrare l’uguaglianza

n∑k=−n

eikt =

2n + 1, se t ≡ 0 (mod 2π),

sin((n + 1/2)t

)sin (t/2)

, altrimenti.

procedendo per induzione rispetto a n.Soluzione. L’uguaglianza da dimostrare sussiste per n = 0, riducendosi in tal casoall’uguaglianza 1 = 1. Ammessa la validita dell’uguaglianza per un assegnato n, si ha

n+1∑k=−n+1

eikt =n∑

k=−n

eikt + ei(n+1)t + e−i(n+1)t =

=sin

((n + 1/2)t

)sin (t/2)

+ 2 cos((n + 1)t

)=

=sin

((n + 1/2)t

)+ 2 cos

((n + 1)t

)sin (t/2)

sin (t/2)=

=sin

((n + 1 + 1/2)t

)sin (t/2)

.

Abbiamo utilizzato l’identita (formula diWerner) 2 cosα sin β = sin(α+β)−sin(α−β).

3.2-4. Dimostrare che se f : R → C e periodica di periodo T e sommabile su [0, T ],allora

∫ T

0f(x) dx =

∫ a+T

af(x) dx, per ogni a reale.

Suggerimento Si spezzi l’integrale sull’intervallo [a, a + T ] nella somma di treintegrali sugli intervalli [a, 0], [0, T ], [T, a + T ].)Soluzione. Si ha∫ T

0

f(x) dx =∫ a

0

f(x) dx +∫ a+T

a

f(x) dx +∫ T

a+T

f(x) dx;

l’ultimo addendo si scrive −∫ a+T

Tf(x) dx = −

∫ a

0f(T + ξ) dξ = −

∫ a

0f(ξ) dξ. Ab-

biamo effettuato il cambiamento di variabile x = T + ξ e abbiamo sfruttato la perio-dicita di f .

3.2-5. Utilizzare il lemma di Riemann-Lebesgue per dimostrare che si ha, per ogniδ > 0,

Page 255: Eserciziario Di Analisi Matematica 2

©c 88-08-07923-6 Esercizi 3

limn→+∞

∫ π

δ

Dn(t) dt = 0.

Dedurne che

limn→+∞

∫ δ

−δ

Dn(t) dt = 1.

Soluzione. Basta osservare che la funzione t → 1/ sin(t/2) e continua, dunque somma-bile, sugli intervalli [δ, π] e [−π,−δ].

3.2-6. Dal risultato

π2

8=

∞∑k=0

1(2k + 1)2

= 1 +132

+152

+ . . .

ottenuto nell’esempio 3.2-2, dedurre che∞∑

k=1

1k2

= 1 +122

+132

+ . . . =π2

6.

Suggerimento Detta s la somma della serie considerata, la somma della seriecostituita dai termini di indice pari vale s/4:

∑∞k=1 1/(2k)2 = s/4.

Soluzione. Infatti si ha s = s/4 + π2/8, da cui il valore di s indicato nel testo.

3.2-7. Se g(x) := | x | e la funzione dell’esempio 3.2-2, e f(x) := sign(x) e la funzionedell’esempio 3.2-1, si ha g′(x) = f(x) per ogni x = kπ, con k intero. Se i coefficientidi Fourier di g vengono ancora indicati con i simboli an e bn, mentre i coefficientianaloghi di f vengono ribattezzati a′

n, b′n, verificare che gli sviluppi di Fourier dif = g′ e di g possono essere dedotti l’uno dall’altro in base alle formule a′

n = n bn,b′n = −n an.Soluzione. Basta ricordare (v. esempi citati) che

an = bn = 0, a′n = 0 b′n = .

3.2-8. Sia f(x) := 0, −π < x < 0, f(x) := 1, 0 < x < π. Verificare che si ha

a0 =1π

∫ π

0

dt = 1,

an = 0, per ogni n > 0, mentre

bn =1π

∫ π

0

sin nt dt =

0, per n pari,2/(nπ), per n dispari.

In definitiva

f(x) =12

+2π

∞∑k=0

1(2k + 1)

sin((2k + 1)x

)=

=12

+2π

(sin x +

13

sin 3x +15

sin 5x + . . .).

Si osservi che la funzione f(x) − 1/2 e la meta della funzione sign(x) dell’esercizioprecedente, e cio giustifica il risultato che abbiamo ottenuto.Soluzione. Si tratta di calcolare, per n ≥ 1, l’integrale∫ π

0

sin nt dt =[− cos nt

n

0=

1 − (−1)n

n=

0, per n pari,2/n, per n dispari.

3.2-9. Sia f la funzione periodica dispari che vale 1 per b < x < π−b, con 0 < b < π/2,e si annulla nei restanti punti dell’intervallo [0, π]. Verificare il seguente sviluppo inserie di Fourier:

f(x) =4π

(cos b sin x +

cos 3b

3sin 3x +

cos 5b

5sin 5x + . . .

).

Page 256: Eserciziario Di Analisi Matematica 2

4 Capitolo 3. Serie di Fourier ©c 88-08-07923-6

Si osservi che per b → 0 si ottiene lo sviluppo della funzione sign (x) (↑ Esempio3.2-1).Soluzione. Si ha

bn =2π

∫ π

0

f(t) sin(nt) dt =2π

∫ π−b

b

sin(nt) dt =

=2

πn[ cos(nb) − cos

(n(π − b)

)].

Ma le formule di addizione forniscono

cos(n(π − b)

)= cos(nπ) cos(nb) = (−1)n cos(nb),

quindi i coefficienti bn sono nulli per n pari, valgono 4 cos(nb)/(πn) per n dispari.

3.2-10. Verificare le uguaglianze∫ π

0

t sin kt dt =π

k(−1)k+1;

∫ π

0

t cos kt dt =

0, per k pari,−2/k2, per k dispari;∫ π

0

t2 cos kt dt =2π

k2(−1)k.

Soluzione. Basta fare un’integrazione per parti (per i primi due integrali) e due inte-grazioni per parti per l’utimo integrale. In realta, dopo aver eseguito un’integrazioneper parti sull’ultimo integrale ci si riporta al primo dei tre integrali considerati.

3.2-11. Dimostrare che si ha∫ +∞

0

sin t

tdt =

π

2in base alle seguenti considerazioni:

i) l’integrale in questione esiste come integrale generalizzato semplicemente (manon assolutamente) convergente, in virtu del criterio di convergenza di Dirichlet: essoafferma che l’integrale generalizzato

limx→+∞

∫ x

0

f(t)g(t) dt

esiste finito se f e dotata di una primitiva limitata (nel nostro caso e la funzione− cos t) e g e monotona e decrescente a 0 all’infinito (nel nostro caso e la funzionet → 1/t);

ii) si ha, per ogni n,∫ π

0

sin((n + 1/2)t

)2 sin(t/2)

dt =π

2;

iii) si ha

limn→+∞

∫ π

0

(1

2 sin(t/2)− 1

t

)sin

((n + 1/2)t

)dt = 0

(dimostrare che la funzione entro parentesi tonde tende a 0 per t → 0);iv) infine∫ π

0

sin (n + 1/2)tt

dt =∫ (n+1/2)π

0

sin x

xdt.

Soluzione. L’identita ii)∫ π

0

sin((n + 1/2)t

)2 sin(t/2)

dt =π

2;

e stata stabilita come una delle proprieta del nucleo di Dirichlet (↑ formula (5),par. 3.2). Per la iii) si tratta di applicare il Lemma di Riemann-Lebesgue alla funzione

Page 257: Eserciziario Di Analisi Matematica 2

©c 88-08-07923-6 Esercizi 5

t → 12 sin(t/2)

− 1t

=t − 2 sin(t/2)2t sin(t/2)

.

Ora si ha

t − 2 sin(t/2) =t3

24+ O(t4), 2t sin(t/2) = t2 + O(t3), t → 0

quindi la funzione in esame e prolungabile con continuita nell’origine, ponendolauguale a 0. Combinando la ii) con la iii) si ottiene

limn→+∞

∫ π

0

sin((n + 1/2)t

)t

dt =π

2.

Infine, per verificare la iv) basta operare il cambiamento di variabile (n+1/2)t = x.

3.3. Serie di Fourier: convergenza uniforme

3.4. Serie di Fourier: convergenza in media quadratica

3.4-1. Verificare il seguente sviluppo in serie per la funzione f(x) := | cos x|:

f(x) =2π

+4π

(cos 2x

1 · 3 − cos 4x

3 · 5 +cos 6x

5 · 7 + . . .

).

Soluzione. Si trova

a0 =2π

∫ π

0

| cos x| dx =2π

[ ∫ π/2

0

cos x dx −∫ π

π/2

cos x dx]

=

=4π

.

Per n ≥ 1 si ha analogamente

an =2π

∫ π

0

| cos x| cos nx dx =2π

[ ∫ π/2

0

cos x cos nx dx −∫ π

π/2

cos x cos nx dx].

Per n = 1 la funzione integranda ammette la primitiva x → (x = sin x cos x)/2, dacui segue facilmente a1 = 0. Per n > 1, con due integrazioni per parti si ottiene laprimitiva

x → sin x cos nx − n cos x sin nx

n2 − 1,

da cui segue

an =

− 4

π

(−1)n/2

(n − 1)(n + 1), per n pari ,

0, per n dispari.

3.4-2. Per la funzione f(x) := max 0, cos x = (cos x)+ verificare lo sviluppo:

f(x) =1π

+cos x

2+

(cos 2x

1 · 3 − cos 4x

3 · 5 +cos 6x

5 · 7 + . . .

).

Fare un calcolo diretto, oppure sfruttare il fatto che (cosx)+ = (cos x + | cos x|)/2.Verificare la convergenza totale, dunque uniforme, della serie scritta; stessa questioneper la serie del precedente esercizio.Soluzione. Sommando cos x allo sviluppo precedentemente ottenuto e dividendo per2, si ottiene quanto richiesto. Si puo osservare che tanto la funzione x → | cos x|quanto la funzione x → (cos x)+ verificano le ipotesi della Proposizione 3.3-1.

3.4-3. Se a > 0 e una costante non intera, verificare il seguente sviluppo in serieuniformemente convergente:

Page 258: Eserciziario Di Analisi Matematica 2

6 Capitolo 3. Serie di Fourier ©c 88-08-07923-6

cos ax =2 sin aπ

π

[12a

+∞∑

k=1

(−1)k a

a2 − k2cos kx

].

Dedurne il seguente sviluppo (in serie non uniformemente convergente)

sin ax =2 sin aπ

π

∞∑k=1

(−1)k k

a2 − k2sin kx,

sfruttando i legami tra i coefficienti di Fourier di una funzione ed i coefficienti analoghidella sua derivata.Soluzione. S’intende che la funzione f e 2π-periodica e vale cos ax per |x| ≤ π.

-10

-5 5

10

1

La figura sopra mostra il caso a = 0.6, mentre quella sotto mostra il caso a = 1.3

-10-5 5

10

-1

1

Si ha

a0 =2π

∫ π

0

cos at dt =2πa

[sin at

0=

2 sin aπ

πa

Per ogni k ≥ 1 si ha poi

ak =2π

∫ π

0

cos at cos kt dt =2π

Ik.

L’integrale Ik puo essere calcolato mediante le formule di Werner, oppure con dueintegrazioni per parti. Seguendo quest’ultimo metodo si ha

Ik =[cos at

sin kt

k

0+

a

k

∫ π

0

sin at sin kt dt =

=a

k

[sin at

cos kt

k

0+

a

k

∫ π

0

cos at cos kt dt

=

= −a sin aπ(−1)k

k2+

a2

k2Ik,

da cui segue

Ik = a sin aπ(−1)k

a2 − k2=⇒ ak =

2a sin aπ

π

(−1)k

a2 − k2.

La funzione f verifica le ipotesi della Prop. 3.3-1. Si osservi che ak = O(1/k2).Essendo poi

sin ax = −1a

(cos ax)′,

i coefficienti bk dello sviluppo della funzione sin ax si ottengono dai coefficienti ak

dello sviluppo di cos ax moltiplicandoli per il fattore k/a.

Page 259: Eserciziario Di Analisi Matematica 2

©c 88-08-07923-6 Esercizi 7

3.4-4. Dall’identita

(x |y) :=14

[‖x + y‖2 − ‖x − y‖2 + i ‖x + iy‖2 − i ‖x − iy‖2

]valida in ogni s.v. complesso con prodotto scalare, dedurre che se φ : V1 → V2 (V1, V2

s.v. complessi) e un’isometria lineare, cioe

‖φ(x)‖2 = ‖x‖1

per ogni x ∈ V1, allora φ conserva i prodotti scalari:(φ(x) |φ(y)

)2

= (x |y)1,

per ogni x, y ∈ V1. (Gli indici in basso stanno a specificare che si tratta dei prodottiscalari e delle norme negli spazi V1 e V2 rispettivamente.)Soluzione. L’identita precedente, scritta in corrispondenza dei vettori φ(x) e φ(y),fornisce infatti(

φ(x) |φ(y))2

=14

[‖φ(x) + φ(y)‖2 − ‖φ(x) − φ(y)‖2+

+ i ‖φ(x) + iφ(y)‖2 − i ‖φ(x) − iφ(y)‖2]

=

=14

[‖φ(x + y)‖2 − ‖φ(x − y)‖2+

+ i ‖φ(x + iy)‖2 − i ‖φ(x − iy)‖2]

=

=14

[‖x + y‖2 − ‖x − y‖2 + i ‖x + iy‖2 − i ‖x − iy‖2

]=

= (x |y)1

3.4-5. Applicando l’identita di Parseval allo sviluppo

|x | =π

2− 4

π

(cos x +

132

cos 3x +152

cos 5x + · · ·)

dedurre l’uguaglianza∑k ≥ 0

1(2k + 1)4

=π4

96

Con un ragionamento simile a quello svolto nell’esercizio 3.2-6, dedurne che∑n≥ 1

1n4

=π4

90

Soluzione. Si ha

‖f‖22 = 2

∫ π

0

x2 dx =23

π3.

L’identita di Parseval si scrive

π

|a0|2

2+

∑k≥1

|ak|2 = ‖f‖2

2 ⇐⇒ π

π2

2+

16π2

∑k≥0

1(2k + 1)4

=

23

π3,

da cui16π2

∑k≥0

1(2k + 1)4

=π2

6=⇒

∑k≥0

1(2k + 1)4

=π4

96.

Si ha poi

s :=∑k≥1

1k4

=∑k≥0

1(2k + 1)4

+∑k≥1

1(2k)4

=π4

96+

116

s,

da cui segue subito il valore fornito per s.

Page 260: Eserciziario Di Analisi Matematica 2

8 Capitolo 3. Serie di Fourier ©c 88-08-07923-6

3.4-6. Applicando l’identita di Parseval allo sviluppo

sgn (x) =4π

(sin x +

13

sin 3x +15

sin 5x + · · ·)

dedurre l’uguaglianza∑k ≥ 0

1(2k + 1)2

=π2

8.

Soluzione. Si ha

‖f‖22 =

∫ π

−π

1 dx = 2π.

L’identita di Parseval si scrive

π∑k ≥ 1

|bk|2 = ‖f‖22 ⇐⇒ π

16π2

∑k ≥ 0

1(2k + 1)2

= 2π

da cui segue subito il risultato.

3.4-7. Applicando l’identita di Parseval allo sviluppo

x = 2(

sin x − 12

sin 2x +13

sin 3x − · · ·)

dedurre l’uguaglianza∑n≥ 1

1n2

=π2

6.

Soluzione. Si ha

‖f‖22 =

∫ π

−π

x2 dx =23

π3.

L’identita di Parseval si scrive

4π∑n≥ 1

1n2

=23

π3,

da cui segue subito il risultato.

3.4-8. Applicando l’identita di Parseval allo sviluppo

x2 =π2

3− 4

(cos x − 1

22cos 2x +

132

cos 3x − · · ·)

dedurre l’uguaglianza∑n≥ 1

1n4

=π4

90.

Soluzione. Si ha

‖f‖22 =

∫ π

−π

x4 dx =25

π5.

L’identita di Parseval si scrive

π[4π4

18+ 16

∑n≥ 1

1n4

]=

25

π5 ⇐⇒ 16∑n≥ 1

1n4

=8π4

45,

da cui segue subito il risultato.

3.5 Serie di Fourier: ulteriori risultati

3.5-1. Sia x → f(x) una funzione periodica di periodo T , avente coefficienti diFourier ck, k ∈ Z. Verificare che la funzione x → f(x − x0) ha coefficienti di Fouriere−ikωx0 ck.

Page 261: Eserciziario Di Analisi Matematica 2

©c 88-08-07923-6 Esercizi 9

Soluzione. Per il coefficiente di Fourier della funzione in esame, sia γk, abbiamo

γk =1T

∫ T

0

f(x − x0) e−ikωx dx =1T

e−ikωx0

∫ T

0

f(t) e−ikωt dt = e−ikωx0 ck.

Abbiamo effettuato il cambiamento di variabile x − x0 = t e abbiamo tenuto contodel fatto che l’integrale di una funzione T -periodica su un intervallo di lunghezza Te invariante rispetto alla collocazione dello stesso intervallo (↑ esercizio 3.2-4). Inparticolare si ha |γk| = |ck|, per ogni k.

3.5-2. Dedurre dal precedente esercizio che le funzioni x → f(x) e x → f(x − x0)hanno lo stesso spettro di ampiezza.Soluzione. Dalle relazioni Ak =

√a2

k + b2k, ck(ak−ibk)/2, segue |Ak| = 2|ck|. Dunque

lo spettro di ampiezza dipende esclusivamente dai valori assoluti dei coefficienti diFourier.

3.5-3. Sia f una funzione periodica di periodo T , avente coefficienti di Fourier ck,k ∈ Z. Dimostrare che se f(t) = f(−t) (proprieta di simmetria hermitiana), dunquein particolare se f e reale pari, allora i coefficienti di Fourier sono reali: ck = ck.Suggerimento Scrivere ck sotto forma integrale e cambiare di segno la variabiled’integrazione.Soluzione. Seguendo il suggerimento si trova

ck =1T

∫ T

0

f(t) eikωt dt =1T

∫ T

0

f(−x) e−ikωx dx =1T

∫ T

0

f(x) e−ikωx dx =

= ck.

Esercizi proposti

3.P-1. Calcolare la somma della serie trigonometrica∑n≥0

cos nx

2n

ni a partire dalla serie geometrica∑n≥0

einx

2n.

Soluzione. La somma della serie trigonometrica e la arte reale della somma dellaserie geometrica∑

n≥0

(eix

2

)n

;

poiche la ragione di tale serie ha come valore assoluto 1/2, essa converge alla somma1

1 − eix/2=

22 − eix

=2

(2 − cos x) − i sin x.

La parte reale della somma calcolata vale4 − 2 cos x

5 − 4 cos x.

3.P-2. Calcolare lo sviluppo in serie di Fourier della funzione periodica di periodo 2π,la cui restrizione all’intervallo [−π, π] vale x per −π/2 ≤ x ≤ π/2, e vale 0 altrimenti(fare uno schizzo).

A quale valore converge la serie di Fourier per x = π/2?Soluzione. Poiche f e una funzione dispari, avremo uno sviluppo del tipo

Page 262: Eserciziario Di Analisi Matematica 2

10 Capitolo 3. Serie di Fourier ©c 88-08-07923-6

f(x) =∑n≥1

bn sin nx.

Le condizioni di Dini sono soddisfatte per x = π/2, quindi la somma della serie intale punto vale π/4:∑

n≥1

bn sin(nπ/2) = π/4

cioe ∑k≥0

b2k+1 sin((2k + 1)π/2) =∑k≥0

b2k+1(−1)k = π/4. (∗)

Abbiamo tenuto conto del fatto che sin(nπ/2) e nullo per n pari, vale (−1)k pern = 2k + 1.

Per i coefficienti bn abbiamo, con un’integrazine per parti,

bn =2π

∫ π/2

0

x sin nx dx = −cos(nπ/2)n

+2π

sin(nπ/2)n2

.

Separando i casi n pari e n dispari, avremo duque

b2k =(−1)k+1

2k, b2k+1 =

(−1)k

(2k + 1)2.

Sostituendo nella (*) si trova dunque

∑k≥0

(−1)k

(2k + 1)2(−1)k =

π

4⇐⇒

∑k≥0

1(2k + 1)2

=π2

8,

risultato gia ottenuto (v. esercizio 3.4-6).

Si veda anche il notebook

http://eulero.ing.unibo.it/~barozzi/MI2/Mathematica/Mathematica-3.2.nb

Page 263: Eserciziario Di Analisi Matematica 2

©c 88-08-07923-6 Esercizi 11

Un elenco di sviluppi in serie di Fourier in forma visuale

1. f(x) =4π

[cos b sin x +

cos 3b

3sin 3x +

cos 5b

5sin 5x + . . .

]

-6 -4 -2 2 4 6

-1

1

π−π

π − bb

La restrizione di f all’intervallo [−π, π] si scrive χ[b,π−b](x) · sgn(x), con 0 ≤ b < π/2.Per b = 0 si ottiene lo sviluppo della funzione segno. La figura seguente mostra iprimi cinque polinomi di Fourier di ordine dispari della funzione segno.

-5

05

-2

-1

0

1

2

5

05

2. f(x) =2π

[ b

2+ sin b cos x +

sin 3b

3cos 3x +

sin 5b

5cos 5x + . . .

](esempio 3.5-1)

-6 -4 -2 2 4 6

1

−π π

−b b

Page 264: Eserciziario Di Analisi Matematica 2

12 Capitolo 3. Serie di Fourier ©c 88-08-07923-6

La restrizione di f all’intervallo [−π, π] si scrive χ[−b,b](x).

3. f(x) =4π

[cos x − cos 3x

3+

cos 5x

5− cos 7x

7+ . . .

]

-6 -4 -2 2 4 6

-1

1

−π π

La restrizione di f all’intervallo [−π, π] si scrive sgn(π/2 − |x|).

4. f(x) = −2[sin x +

sin 2x

2+

sin 3x

3+ . . .

](↓ grafico monometrico)

-6 -4 -2 2 4 6

-3

-2

-1

1

2

3

π

−π

La restrizione di f all’intervallo [−π, π] si scrive x − sgn(x) · π. La figura seguentemostra i primi cinque polinomi di Fourier della funzione considerata.

-5

0

5

-2

0

2

5

0

5

Page 265: Eserciziario Di Analisi Matematica 2

©c 88-08-07923-6 Esercizi 13

5. f(x) =8π2

[sin x − sin 3x

32+

sin 5x

52+ . . .

](↓ grafico monometrico)

-6 -4 -2 2 4 6-1

1

La restrizione di f all’intervallo [−π, π] vale x per |x| ≤ π/2, vale sgn(x) · π − x perπ/2 ≤ |x| ≤ π. La figura seguente mostra i primi cinque polinomi di Fourier dellafunzione considerata.

-50

5-1

0

1

50

5

6. f(x) =b

2π+

2π2

[(1 − cos b) cos x +

1 − cos 2b

22cos 2x +

1 − cos 3b

32cos 3x + . . .

]

-7.5 -5 -2.5 2.5 5 7.5

1

−b b

La restrizione di f all’intervallo [−π, π] vale (1 − |x/b|)+ = max1 − |x/b|, 0.

7. f(x) = (cos x)+ = max0, cos x =

=1π

+cos x

2+

(cos 2x

1 · 3 − cos 4x

3 · 5 +cos 6x

5 · 7 + . . .

).

(esempio 3.4-2)

Page 266: Eserciziario Di Analisi Matematica 2

14 Capitolo 3. Serie di Fourier ©c 88-08-07923-6

-7.5 -5 -2.5 2.5 5 7.5

1

La figura seguente mostra i primi cinque polinomi di Fourier della funzione f .

-5

0

5

0

0.5

1

5

0

5

8. f(x) = | cos x| =2π

+4π

(cos 2x

1 · 3 − cos 4x

3 · 5 +cos 6x

5 · 7 + . . .

). (esempio 3.4-1)

-7.5 -5 -2.5 2.5 5 7.5

1

La figura seguente mostra i primi cinque polinomi di Fourier della funzione f .

-50

50

0.5

1

05

Page 267: Eserciziario Di Analisi Matematica 2

4. Funzioni di variabile complessaEsercizi

[Revisione: dicembre 2007]

http://www.ciram.unibo.it/~barozzi/MI2/PDF/MI2-Cap.4-Ese.pdf

4.1. Il campo complesso

4.2. Funzioni di una variabile complessa

4.3. Funzioni olomorfe

4.3-1. Vogliamo verificare che, per ogni z = x+ iy ∈ C, si ha

limn→∞

(1 +

x+ iy

n

)n

= ex(cos y + i sin y).

Si tratta di verificare che il modulo del primo membro tende a ex, il suo argomentotende a y. Utilizzeremo i seguenti risultati (che supporremo noti):

a) se (an) e una successione reale che tende al limite a, allora

limn→∞

(1 +

an

n

)n

= ea;

b) limt→0(arctan t)/t = 1.

La dimostrazione puo essere articolata nei seguenti punti. Si ha∣∣∣1 +x+ iy

n

∣∣∣ =(1 +

2xn

+x2 + y2

n2

)1/2

,

da cui∣∣∣(1 +x+ iy

n

)n∣∣∣ =[1 +

1n

(2x+

x2 + y2

n

)]n/2

,

dove l’ultima quantita tende a e2x/2 = ex, in virtu del punto a).Poiche la parte reale di 1 + (x + iy)/n tende a 1, essa e positiva per tutti gli nabbastanza grandi, quindi

Arg(1 + (x+ iy)/n

)= arctan

(y/n)(n+ x)/n

= arctany

n+ x.

Ne segue

arg(1 +

x+ iy

n

)n

= nArg(1 +

x+ iy

n

)= n arctan

y

n+ x;

a questo punto basta fare il limite dell’ultima quantita per n → ∞, sfruttando ilpunto b).

Soluzione. La soluzione e gia contenuta nel testo. Ci limitiamo ad osservare che

Arg(1 +

x+ iy

n

)= Arg

(n+ x

n+ i

y

n

)= arctan

( yn· n

n+ x

),

e

Page 268: Eserciziario Di Analisi Matematica 2

2 Capitolo 4. Funzioni di variabile complessa ©c 978-88-08-12546-0

n arctany

x+ n=

ny

n+ x· n+ x

yarctan

y

n+ x.

4.3-2. La formula di De Moivre (↑ formula (8) del paragrafo 4.1) si puo scriverecosnt = (c + is)n, avendo posto c := cos t e s := sin t. Supposto n ≥ 2, si sviluppiil secondo membro con la formula del binomio e si uguaglino a primo e a secondomembro le parti reali, ottenendo l’identita

cosnt =∑

0≤k≤n/2

(n

2k

)(−1)kcn−2ks2k.

Tenendo presente che s2 = 1− c2, si deduca che cosnt si puo scrivere sotto formadi polinomio di grado n nella variabile c:

cosnt =∑

0≤k≤n/2

(n

2k

)(−1)kcn−2k(1 − c2)k =: Tn(c).

Tn e pari per n pari, dispari per n dispari. Si verifichino le formule:

T1(c) = c,

T2(c) = 2c2 − 1,

T3(c) = 4c3 − 3c,

T4(c) = 8c4 − 8c2 + 1.

I polinomi Tn sono noti come polinomi di Cebysev, dal nome del matematico russoP.L. Cebysev (1821-1894). Ritroveremo questi polinomi nell’esempio 8.1-4.

Soluzione. Limitiamoci a verificare l’espressione di T4:

T4(c) =∑

0≤k≤2

(42k

)(−1)kc4−2k(1 − c2)k =

=(

40

)c4 −

(42

)c2(1 − c2) +

(44

)(1 − c2)2 =

= c4 − 6(c2 − c4) + 1 + c4 − 2c2 = 8c4 − 8c2 + 1.

4.3-3. Osservato che l’uguaglianza x = cos t, per x ∈ [−1, 1] equivale a t = arccosx,si ottenga l’uguaglianza

Tn(x) = cos(n arccosx), x ∈ [−1, 1],

e da questa le relazioni Tn(1) = 1, Tn(−1) = (−1)n.

Soluzione. Basta osservare che arccos 1 = 0, arccos(−1) = π, e che cos 0 = 1,cos(nπ) = (−1)n.

-1 -0.5 0.5 1

-1

-0.5

0.5

1

I polinomi di Cebysev di grado non superiore a 4.

4.3-4. A partire dalle formule di addizione

Page 269: Eserciziario Di Analisi Matematica 2

©c 978-88-08-12546-0 Esercizi 3

cos((n+ 1)t

)= cosnt cos t− sinnt sin t,

cos((n− 1)t

)= cosnt cos t+ sinnt sin t

si deduca la relazione ricorrente

cos((n+ 1)t

)= 2 cosnt cos t− cos

((n− 1)t

),

e da questa la relazione

Tn+1(x) = 2xTn(x) − Tn−1(x).

Essa consente il calcolo dei polinomi di Cebysev, a partire dai termini iniziali T0(x) == 1, T1(x) = x.

Soluzione. Sommando membro a membro le due formule di addizione si ottiene laformula

cos((n+ 1)t

)= 2 cosnt cos t− cos

((n− 1)t

);

basta poi ricordare che, ove si ponga t := arccosx, si ha Tn(x) := cos(nt).

4.3-5. Si verifichi che la successione n → cosnt e ortogonale su L2([0, π]

):

∫ π

0

cosnt cosmtdt =

0, se n = m,π, se n = m = 0,π/2, se n = m > 0.

Operando il cambiamento di variabile t = arccosx, x ∈ [−1, 1], si ottengano le re-lazioni∫ 1

−1

Tn(x)Tm(x)√1 − x2

dx =

0, se n = m,π, se n = m = 0,π/2, se n = m > 0.

Esse esprimono ortogonalita dei polinomi di Cebysev sull’intervallo [−1, 1], rispettoalla funzione peso x → 1/

√1 − x2.

Soluzione. Per verificare le prime identita si puo ricorrere alle cosiddette formule diWerner (PCAM, esercizio 2.4-9):

cosnt cosmt =12

[cos

(n+m

2t)

+ cos(n−m

2t)].

Le seconde identita si riportano alle prime mediante il cambiamento di variabile x == cos t.

4.3-6. Verificare, mediante un calcolo diretto, l’uguaglianza

| sin z |2 = sin2 x+ sinh2 y.

Determinare gli zeri della funzione seno in C. Determinare l’uguaglianza analoga perla funzione cos z.

Soluzione. Se z = x+ iy, si ha iz = −y + ix, da cui

sin z =12i

(eiz − e−iz) =12i

(e−y(cosx+ i sinx) − ey(cosx− i sinx)

)=

=12i

(i sinx (ey + e−y) + cosx (e−y − ey)

)=

= sinx cosh y + i cosx sinh y.

Ne segue

| sin z |2 = sin2 x cosh2 y + cos2 x sinh2 y =

= sin2 x cosh2 y + (1 − sin2 x) sinh2 y =

= sin2 x (cosh2 y − sinh2 y) + sinh2 y =

= sin2 x+ sinh2 y.

Page 270: Eserciziario Di Analisi Matematica 2

4 Capitolo 4. Funzioni di variabile complessa ©c 978-88-08-12546-0

Dunque la funzione seno e illimitata in C, in accordo col teorema di Liouville; dasin z = 0 segue sinx = sinh y = 0, dunque y = 0, x = kπ, k intero. In conclusione: glizeri della funzione seno in campo complesso sono soltanto gli zeri della stessa funzionein campo reale.

In modo analogo si ottiene

cos z = cosx cosh y − i sinx sinh y,

da cui | cos z |2 = cos2 x+ sinh2 y.

Esercizi proposti

4.3-P1. Verificare che l’unica funzione f olomorfa in C per cui si abbia f ′ = f ef(0) = 1 e la funzione esponenziale f(z) = f(x, y) = ex(cos y + i sin y):

∀z ∈ C, f ′(z) = f(z)f(0) = 1

=⇒ f(z) = ez = ex(cos y + i sin y)

Soluzione. Se f = u+iv, dalla uguaglianza f ′(z) = fx(x, y) = f deduciamo ux+ivx =u+ iv, dunque ux = u, vx = v. Per ogni fissato y le funzioni x → u(x, y) e x → v(x, y)sono soluzioni dell’equazione differenziale ϕ′(x) = ϕ(x) e pertanto si scrivono

u(x, y) = c1(y) ex, u(x, y) = c2(y) ex,

con c1 e c2 funzioni opportune. D’altra parte le condizioni di Cauchy-Riemann for-niscono

ux = vy ⇐⇒ c1(y) ex = c′2(y) ex ⇐⇒ c1(y) = c′2(y),

uy = −vx ⇐⇒ c′1(y) ex = −c2(y) ex ⇐⇒ c′1(y) = −c2(y).

Finalmente da f(0) = 1 seguono le condizioni iniziali c1(0) = 1, c2(0) = 0. Indefinitiva c1 e c2 sono soluzioni del problema di valori iniziali

c′1(y) = −c2(y),c′2(y) = c1(y),c1(0) = 1, c2(0) = 0

e dunque c1(y) = cos y, c2(y) = sin y.

4.3-P2. Data la funzione u(x, y) = 3x+ 4y, determinare la funzione v(x, y) tale chev(0, 0) = 0 e f(x+ iy) = u(x, y) + iv(x, y) sia intera.Soluzione. Deve essere vy = ux = 3, quindi v(x, y) = 3y + g(x), con g funzioneda determinare. D’altra parte deve essere anche vx = −uy, cioe g′(x) = −4, da cuig(x) = −4x+c. Dunque v(x, y) = 3y−4x+c, e finalmente, per avere v(0, 0) = 0,c = 0.

In definitiva

f(z) = f(x+ iy) = 3x+ 4y + i(3y − 4x) = 3(x+ iy) − 4i(x+ iy) = (3 − 4i)z.

4.4. Serie di potenze

4.4-1. Dimostrare per induzione la seguente formula per le derivate del prodotto didue funzioni

Dn(f(z)g(z)

)=

n∑k=0

(n

k

)Dkf(z)Dn−kg(z).

Soluzione. Per n = 1 abbiamo la formula per la derivazione del prodotto:

D(f(z)g(z)

)= Df(z) g(z) + f(z)Dg(z).

Procediamo per induzione rispetto a n; se la formula in questione sussiste per unassegnato n abbiamo

Page 271: Eserciziario Di Analisi Matematica 2

©c 978-88-08-12546-0 Esercizi 5

D(n+1)(f(z)g(z)

)= D

(Dn

(f(z)g(z)

))= D

( n∑k=0

(n

k

)Dkf(z)Dn−kg(z)

)=

=n∑

k=0

(n

k

)D(k+1)f(z)Dn−kg(z) +

n∑k=0

(n

k

)Dkf(z)Dn−k+1g(z).

Se nella prima delle due sommatorie si pone k + 1 = h ⇐⇒ k = h− 1, e successiva-mente si scrive ancora k al posto di h si ottiene

Dn+1f(z)g(z) +n∑

h=1

(n

h− 1

)Dhf(z)Dn+1−hg(z)+

+n∑

k=1

(n

k

)Dkf(z)Dn+1−kg(z) + f(z)Dn+1g(z) =

= Dn+1f(z)g(z) +n∑

k=1

[(n

k − 1

)+

(n

k

)]Dkf(z)Dn+1−kg(z)+

+ f(z)Dn+1g(z) =

=n+1∑k=0

(n+ 1k

)Dkf(z)Dn+1−kg(z).

Abbiamo sfruttato l’identita(n

k − 1

)+

(n

k

)=

(n+ 1k

).

4.4-2. Sia f una funzione olomorfa in un intorno dell’origine e f(z) =∑

n≥0 an zn

il suo sviluppo di Taylor in tale intorno. Indichiamo simbolicamente con la scritturaf ↔ (an) la corrispondenza tra la funzione f e la successione (an) dei suoi coefficientidi Taylor. Se g e una funzione analoga ad f , e g(z) =

∑n≥0 bn z

n e il suo sviluppo diTaylor, verificare che:

f + g ↔ (an) + (bn) := (an + bn),

λf ↔ λ(an) := (λan), λ ∈ C,

f · g ↔ (an) ∗ (bn) :=( n∑

k=0

ak bn−k

).

L’operazione che alla coppia di successioni (an), (bn) associa la successione

(an) ∗ (bn) =(a0b0, a0b1 + a1b0, a0b2 + a1b1 + a2b0, . . .

),

si chiama convoluzione tra le successioni date. Verificare che se tali successioni sonodefinitivamente nulle (dunque f e g sono polinomi) si ritrova l’ordinaria moltipli-cazione tra polinomi.Suggerimento Si scriva l’identita dell’esercizio precedente nella forma

Dn(f(z)g(z)

)n!

=n∑

k=0

Dkf(z)k!

Dn−kg(z)(n− k)!

.

Soluzione. L’unico cosa da dimostrare e la terza formula, relativa al prodotto di duefunzioni. Basta ricordare che l’n-esimo coefficiente dello sviluppo in serie del prodottofg e [D(n)(fg)(0)]/n! e applicare il risultato dell’esercizio precedente.Dunque se f, g, e h sono tre funzioni analitiche in un intorno dell’origine, f ↔ (an),g ↔ (bn) e h ↔ (cn), ed inoltre (cn) = (an) ∗ (bn), allora h = fg.

4.4-3. Sia f(z) = ez1z, g(z) = ez2z, dove z1 e z2 sono due numeri complessi adarbitrio. Verificare che f(z)g(z) = ez1z ez2z = e(z1+z2)z, da cui, ponendo z = 1,ez1 ez2 = ez1+z2 .

Page 272: Eserciziario Di Analisi Matematica 2

6 Capitolo 4. Funzioni di variabile complessa ©c 978-88-08-12546-0

Soluzione. Con i simboli dell’esercizio precedente abbiamo

ez1z ↔(zn1

n!

), ez2z ↔

(zn2

n!

), e(z1+z2)z ↔

((z1 + z2)n

n!

);

la convoluzione tra le prime due successioni fornisce la terza successione, come abbia-mo verificato nel testo, a pagina 126.

4.4-4. Dall’uguaglianza Dzn/n! = zn−1/(n− 1)!, valida per ogni n > 0, dedurre cheDez = ez.

Soluzione. Basta osservare che, derivando termine a termine la serie esponenziale∑n≥0 z

n/n!, si ottiene nuovamente le serie stessa.

4.4-5. Derivando termine a termine lo sviluppo della funzione 1/(1 − z)2, ottenutonell’esempio 4.4-1, si ottenga lo sviluppo

1(1 − z)3

=∑n≥0

(n+ 1)(n+ 2)2

zn, |z| < 1.

Si ottenga lo stesso risultato moltiplicando tra loro gli sviluppi delle funzioni 1/(1−z)e 1/(1 − z)2.

Soluzione. Derivando termine a termine l’uguaglianza1

(1 − z)2= (1 − z)−2 =

∑n≥0

(n+ 1) zn

si ottiene

2(1 − z)−3 =∑n≥1

(n+ 1)n zn−1,

da cui la formula proposta nel testo, scrivendo n al posto di n− 1. Moltiplicando laserie

∑n≥0(n+ 1) zn per la serie

∑n≥0 z

n si ottiene la serie

∑n≥0

n∑k=0

(k + 1) zk zn−k,

dunque la serie precedentemente ottenuta, ove si ricordi che la somma dei numerinaturali da 1 a n+ 1 vale (n+ 1)(n+ 2)/2.

4.4-6. Dallo sviluppo in serie geometrica di ragione −z1

1 + z=

∑n≥0

(−1)n zn, |z| < 1,

si deduca lo sviluppo1

(1 + z)2=

∑n≥0

(−1)n (n+ 1) zn = 1 − 2z + 3z2 − 4z3 + . . . ,

prima mediante derivazione e successivamente mediante moltiplicazione della serie dipartenza per se stessa.

Soluzione. Limitiamoci alla prima richiesta; derivando la serie data si ottiene−1

(1 + z)2=

∑n≥1

(−1)n n zn−1,

da cui il risultato fornito nel testo, scrivendo n al posto di n− 1.

4.4-7. Dedurre dalla serie geometrica lo sviluppo

1a2 + z2

=∑n≥0

(−1)n z2n

a2n+2,

Page 273: Eserciziario Di Analisi Matematica 2

©c 978-88-08-12546-0 Esercizi 7

valido, per ogni fissato a = 0, per |z| < |a|.Soluzione. Si ha

1a2 + z2

=1

a2(1 + z2/a2)=

1a2

∑n≥0

(−1)n z2n

a2n.

4.4-8. La successione dei numeri di Fibonacci e definita ricorsivamente ponendo

F0 := 0, F1 := 1; Fn := Fn−1 + Fn−2, n ≥ 2.

Verificare che la serie∑n≥0

Fn+1zn = 1 + z + 2z2 + 3z3 + 5z4 + . . .

ha raggio di convergenza R = 2/(1 +√

5).

Suggerimento Posto rn := Fn+1/Fn ≥ 1, verificare la formula ricorsiva rn+1 == 1 + 1/rn, da cui passando il limite . . . .

Soluzione. Scrivendo la formula ricorsiva che genera i numeri di Fibonacci nellaforma Fn+2 := Fn+1 + Fn, si ottiene la relazione rn+1 = 1 + 1/rn, da accoppiarealla condizione iniziale r1 = 1, semplicemente dividendo per Fn+1. La successione deirapporti (rn) e studiata in dettaglio in PCAM, esempio 3.2-3 ed esercizio 3.4-3; perla soluzione degli esercizi si veda il file

http://eulero.ing.unibo.it/~barozzi/PCAM Complementi/

PCAM Soluzioni 3.pdf.

La successione dei termini di indice dispari e strettamente crescente, quella deitermini di indice pari e strettamente decrescente ed entrambe tendono allo stessolimite. Se L e il limite in questione, la relazione rn+1 = 1 + 1/rn per n→ ∞ fornisce

L = 1 +1L

⇐⇒ L2 − L− 1 = 0 =⇒ L =1 +

√5

2.

Abbiamo tenuto conto del fatto che tutti i rapporti rn sono ≥ 1. Il criterio delrapporto applicato alla serie

∑n≥0 Fn+1|z|n conduce al limite

limn→∞

rn+1 |z| =1 +

√5

2|z|,

dunque la serie in esame ha raggio di convergenza R = 2/(1 +√

5).

4.4-9. Detta s(z) la somma della serie considerata nel precedente esercizio, |z| << 2/(1 +

√5), dedurre dalla definizione dei numeri di Fibonacci la relazione

s(z)(1 − z − z2) = 1 =⇒ s(z) =1

1 − z − z2.

Verificare che la funzione razionale 1/(1−z−z2) e olomorfa nel disco |z| < 2/(1+√

5).

Soluzione. Dalle uguaglianze

s(z) = 1 + z + 2z2 + 3z3 + 5z4 + . . .+ Fn+1 zn + . . . ,

zs(z) = z + z2 + 2z3 + 3z4 + . . .+ Fn zn + . . . ,

z2s(z) = z2 + z3 + 2z4 + . . .+ Fn−1 zn + . . . ,

si ottiene la relazione voluta sottraendo la seconda e la terza uguaglianza dalla prima.Il trinomio 1−z−z2 ammette gli zeri (−1±

√5)/2, di cui il secondo e il piu prossimo

all’origine, distando da essa√

5 − 12

=2√

5 + 1.

Page 274: Eserciziario Di Analisi Matematica 2

8 Capitolo 4. Funzioni di variabile complessa ©c 978-88-08-12546-0

4.5. Integrazione in campo complesso

Nel seguito useremo il simbolo

ˇ

∮γ

f(z) dz

per indicare l’integrale di f sul circuito γ, orientato positivamente.

4.5-1. Sia γ : t → z0 + r eit, 0 ≤ t ≤ k · 2π, con k intero ≥ 1, cioe la circonferenzadi centro z0 e raggio r percorsa in senso positivo k volte. Verificare che (↑ Esempio4.5-4)

12πiˇ

∮γ

1z − z0

dz = k.

Soluzione. A parametrizzazione avvenuta si ottiene

12πi

∫ 2kπ

0

1r eit

ir eit dt =12π

∫ 2kπ

0

dt = k.

4.5-2. Sia u(x, y) una funzione di classe C(1) in un aperto contenente il rettangoloD := [a, b] × [c, d] e sia γ la frontiera di D orientata positivamente rispetto ad esso.Verificare che∫∫

D

∂u

∂xdx dy =

∫ d

c

[u(b, y) − u(a, y) ] dy =ˇ

∮γ

u dy,

∫∫D

∂u

∂ydx dy =

∫ b

a

[u(x, d) − u(x, c) ] dx = −ˇ

∮γ

u dx.

Soluzione. Infatti∫∫D

∂u

∂xdx dy =

∫ d

c

(∫ b

a

∂u

∂xdx

)dy =

∫ d

c

[u(x, y)

]x=b

x=ady.

4.5-3. Supponiamo che il dominio D sia descritto nel modo seguente:

D := (x, y) | a ≤ x, ϕ1(x) ≤ y ≤ ϕ2(x)dove ϕ1 e ϕ2 sono funzioni continue. Se u e di classe C(1) in un aperto contenente D,verificare che∫∫

D

∂u

∂ydx dy =

∫ b

a

[u(x, ϕ2(x)) − u(x, ϕ1(x)) ] dx = −ˇ

∮γ

u dx.

Analogamente, se D e descritto come

D := (x, y) | c ≤ y ≤ d, ψ1(y) ≤ x ≤ ψ2(y),dove ψ1 e ψ2 sono funzioni continue, allora∫∫

D

∂u

∂xdx dy =

∫ d

c

[u(ψ2(y), y) − u(ψ1(y), y) ] dy =ˇ

∮γ

u dy.

Soluzione. Infatti∫∫D

∂u

∂ydx dy =

∫ b

a

(∫ ϕ2(x)

ϕ1(x)

∂u

∂ydy

)dx =

∫ b

a

[u(x, y)

]y=ϕ2(x)

y=ϕ1(x)dx.

4.6. Proprieta delle funzioni analitiche

4.6-1. Provare l’identita ez1+z2 = ez1ez2 ,∀z1, z2,∈ C, utilizzando il teorema di unicitadel prolungamento analitico.Suggerimento Fissare z1 ∈ R e considerare le funzioni intere z → ez1+z, z →ez1ez; esse coincidono . . . .

Page 275: Eserciziario Di Analisi Matematica 2

©c 978-88-08-12546-0 Esercizi 9

Soluzione. . . . sull’asse reale, dunque coincidono in tutto il campo complesso.

4.6-2. Utilizzando il teorema fondamentale dell’algebra, mostrare che ogni polinomiop (non costante) trasforma C su se stesso, cioe il piano complesso ha come immaginemediante p se stesso.Suggerimento Per ogni fissato w, l’equazione p(z) = w equivale a . . . .

Soluzione. . . . p(z)−w = 0, dove p(z)−w e ancora un polinomio; dunque il teoremafondamentale assicura che il primo membro si annulla in almeno un punto.

4.7. Punti singolari. Serie bilatere

4.7-1. Determinare (e classificare) le singolarita delle funzioni

1)z

sin z, 2)

sin zz2 − π2

.

Soluzione. La prima funzione ha una sigolarita eliminabile nell’origine e un polosemplice in ciascuno dei punti zk = kπ, k intero non nullo; infatti in tali punti ilnumeratore non si annulla mentre il denominatore presenta uno zero del primo ordine(= zero semplice). Poiche il denomitatore della seconda funzione si scrive (z−π)(z+π),essa presenta due singolarita eliminabili nei punti z = π e z = −π; in tali punti tantoil numeratore quanto il denominatore presentano zeri semplici.

4.7-2. Considerate le funzioni

1) e−1/z2, 2)

e1/z

z, 3) sin

1z, 4) sinh

1z,

5)sin z4

z3, 6)

1 − e−z

z, 7)

e−1/z2

z, 8) z3 sin

1z,

classificare l’origine in quanto singolarita per ciascuna di esse.

Soluzione. Esercizio 1. Da

e−1/z2= 1 − 1

z2+

12! z2

− 13! z6

+ . . .

segue che l’origine e una singolarita essenziale.Esercizio 2. Da

e1/z

z=

1z

+1z2

+1

2! z3+

13! z4

+ . . .

segue che l’origine e una singolarita essenziale.Esercizi 3 e 4. Considerazioni analoghe alle precedenti: basta scrivere 1/z al posto diz nelle serie del seno e del seno iperbolico rispettivamente.Esercizio 5. Da sin z4 ∼ z4, per z → 0, segue che la funzione ha una singolaritaeliminabile, anzi uno zero del primo ordine nell’origine.Esercizio 6. Poiche e−z = 1 − z +O(z2), l’origine e una singolarita eliminabile.Esercizi 7 e 8. Considerazioni analoghe a quelle fatte per gli esercizi 1 e 2: in entrambii casi si tratta di singolarita essenziali.

4.7-3. Calcolare i residui delle funzioni

1)1

z2 − 1, 2)

sin zcos z

, 3)z cos zsin z

, 4) e2/z, 5) z sin1z,

in ciascuna delle loro singolarita.

Soluzione. Esercizio 1. La funzione presenta due poli semplici nei punti z1 = 1,z2 = −1, con residui rispettivamente 1/2 e −1/2. Si puo applicare la formula (3′) delparagrafo 4.7.

Page 276: Eserciziario Di Analisi Matematica 2

10 Capitolo 4. Funzioni di variabile complessa ©c 978-88-08-12546-0

Esercizio 2. La funzione presenta poli semplici in tutti i punti in cui si annulla ildenominatore; la formula citata poco sopra consente di affermare che i residui valgono−1.Esercizio 3. La funzione presenta una singolarita eliminabile nell’origine. In ciascunodei punti zk = kπ, k intero non nullo, c’e un polo semplice con residuo uguale a kπ.Esercizio 4. Da

e2/z = 1 +2z

+2z2

+ . . .

segue che il residuo nell’origine vale 2.Esercizio 5. Da

z sin1z

= 1 − 13! z2

+ . . .

segue che l’origine e una singolarita essenziale con residuo nullo.

4.7-4. Mediante il metodo dei coefficienti indeterminati, dedurre dagli sviluppi dellefunzioni seno e coseno lo sviluppo

tan z =sin zcos z

= z +13z3 +

215z5 +O(z7).

Soluzione. Sappiamo che la tangente e una funzione dispari, dunque lo sviluppocercato e del tipo tan z = a1z + a3z

3 + a5z5 +O(z7). Sara dunque[

a1z + a3z3 + a5z

5 +O(z7)] [

1 − z2

2+z4

24+O(z6)

]= z − z3

6+

z5

120+O(z7).

Sviluppando il prodotto a primo membro, ed uguagliando i coefficienti delle potenzez, z3 e z5 si ottengono le uguaglianze

a1 = 1,

−12a1 + a3 = −1

6,

124

a1 −12a3 + a5 =

1120

,

da cui segue facilmente il risultato dato nel testo.

4.8. Il teorema dei residui

4.8-1 Calcolare (mediante il teorema dei residui) gli integrali

1)∫ 2π

0

sin2 x dx = π,

2)∫ 2π

0

dx

1 + sin2 x=

√2π,

3)∫ 2π

0

dx

2 + sinx cosx=

4π√15,

4)∫ 2π

0

dx

2 + sinx+ cosx=

√2π.

Suggerimento Trasformare ciascun integrale in un integrale sulla circonferenzadi centro l’origine e raggio 1.

Soluzione. La tecnica consiste nell’utilizzare le formule di Eulero, in modo da scri-vere un integrale esteso alla circonferenza unitaria che si trasformerebbe nell’integraleproposto se si parametrizzasse la circonferenza stessa.

Esercizio 1. Preferiamo indicare con la lettera t la variabile d’integrazione; per ilprimo integrale, posto z = eit, si ha

Page 277: Eserciziario Di Analisi Matematica 2

©c 978-88-08-12546-0 Esercizi 11

sin2 t =(z − 1/z

2i

)2

=−z4 + 2z2 − 1

4z2,

quindi∫ 2π

0

sin2 t dt =ˇ

∮|z|=1

−z4 + 2z2 − 14z3 i

dz.

Abbiamo tenuto conto del fatto che da z = eit segue

dz = ieit dt = iz dt ⇐⇒ dt =1izdz.

A questo punto possiamo utilizzare il teorema dei residui. Poiche la funzine ntegrandaammette nell’origine un polo del terzo ordine con residuo uguale a −i/2, si trova perl’ultimo integrale il valore 2πi(−i/2) = π.

Peraltro l’integrale in esame si calcola agevolmente utilizzando la primitiva

x → t− sin t cos t2

.

Esercizio 2. Si trova∫ 2π

0

dt

1 + sin2 t=ˇ

∮|z|=1

4iz1 − 6z2 + z4

dz.

L’equazione biquadratica che si ottiene uguagliando a zero il denominatore am-mette quattro radici (a due a due tra loro opposte) di cui hanno valore assoluto minoredi 1 le radici z1 = 1−

√2, e z2 = −z1. I residui della funzione integranda in tali punti

valgono entrambi −i/(2√

2), dunque per l’integrale si trova 4πi (−i/(2√

2)) =√

2π.

Esercizio 3. Si trova∫ 2π

0

dt

2 + sin t cos t=ˇ

∮|z|=1

4z−1 + 8iz2 + z4

dz.

Con la sostituzione z2 = s si e condotti a studiare l’equazione di secondo grado−1+8is+s2 = 0, che ammette due radici complesse, di cui s0 = i(

√15−4) ha valore

assoluto minore di 1.Siamo dunque condotti ad applicare il teorema dei residui relativamente ai poli

semplici

z1 = ei3π/4

√4 −

√15 =

−1 + i√2

√4 −

√15

e z2 = −z1; poiche i residui in tali poli valgono entrambi −i/√

15, si trova il risultatofornito nel testo.

Esercizio 4. Si trova∫ 2π

0

dt

2 + sin t+ cos t=ˇ

∮|z|=1

1 − i

i+ (2 + 2i)z + z2dz.

Il denominatore si annulla in due punti, di cui z1 = (1 + i)(√

2/2 − 1) ha valoreassoluto minore di 1. Poiche il residuo della funzione integranda in tale punto vale−i/

√2, si ottiene il risultato fornito nel testo.

4.8-2 Con la stessa tecnica per precedente esercizio verificare che

1)∫ 2π

0

dx

a+ cosx=

2π√a2 − 1

, a > 1,

2)∫ 2π

0

dx

1 + a sinx=

2π√1 − a2

, 0 < a < 1,

3)∫ 2π

0

dx

(a+ b cosx)2=

2πa(a2 − b2)3/2

, a > b > 0.

Page 278: Eserciziario Di Analisi Matematica 2

12 Capitolo 4. Funzioni di variabile complessa ©c 978-88-08-12546-0

Soluzione. Indichiamo ogni volta col simbolo f la funzione di variabile complessa cheviene integrata sulla circonferenza unitaria.Esercizio 1. Si ha∫ 2π

0

dx

a+ cosx=ˇ

∮|z|=1

−2iz2 + 2az + 1

dz = 2πi · res(f ;√a2 − 1 − a) =

= 2πi · −i√a2 − 1

=2π√a2 − 1

.

Esercizio 2. Si ha∫ 2π

0

dx

1 + a sinx=ˇ

∮|z|=1

2az2 + 2iz − a

dz = 2πi · res(f ; i

√1 − a2 − 1

a

)=

= 2πi · 1i√

1 − a2=

2π√1 − a2.

Esercizio 3. Si ha∫ 2π

0

dx

(a+ b cosx)2=ˇ

∮|z|=1

−4iz(bz2 + 2az = b)2

dz =

= 2πi · res(f ;

√a2 − b2 − a

b

)= 2πi · −ia

(a2 − b2)3/2=

=2πa

(a2 − b2)3/2.

4.8-3 Utilizzando il teorema dei residui verificare che

1)∫ ∞

−∞

1x2 + x+ 1

dx =∫ ∞

−∞

1x2 − x+ 1

dx =2π√

3,

2)∫ ∞

−∞

1(x2 + x+ 1)2

dx =∫ ∞

−∞

1(x2 − x+ 1)2

dx =4π

3√

3,

3)∫ ∞

0

x2

(1 + x2)2dx =

π

4,

4)∫ ∞

0

x2

(x2 + 4)2(x2 + 9)dx =

π

200,

5)∫ ∞

−∞

dx

x6 + 1=

2π3,

6)∫ ∞

−∞

dx

x4 + x2 + 1=

π√3,

7)∫ ∞

−∞

sinxx(1 + x2)

dx =π(e− 1)

e,

8)∫ ∞

−∞

cosx1 + x2

dx =π

e.

Soluzione. In ciascuno degli esercizi dal numero 1 al numero 6 indichiamo con lalettera f la funzione di variabile complessa che si ottiene scrivendo z ∈ C al posto dix ∈ R. In base al lemma del grande cerchio si hanno i seguenti risultati.Esercizio 1.∫ ∞

−∞

1x2 + x+ 1

dx = 2πi · res(f ; (−1 + i

√3 )/2

)= 2πi · −i√

3=

2π√3.

Esercizio 2. A differenza dell’esercizio precedente, il punto z0 = −1/2 + i√

3/2 e unpolo doppio per la funzione integranda, dunque il relativo residuo si calcola con laformula fornita dalla Proposizione 4.7-1, ottenendo∫ ∞

−∞

1(x2 + x+ 1)2

dx = 2πi · res(f ; (−1 + i

√3 )/2

)= 2πi · −2i

3√

3=

4π3√

3.

Page 279: Eserciziario Di Analisi Matematica 2

©c 978-88-08-12546-0 Esercizi 13

Esercizio 3. Trattandosi di una funzione pari, possiamo calcolare l’integrale su tuttala retta reale. Abbiamo∫ ∞

−∞

x2

(1 + x2)2dx = 2πi · res(f ; i) = 2πi · −i

4=π

2.

Esercizio 4. Trattandosi di una funzione pari, possiamo calcolare l’integrale su tuttala retta reale. Abbiamo∫ ∞

−∞

x2

(x2 + 4)2(x2 + 9)dx = 2πi [res(f ; 2i) + res(f ; 3i)] = 2πi

[−13i200

+3i50

]=

100.

Esercizio 5. Il polinomio z6 + 1 ammette, nel semipiano Im(z) > 0, i tre zeri

z1 = eiπ/6 =√

32

+ i12, z2 = eiπ/2 = i, ei5π/6 = −

√3

2+ i

12.

Per ogni k = 1, 2, 3, si ha il residuo

rk =1

6z5k

=zk

6z6k

=−zk

6,

quindi l’integrale da calcolare vale

2πi (r1 + r2 + r3) =−πi3

(z1 + z2 + z3) =−πi3

· 2i =2π3.

Esercizio 6. La funzione integranda ammette, nel semipiano Im(z) > 0, i due zeri

z1 = eiπ/3 =12

+ i

√3

2, z2 = ei2π/3 = −1

2+ i

√3

2,

con residui, rispettivamente,

r1 = −14

(1 +

i√3

), r2 =

14

(1 − i√

3

);

ne segue il risultato proposto nel testo.Esercizio 7. In questo caso non e conveniente considerare la funzione

f(z) =sin z

z(1 + z2),

in quanto tale funzione e illimitata (in valore assoluto), tanto nel semipiano Im(z) > 0quanto nel semipiano Im(z) < 0. Consideriamo piuttosto la funzione

f(z) =eiz

z(1 + z2),

la cui restrizione all’asse reale valecosx+ i sinxx(1 + x2)

.

Integriamo tale funzione sul circuito Cr,R mostrato in figura.

r R−r−R

i

Tenendo conto del fatto che a funzione x → cosx/(x(1 + x2)) e dispari, dopo averfatto il limite per R → ∞ e r → 0, e applicato i lemmi di 4.8-3 e 4.8-4, si trova

Page 280: Eserciziario Di Analisi Matematica 2

14 Capitolo 4. Funzioni di variabile complessa ©c 978-88-08-12546-0

i

∫ ∞

−∞

sinxx(1 + x2)

dx− iπ · res(f ; 0) = 2πi · res(f ; i),

cioe ∫ ∞

−∞

sinxx(1 + x2)

dx− π = −πe,

da cui il risultato proposto nel testo.Esercizio 8. Analogamente a quanto abbiamo fatto nell’esercizio precedente, convieneconsiderare la funzione

f(z) =eiz

1 + z2.

Integrando sul circuito CR che si ottiene concatenando il segmento [−R,R] con lasemicirconferenza di centro l’origine e raggio R contenuta nel semipiano Im(z) > 0,applicando il Lemma di Jordan si ottiene∫ ∞

−∞

cosx1 + x2

dx = 2πi · res(f ; i) = 2πi · e−1

2i=π

e.

4.8-4 Verificare l’uguaglianza∫ +∞

0

dx

1 + xn=π

n

1sin(π/n)

, n ≥ 2

mediante il teorema dei residui, utilizzando il circuito che congiunge l’origine conR > 1, successivamente con Re2πi/n attraverso un arco di cerchio, e di nuovo conl’origine.Utilizzare il risultato ottenuto per calcolare l’integrale∫ ∞

0

dx

x3 + 8=

√3

18π.

Soluzione. Sia In l’integrale che vogliamo calcolare e sia f(z) := 1/(1 + zn); ilcircuito specificato contiene soltanto il punto z0 = eπi/n che e un polo semplice per fcon residuo

1nzn−1

0

=z0nzn

0

= − 1nz0.

Il segmento che congiunge l’origine col punto Re2πi/n si parametrizza nella forma

t → e2πi/n · t, 0 ≤ t ≤ R,

dunque l’integrale ad esso relativo vale

e2πi/n

∫ R

0

dt

1 + tn= e2πi/n In.

In definitiva, applicando il teorema dei residui e facendo tendere R a +∞, tenendoconto del lemma del grande cerchio si ottiene

In(1 − e2πi/n) = −2πin

eπi/n,

da cui

In = −2πin

eπi/n

1 − e2πi/n=π

n

2ieπi/n − e−πi/n

=π/n

sin(π/n).

In particolare si ottiene

I2 =∫ +∞

0

dx

1 + x2=π

2,

calcolabile elementarmente utilizzando la funzione arcotangente.L’ulteriore integrale proposto si riconduce al calcolo di I3, osservando che

Page 281: Eserciziario Di Analisi Matematica 2

©c 978-88-08-12546-0 Esercizi 15

∫ ∞

0

dx

x3 + 8=

18

∫ ∞

0

dx

(x/2)3 + 1.

4.8-5 Verificare l’uguaglianza∫ ∞

−∞

ex/3

1 + exdx =

2π√3

utilizzando la poligonale che congiunge, nell’ordine, i punti R, R + 2πi, −R + 2πi,−R, R, con R > 0. Dimostrare che gli integrali estesi ai due segmenti paralleli all’asseimmaginario tendono a 0 per R → ∞.Soluzione. All’interno del circuito suggerito la funzione f(z) = ez/3/(1+ez) ammetteil polo semplice z0 = iπ, con residuo

eiπ/3

eiπ= −eiπ/3.

La retta y = 2π ammette la parametrizzazione t → x+ 2πi, r ∈ R, quindi l’integraledi f relativo ad essa vale ei2π/3 I, dove I e l’integrale che vogliamo calcolare. PerR → ∞ si ottiene dunque

I(1 − ei2π/3) = −2πi · eiπ/3,

da cui

I = −2πieiπ/3

1 − ei2π/3= π

2ieiπ/3 − e−iπ/3

sin(π/3)=

2π√3.

Resta da dimostrare che gli integrali estesi ai due segmenti paralleli all’asse im-maginario tendono a 0 per R → ∞. Per il segmento di estremi R e R + 2πi si ha laparametrizzazione t → z(t) = R+ 2πi · t, 0 ≤ t ≤ 1, dunque tale integrale vale

2πi eR/3

∫ 1

0

ei(2π/3)t

1 + eR ei2πtdt.

Il valore assoluto del numeratore della funzione integranda vale 1, mentre il valoreassoluto del denominatore e maggiore o uguale a

∣∣|eR ei2πt|−|1|∣∣ = eR−1. In definitva

il valore assoluto dell’integrale in esame non supera 2πeR/3/(eR − 1).

Esercizi proposti

4.8-P1. Calcolare

I =∫ +∞

0

√x

x2 + 1dx.

Soluzione. Consideriamo la funzione

f(z) =√z

z2 + 1, z ∈ C

e integriamola sul circuito Cr,R gia utilizzato per l’esercizio 4.8-3, numero 7.Per la funzione√z =

0, per z = 0,√|z| eiθ/2, per z = 0,

dove θ ∈ arg(z), scegliamo una determinazione dell’argomento che collochi la semi-retta di discontinuita della stessa funzione nel semipiano Im(z) < 0, ad esempioθ ∈ (−π/2, 3π/2]. Con tale scelta f e olomorfa in un aperto contenente il circuitoCr,R. Il teorema dei residui fornisce

ˇ

∮Cr,R

f(z) dz = 2πi · res(f ; i) = 2πi limz→i

√z

2z= 2πi

eiπ/4

2i= π

1 + i√2.

Per r → 0 l’integrale di f sulla semicirconferenza di raggio r tende a 0 (Lemmadel piccolo cerchio), e lo stesso accade per l’integrale sulla semicirconferenza di raggio

Page 282: Eserciziario Di Analisi Matematica 2

16 Capitolo 4. Funzioni di variabile complessa ©c 978-88-08-12546-0

R, per R → ∞ (Lemma del grande cerchio). Per r → 0 e R → ∞ l’integrale dif sull’intervallo [r,R] fornisce l’integrale I che vogliamo calcolare; per x < 0, postot := |x| = −x, si ha infine∫ −r

−R

f(x) dx = −i∫ r

R

√t

t2 + 1dt = i

∫ R

r

√t

t2 + 1dt→ iI.

Dunque

I(1 + i) = π1 + i√

2=⇒ I =

π√2.

4.8-P2. Calcolare

I =∫ ∞

0

x3 sinx2

x4 + 1dx.

Soluzione. Consideriamo la funzione

f(z) :=z3eiz2

z4 + 1;

essa possiede quattro poli semplici, di cui uno nel primo quadrante:

z0 = eiπ/4 =1√2(1 + i),

avendo ivi come residuo 1/4e. Valutiamo l’integrale di f sul circuito ottenuto conca-tenando:

• il segmento γ1 che congiunge l’origine col punto z = R sull’asse reale (R > 1);

• il quarto di cerchio γ2 (contenuto nel primo quadrante) che congiunge il punto z = Rcol punto z = iR;

• il segmento γ3 che congiunge il punto z = iR con l’origine.

Applicando il teorema dei residui si trova∫γ1

f(z) dz +∫

γ2

f(z) dz +∫

γ3

f(z) dz =2πi4e

. (∗)

Ammettiamo per un istante che l’integrale su γ2 tenda a 0 per R → ∞; l’integrale suγ1 tende a∫ ∞

0

x3 cosx2

x4 + 1dx+ i

∫ ∞

0

x3 sinx2

x4 + 1dx.

L’integrale su γ3 (si tenga conto del verso di percorrenza e si ponga z = iy) tende a

−∫ ∞

0

y3 cos y2

y4 + 1dy + i

∫ ∞

0

y3 sin y2

y4 + 1dx.

Indicando ovunque la variabile d’integrazione con la lettera x, la (*), per R → ∞,fornisce

2i∫ ∞

0

x3 sinx2

x4 + 1dx =

2πi4e

,

quindi l’integrale da calcolare vale π/(4e).L’integrale su γ2 si puo parametrizzare ponendo z = Reit, 0 ≤ t ≤ π/2. Esso diventa

iR

∫ π/2

0

R3ei3t eiR2(cos 2t+i sin 2t)

R4 ei4t − 1dt.

Prendiamo il valore assoluto e sfruttiamo a denominatore la diseguaglianza triangolarenella forma |z1−z2| ≥

∣∣|z1|− |z2|∣∣; si trova che l’integrale scritto non supera, in valore

assoluto,

Page 283: Eserciziario Di Analisi Matematica 2

©c 978-88-08-12546-0 Esercizi 17

R4

R4 − 1

∫ π/2

0

e−R2 sin 2t dt < 2∫ π/2

0

e−R2 sin 2t dt.

Abbiamo sfruttato il fatto che la funzione R → R4/(R4 − 1) e monotona decrescenteper R > 1, e tende a 1 per R → ∞, dunque e certamente minore di 2 per R abbastanzagrande (ad esempio per R > 2).

Per q. o. fissato t ∈ [0, π/2], la funzione integranda nell’ultimo integrale tende a 0per R → ∞, e, al variare di R, le funzioni integrande sono maggiorate dalla funzionecontinua (dunque sommabile) g(t) = e− sin 2t. Il teorema di Lebesgue della conver-genza dominata assicura la liceita del passaggio al limite sotto il segno di integrale.

Page 284: Eserciziario Di Analisi Matematica 2

5. La trasformata di LaplaceEsercizi

Aggiornamento: febbraio 2003

http://www.ciram.unibo.it/~barozzi/MI2/PDF/MI2-Cap.5-Ese.pdf

5.1. Introduzione alla trasformata di Laplace

5.2. Proprieta della trasformata di Laplace

5.2-1. Consideriamo la funzione limitata f(t) = (sin t)/t (↑ esempio 2.2-5) e indi-chiamo con F (s) la sua L-trasformata. In base alla Prop. 5.1-2 si verifichi che

F ′(s) = −L[tf(t)](s) = −L[sin t](s) = − 11 + s2

.

Se ne deduca, per s reale positivo, F (s) = c − arctan s, e finalmente per s → ∞, inbase alla Prop. 5.1-1:

F (s) =∫ ∞

0

e−st sin t

tdt =

π

2− arctan s.

Soluzione. Da

F ′(s) = − 11 + s2

,

segue F (s) = c − arctan s per ogni s con Re(s) > 0. In particolare, la formula scrittavale per ogni s reale positivo. D’altra parte, per s → ∞, F (s) deve tendere a 0, quindila costante c vale necessariamente π/2.

5.2-2. La funzione integralseno e stata definita nell’esempio 2.2-5 come la primitivadella funzione f(t) = (sin t)/t, nulla nell’origine:

Si(x) :=∫ x

0

sin t

tdt.

Dedurre dalla Proposizione 5.2-3 che la L-trasformata dell’integralseno e

G(s) =F (s)

s=

1s

2− arctan s

).

Per poter applicare il teorema del valore finale, occorre sapere che la funzione in-tegralseno tende ad un limite finito per x → +∞ (in realta tale limite e gia statocalcolato nell’esempio 4.8-5, ma vogliamo ricalcolarlo in modo indipendente). Perottenere questo risultato, si fissi a > 0, e si osservi che, per ogni b > a, si ha

Si(b) =∫ b

0

sin t

tdt =

∫ a

0

sin t

tdt +

∫ b

a

sin t

tdt;

si integri per parti il secondo integrale e si dimostri che esso tende ad un limite finitoper b → +∞ (la funzione x → (cos x)/x2 e sommabile sull’intervallo [a,+∞) in quantomaggiorata in valore assoluto da 1/x2).

Page 285: Eserciziario Di Analisi Matematica 2

2 Capitolo 5. La trasformata di Laplace ©c 88-08-07923-6

Finalmente si applichi il teorema del valore finale alla funzione integralseno ottenendo

limx→+∞

Si(x) = lims→0

s G(s) = lims→0

2− arctan s

)=

π

2.

Soluzione. Lo schema della soluzione e contenuto nell’enunciato. Eseguiamo l’inte-grazione per parti suggerita; si ha∫ b

a

sin t

tdt =

cos a

a− cos b

b+

∫ b

a

cos t

t2dt;

l’addendo (cos b)/b tende a 0 per b → +∞, l’ultimo integrale tende all’integrale dellafunzione sommabile t → (cos t)/t2 su [b, +∞).

5.2-3. Sia δh(t) := χ[0,h](t)/h, con h > 0, come nell’esempio 5.1-3. Abbiamo osservatoche l’integrale di δh vale 1 quale che sia h. Sia fh(t) la primitiva di δh(t) nulla pert ≤ 0. Calcolare fh e la relativa trasformata. A che cosa tende fh(t) per h → 0 ?

h h

1

1h

Figura 5.2-8.

Integrando un impulso unitario

si ottiene una rampa unitaria.

Soluzione. Per la primitiva fh(t) si trova (vedi figura a destra)

fh(t) =

0, per t ≤ 0t/h, per 0 < t ≤ h1, per 1 ≤ t .

Per h → 0, fh(t) tende alla funzione nulla per t ≤ 0, uguale a 1 per t > 0, dunquela funzione limite e il gradino unitario (o funzione di Heaviside) a meno del valorenell’origine. Per la trasformata di fh(t) abbiamo, con un’integrazione per parti,∫ h

0

(t/h) e−st dt +∫ ∞

h

e−st dt = −e−hs

s+

1 − e−hs

hs2+

e−hs

s=

=1 − e−hs

hs2

per ogni s con parte reale positiva. Poiche e−hs = 1 − hs + h2s2/2 + . . . , per h → 0la trasformata calcolata tende a 1/s.

5.2-4. Sia f(t) = χ[0,1](t), cioe la funzione uguale a 1 nell’intervallo [0, 1] e nulla fuoridello stesso intervallo. Verificare che

(f ∗ f)(t) =

t, per 0 ≤ t ≤ 1,2 − t per 1 ≤ t ≤ 2,0, altrimenti.

Tracciare un grafico della convoluzione ottenuta.Soluzione. La funzione τ → f(τ) ha come supporto l’intervallo [0, 1], mentre lafunzione τ → f(t− τ) ha come supporto l’intervallo [t− 1, t]. Dunque la convoluzionee nulla quando t e esterno all’intervallo [0, 2], in quanto, per tali valori, i supportidelle due funzioni in questione sono tra loro disgiunti. Per t ∈ [0, 1] essa vale∫ t

0

dτ = t,

Page 286: Eserciziario Di Analisi Matematica 2

©c 88-08-07923-6 Esercizi 3

mentre per t ∈ [1, 2] essa vale∫ 2

t

dτ = 2 − t.

-1 1 2 3

1

5.2-5. Sia δh(t) := χ[0,h](t)/h, con h > 0, come nell’esercizio 5.2-3. Si verifichi che(f ∗ δh)(t) e uguale alla media integrale della funzione f sull’intervallo [t− h, t]. Se fe una funzione continua su R, a che cosa tende (f ∗ δh)(t) per h → 0 ?Soluzione. Infatti, poiche il supporto della funzione τ → δh(t − τ) e l’intervallo[t − h, t], si trova

(f ∗ δh)(t) =1h

∫ t

t−h

f(τ) dτ.

Se f e continua, la media integrale calcolata coincide col valore di f in un puntoopportuno ξ ∈ [t − h, t], e dunque tende a f(t) per h → 0.

5.2-6. Con i simboli dell’esercizio precedente, calcolare esplicitamente (H ∗ δh)(t) etracciarne il grafico. Si suggerisce di fare calcoli separati per 0 ≤ t ≤ h e per t ≥ h.Verificare che la convoluzione ottenuta e continua su R. Confrontare con quanto vistonell’esercizio 5.2-3.Soluzione. Utilizzando il risultato del precedente esercizio, si riconosce subito che laconvoluzione richiesta e nulla per t < 0, e vale 1 per t > h. Per t ∈ [0, h], si ha

(H ∗ δh)(t) =∫ t

0

11h

dτ =t

h.

In definitiva si ottiene la rampa fh(t) considerata nell’esercizio 5.2-3.

5.2-7. Stesso problema dell’esercizio precedente per (t+ ∗ δh)(t). Verificare che laconvoluzione ottenuta e di classe C(1)(R).Soluzione. Sempre in base al risultato dell’esercizio 5.2-5, abbiamo cha la con-voluzione richiesta e nulla per t ≤ 0. Per 0 < t ≤ h essa vale

1h

∫ t

0

τ dτ =t2

2h,

mentre per t ≥ h essa vale

1h

∫ t

t−h

τ dτ =1h

[τ2

2

]t

t−h= t − h

2.

In definitiva

(t+ ∗ δh)(t) =

0, per t ≤ 0

t2

2h, per 0 < t ≤ h

t − h

2, per t > h.

Come si vede, non solo la convoluzione calcolata e continua, ma anche la sua derivataprima e continua:

(t+ ∗ δh)′(t) =

0, per t < 0t

h, per 0 < t < h

1, per t > h.

Page 287: Eserciziario Di Analisi Matematica 2

4 Capitolo 5. La trasformata di Laplace ©c 88-08-07923-6

5.3. Le funzioni beta e gamma di Eulero

5.3-P1. Dedurre dalla relazione Γ(x + 1) = xΓ(x), la relazione

Γ(x + n + 1) = x(x + 1)(x + 2) . . . (x + n) Γ(x)

per ogni naturale n.

Soluzione. Basta procedere per induzione rispetto a n.

5.4. Inversione della trasformata di Laplace

5.4-P1. Mediante il teorema di convoluzione, determinare l’antitrasformata di La-place per ciascuna delle funzioni

F0(s) =1

(s2 + 1)2, F1(s) =

s

(s2 + 1)2, F2(s) =

s2

(s2 + 1)2.

Soluzione. Poiche 1/(s2 + 1) e s/(s2 + 1) sono la trasformate di (sin t)+ e (cos t)+rispettivamente, le antitrasformate richieste sono, nell’ordine,

f0(t) = (sin t)+ ∗ (sin t)+, f1(t) = (sin t)+ ∗ (cos t)+,

f2(t) = (cos t)+ ∗ (cos t)+.

Per t ≥ 0 si trova

f0(t) =∫ t

0

sin τ sin(t − τ) dτ = sin t

∫ t

0

sin τ cos τ dτ − cos t

∫ t

0

sin2 τ dτ =

= sin tsin2 t

2− cos t

t − sin t cos t

2=

=12

(sin t − t cos t).

In modo analogo si trova

f1(t) =12

t sin t, f2(t) =12

(t cos t + sin t).

5.5. Equazioni differenziali ordinarie

5.5-P1 Utilizzare la trasformata di Laplace per risolvere il problema di valori iniziali

y′′ + 2y′ + y = 1

con le condizioni y(0) = −1, y′(0) = 1.

Soluzione. Si trova

Y (s) =1 − s − s2

s(s + 1)2.

Poiche abbiamo un polo semplice per s1 = 0, e un polo doppio in s2 = −1, avremouna decomposizione in fratti semplice del tipo (si veda la Prop. 4.7-1)

Y (s) =1 − s − s2

s(s + 1)2=

A

s+

B

s + 1+

C

(s + 1)2.

La costante A e semplicemente il residuo di Y (s) nel polo s1 = 0, quindi si calcolaimmediatamente

A = lims→0

sY (s) = 1.

L’uguaglianza

1 − s − s2

s(s + 1)2=

1s

+B

s + 1+

C

(s + 1)2

Page 288: Eserciziario Di Analisi Matematica 2

©c 88-08-07923-6 Esercizi 5

si scrive anche−3 − 2s

(s + 1)2=

B

s + 1+

C

(s + 1)2,

dopodiche si possono calcolare i valori B = −2 e C = −1 col metodo dei coefficientiindeterminati o, piu semplicemente, osservando che −3 − 2s = −2(s + 1) − 1. Indefinitiva

Y (s) =1s− 2

s + 1− 1

(s + 1)2

da cui y(t) = 1 − (2 + t)et per t ≥ 0.

5.5-P2 Determinare le risposte impulsive per ciascuna delle equazioni differenzialilineari a coefficienti costanti:

1. y′′ + y = 0;2. y′′ + y′ + y = 0;3. y′′ − y′ + y = 0;4. y′′ − y = 0;5. y′′ + 2y′ + y = 0.

Soluzione. Si tratta di anti-trasformare le funzioni

P1(s) =1

s2 + 1; P2(s) =

1s2 + s + 1

; P3(s) =1

s2 − s + 1;

P4(s) =1

s2 − 1; P5(s) =

1s2 + 2s1

=1

(s + 1)2.

Si trovano, nell’ordine, le soluzioni

1. y1(t) = (sin t)+ ;

2. y2(t) =2√3

(e−t/2 sin

(√3

2t))

+;

3. y3(t) =2√3

(et/2 sin

(√3

2t))

+;

4. y4(t) = (sinh t)+ ;5. y4(t) = (te−t)+ .

Si osservi il legame tra il comportamento delle risposte impulsive calcolate e la collo-cazione, nel piano complesso, degli zeri dei polinomi caratteristici.

Page 289: Eserciziario Di Analisi Matematica 2

6. La trasformata di FourierEsercizi

http://www.ciram.unibo.it/~barozzi/MI2/PDF/MI2-Cap.6-Ese.pdf

6.1. Introduzione alla trasformata di Fourier

6.2-1. Sia f una funzione sommabile e f(ω) :=∫

Re−iωx f(x) dx la sua F-trasformata.

Utilizzare il lemma di Riemann-Legesgue (↑ Prop. 3.2-1) per dimostrare che

lim|ω|→∞

f(ω) = 0.

Se (ωn) e una successione convergente a ω0, verificare che limn→∞ f(ωn) = f(ω0)(continuita di f(ω)) sfruttando il teorema della convergenza dominata di Lebesgue (↑Prop. 2.3-1).Suggerimento Per dimostrare che lim|ω|→∞ f(ω) = 0, si osservi che, scelto ε > 0,si puo trovare in corrispondenza un r > 0 tale che∫ −r

−∞|f(x)| dx +

∫ ∞

r

|f(x)| dx < ε/2.

Per concludere che f(ω) e infinitesima all’infinito, applicare il lemma di Riemann-Legesgue (↑ Prop. 3.2-1) all’integrale

∫ r

−re−iωx f(x) dx.

Soluzione. Riprendendo il ragionamento indicato nel suggerimento, si ha∫R

e−iωx f(x) dx =∫ −r

−∞e−iωx f(x) dx +

∫ r

−r

e−iωx f(x) dx +∫ ∞

r

e−iωx f(x) dx,

da cui, prendendo i valori assoluti,∣∣∣∣∫R

e−iωx f(x) dx

∣∣∣∣ ≤ ε/2 +∣∣∣∣∫ r

−r

e−iωx f(x) dx

∣∣∣∣ .

Ma il lemma di Riemann-Legesgue ci assicura che l’ultimo integrale scritto si puorendere in valore assoluto minore di ε/2 a patto che |ω| sia abbastanza grande.

Per verificare che

limn→∞

∫R

e−iωnx f(x) dx =∫

R

e−iω0x f(x) dx

basta osservare che le funzioni fn(x) = e−iωnx f(x) sono tutte maggiorate in valoreassoluto dalla funzione sommabile |f(x)|, dunque e applicabile il teorema di Lebesgue.

6.2-2. Con una tecnica simile a quella mostrata nell’esempio 6.1-3, calcolare leseguenti trasformate:

F[ 1x2 ± x + 1

](ω) =

2π√3

e±i ω/2 e−(√

3/2)|ω|,

F[ 1x4 + 1

](ω) =

π√2

e−|ω|/√

2(cos(ω/

√2) + sin(|ω|/

√2)

).

Page 290: Eserciziario Di Analisi Matematica 2

2 Capitolo 6. La trasformata di Fourier ©c 88-08-07923-6

Ritrovare il primo risultato combinando la trasformata della funzione x → 1/(x2+a2)(↑ Tabella 6.2-1) con la Prop. 6.2-2. Dedurre dall’ultimo risultato le trasformate dellefunzioni

x

x4 + 1,

x2

x4 + 1.

Soluzione. La funzione

f(z) =e−iωz

z2 + z + 1

presenta due poli semplici nei punti z1,2 = −1/2 ± i√

3/2, con residui

r1,2 =e(±

√3/2+i/2)ω

±i√

3,

dunque

F[ 1x2 + x + 1

](ω) =

2πi · r1 = 2π√

3ei ω/2 e(

√3/2)ω, per ω < 0,

2πi · r2 = 2π√3

ei ω/2 e−(√

3/2)ω, per ω > 0.

Alternativamente, poiche sappiamo che la trasformata di 1/(a2 + x2) eπ

ae−a|ω|, a > 0

utilizzando la Proposizione 6.2-2 troviamo che la trasformata di1

x2 + x + 1=

1(x + 1/2)2 + (

√3/2)2

e ancora2π√

3ei ω/2 e−(

√3/2)|ω|.

Per quanto riguarda la funzione x → 1/(x4 +1), si puo procedere in modo analogoa quello che abbiamo seguito per l’esercizio precedente, osservando che la funzione

f(z) =e−iωz

z4 + 1presenta nel semipiano Im(z) > 0 i due poli semplici

z1 = eiπ/4 =1 + i√

2, z2 = ei3π/4 =

−1 + i√2

e nel semipiano Im(z) < 0 i due poli semplici

z3 = ei5π/4 =−1 − i√

2, z4 = ei7π/4 =

1 − i√2

.

In ciascun polo zk il residuo vale

rk =e−iωzk

4z3k

= −zk

4e−iωzk ,

in quanto z4k = −1.

Dunque per ω > 0 si ha

F[ 1x4 + 1

](ω) = −2πi(r3 + r4) =

2[z3e

−iωz3 + z4e−iωz4 ] =

=iπ

2√

2e−ω/

√2[− (1 + i)

(cos(ω/

√2) + i sin(ω/

√2)

)+

= + (1 − i)(cos(ω/

√2) − i sin(ω/

√2)

)]=

=π√2

e−ω/√

2(cos(ω/

√2) + sin(ω/

√2)

).

Page 291: Eserciziario Di Analisi Matematica 2

©c 88-08-07923-6 Esercizi 3

Scrivendo |ω| al posto di ω e tenendo conto della parita del coseno, si ottiene ilrisultato.

Per calcolare poi la trasformata delle funzioni x/(x4 + 1) e x2/(x4 + 1) si puoapplicare la Proposizione 6.2-5. Si trova, ad esempio, per la trasformata della primafunzione (che e reale e dispari):

−iπe−|ω|/√

2 sin(ω/√

2).

6.2-3. Generalizzare la prima funzione del precedente esercizio, mostrando che, perogni α reale e per ogni β > 0, si ha

F[

1(x − α)2 + β2

](ω) =

π

βe−i αω e−β|ω|.

Soluzione. Basta osservare che la funzione

f(z) =e−iωz

(z − α)2 + β2

presenta poli semplici nei punti z1,2 = α ± iβ, con residui

r1,2 =e(±β−iα)ω

±2iβ.

6.2-4. Verificare che la funzione

f(x) =

e−ax, per x ≥ 0,0, altrimenti,

con a > 0, ha come F -trasformata 1/(a + iω). Dedurne le trasformate delle funzionix → xf(x), x → x2f(x), in generale x → xnf(x), per ogni naturale n.Soluzione. Si ha

f(ω) =∫ ∞

0

e−iωxe−ax d =[ e−(a+iω)x

−(a + iω)

]∞0

=1

(a + iω).

Per calcolare le trasformate di xf(x), x2f(x), . . . , si puo applicare la proposizione6.2-5, ottenendo

F [xf(x)] (ω) = if ′(ω) =1

(a + iω)2,

F[x2f(x)

](ω) = if ′′(ω) =

2(a + iω)3

,

e in generale

F[x2f(x)

](ω) =

n!(a + iω)n+1

.

Piu semplicemente si puo osservare che la trasformata di Laplace del segnale xn+

essendo n!/sn+1, per Re(s) > 0, la restrizione di tale trasformata alla retta a + iω,ω ∈ R, ci fornisce la F -trasformata del segnale f(x) = (xne−ax)+, secondo quantoabbiamo osservato a pag. 244 del testo.

6.2-5. Si consideri l’impulso unitario

ph(x) :=1h

χ[−h/2,h/2](x) =

1h

, se |x| ≤ h/2

0, altrimenti.Verificare, mediante un calcolo diretto, che

(ph ∗ ph)(x) = th(x) :=1h

(1 − |x|

h

)+

=

1h

(1 − |x|

h

), se |x| ≤ h

0, altrimenti.

Page 292: Eserciziario Di Analisi Matematica 2

4 Capitolo 6. La trasformata di Fourier ©c 88-08-07923-6

Ritrovare lo stesso risultato utilizzando la trasformata di Fourier, sfruttando le trasfor-mate calcolate negli esempi 6.1-1 e 6.2-5.Soluzione. La funzione ξ → ph(ξ) ha come supporto l’intervallo [−h/2, h/2], lafunzione ξ → ph(x− ξ) ha come supporto l’intervallo [x−h/2, x+h/2]; l’intersezionetra tali supporti e dunque

[−h/2, h/2] ∩ [x − h/2, x + h/2] =

∅, se x < −h,[−h/2, x + h/2], se −h ≤ x ≤ 0,[x − h/2, h/2], se 0 ≤ x ≤ h,∅, se h < x.

−h −h/2 0 h/2 h −h −h/2 0 h/2 h

−h −h/2 0 h/2 h −h −h/2 0 h/2 h

x x

x x

Figura 6.E-1. Mutue posizioni dei supporti delle funzioni ξ → ph(ξ) e ξ → ph(x − ξ).

Di conseguenza, poiche il prodotto ph(ξ) ph(x−ξ) vale 1/h2 nei punti dell’intersezioneappena calcolata, mentre vale 0 nei restanti punti, si ha

(ph ∗ ph)(x) =

0, se x < −h,h + x

h2, se −h ≤ x ≤ 0,

h − x

h2, se 0 ≤ x ≤ h,

0, se h < x.

Questo e precisamente il risultato annunciato; salvo le notazioni. Si osservi che laconvoluzione appena calcolata si scrive anche, utilizzando i simboli dell’esempio 6.2-5, gh(x)/h2.

-1 1

1

-1 1

1

-1 1

1

Figura 6.E-2. Da sinistra a destra: grafici degli impulsi x → ph(x), x → gh(x) e x →gh(x)/h2 = (ph ∗ ph)(x), con h = 0.7.

D’altra parte ph ha come F -trasformata (v. esempio 6.1-1)

Page 293: Eserciziario Di Analisi Matematica 2

©c 88-08-07923-6 Esercizi 5

2h

sin(hω/2)ω

dunque ph ∗ ph ha come F -trasformata

4h2

(sin(hω/2)

ω

)2

che sappiamo essere la F -trasformata di gh(x)/h2.

6.2-6. Con i simboli del precedente esercizio, verificare che

(ph ∗ H)(x) =

0, per x < −h/2x/h + 1/2, per −h/2 ≤ x ≤ h/21, per x ≥ h/2;

(th ∗ H)(x) =

0, per x < −h

(x + h)2/(2h2), per −h ≤ x ≤ 0(−x2 + 2hx + h2)/(2h2), per 0 ≤ x ≤ h

1, per x ≥ h.

(th ∗ ph)(x) =

0, per |x| > 3h/2(3h + 2x)2/(8h2), per −3h/2 ≤ x ≤ −h/23/4 − x2, per −h/2 ≤ x ≤ h/2(3h − 2x)2/(8h2), per h/2 ≤ x ≤ 3h/2.

H(x) e la funzione di Heaviside. Si osservi che th ∗ ph = th ∗ th ∗ th. Verificare che(th ∗ H)(x) e (th ∗ ph)(x) sono funzioni di classe C(1)(R).Soluzione. Si osservi che, per ogni funzioni sommabile f , si ha

(f ∗ H)(x) =∫

R

f(ξ)H(x − ξ) dξ =∫ x

−∞f(ξ) dξ.

Pertanto

(ph ∗ H)(x) =∫ x

−∞ph(ξ) dξ =

0, per x < −h/2(x +

h

2

) 1h

=x

h+

12, per −h/2 ≤ x ≤ h/2

1, per x ≥ h/2.

Analogamente

(th ∗ H)(x) =∫ x

−∞th(ξ) dξ =

0, per x < −h

(x + h)2/(2h2), per −h ≤ x ≤ 0(−x2 + 2hx + h2)/(2h2), per 0 ≤ x ≤ h

1, per x ≥ h.

Si tenga conto che, per −h ≤ x ≤ 0, si ha∫ x

−∞th(ξ) dξ =

1h2

∫ x

−h

(ξ + h) dξ,

mentre per 0 ≤ x ≤ h si ha∫ x

−∞th(ξ) dξ =

12

+1h2

∫ x

0

(h − ξ) dξ.

Se le funzioni e ph e th vengono considerate come densita di due varibili aleatoriecontinue, le funzioni ph ∗ H e th ∗ H sono le rispettive funzioni di ripartizione (ofunzioni di distribuzione; in inglese: cumulative distribution functions).

Page 294: Eserciziario Di Analisi Matematica 2

6 Capitolo 6. La trasformata di Fourier ©c 88-08-07923-6

-2 -1 1 2

0.5

1

-2 -1 1 2

0.5

1

Figura 6.E-3. Grafico della funzione (ph ∗ H)(x) (a sinistra), della funzione (th ∗ H)(x) (a

destra); in figura h = 0.8. I grafici non sono monometrici.

Il calcolo per la convoluzione th ∗ ph e analogo, tenendo conto del fatto che ξ →th(ξ) ha come supporto l’intervallo [−h, h], mentre ξ → ph(ξ) ha come supportol’intervallo [−h/2, h/2].

6.2-7. Siano f e g due funzioni con i supporti contenuti, rispettivamente, negliintervalli [a, b] e [c, d], vale a dire f e nulla fuori di [a, b], g e nulla fuori di [c, d];verificare che f ∗ g ha il supporto contenuto nell’intervallo [a + c, b + d]:

(supp f ⊆ [a, b]) ∧ (supp g ⊆ [c, d]) =⇒ supp (f ∗ h) ⊆ [a + c, b + d].

Suggerimento Per ogni fissato x, il supporto della funzione ξ → g(x−ξ) e contenutonell’intervallo [x − d, x − c].Soluzione. La funzione integranda della convoluzione, ξ → f(ξ)g(x − ξ), e identi-camente nulla per quei valori di x per i quali gli intervalli [a, b] e [x − d, x − c] sonodisgiunti, dunque per

x − c < a ⇐⇒ x < a + x, x − d > b ⇐⇒ x > b + d.

6.2-8. Siano f e g due impulsi unitari, nel senso che si tratta di due funzioni aventicome supporto un intervallo limitato e integrale uguale a 1:

∫R

f(x) dx =∫

Rg(x) dx =

= 1. Verificare che f ∗ g e ancora un impulso unitario.Suggerimento Utilizzare la trasformata di Fourier per verificare che l’integrale dif ∗ g vale 1.Soluzione. In base al precedente esercizio f ∗ g e ancora una funzione a supportocompatto. Posto h = f ∗ g, si ha poi

∫R

h(x) dx = h(0) = f(0) g(0) = 1.

6.2-9. Verificare che la convoluzione tra due funzioni reali pari e ancora reale pari.Soluzione. Con gli stessi simboli del precedente esercizio, si ha che h(ω) e unafunzione reale pari in quanto prodotto delle funzioni reali pari f(ω) e g(ω). Ne segueche anche h e reale pari in virtu della formula di dualita (v. formula (4′) a pag. 244):

h (ω) = 2πh(−ω).

Peraltro e immediato che la convoluzione tra due funzioni reali sia reale, mentre none altrettanto immediato che la convoluzione tra due funzioni pari sia ancora pari.Occorre un calcolo: se f e g sono pari, allora da (f ∗ g)(x) =

∫R

f(ξ)g(x− ξ) dξ segue

(f ∗ g)(−x) =∫

R

f(ξ) g(−x − ξ) dξ = (f e g sono pari)

=∫

R

f(−ξ) g(x + ξ) dξ = (t = −ξ)

= −∫ −∞

∞f(t) g(x − t) dt =

∫R

f(t) g(x − t) dt = (f ∗ g)(x).

Page 295: Eserciziario Di Analisi Matematica 2

©c 88-08-07923-6 Esercizi 7

-2 -1 1 2

0.5

-2 -1 1 2

0.5

Figura 6.E-4. A sinistra il grafico della funzione (p1 ∗ t1)(x) (↑ esercizio 6.2-6), a destra il

grafico della funzione (t1 ∗ t1)(x) (↑ esercizio 6.2-10). La prima funzione e di classe C(1)(R),

la seconda e di classe C(2)(R). I grafici non sono monometrici.

6.2-10. Coi simboli dei precedenti esercizi, verificare che (t1 ∗ t1)(x) e la funzionepari che, per x ≥ 0, vale

(t1 ∗ t1)(x) =

x3/2 − x2 + 2/3, per 0 ≤ x ≤ 1−x3/6 + x2 − 2x + 4/3, per 1 ≤ x ≤ 20, per 2 ≤ x.

Si osservi che t1 ∗ t1 = p1 ∗ p1 ∗ p1 ∗ p1. Verificare che (t1 ∗ t1)(x) e una funzione diclasse C(2)(R); si tratta di una cosiddetta funzione spline cubica.Soluzione. In base al precedente esercizio ci basta calcolare (t1 ∗ t1)(x) per x ≥ 0.La figura seguente mostra le mutue posizioni della funzioni ξ → t1(ξ) e ξ → t1(x− ξ)a seconda che sia 0 ≤ x ≤ 1, 1 ≤ x ≤ 2 ed infine 2 ≤ x.

-1 1 2 3

1

-1 1 2 3

1

-1 1 2 3

1

x x

x

Figura 6.E-5. Mutue posizioni delle funzioni ξ → t1(ξ) e ξ → t1(x − ξ).

Se 0 ≤ x ≤ 1, allora

(t1 ∗ t1)(x) =∫ 0

x−1

(1 + ξ)(1 − x + ξ) dξ +∫ x

0

(1 − ξ)(1 − x + ξ) dξ+

=∫ 1

x

(1 − ξ)(1 + x − ξ) dξ,

Page 296: Eserciziario Di Analisi Matematica 2

8 Capitolo 6. La trasformata di Fourier ©c 88-08-07923-6

mentre se 1 ≤ x ≤ 2, allora

(t1 ∗ t1)(x) =∫ 1

x−1

(1 − ξ)(1 − x + ξ) dξ,

da cui segue il risultto con un calcolo elementare. Basta poi derivare due volte leespressioni ottenute negli intervalli 0 ≤ x ≤ 1, 1 ≤ x ≤ 2 e 2 ≤ x, per verificare chetanto le derivate prime quanto le derivate seconde si raccordano con continuita neipunti di ascissa intera.

6.2-11. Verificare, mediante un calcolo diretto, che la convoluzione della funzione1√2π

e−x2/2

(densita di una variabile aleatoria normale standard) con se stessa e (↑ esempio 6.2-7)

12√

πe−x2/4.

Suggerimento La convoluzione in questione si scrive

12π

∫R

e−t2/2 e−(x−t)2/2 dt =e−x2/2

∫R

e−(t2−xt) dt;

utilizzare il “completamento del quadrato”, cioe l’identita t2−xt = (t−x/2)2−x2/4.Soluzione. Proseguendo in base al suggerimento si trova

12π

∫R

e−t2/2 e−(x−t)2/2 dt =e−x2/2ex2/4

∫R

e−ξ2dξ =

12√

πe−x2/4 dt,

avendo operato il cambiamento di variabile ξ = t − x/2.

Esercizi proposti

6.2-P1. Calcolare la trasformata di Fourier della funzione reale e pari x → f(x) chevale cos x per |x| ≤ π/2 e vale 0 nei restanti punti dell’asse reale.Soluzione. Trattandosi di un segnale reale pari, la F -trasformata si riduce ad unacoseno-trasformata

f(ω) =∫ π/2

−π/2

cos(ωx) cos x dx = 2∫ π/2

0

cos(ωx) cos x dx.

L’ultimo integrale puo essere calcolato con due integrazioni per parti, oppure trasfor-mando l’espressione cos(ωx) cos x mediante le cosiddette formule di Werner. Seguen-do la prima strada si ha

I =∫ π/2

0

cos(ωx) cos x dx =[sin x cos(ωx)

]π/2

0+ ω

∫ π/2

0

sin x sin(ωx) dx =

= cos(πω/2) + ω[[

− cos x sin(ωx)]π/2

0+ ω

∫ π/2

0

cos x cos(ωx) dx]

=

= cos(πω/2) + ω2I.

Si trova dunque

f(ω) =2 cos(πω/2)

1 − ω2,

che e ancora una funzione reale pari. Si osservi che la trasformata ottenuta ha singo-larita eliminabili nei punti ω = ±1, in quanto tali valori sono zeri semplici tanto delnumeratore quanto del denominatore.

Page 297: Eserciziario Di Analisi Matematica 2

©c 88-08-07923-6 Esercizi 9

Alternativamente si puo osservare che f(x) = χ[−π/2,π/2] cos x, e quindi, essendonota la F -trasformata di χ[−π/2,π/2], si puo applicare la formula fornita nell’esempio6.2-3. Si trova dunque l’espressione

f(ω) =12

(2 sin[(π/2)(ω − 1)]

ω − 1+

2 sin[(π/2)(ω + 1)]ω + 1

).

Applicando le formule di addizione alle funzioni a numeratore si ottiene poi

f(ω) = cos(πω/2)( −1

ω − 1+

1ω + 1

)=

2 cos(πω/2)1 − ω2

.

6.2-P2. Calcolare la trasformata di Fourier della funzione reale e dispari x → f(x)che vale sin x per |x| ≤ π e vale 0 nei restanti punti dell’asse reale.Soluzione. L’esercizio puo essere risolto con la stessa tecnica usata nel precedente.Si trova la trasformata (puramente immaginaria e dispari)

f(ω) =2 i sin(π ω)

ω2 − 1.

6.2-P3. Mediante un calcolo diretto calcolare la F -trasformata della funzione f(x) == χ[0,a](x) (impulso di ampiezza unitaria e durata a > 0). Ottenere poi la trasformatadelle funzioni x → χ[0,a](x) e x → χ[0,a](x) − χ[−a,0](x) in base al fatto che questesono rispettivamente la parte pari e la parte dispari della funzione x → 2χ[0,a](x).Soluzione. Per la funzione data si trova la trasformata

f(ω) =∫ a

0

e−iωx dx =[− e−iωx

]=

1 − e−iaω

iω=

sin(aω)ω

+ icos(aω) − 1

ω.

Per la funzione x → χ[0,a](x) si trova dunque la trasformata

2 Re[f(ω)] =2 sin(aω)

ω,

mentre per la funzione x → χ[0,a](x) − χ[−a,0](x) si trova la trasformata

2i Im[f(ω)] = 2icos(aω) − 1

ω= i

4 sin2(aω/2)ω

.

Abbiamo sfruttato l’identita 1 − cos t = 2 sin2(t/2). Si confronti con gli esempi 6.1-1e 6.2-2.

6.2-P4. Sia f una funzione sommabile e f(ω) = X(ω)+ iY (ω) la sua F -trasformata,con X e Y reali. Dmostrare che se X e pari e Y e dispari, allora |f(ω)| e pari eArg[f(ω)] e “sostanzialmente” dispari, nel senso che, tranne gli eventuali valori di ω

per cui f(ω) e reale negativa(⇐⇒ (X(ω) < 0) ∧ (Y (ω) = 0)

), si ha

Arg[f(−ω)] = −Arg[f(ω)].

Soluzione. Infatti

|f(−ω)|2 = X(−ω)2 + Y (−ω)2 = X(ω)2 +(− Y (ω)

)2 = X(ω)2 + Y (ω)2.

Si osservi poi che, nelle ipotesi ammesse, X(−ω) + iY (−ω) e il coniugato di X(ω) +iY (ω), e dunque ha come argomento principale l’opposto dell’argomento principaledi quest’ultimo, a meno che esso non sia reale negativo:

X(ω) + iY (ω) = X(ω) − iY (ω) = X(ω) < 0,

quindi Arg[f(−ω)] = Arg[f(ω)] = π.

6.2-P5. Utilizzare la tecnica mostrata nell’esercizo 6.2-8 per dimstrare che la con-voluzione tra due densita di probabilita (funzioni non negative con integrale unitariosu R) e ancora una densita di probabilita.Soluzione. A quanto mostrato nella soluzione dell’esercizio 6.2-8 basta aggiungerel’osservazione che la convoluzione tra due funzioni non negative e ancora una funzionenon negativa.

Page 298: Eserciziario Di Analisi Matematica 2

7. DistribuzioniEsercizi

[Revisione: ottobre 2004]

http://www.ciram.unibo.it/~barozzi/MI2/PDF/MI2-Cap.7-Ese.pdf

7.1. Il concetto di distribuzione

7.1-1. Consideriamo la funzione

f(x) := (e−1/x)+ =

e−1/x, per x > 0 ,

0, altrimenti.

Si dimostri che essa e di classe C(∞)(R), verificando innanzitutto che limx→0+ f(x) = 0, e

successivamente che una relazione analoga vale per ciascuna delle derivate.

Suggerimento Si dimostri, procedendo per induzione, che per x > 0 ciascuna delle

derivate della funzione f si puo scrivere come prodotto pn(1/x) f(x), dove pn e un polinomio

opportuno nella variabile 1/x.

Soluzione. Osserviamo innanzitutto che se p e un qualsivoglia polinomio si ha

limx→0+

p(1/x)f(x) = limt→+∞

p(t)

et= 0, (t := 1/x)

come si riconosce applicando la regola di L’Hopital tante volte quant’e il grado di p.

Cio posto, chiaramente la proposizione sussiste per n = 0 ponendo p0 = 1. Se per un

assegnato n si ha Dnf(x) = pn(1/x) f(x), con pn polinomio opportuno, allora

Dn+1f(x) = −p′n(1/x)

1

x2f(x) − pn(1/x)

1

x2f(x) = pn+1(1/x) f(x)

avendo posto

pn+1(1/x) := −p′n(1/x)

1

x2− pn(1/x)

1

x2.

Abbiamo utilizzato la relazione f ′(x) = −(1/x2) f(x).

7.1-2. In base al risultato del precedente esercizio, verificare che la funzione

φ(x) :=

e1/(x2−1), per |x| < 1,

0, altrimenti,

gia ripetutamente considerata, appartiene allo spazio D(R). Si scelga c :=∫

Rφ(x) dx e si

consideri la funzione

v(x) :=1

c

∫ x

−∞φ(t) dt.

Si verifichi che v e una funzione crescente, di classe C(∞)(R), nulla per x ≤ −1, uguale a 1

per x ≥ 1. Si esamini la funzione vε(x) := v(x/ε), con ε > 0.

Soluzione. La funzione φ e di classe C(∞)(R) in quanto composta mediante funzioni della

stessa classe. Quanto alla v, essa e crescente in quanto primitiva della funzione non negativa

φ. La funzione vε(x) := v(x/ε) e qualitativamente simile alla v, ma e strettamente crescente

Page 299: Eserciziario Di Analisi Matematica 2

2 Capitolo 7. Distribuzioni ©c 88-08-07923-6

Figura 7.E-1.

La funzione v(x) e crescente,

nulla per x ≤ −1, uguale

a 1 per x ≥ 1.-2 -1 1 2

0.5

1

nell’intervallo [−ε, ε], mentre vale 0 per x ≤ −ε e vale 1 per x ≥ ε. Il suo grafico si ottiene

da quella di v comprimendo quest’ultimo di un fattore 1/ε nel senso dell’asse delle ascisse.

7.1-3. Dato l’intervallo [a, b], con a < b, si verifichi che la funzione (i simboli sono quelli del

precedente esercizio)

v(x) := vε(x − a) · vε(b − x)

ha come supporto l’intervallo [a − ε, b + ε] e vale 1 nell’intervallo [a + ε, b − ε] (si suppone

che sia 2ε < b − a).

Soluzione. Basta osservare che il grafico di vε(x−a) si ottiene da quello di vε(x) traslandolo

della quantita a, mentre il grafico di vε(b − x) si ottiene da quello di vε(x) traslandolo della

quantita b e poi simmetrizzandolo rispetto alla parallela all’asse delle ordinate passante per

il punto di ascissa b.

1 2 3 4

1

1 2 3 4

1

1 2 3 4

1

Figura 7.E-2. Dall’altro al basso: grafici delle funzioni vε(x − a), vε(b − x) e del loro

prodotto vε(x − a) · vε(b − x), per a = 0.5, b = 3, ε = 0.3.

7.1-4. Si consideri la funzione f(x) := (1 − |x − 1|)+ e se ne tracci il grafico. In base a

quanto visto nell’esempio 7.1-7, si verifichi che la successione fk(x) := kf(kx) tende a δ(x)

per k → ∞, mentre la stessa successione tende puntualmente a 0 per ogni x reale. Si riveda

l’esempio 2.1-3 con hn = n.

Page 300: Eserciziario Di Analisi Matematica 2

©c 88-08-07923-6 Esercizi 3

1 2

1

Figura 7.E-3.

Grafico della funzione

f(x) := (1 − |x − 1|)+.

Soluzione. Per ogni x ≤ 0, si ha fk(x) = 0 per ogni k, mentre, per ogni fissato x > 0, si ha

fk(x) = 0 per ogni k ≥ 2/x, in quanto la funzione fk ha come supporto l’intervallo [0, 2/k].

Per quanto riguarda la convergenza alla distribuzione δ, si tratta semplicemente di un caso

particolare di quanto dimostrato nell’esempio 7.1-7, scrivendo k al posto di λ.

Esercizi proposti

7.1-P1. Se u(x) e la funzione gradino unitario, u(x)−χ[0,+∞)(x), verificare che la successione

fn(x) := u(x − n) tende a 0 in L1loc(R).

Soluzione. Scelto ad arbitrio l’intervallo [a, b], risulta fn(x) = 0 su [a, b] per tutti gli n > b.

7.1-P2. Sia fn(x) := χ[0,n](x); verificare che fn tende a u(x) in L1loc(R).

Soluzione. Scelto ad arbitrio l’intervallo [a, b], risulta fn(x) = u(x) su [a, b] per tutti gli

n > b.

7.1-P3. Posto

fn(x) :=

0, per x ≤ 0

nx, per 0 ≤ x ≤ 1/n,

1, per x ≥ 1/n,

verificare che fn tende a u(x) in L1loc(R).

Soluzione. Le funzioni fn e u coincidono fuori dell’intervallo [0, 1/n]. Su un qualunque

intervallo [a, b] che contanga, in tutto o in parte, l’intervallo [0, 1/n] si ha∫ b

a

|fn(x) − u(x)| dx ≤∫ 1/n

0

|fn(x) − u(x)| dx =

∫ 1/n

0

(1 − nx) dx =1

2n.

7.1-P4. Dalla relazione eikx → 0 (in D′(R)) per k → ∞, stabilita nell’esempio 7.1-5, dedurre

che sin2(kx) → 1/2 utilizzando le formule di Eulero.

Soluzione. Si ha

sin2(kx) =

(eikx − e−ikx

2i

)2

= −ei2kx + e−i2kx

4+

1

2→ 1

2.

7.1-P5. verificare che la successione k → sin(kx) (che tende a 0 nel senso delle distribuzioni)

non tende a 0 in L1loc(R).

Soluzione. Si ha, per esempio∫ π

0

| sin(kx)| dx =

k=n−1∑k=0

∫ (k+1)π/n

kπ/n

| sin(kx)| dx = n2

n= 2.

Page 301: Eserciziario Di Analisi Matematica 2

4 Capitolo 7. Distribuzioni ©c 88-08-07923-6

1 2 3

1

Figura 7.E-4.

Grafico della funzione

x → | sin(5x)| sull’intervallo

[0, π].

7.2. Operazioni sulle distribuzioni

7.2-1. Si consideri la distribuzione f(x) :=∑∞

k=−∞ δ(x − k); per ogni funzione test v si

ha 〈f(x), v(x)〉 =∑∞

k=−∞ v(k), dove la somma indicata contiene al piu un numero finito di

termini diversi da 0, quelli corrispondenti agli interi k che appartengono al supporto di v. Si

verifichi che f e periodica di periodo 1: f(x) = f(x + 1).

Soluzione. Si ha 〈f(x+1), v(x)〉 = 〈f(x), v(x−1)〉 =∑∞

k=−∞ v(k−1); ma l’ultima somma,

scrivendo h al posto di k − 1 e poi ancora k al posto di h, si scrive ancora∑∞

k=−∞ v(k). In

conclusione: tanto 〈f(x), v(x)〉 quanto 〈f(x + 1), v(x)〉 forniscono la somma dei valori che la

funzione test v assume nei punti di ascissa intera appartenenti al proprio supporto.

7.2-2. Verificare che la distribuzione v.p.(1/x) (↑ esempio 7.1-8) e dispari.

Soluzione. Si ha⟨v.p.

1

−x, v(x)

⟩=

⟨v.p.

1

x, v(−x)

⟩=

∫ r

−r

v(−x) − v(0)

xdx

dove r e abbastanza grande perche l’intervallo [−r, r] contenga il supporto di v. Ponendo t =

−x, e successivamente indicando ancora con x la variabile di integrazione l’ultima quantita

si scrive∫ −r

r

v(t) − v(0)

tdt = −

∫ r

−r

v(x) − v(0)

xdx = −

⟨v.p.

1

x, v(x)

⟩.

7.2-3. Verificare che se f e una distribuzione pari, essa si annulla in corrispondenza di ogni

funzione test v dispari, e viceversa.

Soluzione. Sia f pari, dunque f(x) = f(−x). Allora per ogni funzione test v si ha

〈f(x), v(x)〉 = 〈f(−x), v(x)〉 = 〈f(x), v(−x)〉;ma se v e dispari, v(−x) = −v(x), dunque l’ultima quantita e uguale a −〈f(x), v(x)〉. In

conclusione:

〈f(x), v(x)〉 = −〈f(x), v(x)〉 =⇒ 〈f(x), v(x)〉 = 0.

7.2-4. Verificare che se f e una distribuzione pari, f ′ e dispari e viceversa.

Soluzione. Utilizzeremo:

(a) la composizione di una distribuzione f con la funzione x → −x (simmetria rispetto

all’origine);

(b) la derivata di una distribuzione.

Si ha

〈f ′(−x), v(x)〉 a=〈f ′(x), v(−x)〉 =

b=−〈f(x),

(v(−x)

)′〉 = 〈f(x), v′(−x)〉 =

a=〈f(−x), v′(x)〉 = 〈f(x), v′(x)〉 = (f e pari)

b=−〈f ′(x), v(x)〉.

Page 302: Eserciziario Di Analisi Matematica 2

©c 88-08-07923-6 Esercizi 5

Dunque f ′(−x) = −f ′(x).

7.2-5. Se f e una distribuzione e ψ una funzione di classe C(∞), verificare la formula

(ψ f)′ = ψ′ f + ψ f ′.

Soluzione. Per ogni funzione test v si ha:

〈(ψ f)′, v〉 = −〈ψ f, v′〉 = −〈f, ψ v′〉;

analogamente

〈ψ′ f, v〉 = 〈f, ψ′ v〉,

〈ψ f ′, v〉 = 〈f ′, ψ v〉 = −〈f, (ψ v)′〉 =

= −〈f, ψ′ v〉 − 〈f, ψ v′〉,da cui il risultato sommando membro a membro.

7.2-6. Consideriamo la famiglia di funzioni localmente sommabili

fa(x) :=x

a2 + x2, a = 0.

Figura 7.E-5.

La famiglia di funzioni

fa(x) := x/(a2 + x2) tende

alla distribuzione v.p. (1/x)

per a → 0.

-2 -1 1 2

-1

1

2

a = 1

a = 2/3

a = 1/3

Dimostrare che

lima→0

fa(x) = v.p.1

x.

Suggerimento Si proceda, in modo analogo a quanto visto nell’esempio 7.2.-2, osservando

che, per ogni funzione test v con supporto contenuto nell’intervallo [−r, r], si puo scrivere

〈fa(x), v(x)〉 =

∫ r

−r

x

a2 + x2v(x) dx =

∫ r

−r

x

a2 + x2

(v(x) − v(0)

)dx =

=

∫ r

−r

x2

a2 + x2

v(x) − v(0)

xdx.

Nell’ultimo integrale scritto si passi al limite sotto il segno di integrale applicando il teorema

della convergenza dominata di Lebesgue (↑ Prop 2.3-1).

Soluzione. Proseguendo nel ragionamento suggerito, basta osservare che la famiglia di

funzioni

x → x2

a2 + x2

v(x) − v(0)

x

ammette la maggiorante x →∣∣[v(x) − v(0)]/x

∣∣, che e sommabile, in quanto continua,

sull’intervallo [−r, r].

Page 303: Eserciziario Di Analisi Matematica 2

6 Capitolo 7. Distribuzioni ©c 88-08-07923-6

-2 -1 1 2

-2

-1

1

2

-2 -1 1 2

-2

-1

1

2

Figura 7.E-6. A sinistra viene mostrato il grafico di una funzione test v, a destra il grafico

della funzione x → (v(x)−v(0)/x, sovrapposto a quello della funzione dispari x → −v(0)/x.

Esercizi proposti

7.2-P1. Utilizzare l’esercizio 7.2-5 per calcolare la distribuzione xδ′(x).

Soluzione. Sappiamo che xδ(x) = 0, dunque

0 =(xδ(x)

)′= δ(x) + xδ′(x) =⇒ xδ′(x) = −δ(x).

D’altra parte un calcolo diretto fornisce, per ogni funzione test v,

〈xδ′(x), v(x)〉 = 〈δ′(x), x v(x)〉 = −〈δ(x),(x v(x)

)′〉 =

= −〈δ(x), v(x)〉 − 〈δ(x), x v′(x)〉 = −〈δ(x), v(x)〉.

7.2-P2. Per la funzione f(x) := xn+/n!, verificare che f (k)(x) = xn−k

+ /(n − k)! per ogni

k ≤ n, mentre f (n+1)(x) = δ(x).

Soluzione. La funzione f ha derivate continue fino all’ordine n−1, essendo f (n−1)(x) = x+,

funzione che, a sua volta, ha come derivata q.o. la funzione x → x0+, e quest’ultima coincide

q.o. col gradino unitario u(x).

7.2-P3. Consideriamo la famiglia di funzioni

δh(x) :=1

hχ([−h/2,h/2])(x),

che sappiamo tendere alla δ di Dirac per h → 0. In precedenza (v. esercizio 6.2-5 e seguenti)

abbiamo usato anche il simbolo ph(x) per la stessa funzione. Si verifichi che, se f e una

funzione polinomiale di primo grado, f(x) = ax + b, allora

(δh ∗ f)(x) = f(x), ∀h > 0.

Analogamente, se f(x) = ax2 + bx + c e una funzione polinomiale di secondo grado,

(δh ∗ f)(x) = f(x) + ah2

12, ∀h > 0.

Soluzione. Per ogni funzione f sommabile si ha

(δh ∗ f)(x) =1

h

∫ x+h/2

x−h/2

f(t) dt,

cioe (δh ∗f)(x) e la media integrale di f sull’intervallo [x−h/2, x+h/2]. Se f e un polinomio

di primo grado, si trova subito∫ x+h/2

x−h/2

(at + b) dt = a[t2

2

]x+h/2

x−h/2+ b = axh + bh = f(x) · h.

Geometricamente: il sottografico di f , relativamente all’intervallo [x − h/2, x + h/2], e un

trapezio di area f(x) · h.

Se poi f(x) e un polinomio di secondo grado, scriviamo f(x) = aq(x) + p1(x) dove

q(x) = x2 e p1(x) e il polinomio di primo grado bx + c: Avremo che

Page 304: Eserciziario Di Analisi Matematica 2

©c 88-08-07923-6 Esercizi 7

(δh ∗ f)(x) = a(δh ∗ q)(x) + (δh ∗ p1)(x) = a(δh ∗ q)(x) + p1(x),

per quanto gia sappiamo sulla convoluzione di δh con un polinomio di primo grado. In

definitiva si tratta di verificare che

(δh ∗ q)(x) = x2 +h2

12.

Infatti

1

h

∫ x+h/2

x−h/2

t2 dt =1

3h

[t3

]x+h/2

x−h/2= x2 +

h2

12.

7.2-P4. Con gli stessi simboli dei precedenti esercizi, si dimostri che

(δh ∗ sin)(x) =sin(h/2)

h/2sin x, (δh ∗ cos)(x) =

sin(h/2)

h/2cos x.

Soluzione. Si ha∫ x+h/2

x−h/2

sin t dt = −[cos t

]x+h/2

x−h/2= cos(x − h/2) − cos(x + h/2) = 2 sin x sin(h/2),

in quanto le formule di addizione forniscono

cos(x ± h/2) = cos x cos(h/2) ∓ sin x sin(h/2).

Geometricamente: il grafico della funzione (δh∗sin)(x) e il grafico della funzione seno, ridotto

in ampiezza del fattore sin(h/2)/(h/2); si osservi che tale fattore tende crescendo a 1 per

h → 0. Calcolo analogo per la funzione coseno.

7.2-P5. Con gli stessi simboli del precedente esercizio, verificare che se f(x) = |x|, allora

(δh ∗ f)(x) =

|x|, per |x| > h/2,

x2

h+

h

4, per |x| ≤ h/2.

Verificare che (δh ∗ f)(x) e una funzione di classe C(1)(R).

Soluzione. Per |x| > h/2 la restrizione di f all’intervallo [x − h/2, x + h/2] e un polinomio

di primo grado, dunque il risultato segue dal precedente esercizio. Se poi |x| ≤ h/2, si ha∫ x+h/2

x−h/2

|t| dt = −∫ 0

x−h/2

t dt +

∫ x+h/2

0

t dt = −[t2

2

]0

x−h/2+

[t2

2

]x+h/2

0=

=1

2

[(x +

h

2

)2

+(x +

h

2

)2]= x2 +

h2

4.

-2 -1 1 2

1

Figura 7.E-7.

Grafico della

convoluzione (δh ∗ f)(x)

per h = 0.5.

Page 305: Eserciziario Di Analisi Matematica 2

8 Capitolo 7. Distribuzioni ©c 88-08-07923-6

Per quanto riguarda la derivata della convoluzione calcolata, basta osservare che, per |x| <

h/2, tale derivata vale 2x/h; essa tende a 1 per x → 1−, e tende a −1 per x → −1+.

7.3. Distribuzioni temperate

Esercizi proposti

7.3-P1. Dimostrare che se una successione di funzioni vk(x) ∈ D(R) converge a 0 nel senso

della Definizione 7.1-2, cioe nel senso proprio dello spazio D(R), essa converge a 0 anche nel

senso dello spazio S(R).

Soluzione. Nelle ipotesi ammesse tutti i supporti delle funzioni vk sono contenuti in un

opportuno intervallo [−r, r], dunque per ogni coppia di numeri naturali p e q si ha

maxx∈R

|xpDqvk(x)| = max|x|≤r

|xpDqvk(x)| ≤ rp‖vk‖∞ → 0.

7.3-P2. Dimostrare che se una successione di funzioni vk(x) converge a 0 in S(R), lo stesso

vale per la successione p(x)vk(x), quale che sia il polinomio p.

Soluzione. Basta dimostrare l’affermazione nel caso in cui p e un monomio, diciamo xn

con n naturale. Si tratta di dimostrare che, per ogni p, q ∈ N, la successione di funzioni

k → xpDq[xnvk(x)] tende uniformemente a 0 in R. Ora si ha

xpDq[xnvk(x)] = xp

q∑h=0

(q

h

)Dhxn Dq−hvk(x) =

min(q,n)∑h=0

c(n, q, h) xp+n−h Dq−hvk(x),

dove c(n, q, h) indica un coefficiente dipendente da n, q e h. Infatti Dhxn vale 0 se h > n, e

un monomio di grado n − h in caso contrario. Dunque

‖xpDq[xnvk(x)]‖∞ ≤min(q,n)∑

h=0

|c(n, q, h)| ‖xp+n−h Dq−hvk(x)‖∞ → 0.

7.3-P3. Dimostrare direttamente, in base alla definizione di F -trasformata di una distribu-

zione temperata, che la trasformata di δ(x − x0) e e−ix0ξ.

Soluzione. Si ha 〈F [δ(x−x0)](ξ), v(ξ)〉 = 〈δ(x−x0), v(x)〉 = v(x0); poiche abbiamo indicato

con la lettera x l’argomento della trasformata della funzione test v, abbiamo per tale trasfor-

mata un’espressione del tipo∫

Re−ixξ v(ξ) dξ. Dunque 〈δ(x − x0), v(x)〉 =

∫R

e−ix0ξ v(ξ) dξ,

quindi la trasformata richiesta coincide con e−ix0ξ.

Si osservi che ξ → e−ix0ξ e una funzione limitata, e come tale e (piu esattamente: induce)

una distribuzione temperata.

7.3-P4. Nota la trasformata di Fourier della funzione cos 2x, calcolare la trasformata delle

funzioni sin2 x e cos2 x sfruttando le formule

sin2 x =1 − cos 2x

2, cos2 x =

1 + cos 2x

2.

Alternativamente, nota la trasformata della funzione eikx, con k reale, ottenere le stesse

trasformate utilizzando le formule di Eulero.

Soluzione. La trasformata di cos 2x vale

π[δ(ξ + 2) + δ(ξ − 2)];

troveremo dunque per la trasformata di sin2 x

1

2[2πδ(ξ) − πδ(ξ + 2) − πδ(ξ − 2)] = π [δ(ξ) − 1

2δ(ξ + 2) − 1

2δ(ξ − 2)].

Alternativamente, da

sin2 x =

(eix − e−ix

2i

)2

= −ei2x + e−i2x

4+

1

2

Page 306: Eserciziario Di Analisi Matematica 2

©c 88-08-07923-6 Esercizi 9

segue lo stesso risultato, in quanto la trasformata di e±i2x e 2π δ(ξ ± 2).

7.3-P5. Abbiamo dimostrato (v. pag. 285, esempio 7.3-6) che se una distribuzione temperata

f e tale che xf(x) = 0, allora f(x) = cδ(x) con c costante. Dimostrare, analogamente, che

se f e tale che x2f(x) = 0 allora f(x) = c0δ(x) + c1δ′(x), con c0 e c1 costanti opportune.

In breve: la distribuzioni temperate per cui x2f(x) = 0 sono tutte (e soltanto) quelle del

tipo f(x) = c0δ(x)+c1δ′(x). Il risultato si estende per induzione al caso generale: xnf(x) = 0

se e solo se f e una combinazione lineare della δ e delle sua derivate fino all’ordine n − 1.

Soluzione. Da x(xf(x)) = 0, segue xf(x) = c δ(x). F -trasformando si trova i f ′(ξ) = c,

cioe f ′(ξ) = −c i, da cui f(ξ) = −c i ξ + c0 = c1 i ξ ·1+ c0 ·1 = c1δ′(ξ)+ c0δ(ξ), avendo posto

c1 = −c, da cui finalmente segue il risultato anti-trasformando.

7.3-P6. Calcolare la trasformata di f(x) = χ[−a,a](x) a partire dal fatto che f ′(x) =

= 0 + δ(x + a) − δ(x − a) = δ(x + a) − δ(x − a).

Soluzione. Trasformando l’ultima uguaglianza si ottiene

iξ f(ξ) = eiaξ − e−iaξ.

Le distribuzioni che risolvono l’equazione scritta sono tutte (e soltanto) quelle del tipo

ξ → eiaξ − e−iaξ

iξ+ c δ(ξ),

ma poiche f(ξ), in quanto trasformata di una funzione sommabile, e continua, si ha neces-

sariamente c = 0, dunque

f(ξ) =2 sin(aξ)

ξ.

7.3-P7. Calcolare la trasformata della funzione f(x) = x χ[−1,1](x), a partire dal fatto che

la sua derivata e esprimibile mediante la funzione χ[−1,1](x) e la delta di Dirac.

Soluzione. Si trova f ′(x) = χ[−1,1](x) − δ(x + 1) − δ(x − 1), distribuzione che ha come

trasformata

2sin ξ

ξ− eiξ − e−iξ = 2

(sin ξ

ξ− cos ξ

).

Ragionando come nel precedente esercizio si trova

f(ξ) =2

i

sin ξ − ξ cos ξ

ξ2.

Si osservi che la trasformata e puramente immaginaria e dispari, in quanto f e reale dispari.

Tale trasformata si annulla nell’origine in quanto sin ξ − ξ cos ξ = O(ξ3), per ξ → 0.

Suggeriamo di ricalcolare direttamente la trasformata in esame come −2i∫ 1

0sin(ξx) x dx.

7.3-P8. Calcolare la F -trasformata della funzione f(x) := (1 − |x|)+ (v. esempio 6.2-5) a

partire dal fatto che f ′′(x) = δ(x + 1) − 2δ(x) + δ(x − 1).

Soluzione. Si ha Df(x) = χ(−1,0)(x)−χ(0,1)(x), da cui f ′′(x) = 0+δ(x+1)−2δ(x)+δ(x−1).

Trasformando l’ultima uguaglianza si ottiene

−ξ2f(ξ) = eiξ − 2 + e−iξ = −2(1 − cos ξ).

per quanto sappiamo dal precedente esercizio 7.3-P5, sono soluzione dell’equazione scritta

tutte (e soltanto) le distribuzioni

ξ → 21 − cos ξ

ξ2+ c0δ(ξ) + c1δ

′(ξ);

ma poiche f e sommabile, la sua trasformata e una funzione continua, dunque

f(ξ) = 21 − cos ξ

ξ2= 4

sin2(ξ/2)

ξ2=

(sin(ξ/2)

ξ/2

)2

.

Page 307: Eserciziario Di Analisi Matematica 2

10 Capitolo 7. Distribuzioni ©c 88-08-07923-6

7.3-P9. Calcolare la F -trasformata della funzione f(x) := (1 − x2)+ a partire dal fatto che

f ′′(x) = −2χ(−1,1)(x) + 2δ(x + 1) + 2δ(x − 1). Ritrovare lo stesso risultato mediante un

calcolo diretto.

Soluzione. Si osservi che si ha Df(x) = −2x χ(−1,1)(x), da cui

f ′′(x) = −2χ(−1,1)(x) + 2δ(x + 1) + 2δ(x − 1).

-1 1

1

-1

1

1

Figura 7.E-8. A sinistra il grafico della funzione f(x) := (1− x2)+, a destra il grafico della

derivata prima.

Trasformando l’ultima uguaglianza si ottiene

−ξ2f(ξ) = −4sin ξ

ξ+ 2eiξ + 2e−iξ = −4

sin ξ

ξ+ 4 cos ξ.

Ragionando come nel precedente esercizio si trova

f(ξ) = 4sin ξ − ξ cos ξ

ξ3.

Si osservi che, poiche sin t = t − t3/6 + O(t5) mentre cos t = 1 − t2/2 + O(t4), il numeratore

ha uno zero del terzo ordine nell’origine, al pari del denominatore.

Poiche la funzione data e reale e pari, la sua trasformata si riduce all’integrale

2

∫ 1

0

(1 − x2) cos(ξx) dx,

che puo essere calcolato con due integrazioni per parti.

7.3-P10. Si vuole calcolare la F -trasformata della funzione f(x) := x χ[0,1](x), che vale x

per x ∈ [0, 1] e 0 altrimenti. Eseguire innanzitutto un calcolo in base alla definizione, poi

ritrovare lo stesso risultato calcolando la derivata seconda di f (nel senso delle distribuzioni)

e trasformando quest’ultima.

Soluzione. Si trova, con un calcolo diretto,

f(ξ) =e−iξ(1 + iξ) − 1

ξ2

La derivata di f e esprimibile mediante χ[0,1] e δ:

f ′(x) = χ[0,1](x) − δ(x − 1).

Un’ulteriore derivazione fornisce

f ′′(x) = δ(x) − δ(x − 1) − δ′(x − 1),

Page 308: Eserciziario Di Analisi Matematica 2

©c 88-08-07923-6 Esercizi 11

da cui F [f ′′](ω) = 1−e−iξ−iξ e−iξ. Poiche f e sommabile si trova finalmente f(ξ) dividendo

per −ξ2, ottenendo nuovamente il risultato iniziale.

Si osservi che da

e−iξ = 1 − iξ − ξ2/2 + O(ξ3)

segue

e−iξ(1 + iξ) = 1 + ξ2/2 + O(ξ3)

quindi limξ→0 f(ξ) = 1/2.

7.3-P11. Calcolare la F -trasformata della funzione cos x deducendola da quella della fun-

zione sin x in base alla regola sulla trasformata della derivata.

Suggerimento Sfruttare il fatto che (ξ ± 1) δ(ξ ± 1) = 0.

Page 309: Eserciziario Di Analisi Matematica 2

8. ApplicazioniEsercizi

[Aggiornamento: febbraio 2005]

http://newton.ciram.unibo.it/~barozzi/MI2/PDF/MI2-Cap.8-Ese.pdf

8.1. Problemi ai limiti per equazioni differenziali omogenee

8.1-1. Dimostrare che l’equazione differenziale p0(x) y′′(x)+p1(x) y′(x)+p2(x) y(x) == 0 puo essere posta nella forma autoaggiunta (1) moltiplicandola per un’opportunafunzione = 0.

Suggerimento. Sia f(x) la funzione che funge da moltiplicatore; allora dev’essere(f(x) p0(x)

)′ = f(x)p1(x) . . . . Si ottiene un’equazione differenziale lineare del primoordine la cui soluzione e f(x) = exp

( ∫ x

x0[p1(t)/p0(t)] dt

)/p0(x).

Soluzione. Non e restrittivo suppporre p0(x) > 0. L’uguaglianza (fp0)′ = fp1 siscrive anche

f ′ =p1 − p′o

p0f

equazione differenziale la cui soluzione e

f(x) = c exp(∫ x

x0

p1(t) − p′0(t)p0(t)

dt

)=

= c exp(∫ x

x0

p1(t)p0(t)

dt − logp0(x)p0(x0)

)=

=c p0(x0)p0(x)

exp(∫ x

x0

p1(t)p0(t)

dt

)dq cui segue la soluzione indicata nel suggerimento scegliendo opportunamente lacostante c.

8.1-2. Trovare autovalori ed autofunzioni del problema di valori ai limiti

y′′ + λy = 0, y′(0) = 0, y(π) = 0.

Soluzione. Per λ > 0, poniamo λ = ω2 ⇐⇒ ω =√

λ; l’equazione in esame ammettela soluzione generale y(x) = c1 cos(ω x)+ c2 sin(ω x); dunque y′(x) = −c1ω sin(ω x)+c2ω cos(π) = 0. La condizione y′(0) = 0 diventa c2 = 0; la condizione y(π) = 0 siscrive dunque

cos(ω π) = 0 ⇐⇒ ω π =π

2+ kπ =

2k + 12

π, k ∈ N.

Abbiamo dunque gli autovalori λk = (2k + 1)2/4, k ∈ N.Per λ = 0 l’equazione in esame ammette la soluzione generale y(x) = c1 +c2x, con

derivata y′(x) = c2. La condizione y′(0) = 0 implica c2 = 0; la condizione y(π) =0implica c1 = 0. Dunque 0 non e un autovalore.

Page 310: Eserciziario Di Analisi Matematica 2

2 Capitolo 8. Applicazioni ©c 88-08-07923-6

Per λ < 0, poniamo λ = −ω2 ⇐⇒ ω =√|λ|; l’equazione in esame ammette la

soluzione generale

y(x) = c1eω x + c2e

−ω x;

le condizioni ai limiti conducono al sistema

c1 + c2 = 0

c1eω π − c2e

−ω π = 0

che ammette soltanto la soluzione nulla. Dunque non esistono autovalori negativi.

8.1-3. Trovare autovalori ed autofunzioni del problema di valori ai limiti

y′′ + λy = 0, y′(0) = 0, y′(π) = 0.

Soluzione. Per λ > 0, poniamo λ = ω2 ⇐⇒ ω =√

λ; l’equazione in esame ammettela soluzione generale y(x) = c1 cos(ω x)+ c2 sin(ω x); dunque y′(x) = −c1ω sin(ω x)+c2ω cos(π) = 0. La condizione y′(0) = 0 diverta c2 = 0; la condizione y′(π) = 0 siscrive

sin(ω π) = 0 ⇐⇒ ω = k ⇐⇒ λ = k2, k ∈ N∗.

Si hanno dunque gli autovalori λk = k2, k ∈ N∗, con le corrispondenti autofunzioni

yk(x) = cos(kx).Per λ = 0 l’equazione in esame ammette la soluzione generale y(x) = c1 + c2x,

con derivata y′(x) = c2. Abbiamo dunque la condizione c2 = 0, e l’autofunzioney0(x) = 1.

Per λ < 0, poniamo λ = −ω2 ⇐⇒ ω =√|λ|; l’equazione in esame ammette la

soluzione generale

y(x) = c1eω x + c2e

−ω x;

le condizioni ai limiti conducono al sistema

c1 − c2 = 0

c1eω π − c2e

−ω π = 0

che ammette soltanto la soluzione nulla. Dunque non esistono autovalori negativi.

Esercizi proposti

8.1-P.1. Verificare che la successione sin kx, k ∈ N∗, e ortogonale nello spazio L2[0, π].

Suggerimento Utilizzare l’identita sinα sinβ = (−1/2)[cos(α + β) − cos(α − β)].Alternativamente, sfruttare le formule di Eulero tenendo conto del fatto che, per nintero, si ha∫ π

0

einx dx =

π, se n = 00, se n = 2k, k ∈ N

∗,2i/n, se n = 2k + 1, k ∈ N.

Soluzione. Si tratta di calcolare, per h, k ∈ N∗, l’integrale∫ π

0

sin(hx) sin(kx) dx = −12

∫ π

0

[cos((h + k)x) − cos((h − k)x)] dx.

Se h = k, una primitiva della funzione integranda e

sin((h + k)x)h + k

− sin((h − k)x)h − k

,

da cui cui segue subito l’annullamento dell’integrale in questione. Se poi h = k siottiene

Page 311: Eserciziario Di Analisi Matematica 2

©c 88-08-07923-6 Esercizi 3

∫ π

0

sin2(hx) dx = −12

∫ π

0

[cos(2hx) − 1] dx =π

2.

Le analoghe relazioni di ortogonalita∫ π

0

cos(hx) cos(kx) dx =

0, se h = k,π, se h = k = 0,π/2, se h = k > 0

sono state verificate nell’esercizio 4.3-5.

8.2 Polinomi ortogonali

8.2-1. Mediante ripetuta integrazione per parti mostrare che, per ogni coppia dinumeri naturali n e m, si ha∫ b

a

(b − x)m

m!(x − a)n

n!dx =

(b − a)m+n+1

(m + n + 1)!;

in particolare (a = −1, b = 1, m = n):∫ 1

−1

(1 − x2)n dx = 22 · 4 · . . . · 2n

1 · 3 · . . . · (2n + 1)= 2

(2n)!!(2n + 1)!!

.

Suggerimento Una primitiva di (x − a)n/n! e (x − a)n+1/(n + 1)!; la derivata di(b − x)m/m! e −(b − x)m−1/(m − 1)!.

8.2-2. Per stabilire la formula (5′) si tratta di calcolare∫ 1

−1

P 2n(x) dx =

1[(2n)!!]2

∫ 1

−1

Dn(x2 − 1)n Dn(x2 − 1)n dx.

Con un’integrazione per parti mostrare che∫ 1

−1

Dn(x2 − 1)n Dn(x2 − 1)n dx =

=[Dn−1(x2 − 1)n Dn(x2 − 1)n

]1

−1−

∫ 1

−1

Dn−1(x2 − 1)n Dn+1(x2 − 1)n dx =

−∫ 1

−1

Dn−1(x2 − 1)n Dn+1(x2 − 1)n dx.

Cosı proseguendo si trovi, dopo n integrazioni per parti,∫ 1

−1

Dn(x2 − 1)n Dn(x2 − 1)n dx =

= (−1)n

∫ 1

−1

Dn−n(x2 − 1)n Dn+n(x2 − 1)n dx =

= (−1)n(2n)!∫ 1

−1

(x2 − 1)n dx = (2n)!∫ 1

−1

(1 − x2)n dx =

= 2(2n)! (2n)!!(2n + 1)!!

,

dove si e utilizzato il risultato del precedente esercizio.Suggerimento Poiche (x2 − 1)n = (x − 1)n (x + 1)n ha uno zero di ordine n tantoin 1 quanto in −1, le derivate di tale polinomio di ordine inferiore a n si annullano intali punti. Si sfrutti il fatto che la derivata di ordine 2n di (x2 − 1)n coincide con laderivata del termine direttivo x2n.

8.2-3. Combinare i risultati dei due precedenti esercizi per ottenere nuovamente laformula (4) relativa alla norma di Pn.

8.2-4. Si consideri nuovamente la rappresentazione integrale

Page 312: Eserciziario Di Analisi Matematica 2

4 Capitolo 8. Applicazioni ©c 88-08-07923-6

Pn(x) :=1

2n · 2πi ˇ

∮γ(x)

(s2 − 1)n

(s − x)n+1ds

che abbiamo utilizzato per dimostrare la Proposizione 8.2-3. Si verifichi che sceglien-do come circuito γ(x) la circonferenza di centro x e raggio

√1 − x2 (v. figura 8.2-6),

essa si scrive

Pn(x) :=12π

∫ 2π

0

(x + i

√1 − x2 sinϕ

)ndϕ.

Suggerimento Dalla parametrizzazione s := x +√

1 − x2 eiϕ, 0 ≤ ϕ ≤ 2π, dellacirconferenza γ(x), segue ds = i

√1 − x2 eiϕ dϕ, s − x =

√1 − x2 eiϕ, e

s2 − 1 = x2 + (1 − x2) ei2ϕ + 2x√

1 − x2 eiϕ − 1 =

= (1 − x2) ei2ϕ + 2x√

1 − x2 eiϕ − (1 − x2) =

=√

1 − x2 eiϕ[2x +

√1 − x2 (eiϕ − (e−iϕ)

]=

= 2√

1 − x2 eiϕ[x + i

√1 − x2 sinϕ

].

Soluzione. Basta proseguire i calcoli iniziati nel suggerimento, tenendo conto delfatto che s − x =

√1 − x2 eiϕ.

1

1

x

s

x + i√

1 − x2

x − i√

1 − x2

Figura 8.2-6.

Per ogni x ∈ [−1, 1] il punto

x + i√

1 − x2 sin ϕ appartiene

al segmento di estremi

x + i√

1 − x2, x + i√

1 − x2,

contenuto nel disco di centro

l’origine e raggio 1.

8.2-5. Dedurre dalla rappresentazione integrale ottenuta nel precedente esercizio ladisuguaglianza |Pn(x)| ≤ 1 per ogni naturale n e per ogni x ∈ [−1, 1]. Si riveda, inproposito, la figura 8.1-1.

Suggerimento Verificare che |x + i√

1 − x2 sinϕ| ≤ 1 per x ∈ [−1, 1], calcolando ilquadrato del valore assoluto in esame.

Soluzione. Si ha

|x + i√

1 − x2 sin ϕ|2 = x2 + (1 − x2) sin2 ϕ ≤ x2 + 1 − x2 = 1.

8.2-6. Dedurre dalla formula ricorsiva (18) che i coefficienti dei polinomi di Cebysevsono interi. Discorso analogo per i polinomi di Hermite, a partire dalla formula (25).

8.2-7. Dimostrare che il polinomio di Cebysev Tn(x), per n ≥ 1, si annulla nei punti

xk = cos(2k + 1)π

2n, k = 0, 1, 2, . . . , n − 1,

Soluzione. Si ha Tn(xk) = cos(n arccos(xk)) = cos[(2k + 1)π/2] = 0.

8.3. Problemi ai limiti per equazioni differenziali non omogeneeLa funzione di Green

8.3-1. Siano y1 e y2 due soluzioni dell’equazione differenziale lineare omogenea−(py′)′ + qy = 0, che si puo anche scrivere py′′ = −p′y′ + qy. Dimostrare che per ilrelativo wronskiano

Page 313: Eserciziario Di Analisi Matematica 2

©c 88-08-07923-6 8.3 Equazioni differenziali non omogenee 5

w(x) =

∣∣∣∣∣∣y1(x) y2(x)

y′1(x) y′

2(x)

∣∣∣∣∣∣si ha p(x) w(x) = costante, in quanto

(p(x) w(x)

)′ = 0.Suggerimento Si tratta di dimostrare che pw′ = −p′w. Ora

w′(x) =

∣∣∣∣∣∣y′1(x) y′

2(x)

y′1(x) y′

2(x)

∣∣∣∣∣∣ +

∣∣∣∣∣∣y1(x) y2(x)

y′′1 (x) y′′

2 (x)

∣∣∣∣∣∣ =

∣∣∣∣∣∣y1(x) y2(x)

y′′1 (x) y′′

2 (x)

∣∣∣∣∣∣ .

Si moltiplichino entrambi i membri per p, scrivendo py′′1 e py′′

2 nella seconda rigadell’ultimo determinante, e si sfruttino le uguaglianze py′′

k = −p′y′k + qyk, k = 1, 2.

Soluzione. Proseguendo secondo quanto suggerito si trova

pw′ =

∣∣∣∣∣∣y1 y2

py′′1 py′′

2

∣∣∣∣∣∣ =

∣∣∣∣∣∣y1 y2

−p′y′1 + qy1 −p′y′

2 + qy2

∣∣∣∣∣∣ = −p′

∣∣∣∣∣∣y1 y2

y′1 y′

2

∣∣∣∣∣∣ = −p′w.

Si tenga presente che un determinante con due righe uguali e nullo.

8.3-2. Dedurre dal precedente esercizio che, se p ha segno costante sull’intervallo didefinizione [a, b], ad esempio p(x) > 0, allora o il wronskiano w e identicamente nullo,oppure e anch’esso di segno costante. La prima alternativa si verifica se e solo se y1

e y2 sono linearmente dipendenti.

8.3-3. Si consideri il problema ai limiti dell’esempio 8.3-1, cioe −y′′ = f , y(0) == y(1) = 0. Si scriva esplicitamente la formula risolutiva (8) e si verifichi che, incorrispondenza dei termini noti f indicati nella tabella seguente, si hanno le soluzioniindicate a fianco di ciascuno.

f(x) y(x) f(x) y(x)

1 (x − x2)/2 x (x − x3)/6

x2 (x − x4)/12 x3 (x − x5)/20

Potete fare una congettura sulla soluzione corrispondente al termine noto f(x) = xn?

Soluzione. Scegliendo come soluzioni dell’equazione omogenea y1(x) = x e y2(x) =1 − x, il cui wronskiano vale −1, si ottiene la formula risolutiva

y(x) = (1 − x)∫ x

0

ξ f(ξ) dξ + x

∫ 1

x

(1 − ξ) f(ξ) dξ.

Per f(x) = xn essa fornisce la soluzione

x − xn+2

(n + 1)(n + 2).

8.3-4. Relativamente allo stesso problema ai limiti del precedente esercizio, calcolarele soluzioni corrispondenti ai termini noti f(x) = sin(πx), f(x) = cos(πx), f(x) = ex.In ciascun caso verificare la soluzione ottenuta.

Soluzione. Si trovano, nell’ordine, le soluzioni sin(πx)/π2, (cos(πx) + 2x − 1)/π2,1 + (e − 1)x − ex.

8.3-5. Verificare che tutte le soluzioni calcolate nei due precedenti esercizi si pre-sentano come somma di una soluzione dell’equazione omogenea e di una soluzionedell’equazione non omogenea.

8.3-6. Si consideri il problema ai limiti −y′′ = f , y(0) + y′(0) = 0, y(1) + y′(1) = 0.Dopo aver aver verificato che il corrispondente problema per l’equazione omogenea

Page 314: Eserciziario Di Analisi Matematica 2

6 Capitolo 8. Applicazioni ©c 88-08-07923-6

ammette soltanto la soluzione nulla, si costruisca la funzione di Green, si scriva laformula risolutiva (8) e si verifichi che, in corrispondenza dei termini noti f indicatinella tabella seguente, si hanno le soluzioni indicate a fianco di ciascuno.

f(x) y(x) f(x) y(x)

1 (−x2 + 3x − 3)/2 x (−x3 + 4x − 4)/6

x2 (−x4 + 5x − 5)/12 x3 (−x5 + 6x − 6)/20

Potete fare una congettura sulla soluzione corrispondente al termine noto f(x) = xn?

Soluzione. La soluzione generale dell’equazione omogenea y′′ = 0 si scrive y(x) =c1 + c2x, quindi le condizioni ai limiti diventano c1 + c2 = 0, c1 + 2c2 = 0, sistemache ammette soltanto la soluzione nulla. Per soddisfare separatamente le condizioniai limiti possiamo scegliere y1(x) = 1− x (cioe c1 = 1, c2 = −1) e y2(x) = 2− x (cioec1 = 2, c2 = −1). Poiche il wronskiano vale 1, la formula risolutiva si scrive

y(x) = (x − 2)∫ x

0

(1 − ξ) f(ξ) dξ + (x − 1)∫ 1

x

(2 − ξ) f(ξ) dξ.

Relativamente al termine noto f(x) = xn essa fornisce la soluzione

y(x) =−xn+2 + (n + 3)x − (n + 3)

(n + 1)(n + 2).

8.4. L’equazione del calore

8.4-1. Verificare che le soluzioni dell’equazione del calore che sono indipendenti daltempo sono tutte (e soltanto) quello del tipo u(x, t) = ax + b.

Soluzione. Se u e indipendente da t, u(x, t) = u(x), l’equazione del calore diventauxx = 0.

8.4-2. Verificare che i polinomi del tipo u(x, t) = ax2 + bx + c + 2at (in particolare ipolinomi di grado ≤ 1 nella sola x) sono soluzioni dell’equazione del calore ut = uxx.

Soluzione. si trova ut = uxx = 2a.

8.4-3. Sotto quale condizione sui coefficienti α e β le due funzioni eαt cos(βx) eeαt sin(βx) sono soluzioni dell’equazione del calore ut = uxx ?

Soluzione. Sotto la condizione α = −β2.

8.4-4. Verificare che la funzione

u(x, t) =1√t

exp(− x2

4t

)soddisfa l’equazione del calore ut = uxx per t > 0.

Soluzione. Si trova

ut(x, y) =[− 1

2t−3/2 +

14

t−5/2 x2]

exp(− x2

4t

);

ux(x, y) = −12

t−3/2 x exp(− x2

4t

),

uxx(x, y) =[− 1

2t−3/2 +

14

t−5/2 x2]

exp(− x2

4t

).

8.4-5. Verificare che la funzione

Page 315: Eserciziario Di Analisi Matematica 2

©c 88-08-07923-6 8.4 L’equazione del calore 7

u(x, y, t) =1t

exp(− x2 + y2

4t

)soddisfa l’equazione del calore ut = uxx + uyy per t > 0.

Soluzione. Si trova

ut(x, y) =[− t−2 +

14

t−3 (x2 + y2)]

exp(− x2 + y2

4t

);

ux(x, y) = −12t−2 x exp

(− x2 + y2

4t

),

uxx(x, y) =[− 1

2t−2 +

14

t−3 x2]

exp(− x2 + y2

4t

),

e analogamente

uyy(x, y) =[− 1

2t−2 +

14

t−3 y2]

exp(− x2 + y2

4t

).

8.4-6. Applicare il metodo di separazione delle variabili alla soluzione del problema

ut = uxx, x ∈ [0, L], t > 0, u(x, 0) = u0(x), ux(0, t) = ux(L, t) = 0,

corrispondente al caso di una sbarra conduttrice i cui estremi sono isolati.

0

0.51

1.520

1

2

3

4

0.51

1.52

Figura 8.4-5.

Soluzione approssimata

del problema posto

nell’esercizio 8.4-6, con i dati

L = 2, u0(x) = 4x − x3.

Verificare che si ottiene la soluzione sotto la forma

u(x, t) =a0

2+

∑n>0

an cos(nπ

Lx)

exp[−

(nπ

L

)2

t],

dove an = (2/L)∫ L

0u0(t) cos(nπ/L t) dt. Quanto vale il limite limt→∞ u(x, t)?

Soluzione. Procediamo in modo analogo a quanto fatto nel testo a pagine 320− 321.Posto u(x, y) = X(x)T (t), sostituendo nell’equazione uxx = ut si perviene alle dueequazioni differenziali

X ′′ = X, T ′ = kT.

Le condizioni ai limiti per la prima equazione, cioe X ′(0) = X ′(L) = 0, possono esseresoddisfatte solo per k ≤ 0, dunque k = −ω2, con ω ≥ 0. Poiche la soluzione generaledell’equazione X ′′ + ω2X = 0 si scrive

X(x) = c1 cos(ωx) + c2 sin(ωx),

da cui

X ′(x) = −c1ω sin(ωx) + c2ω cos(ωx),

Page 316: Eserciziario Di Analisi Matematica 2

8 Capitolo 8. Applicazioni ©c 88-08-07923-6

ponendo X ′(0) = 0 si ottiene c2 = 0, dunque X(x) = c1 cos(ωx). La condizioneX ′(L) = 0, cioe sin(ωL) = 0, implica che i valori accettabili per ω sono

ωn =nπ

L, n = 0, 1, 2, . . . .

Possiamo dunque scrivere una soluzione dell’equazione del calore che verifica lecondizioni ux(0, t) = ux(L, t) = 0 nella forma

u(x, t) =a0

2+

∑n≥1

an cos(nπ

Lx)

exp[−

(nπ

L

)2

t].

Per soddisfare la condizione iniziale u(x, 0) = u0(x) possiamo prolungare perparita la funzione u0 sull’intervallo [−L, L]:

∀x ∈ [0, L] : u0(−x) := u0(x),

dopodiche la condizione iniziale si scrive

u(x, 0) =a0

2+

∑n≥1

an cos(nπ

Lx)

= u0(x);

essa viene soddisfatta scegliendo come an i coefficienti di Fourier di u0.Per t → +∞ la funzione u(x, t) tende ad a0/2.

8.6. Il metodo delle serie di potenze

8.6-1. I polinomi di Cebysev sono autofunzioni dell’equazione differenziale

(1 − x2) y′′ − xy′ + λy = 0,

e piu precisamente Tn corrisponde all’autovalore λn = n2. Procedendo come nel-l’esempio 8.6-5, si utilizzi il metodo delle serie di potenze per dedurre la formula

ak+2 = − n2 − k2

(k + 1)(k + 2)ak

per il calcolo dei coefficienti di Tn. Sapendo che il termine direttivo di Tn vale 2n−1,ponendo k = n − 2h si ottenga il seguente schema

an := 2n−1,

an−2h := − (n − 2h + 1)(n − 2h + 2)4h (n − h)

an−2(h−1), h = 1, 2, . . . , n/2.

8.6-2. I polinomi di Laguerre sono autofunzioni dell’equazione differenziale

x y′′ + (1 − x) y′ + λy = 0,

e piu precisamente Ln corrisponde all’autovalore λn = n. Utilizzando il metodo delleserie potenze si deduca la formula

ak+1 = − n − k

(k + 1)2ak,

che, unita alla condizione iniziale a0 = 1, consente il calcolo ricorsivo dei coefficientidi Ln.

8.6-3. I polinomi di Hermite sono autofunzioni dell’equazione differenziale

y′′ − 2x y′ + λy = 0,

e piu precisamente Hn corrisponde all’autovalore λn = 2n. Procedendo come neiprecedenti esercizi, si ottenga la formula

ak+2 = − 2(n − k)(k + 1)(k + 2)

ak.

Page 317: Eserciziario Di Analisi Matematica 2

©c 88-08-07923-6 8.4 L’equazione del calore 9

Si ponga poi k = n − 2h, e si scriva uno schema ricorsivo che consenta il calcolo deicoefficienti del polinomio Hn a partire dal coefficiente direttivo dn = 2n.

Esercizi proposti

8.6-P.1. Si chiama equazione ipergeometrica (v. Appendice 8-E) l’equazione differen-ziale

x(1 − x) y′′ + [c − (a + b + 1)x] y′ − ab y = 0,

dove a, b e c sono parametri reali. Dimostrare che il metodo delle serie di potenzeconduce alla formula ricorsiva

ak+1 =(a + k)(b + k)(1 + k)(c + k)

ak,

e pertanto, se si pone a0 = 1, alla serie ipergeometrica

1 +∑k≥1

a(a + 1) . . . (a + k − 1) b(b + 1) . . . (b + k − 1)k! c(c + 1) . . . (c + k − 1)

xk.

Se si introduce il simbolo (x)k := x(x + 1)(x + 2) . . . (x + k − 1), x ∈ R, k ∈ N∗, con

la convenzione (x)0 := 1, allora si puo scrivere la serie ipergeometrica nella forma

F (a, b, c;x) =∑k≥0

(a)k (b)k

k! (c)kxk.

Si dimostri che il raggio di convergenza della serie scritta e 1. Si osservi che k! = (1)k.

Soluzione. Posto y(x) =∑

k≥0 akxk, sostituendo nell’equazione ipergeometrica sitrova

x(1− x)∑k≥2

k(k − 1) akxk−2 + [c− (a + b + 1)x]∑k≥1

k akxk−1 − ab∑k≥0

akxk = 0

uguaglianza che puo essere riscritta successivamente

−∑k≥2

k(k − 1) akxk +∑k≥2

k(k − 1) akxk−1+

+ c∑k≥1

kakxk−1 + (a + b + 1)∑k≥1

kakxk − ab∑k≥0

akxk = 0,

−∑k≥0

k(k − 1) akxk +∑k≥0

k(k + 1) ak+1xk+

+c∑k≥1

(k + 1) ak+1xk + (a + b + 1)

∑k≥0

k akxk − ab∑k≥0

akxk = 0.

Uguagliando a 0 il coefficiente di xk si trova

ak+1(k + 1)(c + k) = ak[k(k − 1) + k(a + b + 1) + ab]

da cui segue la formula ricorsiva indicata.Per calcolare il raggio di convergenza della serie ipergeometrica possiamo calcolare

il limite

limk→∞

ak

ak+1

a patto che esso esista. Si rivedano le pagine 136 e 137 del testo. Ma il limite delrapporto indicato esiste e vale 1 in quanto esso si scrive

(1 + k)(c + k)(a + k)(b + k)

.

8.6-P.2. Verificare le identita:

Page 318: Eserciziario Di Analisi Matematica 2

10 Capitolo 8. Applicazioni ©c 88-08-07923-6

F (1, 1, 1;x) = 1 + x + . . . + xn + . . . =1

1 − x, F (−n, 1, 1;x) = (1 − x)n.

Soluzione. Per a = b = c = 1 il coefficiente di xk vale(1)k(1)k

(1)k(1)k= 1.

Per a = −n, con n naturale e b = c = 1, il coefficiente di xk vale

(−n)k

k!=

−n(−n + 1) . . . (−n + k − 1)k!

=

= (−1)k n(n − 1)(n − 2) . . . (n − k + 1)k!

= (−1)k

(n

k

).

8.6-P.3. Calcolare la serie di Taylor della funzione f(x) = arctanx:

f(x) = arctanx =∑k≥0

ak xk

sapendo che a0 = 0 e si ha

f ′(x) =1

1 + x2⇐⇒ (1 + x2) f ′(x) = 1.

Soluzione. Si ha innanzitutto a0 = f(0) = 0. Si ha poi f ′(x) =∑

k≥1 k ak xk−1.Sostituendo nell’ultima equazione scritta si ha, in un primo tempo,∑

k≥1

k ak xk−1 +∑k≥1

k ak xk+1 = 1,

uguaglianza che si puo riscrivere∑k≥0

(k + 1) ak+1 xk +∑k≥2

(k − 1) ak−1 xk = 1.

Abbiamo dunque, per k = 0, a1 = 1, per k = 1, a2 = 0 e, per k ≥ 2, la formularicorsiva

ak+1 = −k − 1k + 1

ak−1.

Dunque tutti i coefficienti ak di indice pari sono nulli (in accordo col fatto che lafunzion arcotangente e dispari), mentre si ha

a3 = −13

a1 = −13, a5 = −3

5a3 =

15, . . . ;

in generale a2k+1 = (−1)k/(2k + 1), cioe

arctanx = x − x3

3+

x5

5− x7

7+ . . . .

8.6-P.4. Calcolare la serie di Taylor della funzione f(x) = ln(1 + x):

f(x) = ln(1 + x) =∑k≥0

ak xk

sapendo che a0 = 0 e si ha

f ′(x) =1

1 + x⇐⇒ (1 + x) f ′(x) = 1.

Soluzione. Si ha innanzitutto a0 = f(0) = 0. Si ha poi f ′(x) =∑

k≥1 k ak xk−1 =∑k≥0(k + 1) ak+1 xk. Sostituendo nell’ultima equazione scritta si ha dunque∑

k≥0

[(k + 1) ak+1 + k ak]xk = 1,

Page 319: Eserciziario Di Analisi Matematica 2

©c 88-08-07923-6 8.4 L’equazione del calore 11

da cui segue, per k = 0, a1 = 1, e, per k > 0, la formula ricorsiva

ak+1 = − k

k + 1ak.

Dunque

a2 = −12, a3 = −2

3·(− 1

2

)=

13, a4 = −3

4· 13

= −14, . . . ;

in generale ak = (−1)k+1/k, cioe

ln(1 + x) = x − x2

2+

x3

3− x4

4+ . . . .

8.6-P.5. Calcolare la serie di Taylor della funzione f(x) = ln[(1 + x)/(1 − x)]:

f(x) = ln1 + x

1 − x=

∑k≥0

ak xk

sapendo che a0 = 0 e si ha

f ′(x) =2

(1 − x)2⇐⇒ (1 − x2) f ′(x) = 2.

Soluzione. Posto f(x) =∑

k≥1 akxk, si trova

(1 − x2)∑k≥1

kakxk−1 = 2,

uguaglianza che si riscrive successivamente∑k≥1

kakxk−1 −∑k≥1

kakxk+1 = 2,

∑k≥0

(k + 1) ak+1xk −

∑k≥2

(k − 1) ak−1xk = 2.

Uguagliando a primo e a secondo membro i termini di grado 0 e 1 si ottiene

a1 = 2, 2a2 = 0 =⇒ a2 = 0;

per ogni k ≥ 2, uguagliando i termini di grado k si ottiene

k + 1) ak+1 = (k − 1) ak−1 =⇒ ak+1 =k − 1k + 1

ak−1.

Ne segue che tutti i coefficienti di indice pari sono nulli, mentre si ha

a3 =13

a1 =23, a5 =

35

a3 =25,

e, in generale, a2k+1 = 2/(2k + 1). In conclusione

f(x) = ln1 + x

1 − x= 2

∑k≥0

x2k+1

2k + 1.

8.6-P.6. Con riferimento ai precedenti esercizi, verificare che

ln(1 + x) = x F (1, 1, 2;−x), ln1 + x

1 − x= 2x F (1/2, 1, 3/2; x2).

arctanx = x F (1/2, 1, 3/2;−x2), arcsin x = x F (1/2, 1/2, 3/2;x2).

Soluzione. Occupiamoci della prima uguaglianza. Si ha

x F (1, 1, 2;−x) = x∑k≥0

(1)k (1)k

(1)k (2)k(−1)k xk =

∑k≥0

k!(k + 1)!

(−1)k xk+1 =

=∑k≥0

(−1)k xk+1

k + 1= x − x2

2+

x3

3+ . . . .

Page 320: Eserciziario Di Analisi Matematica 2

12 Capitolo 8. Applicazioni ©c 88-08-07923-6

8.6-P.7. Risolvere per serie il problema di valori iniziali (1 + x2)y′ = 2xy, y(0) = 1,verificando che la soluzione e un polinomio.

Soluzione. Posto y(x) =∑

k≥0 ak xk, si trova y′(x) =∑

k≥1 kak xk−1, quindi sosti-tuendo nell’equazione considerata∑

k≥1

kak xk−1 +∑k≥1

kak xk+1 = 2∑k≥0

ak xk+1,

uguaglianza che si puo scrivere in forma equivalente∑k≥0

(k + 1)ak+1 xk +∑k≥2

(k − 1)ak−1 xk − 2∑k≥1

ak−1 xk =

= a1 + (2a2 − 2a0) x +∑k≥2

[(k + 1)ak+1 − (k − 3) ak−1]xk = 0. (∗)

Si ha innanzitutto a0 = y(0) = 1; uguagliando a 0 i coefficienti dei termini di grado0 e 1 nella serie a primo membro di (*) si ha a1 = 0, 2a2 − 2a0 = 0 =⇒ a2 = 1,mentre uguagliando a 0 il coefficiente di xk, per k ≥ 2, si ha la formula ricorsiva

ak+1 = −k − 3k + 1

ak−1.

Da essa segue che tutti i coefficienti di indice dispari sono nulli, e poiche (ponendok = 3) si ha a4 = 0, sono nulli anche tutti i coefficienti di ordine pari a partire da a4

stesso. Ne segue che la soluzione e y(x) = 1+x2, la cui correttezza e subito verificata.

8.6-P.8. Generalizzare il precedente esercizio risolvendo per serie il problema di valoriiniziali (1 + x2)y′ = 2 p x y, y(0) = 1, con p reale, verificando che a0 = 1, i coefficientiak sono nulli per k dispari e, per ogni k > 0,

a2k =p(p − 1) . . . (p − k + 1)

k!=

(p

k

),

quindi, ricordando la serie binomiale, y(x) = (1 + x2)p.

8.6-P.9. Risolvere per serie il problema di valori iniziali y′′ + xy′ + y = 0, y(0) = 1,y′(0) = 0, verificando che la soluzione e esprimibile mediante la funzione esponenziale:y(x) = eφ(x), con φ funzione opportuna..

Soluzione. Posto y(x) =∑

n≥0 an xn, si trova a0 = 1, a1 = 0, e, per n ≥ 0, la formularicorsiva

an+2 = − an

n + 2.

Dunque per i coefficienti di indice dispari si ha a2n+1 = 0 e per quelli di indice pari

a2n =1

(2n)!!=

12n n!

.

Ne segue che la soluzione si scrive

y(x) =∑n≥0

1n!

(−x2

2

)n

;

confrontando con la serie esponenziale si riconosce che y(x) = exp(−x2/2).

8.7. Le funzioni di Bessel

8.7-P.1. Risolvere per serie l’equazione differenziale xy′′+y′+y = 0, x ∈ R. Mostrareche la soluzione ottenuta con la condizione iniziale y(0) = 1 e esprimibile, per x ≥ 0,mediante la funzione di Bessel J0 nella forma J0(φ(x)), con φ funzione opportuna.

Soluzione. Posto y(x) =∑

k≥0 ak xk,si trova

Page 321: Eserciziario Di Analisi Matematica 2

©c 88-08-07923-6 8.7 Le funzioni di Bessel 13

∑k≥2

k(k − 1) akxk−1 +∑k≥1

kak xk−1 +∑k≥0

ak xk = 0.

Nella prima somma l’indice puo partire da 1, quindi le prime due somme si compat-tano nell’unica somma

∑k≥1 k2ak xk−1 che puo riscriversi

∑k≥0(k + 1)2ak+1 xk. In

conclusione abbiamo l’uguaglianza∑k≥0

[(k + 1)2ak+1 + ak]xk = 0,

da cui la formula ricorsiva

ak+1 = − ak

(k + 1)2

Da a0 = 1, segue dunque

a1 = −1, a2 =122

, a3 = − 1(2 · 3)2

,

in generale ak = (−1)k/(k!)2, quindi

y(x) =∑k≥0

(−1)k

(k!)2xk.

Ora

J0(φ(x)) =∑k≥0

(−1)k

(k!)2

(φ(x)

2

)2k

,

espressione che coincide con la precedente a patto che sia φ(x)2/4 = x, cioe si scelgaφ(x) = 2

√x.

Operatori differenziali nel piano e nello spazio

Esercizi proposti

1. Verificare che la funzione u(x, y) = log(x2 + y2) e soluzione dell’equazione diLaplace ∇2u = uxx + uyy = 0, in tutti i punti del piano distinti dall’origine, cioe perx2 + y2 > 0.

Soluzione. Infatti

ux =x

x2 + y2, uy =

y

x2 + y2,

uxx =x2 + y2 − 2x2

(x2 + y2)2=

y2 − x2

(x2 + y2)2, uyy =

x2 − y2

(x2 + y2)2.

2. Verificare che la funzione u(x, y, z) = 1/√

x2 + y2 + z2, e soluzione dell’equazionedi Laplace ∇2u = uxx +uyy +uzz = 0, in tutti i punti dello spazio distinti dall’origine,cioe per x2 + y2 + z2 > 0.

Soluzione. Se P e il punto di coordinate (x, y, z), introduciamo la funzione r(x, y, z) =√x2 + y2 + z2; essa e semplicemente la norma euclidea del vettore di posizione OP :

r(x, y, z) =∥∥OP

∥∥2.

Si ha facilmente:

rx =x

r, ry =

y

r, rz =

z

r.

Allora per la funzione u = 1/r si trova

ux = −rx

r2= − x

r3, uxx = − 1

r3+

3xrx

r4= − 1

r3+

3x2

r5,

Page 322: Eserciziario Di Analisi Matematica 2

14 Capitolo 8. Applicazioni ©c 88-08-07923-6

e analogamente

uyy = − 1r3

+3y2

r5, uzz = − 1

r3+

3z2

r5.

In definitiva:

∇2u = − 3r3

+3(x2 + y2 + z + 2)

r5= − 3

r3+

3r2

r5= 0.

3. Si consideri la famiglia di funzioni, da R3 a C, u(x, y, z) := eax+by+cz, con a, b e c

costanti complesse non tutte nulle. Si chiede quale relazione debbano soddisfare talicostanti affinche u sia soluzione dell’equazione ∇2u = uxx + uyy + uzz = 0.

Soluzione. Si trova subito ∇2u = (a2+b2+c2)u, quindi la condizione e a2+b2+c2 = 0.Esistono infinite terne di valori che soddisfano tale condizione: ad esempio si possonoscegliere a e b reali ad arbitrio, e ricavare c = i

√a2 + b2. Le scelte a = 3, b = 4, c = 5i

sono ammissibili, fornendo la funzione u(x, y, z) = e3x+4y+iz.Scegliendo parte reale e parte immaginaria di u si ottengono le due funzioni armonichereali u1(x, y, z) = e3x+4y cos(5z), u2(x, y, z) = e3x+4y sin(5z)